bookshelf/Bookshelf/Enderton/Set.tex

9933 lines
330 KiB
TeX
Raw Normal View History

\documentclass{report}
2023-06-04 23:34:42 +00:00
\usepackage{graphicx}
\graphicspath{{./Set/images/}}
\input{../../preamble}
\makecode{../..}
\externaldocument[S:]{../../Common/Real/Sequence}
\newcommand{\ineq}{\,\mathop{\underline{\in}}\,}
\begin{document}
\header{Elements of Set Theory}{Herbert B. Enderton}
\tableofcontents
\begingroup
\renewcommand\thechapter{R}
\setcounter{chapter}{0}
\addtocounter{chapter}{-1}
\chapter{Reference}%
\hyperlabel{chap:reference}
\section{\defined{Addition}}%
2023-08-09 13:39:41 +00:00
\hyperlabel{ref:addition}
2023-08-09 13:39:41 +00:00
For each $m \in \omega$, there exists (by the
\nameref{sub:recursion-theorem-natural-numbers}) a unique
\nameref{ref:function} $A_m \colon \omega \rightarrow \omega$ for which
\begin{align*}
A_m(0) & = m, \\
A_m(n^+) & = A_m(n)^+ & \text{for } n \text{ in } \omega.
\end{align*}
\textbf{Addition} ($+$) is the \nameref{ref:binary-operation} on $\omega$ such
that for any $m$ and $n$ in $\omega$, $$m + n = A_m(n).$$
\lean{Init/Prelude}
{Add.add}
\section{\defined{Axiom of Choice, First Form}}%
\hyperlabel{ref:axiom-of-choice-1}
2023-08-09 13:39:41 +00:00
For any relation $R$ there is a function $H \subseteq R$ with
$\dom{H} = \dom{R}$.
\lean*{Init/Prelude}
{Classical.choice}
\section{\defined{Axiom of Choice, Second Form}}%
\hyperlabel{ref:axiom-of-choice-2}
2023-06-29 20:05:08 +00:00
2023-08-09 13:39:41 +00:00
For any set $I$ and any function $H$ with domain $I$, if $H(i) \neq \emptyset$
for all $i \in I$, then $$\bigtimes_{i \in I} H(i) \neq \emptyset.$$
\lean{Init/Prelude}
{Classical.choice}
\section{\defined{Binary Operation}}%
\hyperlabel{ref:binary-operation}
2023-08-09 13:39:41 +00:00
A \textbf{binary operation} on a set $A$ is a \nameref{ref:function} from
$A \times A$ into $A$.
\section{\defined{Cardinal Arithmetic}}%
\hyperlabel{sec:cardinal-arithmetic}
Let $\kappa$ and $\lambda$ be any cardinal numbers.
\begin{enumerate}[(a)]
\item $\kappa + \lambda = \card{(K \cup L)}$, where $K$ and $L$ are any
disjoint sets of cardinality $\kappa$ and $\lambda$, respectively.
\item $\kappa \cdot \lambda = \card{(K \times L)}$, where $K$ and $L$ are
any sets of cardinality $\kappa$ and $\lambda$, respectively.
\item $\kappa^\lambda = \card{(^L{K})}$, where $K$ and $L$ are any sets of
cardinality $\kappa$ and $\lambda$, respectively.
\end{enumerate}
\lean{Mathlib/SetTheory/Cardinal/Basic}
{Cardinal.add\_def}
\lean{Mathlib/SetTheory/Cardinal/Basic}
{Cardinal.mul\_def}
\lean{Mathlib/SetTheory/Cardinal/Basic}
{Cardinal.power\_def}
\section{\defined{Cardinal Number}}%
\hyperlabel{ref:cardinal-number}
For any set $C$, the \textbf{cardinal number} of set $C$ is denoted as
$\card{C}$.
Furthermore,
\begin{enumerate}[(a)]
\item For any sets $A$ and $B$,
$$\card{A} = \card{B} \quad\text{iff}\quad A \equin B.$$
\item For a finite set $A$, $\card{A}$ is the \nameref{ref:natural-number}
$n$ for which $A \equin n$.
\end{enumerate}
\lean{Mathlib/Data/Finset/Card}
{Finset.card}
\lean{Mathlib/SetTheory/Cardinal/Basic}
{Cardinal}
2023-07-08 02:30:53 +00:00
\section{\defined{Cartesian Product}}%
\hyperlabel{ref:cartesian-product}
2023-08-09 13:39:41 +00:00
Let $I$ be a set and let $H$ be a \nameref{ref:function} whose domain includes
$I$.
Then for each $i$ in $I$ we have the set $H(i)$.
We define the \textbf{cartesian product} of the $H(i)$'s as
$$\bigtimes_{i \in I} H(i) = \{f \mid
f \text{ is a function with domain } I \text{ and }
(\forall i \in I) f(i) \in H(i)\}.$$
2023-07-08 02:30:53 +00:00
2023-08-09 13:39:41 +00:00
\lean{Mathlib/Data/Set/Prod}{Set.prod}
2023-07-08 02:30:53 +00:00
\section{\defined{Compatible}}%
\hyperlabel{ref:compatible}
2023-08-09 13:39:41 +00:00
A \nameref{ref:function} $F$ is \textbf{compatible} with relation $R$ if and
only if for all $x$ and $y$ in $A$,
$$xRy \Rightarrow F(x)RF(y).$$
\lean{Init/Core}
{Quotient.lift}
\section{\defined{Composition}}%
\hyperlabel{ref:composition}
2023-06-23 20:47:25 +00:00
2023-08-09 13:39:41 +00:00
The \textbf{composition} of sets $F$ and $G$ is
$$F \circ G = \{\tuple{u, v} \mid \exists t(uGt \land tFv)\}.$$
\code{Bookshelf/Enderton/Set/Relation}
{Set.Relation.comp}
\lean{Mathlib/Data/Rel}
{Rel.comp}
\section{\defined{Connected}}%
\hyperlabel{ref:connected}
2023-08-09 13:39:41 +00:00
A binary relation $R$ on $A$ is \textbf{connected} if for distinct
$x, y \in A$, either $xRy$ or $yRx$.
\lean*{Common/Algebra/Classes}{IsConnected}
2023-06-23 20:47:25 +00:00
\section{\defined{Domain}}%
\hyperlabel{ref:domain}
2023-06-23 20:47:25 +00:00
2023-08-09 13:39:41 +00:00
The \textbf{domain} of set $R$, denoted $\dom{R}$, is given by
$$x \in \dom{R} \iff \exists y \tuple{x, y} \in R.$$
2023-06-23 20:47:25 +00:00
\code{Bookshelf/Enderton/Set/Relation}
{Set.Relation.dom}
2023-06-23 20:47:25 +00:00
\lean{Mathlib/Data/Rel}
{Rel.dom}
\section{\defined{Empty Set Axiom}}%
\hyperlabel{ref:empty-set-axiom}
2023-05-19 15:54:23 +00:00
2023-08-09 13:39:41 +00:00
There is a set having no members: $$\exists B, \forall x, x \not\in B.$$
2023-08-09 13:39:41 +00:00
\lean{Mathlib/Init/Set}{Set.emptyCollection}
\section{\defined{Equinumerous}}%
\hyperlabel{ref:equinumerous}
A set $A$ is \textbf{equinumerous} to a set $B$ (written $A \equin B$) if and
only if there is a one-to-one \nameref{ref:function} from $A$ onto $B$.
In other words, there exists a one-to-one correspondence between $A$ and $B$.
\lean*{Mathlib/Init/Function}
{Function.Bijective}
\lean{Mathlib/Data/Set/Function}
{Set.BijOn}
\lean{Mathlib/Logic/Equiv/Defs}
{Equiv}
\section{\defined{Equivalence Class}}%
\hyperlabel{ref:equivalence-class}
2023-08-09 13:39:41 +00:00
The set $[x]_R$ is defined by $$[x]_R = \{t \mid xRt\}.$$
If $R$ is an \nameref{ref:equivalence-relation} and $x \in \fld{R}$, then
$[x]_R$ is called the \textbf{equivalence class} of $x$
(\textbf{modulo $R$}).
If the relation $R$ is fixed by the context, we may write just $[x]$.
\code*{Bookshelf/Enderton/Set/Relation}
{Set.Relation.modEquiv}
\section{\defined{Equivalence Relation}}%
\hyperlabel{ref:equivalence-relation}
2023-08-09 13:39:41 +00:00
Relation $R$ is an \textbf{equivalence relation} on set $A$ if and only if
$R$ is a binary \nameref{ref:relation} on $A$ that is
\nameref{ref:reflexive} on $A$, \nameref{ref:symmetric}, and
\nameref{ref:transitive}.
\code*{Bookshelf/Enderton/Set/Relation}
{Set.Relation.isEquivalence}
\section{\defined{Exponentiation}}%
\hyperlabel{ref:exponentiation}
2023-08-09 13:39:41 +00:00
For each $m \in \omega$, there exists (by the
\nameref{sub:recursion-theorem-natural-numbers}) a unique
\nameref{ref:function} $E_m \colon \omega \rightarrow \omega$ for which
\begin{align*}
E_m(0) & = 1, \\
E_m(n^+) & = E_m(n) \cdot m & \text{for } n \text{ in } \omega.
\end{align*}
\textbf{Exponentiation} is the \nameref{ref:binary-operation} on $\omega$
such that for any $m$ and $n$ in $\omega$, $$m^n = E_m(n).$$
\lean{Init/Prelude}
{Pow.pow}
2023-05-19 15:54:23 +00:00
\section{\defined{Extensionality Axiom}}%
\hyperlabel{ref:extensionality-axiom}
2023-05-19 15:54:23 +00:00
2023-08-09 13:39:41 +00:00
If two sets have exactly the same members, then they are equal:
$$\forall A, \forall B,
\left[\forall x, (x \in A \iff x \in B) \Rightarrow A = B\right].$$
2023-05-19 15:54:23 +00:00
\lean{Mathlib/Init/Set}
{Set.ext}
2023-05-19 15:54:23 +00:00
2023-06-23 20:47:25 +00:00
\section{\defined{Field}}%
\hyperlabel{ref:field}
2023-06-23 20:47:25 +00:00
2023-08-09 13:39:41 +00:00
Given \nameref{ref:relation} $R$, the \textbf{field} of $R$, denoted $\fld{R}$,
is given by $$\fld{R} = \dom{R} \cup \ran{R}.$$
2023-06-23 20:47:25 +00:00
\lean{Bookshelf/Enderton/Set/Relation}
{Set.Relation.fld}
2023-06-23 20:47:25 +00:00
\section{\defined{Finite Set}}%
\hyperlabel{ref:finite-set}
A set is \textbf{finite} if and only if it is \nameref{ref:equinumerous} to a
\nameref{ref:natural-number}.
\code*{Common/Set/Finite}
{Set.finite\_iff\_equinumerous\_nat}
\lean{Mathlib/Data/Set/Finite}
{Set.Finite}
\section{\defined{Function}}%
\hyperlabel{ref:function}
2023-08-09 13:39:41 +00:00
A \textbf{function} is a relation $F$ such that for each $x$ in $\dom{F}$
there is only one $y$ such that $xFy$.
In other words, $F$ is \textbf{single-valued}.
2023-06-24 15:52:45 +00:00
2023-08-09 13:39:41 +00:00
We say that $F$ is a function \textbf{from $A$ into $B$} or that $F$
\textbf{maps $A$ into $B$} (written $F \colon A \rightarrow B$) iff $F$ is a
function, $\dom{F} = A$, and $\ran{F} \subseteq B$.
If $\ran{F} = B$, then $F$ is a function from \textbf{$A$ onto $B$}.
A function $F$ is \textbf{one-to-one} iff for each $y \in \ran{F}$ there is
only one $x$ such that $xFy$.
One-to-one functions are sometimes called \textbf{injections}.
A one-to-one function from $A$ onto $B$ is a
\textbf{one-to-one correspondence} between $A$ and $B$.
\code*{Bookshelf/Enderton/Set/Relation}
{Set.Relation.isSingleValued}
\code{Bookshelf/Enderton/Set/Relation}
{Set.Relation.isSingleRooted}
\code{Bookshelf/Enderton/Set/Relation}
{Set.Relation.isOneToOne}
\lean{Mathlib/Data/Set/Function}
{Set.MapsTo}
\lean{Mathlib/Data/Set/Function}
{Set.InjOn}
\lean{Mathlib/Data/Set/Function}
{Set.SurjOn}
\lean{Mathlib/Data/Set/Function}
{Set.BijOn}
\section{\defined{Image}}%
\hyperlabel{ref:image}
2023-06-23 20:47:25 +00:00
2023-08-09 13:39:41 +00:00
Let $A$ and $F$ be arbitrary sets.
The \textbf{image of $A$ under $F$} is the set
\begin{align*}
\img{F}{A}
& = \ran{(F \restriction A)} \\
& = \{v \mid (\exists u \in A) uFv\}.
\end{align*}
\code{Bookshelf/Enderton/Set/Relation}
{Set.Relation.image}
\lean{Mathlib/Data/Rel}
{Rel.image}
\section{\defined{Inductive Set}}%
\hyperlabel{ref:inductive-set}
2023-08-09 13:39:41 +00:00
A set $A$ is said to be \textit{inductive} if and only if $\emptyset \in A$
and it is "closed under \nameref{ref:successor}", i.e.
$$(\forall a \in A) a^+ \in A.$$
\lean{Prelude}
{Nat}
\lean{Mathlib/Init/Set}
{Set.univ}
2023-08-09 13:39:41 +00:00
\begin{note}
Induction is baked into Lean's type system.
In particular, the $\emptyset$ and "closed under successor" properties are
analagous to base and recursive constructors of an inductive data type
respectively.
\end{note}
\section{\defined{Infinite Set}}%
\hyperlabel{ref:infinite-set}
A set is \textbf{infinite} if and only if it is not a
\nameref{ref:finite-set}.
\lean*{Mathlib/Data/Set/Finite}
{Set.Infinite}
\section{\defined{Infinity Axiom}}%
\hyperlabel{ref:infinity-axiom}
2023-08-09 13:39:41 +00:00
There exists an \nameref{ref:inductive-set}:
$$(\exists A)\left[
\emptyset \in A \land (\forall a \in A) a^+ \in A\right].$$
\lean{Prelude}
{Nat}
\lean{Mathlib/Init/Set}
{Set.univ}
\section{\defined{Inverse}}%
\hyperlabel{ref:inverse}
2023-06-23 20:47:25 +00:00
2023-08-09 13:39:41 +00:00
The \textbf{inverse} of a set $F$ is the set
$$F^{-1} = \{\tuple{u, v} \mid vFu\}.$$
2023-06-23 20:47:25 +00:00
\code{Bookshelf/Enderton/Set/Relation}
{Set.Relation.inv}
\lean{Mathlib/Data/Rel}
{Rel.inv}
\section{\defined{Irreflexive}}%
\hyperlabel{ref:irreflexive}
2023-08-09 13:39:41 +00:00
A binary relation $R$ on set $A$ is \textbf{irreflexive} if there is no
$x \in A$ for which $xRx$.
\lean*{Mathlib/Init/Algebra/Classes}
{IsIrrefl}
\section{\defined{Linear Ordering}}
\hyperlabel{ref:linear-ordering}
2023-08-09 13:39:41 +00:00
Let $A$ be any set.
A \textbf{linear ordering} on $A$ (also called a \textbf{total ordering} on
$A$) is a binary relation $R$ on $A$ (i.e., $R \subseteq A \times A$)
meeting the following two conditions:
2023-08-09 13:39:41 +00:00
\begin{enumerate}[(a)]
\item $R$ is \nameref{ref:transitive}.
\item $R$ is \nameref{ref:trichotomous}.
\end{enumerate}
\lean{Mathlib/Init/Algebra/Classes}
{IsStrictTotalOrder}
2023-08-09 13:39:41 +00:00
\begin{note}
This definition does not agree with how Lean defines a linear order.
\vspace{6pt}
Trichotomy is equivalent to asymmetry and connectivity and asymmetry is
equivalent to antisymmetry and irreflexivity.
Thus a linear order, as defined by Enderton, is a binary relation with the
following four properties:
2023-08-09 13:39:41 +00:00
\vspace{6pt}
\begin{enumerate}[(i)]
\item Irreflexivity
\item Antisymmetry
\item Connectivity (i.e. totality)
\item Transitivity
\end{enumerate}
\end{note}
\section{\defined{Multiplication}}%
\hyperlabel{ref:multiplication}
2023-08-09 13:39:41 +00:00
For each $m \in \omega$, there exists (by the
\nameref{sub:recursion-theorem-natural-numbers}) a unique
\nameref{ref:function} $M_m \colon \omega \rightarrow \omega$ for which
\begin{align*}
M_m(0) & = 0, \\
M_m(n^+) & = M_m(n) + m.
\end{align*}
\textbf{Multiplication} ($\cdot$) is the \nameref{ref:binary-operation} on
$\omega$ such that for any $m$ and $n$ in $\omega$, $$m \cdot n = M_m(n).$$
\lean{Init/Prelude}
{Mul.mul}
\section{\defined{Natural Map}}%
\hyperlabel{ref:natural-map}
Let $R$ be an \nameref{ref:equivalence-relation} on $A$.
Then the \textbf{natural map} (or \textbf{canonical map})
$\phi \colon A \rightarrow A / R$ is defined as $$\phi(x) = [x]_R$$ for
$x \in A$.
\lean*{Init/Core}
{Quotient.lift}
\section{\defined{Natural Number}}%
\hyperlabel{ref:natural-number}
2023-08-09 13:39:41 +00:00
A \textbf{natural number} is a set that belongs to every inductive set.
The set of all natural numbers exists by virtue of \nameref{sub:theorem-4a}.
This set is denoted as $\omega$.
\lean*{Prelude}
{Nat}
2023-06-07 02:16:06 +00:00
\section{\defined{Ordered Pair}}%
\hyperlabel{ref:ordered-pair}
2023-06-07 02:16:06 +00:00
For any sets $u$ and $v$, the \textbf{ordered pair} $\tuple{u, v}$ is
2023-08-09 13:39:41 +00:00
the set $\{\{u\}, \{u, v\}\}$.
\code*{Bookshelf/Enderton/Set/OrderedPair}
{OrderedPair}
\lean{Prelude}
{Prod}
2023-06-07 02:16:06 +00:00
\section{\defined{Ordering on \texorpdfstring{$\omega$}{Natural Numbers}}}%
\hyperlabel{ref:ordering-natural-numbers}
2023-08-09 13:39:41 +00:00
For \nameref{ref:natural-number}s $m$ and $n$, define $m$ to be
\textbf{less than} $n$ if and only if $m \in n$.
That is, $$m < n \iff m \in n.$$
Likewise, define $m$ to be \textbf{less than or equal to} $n$ if and only if
$m \in n \lor m = n$.
That is,
\begin{align*}
m \leq n
& \iff m \ineq n \\
& \iff m < n \lor m = n.
\end{align*}
\lean{Init/Prelude}
{Nat.lt}
\section{\defined{Pair Set}}%
\hyperlabel{ref:pair-set}
2023-08-09 13:39:41 +00:00
For any sets $u$ and $v$, the \textbf{pair set $\{u, v\}$} is the set whose
only members are $u$ and $v$.
\lean*{Mathlib/Init/Set}
{Set.insert}
\lean{Mathlib/Init/Set}
{Set.singleton}
\section{\defined{Pairing Axiom}}%
\hyperlabel{ref:pairing-axiom}
2023-05-19 15:54:23 +00:00
2023-08-09 13:39:41 +00:00
For any sets $u$ and $v$, there is a set having as members just $u$ and $v$:
$$\forall u, \forall v, \exists B, \forall x,
(x \in B \iff x = u \text{ or } x = v).$$
\lean{Mathlib/Init/Set}
{Set.insert}
\lean{Mathlib/Init/Set}
{Set.singleton}
\section{\defined{Partition}}%
\hyperlabel{ref:partition}
2023-08-09 13:39:41 +00:00
A \textbf{partition} $\Pi$ of a set $A$ is a set of nonempty subsets of $A$
that is disjoint and exhaustive, i.e.
\begin{enumerate}[(a)]
\item no two different sets in $\Pi$ have any common elements, and
\item each element of $A$ is in some set in $\Pi$.
\end{enumerate}
\code{Bookshelf/Enderton/Set/Relation}
{Set.Relation.Partition}
\lean{Mathlib/Data/Setoid/Partition}
{Setoid.IsPartition}
2023-07-21 19:40:38 +00:00
\section{\defined{Peano System}}%
\hyperlabel{ref:peano-system}
2023-08-09 13:39:41 +00:00
A \textbf{Peano system} is a triple $\langle N, S, e \rangle$ consisting of a
set $N$, a function $S \colon N \rightarrow N$, and a member $e \in N$ such
that the following three conditions are met:
\begin{enumerate}[(i)]
\item $e \not\in \ran{S}$.
\item $S$ is one-to-one.
\item Every subset $A$ of $N$ containing $e$ and closed under $S$ is $N$
itself.
\end{enumerate}
2023-07-21 19:40:38 +00:00
\code{Common/Set/Peano}
{Peano.System}
2023-07-21 19:40:38 +00:00
\section{\defined{Power Set}}%
\hyperlabel{ref:power-set}
2023-08-09 13:39:41 +00:00
For any set $a$, the \textbf{power set $\powerset{a}$} is the set whose
members are exactly the subsets of $a$.
\lean*{Mathlib/Init/Set}
{Set.powerset}
\section{\defined{Power Set Axiom}}%
\hyperlabel{ref:power-set-axiom}
2023-08-09 13:39:41 +00:00
For any set $a$, there is a set whose members are exactly the subsets of $a$:
$$\forall a, \exists B, \forall x, (x \in B \iff x \subseteq a).$$
\lean{Mathlib/Init/Set}
{Set.powerset}
\section{\defined{Proper Subset}}%
\hyperlabel{ref:proper-subset}
2023-08-09 13:39:41 +00:00
A set $A$ is said to be a \textbf{proper subset} of $B$ ($A \subset B$) if and
only if it is a subset of $B$ that is unequal to $B$.
$$A \subset B \iff A \subseteq B \land A \neq B.$$
\lean{Std/Classes/SetNotation}
{HasSSubset}
\section{\defined{Quotient Set}}%
\hyperlabel{ref:quotient-set}
2023-08-09 13:39:41 +00:00
If $R$ is an \nameref{ref:equivalence-relation} on set $A$, then we can define
the \textbf{quotient set} $$A / R = \{[x]_R \mid x \in A\}$$ whose members
are the equivalence classes.
The expression $A / R$ is read "$A$ modulo $R$.
\lean*{Init/Core}
{Quotient}
2023-06-23 20:47:25 +00:00
\section{\defined{Range}}%
\hyperlabel{ref:range}
2023-06-23 20:47:25 +00:00
2023-08-09 13:39:41 +00:00
The \textbf{range} of set $R$, denoted $\ran{R}$, is given by
$$x \in \ran{R} \iff \exists t \tuple{t, x} \in R.$$
\code{Bookshelf/Enderton/Set/Relation}
{Set.Relation.ran}
2023-06-23 20:47:25 +00:00
\lean{Mathlib/Data/Rel}
{Rel.codom}
\section{\defined{Reflexive}}%
\hyperlabel{ref:reflexive}
2023-08-09 13:39:41 +00:00
A binary relation $R$ is \textbf{reflexive} on $A$ if and only if $xRx$ for
all $x \in A$.
\code*{Bookshelf/Enderton/Set/Relation}
{Set.Relation.isReflexive}
\lean{Mathlib/Init/Algebra/Classes}
{IsRefl}
\section{\defined{Relation}}%
\hyperlabel{ref:relation}
2023-08-09 13:39:41 +00:00
A \textbf{relation} is a set of \nameref{ref:ordered-pair}s.
\code*{Bookshelf/Enderton/Set/Relation}
{Set.Relation}
\lean{Mathlib/Data/Rel}{Rel}
\section{\defined{Restriction}}%
\hyperlabel{ref:restriction}
2023-08-09 13:39:41 +00:00
The \textbf{restriction} of a set $F$ to set $A$ is the set
$$F \restriction A = \{\tuple{u, v} \mid uFv \land u \in A\}.$$
\code{Bookshelf/Enderton/Set/Relation}
{Set.Relation.restriction}
\section{\defined{Successor}}%
2023-08-09 13:39:41 +00:00
\hyperlabel{ref:successor}
2023-08-09 13:39:41 +00:00
For any set $a$, its \textbf{successor} is defined by $$a^+ = a \cup \{a\}.$$
\lean{Prelude}
{Nat.succ}
2023-08-09 13:39:41 +00:00
\begin{note}
The corresponding Lean definition refers to the `Nat.succ` constructor.
2023-08-09 13:39:41 +00:00
This is not represented internally as a union of sets, but serves the same
role.
\end{note}
\section{\defined{Subset Axioms}}%
\hyperlabel{ref:subset-axioms}
2023-08-09 13:39:41 +00:00
For each formula $\phi$ not containing $B$, the following is an axiom:
$$\forall t_1, \cdots \forall t_k, \forall c,
\exists B, \forall x, (x \in B \iff x \in c \land \phi).$$
\lean{Mathlib/Init/Set}
{Set.Subset}
\section{\defined{Symmetric}}%
\hyperlabel{ref:symmetric}
2023-08-09 13:39:41 +00:00
A binary relation $R$ is \textbf{symmetric} if and only if whenever $xRy$ then
$yRx$.
\code*{Bookshelf/Enderton/Set/Relation}
{Set.Relation.isSymmetric}
\section{\defined{Symmetric Difference}}%
\hyperlabel{ref:symmetric-difference}
2023-08-09 13:39:41 +00:00
The \textbf{symmetric difference} $A + B$ of sets $A$ and $B$ is the set
$(A - B) \cup (B - A)$.
\lean*{Mathlib/Data/Set/Basic}
{symmDiff\_def}
\section{\defined{Transitive}}%
\hyperlabel{ref:transitive}
2023-08-09 13:39:41 +00:00
A binary relation $R$ is \textbf{transitive} if and only if whenever $xRy$ and
$yRz$, then $xRz$.
\code*{Bookshelf/Enderton/Set/Relation}
{Set.Relation.isTransitive}
\lean{Mathlib/Init/Algebra/Classes}
{IsTrans}
\section{\defined{Transitive Set}}%
\hyperlabel{ref:transitive-set}
2023-08-09 13:39:41 +00:00
A set $A$ is said to be \textbf{transitive} if and only if every member of a
member of $A$ is a member of $A$ itself.
That is, $\bigcup A \subseteq A$.
\section{\defined{Trichotomous}}%
\hyperlabel{ref:trichotomous}
2023-08-09 13:39:41 +00:00
A binary relation $R$ on set $A$ is \textbf{trichotomous} if for any
$x, y \in A$, exactly one of the three alternatives
$$xRy, \quad x = y, \quad yRx$$
holds.
\lean*{Mathlib/Init/Algebra/Classes}
{IsTrichotomous}
\section{\defined{Union Axiom}}%
\hyperlabel{ref:union-axiom}
2023-08-09 13:39:41 +00:00
For any set $A$, there exists a set $B$ whose elements are exactly the members
of the members of $A$:
$$\forall A, \exists B, \forall x
\left[ x \in B \iff (\exists b \in A) x \in b \right]$$
\lean{Mathlib/Data/Set/Lattice}
{Set.sUnion}
\section{\defined{Union Axiom, Preliminary Form}}%
\hyperlabel{ref:union-axiom-preliminary-form}
2023-08-09 13:39:41 +00:00
For any sets $a$ and $b$, there is a set whose members are those sets
belonging either to $a$ or to $b$ (or both):
$$\forall a, \forall b, \exists B, \forall x,
(x \in B \iff x \in a \text{ or } x \in b).$$
\lean{Mathlib/Init/Set}
{Set.union}
\endgroup
\chapter{Introduction}%
\hyperlabel{chap:introduction}
2023-06-29 20:53:36 +00:00
\section{Exercises 1}%
\hyperlabel{sec:exercises-1}
2023-05-19 15:25:37 +00:00
\subsection{\verified{Exercise 1.1}}%
\hyperlabel{sub:exercise-1.1}
2023-08-09 13:39:41 +00:00
Which of the following become true when "$\in$" is inserted in place of the
blank?
Which become true when "$\subseteq$" is inserted?
2023-05-19 15:25:37 +00:00
\subsubsection{\verified{Exercise 1.1a}}%
\hyperlabel{ssub:exercise-1.1a}
2023-08-09 13:39:41 +00:00
$$\{\emptyset\} \_\_\_\_ \{\emptyset, \{\emptyset\}\}.$$
\code{Bookshelf/Enderton/Set/Chapter\_1}
2023-05-19 15:25:37 +00:00
{Enderton.Set.Chapter\_1.exercise\_1\_1a}
2023-05-18 22:02:15 +00:00
2023-08-09 13:39:41 +00:00
\begin{proof}
Because the \textit{object} $\{\emptyset\}$ is a member of the right-hand
set, the statement is \textbf{true} in the case of "$\in$".
2023-08-09 13:39:41 +00:00
Because the \textit{members} of $\{\emptyset\}$ are all members of the
right-hand set, the statement is also \textbf{true} in the case of
"$\subseteq$".
\end{proof}
2023-05-19 15:25:37 +00:00
\subsubsection{\verified{Exercise 1.1b}}%
\hyperlabel{ssub:exercise-1.11b}
2023-08-09 13:39:41 +00:00
$$\{\emptyset\} \_\_\_\_ \{\emptyset, \{\{\emptyset\}\}\}.$$
\code{Bookshelf/Enderton/Set/Chapter\_1}
2023-05-19 15:25:37 +00:00
{Enderton.Set.Chapter\_1.exercise\_1\_1b}
2023-05-18 22:02:15 +00:00
2023-08-09 13:39:41 +00:00
\begin{proof}
Because the \textit{object} $\{\emptyset\}$ is not a member of the
right-hand set, the statement is \textbf{false} in the case of "$\in$".
2023-08-09 13:39:41 +00:00
Because the \textit{members} of $\{\emptyset\}$ are all members of the
right-hand set, the statement is \textbf{true} in the case of
"$\subseteq$".
\end{proof}
2023-05-19 15:25:37 +00:00
\subsubsection{\verified{Exercise 1.1c}}%
\hyperlabel{ssub:exercise-1.1c}
2023-08-09 13:39:41 +00:00
$$\{\{\emptyset\}\} \_\_\_\_ \{\emptyset, \{\emptyset\}\}.$$
\code{Bookshelf/Enderton/Set/Chapter\_1}
2023-05-19 15:25:37 +00:00
{Enderton.Set.Chapter\_1.exercise\_1\_1c}
2023-05-18 22:02:15 +00:00
2023-08-09 13:39:41 +00:00
\begin{proof}
Because the \textit{object} $\{\{\emptyset\}\}$ is not a member of the
right-hand set, the statement is \textbf{false} in the case of "$\in$".
2023-08-09 13:39:41 +00:00
Because the \textit{members} of $\{\{\emptyset\}\}$ are all members of the
right-hand set, the statement is \textbf{true} in the case of
"$\subseteq$".
\end{proof}
2023-05-19 15:25:37 +00:00
\subsubsection{\verified{Exercise 1.1d}}%
\hyperlabel{ssub:exercise-1.1d}
2023-08-09 13:39:41 +00:00
$$\{\{\emptyset\}\} \_\_\_\_ \{\emptyset, \{\{\emptyset\}\}\}.$$
\code{Bookshelf/Enderton/Set/Chapter\_1}
2023-05-19 15:25:37 +00:00
{Enderton.Set.Chapter\_1.exercise\_1\_1d}
2023-05-18 22:02:15 +00:00
2023-08-09 13:39:41 +00:00
\begin{proof}
Because the \textit{object} $\{\{\emptyset\}\}$ is a member of the
right-hand set, the statement is \textbf{true} in the case of "$\in$".
2023-08-09 13:39:41 +00:00
Because the \textit{members} of $\{\{\emptyset\}\}$ are not all members of
the right-hand set, the statement is \textbf{false} in the case of
"$\subseteq$".
\end{proof}
2023-05-19 15:25:37 +00:00
\subsubsection{\verified{Exercise 1.1e}}%
\hyperlabel{ssub:exercise-1.1e}
2023-08-09 13:39:41 +00:00
$$\{\{\emptyset\}\} \_\_ \{\emptyset, \{\emptyset, \{\emptyset\}\}\}.$$
\code{Bookshelf/Enderton/Set/Chapter\_1}
2023-05-19 15:25:37 +00:00
{Enderton.Set.Chapter\_1.exercise\_1\_1e}
2023-05-18 22:02:15 +00:00
2023-08-09 13:39:41 +00:00
\begin{proof}
Because the \textit{object} $\{\{\emptyset\}\}$ is not a member of the
right-hand set, the statement is \textbf{false} in the case of "$\in$".
2023-08-09 13:39:41 +00:00
Because the \textit{members} of $\{\{\emptyset\}\}$ are not all members of
the right-hand set, the statement is \textbf{false} in the case of
"$\subseteq$".
\end{proof}
2023-05-19 15:25:37 +00:00
\subsection{\verified{Exercise 1.2}}%
\hyperlabel{sub:exercise-1.2}
2023-08-09 13:39:41 +00:00
Show that no two of the three sets $\emptyset$, $\{\emptyset\}$, and
$\{\{\emptyset\}\}$ are equal to each other.
2023-08-09 13:39:41 +00:00
\code*{Bookshelf/Enderton/Set/Chapter\_1}
2023-05-19 15:25:37 +00:00
{Enderton.Set.Chapter\_1.exercise\_1\_2}
2023-05-18 22:02:15 +00:00
2023-08-09 13:39:41 +00:00
\begin{proof}
By the \nameref{ref:extensionality-axiom}, $\emptyset$ is only equal to
$\emptyset$.
This immediately shows it is not equal to the other two.
Now consider object $\emptyset$.
This object is a member of $\{\emptyset\}$ but is not a member of
$\{\{\emptyset\}\}$.
Again, by the \nameref{ref:extensionality-axiom}, these two sets must be
different.
\end{proof}
2023-05-19 15:25:37 +00:00
\subsection{\verified{Exercise 1.3}}%
\hyperlabel{sub:exercise-1.3}
2023-08-09 13:39:41 +00:00
Show that if $B \subseteq C$, then $\powerset{B} \subseteq \powerset{C}$.
2023-08-09 13:39:41 +00:00
\code*{Bookshelf/Enderton/Set/Chapter\_1}
2023-05-19 15:25:37 +00:00
{Enderton.Set.Chapter\_1.exercise\_1\_3}
2023-05-18 22:02:15 +00:00
2023-08-09 13:39:41 +00:00
\begin{proof}
Let $x \in \powerset{B}$.
By definition of the \nameref{ref:power-set}, $x$ is a subset of $B$.
By hypothesis, $B \subseteq C$.
Then $x \subseteq C$.
Again by definition of the \nameref{ref:power-set}, it follows
$x \in \powerset{C}$.
\end{proof}
2023-05-19 15:25:37 +00:00
\subsection{\verified{Exercise 1.4}}%
\hyperlabel{sub:exercise-1.4}
2023-08-09 13:39:41 +00:00
Assume that $x$ and $y$ are members of a set $B$.
Show that $\{\{x\}, \{x, y\}\} \in \powerset{\powerset{B}}.$
2023-08-09 13:39:41 +00:00
\code*{Bookshelf/Enderton/Set/Chapter\_1}
2023-05-19 15:25:37 +00:00
{Enderton.Set.Chapter\_1.exercise\_1\_4}
2023-05-18 22:02:15 +00:00
2023-08-09 13:39:41 +00:00
\begin{proof}
Let $x$ and $y$ be members of set $B$.
Then $\{x\}$ and $\{x, y\}$ are subsets of $B$.
By definition of the \nameref{ref:power-set}, $\{x\}$ and $\{x, y\}$ are
members of $\powerset{B}$.
Then $\{\{x\}, \{x, y\}\}$ is a subset of $\powerset{B}$.
By definition of the \nameref{ref:power-set}, $\{\{x\}, \{x, y\}\}$ is a
member of $\powerset{\powerset{B}}$.
\end{proof}
2023-05-19 15:25:37 +00:00
\subsection{\unverified{Exercise 1.5}}%
\hyperlabel{sub:exercise-1.5}
2023-05-19 15:25:37 +00:00
2023-08-09 13:39:41 +00:00
Define the rank of a set $c$ to be the least $\alpha$ such that
$c \subseteq V_\alpha$.
Compute the rank of $\{\{\emptyset\}\}$.
Compute the rank of
$\{\emptyset, \{\emptyset\}, \{\emptyset, \{\emptyset\}\}\}$.
\begin{proof}
We first compute the values of $V_n$ for $0 \leq n \leq 3$ under the
assumption the set of atoms $A$ at the bottom of the hierarchy is empty.
\begin{align*}
V_0 & = \emptyset \\
V_1 & = V_0 \cup \powerset{V_0} \\
& = \emptyset \cup \{\emptyset\} \\
& = \{\emptyset\} \\
V_2 & = V_1 \cup \powerset{V_1} \\
& = \{\emptyset\} \cup \powerset{\{\emptyset\}} \\
& = \{\emptyset\} \cup \{\emptyset, \{\emptyset\}\} \\
& = \{\emptyset, \{\emptyset\}\} \\
V_3 & = V_2 \cup \powerset{V_2} \\
& = \{\emptyset, \{\emptyset\}\} \cup
\powerset{\{\emptyset, \{\emptyset\}\}} \\
& = \{\emptyset, \{\emptyset\}\} \cup
\{\emptyset,
\{\emptyset\},
\{\{\emptyset\}\},
\{\emptyset, \{\emptyset\}\}\} \\
& = \{\emptyset,
\{\emptyset\},
\{\{\emptyset\}\},
\{\emptyset, \{\emptyset\}\}\}
\end{align*}
It then immediately follows $\{\{\emptyset\}\}$ has rank $2$ and
$\{\emptyset, \{\emptyset\}, \{\emptyset, \{\emptyset\}\}\}$ has rank $3$.
\end{proof}
2023-05-19 15:25:37 +00:00
\subsection{\unverified{Exercise 1.6}}%
\hyperlabel{sub:exercise-1.6}
2023-05-19 15:25:37 +00:00
2023-08-09 13:39:41 +00:00
We have stated that $V_{\alpha + 1} = A \cup \powerset{V_\alpha}$.
Prove this at least for $\alpha < 3$.
2023-05-19 15:25:37 +00:00
2023-08-09 13:39:41 +00:00
\begin{proof}
2023-05-19 15:25:37 +00:00
2023-08-09 13:39:41 +00:00
Let $A$ be the set of atoms in our set hierarchy.
Let $P(n)$ be the predicate, "$V_{n + 1} = A \cup \powerset{V_n}$."
We prove $P(n)$ holds true for all natural numbers $n \geq 1$ via induction.
2023-05-19 15:25:37 +00:00
2023-08-09 13:39:41 +00:00
\paragraph{Base Case}%
2023-05-19 15:25:37 +00:00
2023-08-09 13:39:41 +00:00
Let $n = 1$.
By definition, $V_1 = V_0 \cup \powerset{V_0}$.
By definition, $V_0 = A$.
Therefore $V_1 = A \cup \powerset{V_0}$.
This proves $P(1)$ holds true.
2023-05-19 15:25:37 +00:00
\paragraph{Inductive Step}%
2023-05-19 15:25:37 +00:00
2023-08-09 13:39:41 +00:00
Suppose $P(n)$ holds true for some $n \geq 1$.
Consider $V_{n+1}$.
By definition, $V_{n+1} = V_n \cup \powerset{V_n}$.
Therefore, by the induction hypothesis,
\begin{align}
V_{n+1}
& = V_n \cup \powerset{V_n}
\nonumber \\
& = (A \cup \powerset{V_{n-1}}) \cup \powerset{V_n}
\nonumber \\
& = A \cup (\powerset{V_{n-1}} \cup \powerset{V_n})
\hyperlabel{sub:exercise-1.6-eq1}
\end{align}
But $V_{n-1}$ is a subset of $V_n$.
\nameref{sub:exercise-1.3} then implies
$\powerset{V_{n-1}} \subseteq \powerset{V_n}$.
This means \eqref{sub:exercise-1.6-eq1} can be simplified to
$$V_{n+1} = A \cup \powerset{V_n},$$
proving $P(n+1)$ holds true.
2023-05-19 15:25:37 +00:00
2023-08-09 13:39:41 +00:00
\paragraph{Conclusion}%
2023-05-19 15:25:37 +00:00
2023-08-09 13:39:41 +00:00
By mathematical induction, it follows for all $n \geq 1$, $P(n)$ is true.
2023-05-19 15:25:37 +00:00
2023-08-09 13:39:41 +00:00
\end{proof}
2023-05-19 15:25:37 +00:00
\subsection{\unverified{Exercise 1.7}}%
\hyperlabel{sub:exercise-1.7}
2023-05-19 15:25:37 +00:00
2023-08-09 13:39:41 +00:00
List all the members of $V_3$.
List all the members of $V_4$.
(It is to be assumed here that there are no atoms.)
\begin{proof}
As seen in the proof of \nameref{sub:exercise-1.5},
$$V_3 = \{
\emptyset,
\{\emptyset\},
\{\{\emptyset\}\},
\{\emptyset, \{\emptyset\}\}
\}.$$
By \nameref{sub:exercise-1.6}, $V_4 = \powerset{V_3}$ (since it is assumed
there are no atoms).
Thus
\begin{align*}
& V_4 = \{ \\
& \qquad \emptyset, \\
& \qquad \{\emptyset\}, \\
& \qquad \{\{\emptyset\}\}, \\
& \qquad \{\{\{\emptyset\}\}\}, \\
& \qquad \{\{\emptyset, \{\emptyset\}\}\}, \\
& \qquad \{\emptyset, \{\emptyset\}\}, \\
& \qquad \{\emptyset, \{\{\emptyset\}\}\}, \\
& \qquad \{\emptyset, \{\emptyset, \{\emptyset\}\}\}, \\
& \qquad \{\{\emptyset\}, \{\{\emptyset\}\}\}, \\
& \qquad \{\{\emptyset\}, \{\emptyset, \{\emptyset\}\}\}, \\
& \qquad \{\{\{\emptyset\}\}, \{\emptyset, \{\emptyset\}\}\}, \\
& \qquad \{\emptyset, \{\emptyset\}, \{\{\emptyset\}\}\}, \\
& \qquad \{\emptyset, \{\emptyset\}, \{\emptyset, \{\emptyset\}\}\}, \\
& \qquad \{\emptyset, \{\{\emptyset\}\}, \{\emptyset, \{\emptyset\}\}\} \\
& \qquad \{\{\emptyset\}, \{\{\emptyset\}\}, \{\emptyset, \{\emptyset\}\}\}, \\
& \qquad \{\emptyset, \{\emptyset\}, \{\{\emptyset\}\}, \{\emptyset, \{\emptyset\}\}\} \\
& \}.
\end{align*}
\end{proof}
\chapter{Axioms and Operations}%
\hyperlabel{chap:axioms-operations}
\section{Axioms}%
\hyperlabel{sec:axioms}
\subsection{\unverified{Theorem 2A}}%
\hyperlabel{sub:theorem-2a}
2023-08-09 13:39:41 +00:00
\begin{theorem}[2A]
There is no set to which every set belongs.
\end{theorem}
\begin{note}
This was revisited after reading Enderton's proof prior.
\end{note}
2023-08-09 13:39:41 +00:00
\begin{proof}
Let $A$ be an arbitrary set.
Define $B = \{ x \in A \mid x \not\in x \}$.
By the \nameref{ref:subset-axioms}, $B$ is a set.
Then $$B \in B \iff B \in A \land B \not\in B.$$
If $B \in A$, then $B \in B \iff B \not\in B$, a contradiction.
Thus $B \not\in A$.
Since this process holds for any set $A$, there must exist no set to which
every set belongs.
\end{proof}
\subsection{\unverified{Theorem 2B}}%
\hyperlabel{sub:theorem-2b}
2023-08-09 13:39:41 +00:00
\begin{theorem}[2B]
For any nonempty set $A$, there exists a unique set $B$ such that for any
$x$, $$x \in B \iff x \text{ belongs to every member of } A.$$
\end{theorem}
\begin{proof}
Suppose $A$ is a nonempty set.
This ensures the statement we are trying to prove does not vacuously hold for
all sets $x$ (which would yield a contradiction due to
\nameref{sub:theorem-2b}).
By the \nameref{ref:union-axiom}, $\bigcup A$ is a set.
Define $$B = \{ x \in \bigcup A \mid (\forall b \in A), x \in b \}.$$
By the \nameref{ref:subset-axioms}, $B$ is indeed a set.
By construction,
$$\forall x, x \in B \iff x \text{ belongs to every member of } A.$$
By the \nameref{ref:extensionality-axiom}, $B$ is unique.
\end{proof}
2023-06-29 20:53:36 +00:00
\section{Algebra of Sets}%
\hyperlabel{sec:algebra-sets}
\subsection{\verified{Commutative Laws}}%
\hyperlabel{sub:commutative-laws}
2023-08-09 13:39:41 +00:00
For any sets $A$ and $B$,
\begin{enumerate}[(i)]
\item $A \cup B = B \cup A$
\item $A \cap B = B \cap A$
\end{enumerate}
2023-08-09 13:39:41 +00:00
\code{Bookshelf/Enderton/Set/Chapter\_2}
{Enderton.Set.Chapter\_2.commutative\_law\_i}
\lean{Mathlib/Data/Set/Basic}{Set.union\_comm}
2023-08-09 13:39:41 +00:00
\code{Bookshelf/Enderton/Set/Chapter\_2}
{Enderton.Set.Chapter\_2.commutative\_law\_ii}
\lean{Mathlib/Data/Set/Basic}{Set.inter\_comm}
2023-08-09 13:39:41 +00:00
\begin{proof}
2023-08-09 13:39:41 +00:00
Let $A$ and $B$ be sets.
2023-08-09 13:39:41 +00:00
\paragraph{(i)}%
2023-08-09 13:39:41 +00:00
By the definition of the union of sets,
\begin{align*}
A \cup B
& = \{ x \mid x \in A \lor x \in B \} \\
& = \{ x \mid x \in B \lor x \in A \} \\
& = B \cup A.
\end{align*}
2023-08-09 13:39:41 +00:00
\paragraph{(ii)}%
2023-08-09 13:39:41 +00:00
By the definition of the intersection of sets,
\begin{align*}
A \cap B
& = \{ x \mid x \in A \land x \in B \} \\
& = \{ x \mid x \in B \land x \in A \} \\
& = B \land A.
\end{align*}
2023-08-09 13:39:41 +00:00
\end{proof}
2023-08-09 13:39:41 +00:00
\subsection{\verified{Associative Laws}}%
\hyperlabel{sub:associative-laws}
2023-08-09 13:39:41 +00:00
For any sets $A$, $B$ and $C$,
\begin{enumerate}[(i)]
\item $A \cup (B \cup C) = (A \cup B) \cup C$
\item $A \cap (B \cap C) = (A \cap B) \cap C$
\end{enumerate}
2023-08-09 13:39:41 +00:00
\code{Bookshelf/Enderton/Set/Chapter\_2}
{Enderton.Set.Chapter\_2.associative\_law\_i}
\lean{Mathlib/Data/Set/Basic}{Set.union\_assoc}
2023-08-09 13:39:41 +00:00
\code{Bookshelf/Enderton/Set/Chapter\_2}
{Enderton.Set.Chapter\_2.associative\_law\_ii}
\lean{Mathlib/Data/Set/Basic}{Set.inter\_assoc}
2023-08-09 13:39:41 +00:00
\begin{proof}
2023-08-09 13:39:41 +00:00
Let $A$, $B$, and $C$ be sets.
2023-08-09 13:39:41 +00:00
\paragraph{(i)}%
2023-08-09 13:39:41 +00:00
By the definition of the union of sets,
\begin{align*}
A \cup (B \cup C)
& = \{ x \mid x \in A \lor x \in (B \cup C) \} \\
& = \{ x \mid x \in A \lor (x \in B \lor x \in C) \} \\
& = \{ x \mid (x \in A \lor x \in B) \lor x \in C \} \\
& = \{ x \mid x \in (A \cup B) \lor x \in C \} \\
& = (A \cup B) \cup C.
\end{align*}
2023-08-09 13:39:41 +00:00
\paragraph{(ii)}%
2023-08-09 13:39:41 +00:00
By the definition of the intersection of sets,
\begin{align*}
A \cap (B \cap C)
& = \{ x \mid x \in A \land x \in (B \cap C) \} \\
& = \{ x \mid x \in A \land (x \in B \land x \in C) \} \\
& = \{ x \mid (x \in A \land x \in B) \land x \in C \} \\
& = \{ x \mid x \in (A \cap B) \land x \in C \} \\
& = (A \cap B) \cap C.
\end{align*}
2023-08-09 13:39:41 +00:00
\end{proof}
2023-08-09 13:39:41 +00:00
\subsection{\verified{Distributive Laws}}%
\hyperlabel{sub:distributive-laws}
2023-08-09 13:39:41 +00:00
For any sets $A$, $B$, and $C$,
\begin{enumerate}[(i)]
\item $A \cap (B \cup C) = (A \cap B) \cup (A \cap C)$
\item $A \cup (B \cap C) = (A \cup B) \cap (A \cup C)$
\end{enumerate}
2023-08-09 13:39:41 +00:00
\code{Bookshelf/Enderton/Set/Chapter\_2}
{Enderton.Set.Chapter\_2.distributive\_law\_i}
\lean{Mathlib/Data/Set/Basic}{Set.inter\_distrib\_left}
2023-08-09 13:39:41 +00:00
\code{Bookshelf/Enderton/Set/Chapter\_2}
{Enderton.Set.Chapter\_2.distributive\_law\_ii}
\lean{Mathlib/Data/Set/Basic}{Set.union\_distrib\_left}
2023-08-09 13:39:41 +00:00
\begin{proof}
2023-08-09 13:39:41 +00:00
Let $A$, $B$, and $C$ be sets.
2023-08-09 13:39:41 +00:00
\paragraph{(i)}%
2023-08-09 13:39:41 +00:00
By the definition of the union and intersection of sets,
\begin{align*}
A \cap (B \cup C)
& = \{ x \mid x \in A \land x \in B \cup C \} \\
& = \{ x \mid x \in A \land (x \in B \lor x \in C) \} \\
& = \{ x \mid (x \in A \land x \in B) \lor
(x \in A \land x \in C) \} \\
& = \{ x \mid x \in A \cap B \lor x \in A \cap C \} \\
& = (A \cap B) \cup (A \cap C).
\end{align*}
2023-08-09 13:39:41 +00:00
\paragraph{(ii)}%
2023-08-09 13:39:41 +00:00
By the definition of the union and intersection of sets,
\begin{align*}
A \cup (B \cap C)
& = \{ x \mid x \in A \lor x \in B \cap C \} \\
& = \{ x \mid x \in A \lor (x \in B \land x \in C) \} \\
& = \{ x \mid (x \in A \lor x \in B) \land
(x \in A \lor x \in C) \} \\
& = \{ x \mid x \in A \cup B \land x \in A \cup C \} \\
& = (A \cup B) \cap (A \cup C).
\end{align*}
2023-08-09 13:39:41 +00:00
\end{proof}
2023-08-09 13:39:41 +00:00
\subsection{\verified{De Morgan's Laws}}%
\hyperlabel{sub:de-morgans-laws}
2023-08-09 13:39:41 +00:00
For any sets $A$, $B$, and $C$,
2023-06-29 20:53:36 +00:00
\begin{enumerate}[(i)]
\item $C - (A \cup B) = (C - A) \cap (C - B)$
\item $C - (A \cap B) = (C - A) \cup (C - B)$
\end{enumerate}
2023-08-09 13:39:41 +00:00
\code{Bookshelf/Enderton/Set/Chapter\_2}
{Enderton.Set.Chapter\_2.de\_morgans\_law\_i}
\lean{Mathlib/Data/Set/Basic}{Set.diff\_inter\_diff}
2023-08-09 13:39:41 +00:00
\code{Bookshelf/Enderton/Set/Chapter\_2}
{Enderton.Set.Chapter\_2.de\_morgans\_law\_ii}
\lean{Mathlib/Data/Set/Basic}{Set.diff\_inter}
2023-08-09 13:39:41 +00:00
\begin{proof}
2023-08-09 13:39:41 +00:00
Let $A$, $B$, and $C$ be sets.
2023-08-09 13:39:41 +00:00
\paragraph{(i)}%
2023-08-09 13:39:41 +00:00
By definition of the union, intersection, and relative complements of
sets,
\begin{align*}
C - (A \cup B)
& = \{ x \mid x \in C \land x \not\in A \cup B \} \\
& = \{ x \mid x \in C \land \neg(x \in A \lor x \in B) \} \\
& = \{ x \mid x \in C \land (x \not\in A \land x \not\in B) \} \\
& = \{ x \mid (x \in C \land x \not\in A) \land
(x \in C \land x \not\in B) \} \\
& = \{ x \mid x \in (C - A) \land x \in (C - B) \} \\
& = (C - A) \cap (C - B).
\end{align*}
2023-08-09 13:39:41 +00:00
\paragraph{(ii)}%
2023-08-09 13:39:41 +00:00
By definition of the union, intersection, and relative complements of
sets,
\begin{align*}
C - (A \cap B)
& = \{ x \mid x \in C \land x \not\in A \cap B \} \\
& = \{ x \mid x \in C \land \neg(x \in A \land x \in B) \} \\
& = \{ x \mid x \in C \land (x \not\in A \lor x \not\in B) \} \\
& = \{ x \mid (x \in C \land x \not\in A) \lor
(x \in C \land x \not\in B) \} \\
& = \{ x \mid x \in C - A \lor x \in C - B \} \\
& = (C - A) \cup (C - B).
\end{align*}
2023-08-09 13:39:41 +00:00
\end{proof}
2023-08-09 13:39:41 +00:00
\subsection{\verified{%
Identities Involving \texorpdfstring{$\emptyset$}{the Empty Set}}}%
\hyperlabel{sub:identitives-involving-empty-set}
2023-08-09 13:39:41 +00:00
For any set $A$,
2023-06-29 20:53:36 +00:00
\begin{enumerate}[(i)]
\item $A \cup \emptyset = A$
\item $A \cap \emptyset = \emptyset$
\item $A \cap (C - A) = \emptyset$
\end{enumerate}
2023-08-09 13:39:41 +00:00
\code{Bookshelf/Enderton/Set/Chapter\_2}
{Enderton.Set.Chapter\_2.emptyset\_identity\_i}
\lean{Mathlib/Data/Set/Basic}{Set.union\_empty}
2023-08-09 13:39:41 +00:00
\code{Bookshelf/Enderton/Set/Chapter\_2}
{Enderton.Set.Chapter\_2.emptyset\_identity\_ii}
\lean{Mathlib/Data/Set/Basic}{Set.inter\_empty}
2023-08-09 13:39:41 +00:00
\code{Bookshelf/Enderton/Set/Chapter\_2}
{Enderton.Set.Chapter\_2.emptyset\_identity\_iii}
\lean{Mathlib/Data/Set/Basic}{Set.inter\_diff\_self}
2023-08-09 13:39:41 +00:00
\begin{proof}
2023-08-09 13:39:41 +00:00
Let $A$ be an arbitrary set.
\paragraph{(i)}%
By definition of the emptyset and union of sets,
\begin{align*}
A \cup \emptyset
& = \{ x \mid x \in A \lor x \in \emptyset \} \\
& = \{ x \mid x \in A \lor F \} \\
& = \{ x \mid x \in A \} \\
& = A.
\end{align*}
\paragraph{(ii)}%
By definition of the emptyset and intersection of sets,
\begin{align*}
A \cap \emptyset
& = \{ x \mid x \in A \land x \in \emptyset \} \\
& = \{ x \mid x \in A \land F \} \\
& = \{ x \mid F \} \\
& = \emptyset.
\end{align*}
2023-08-09 13:39:41 +00:00
\paragraph{(iii)}%
2023-08-09 13:39:41 +00:00
By definition of the emptyset, and the intersection and relative
complement of sets,
\begin{align*}
A \cap (C - A)
& = \{ x \mid x \in A \land x \in C - A \} \\
& = \{ x \mid x \in A \land (x \in C \land x \not\in A) \} \\
& = \{ x \mid x \in C \land F \} \\
& = \{ x \mid F \} \\
& = \emptyset.
\end{align*}
\end{proof}
2023-08-09 18:59:23 +00:00
\subsection{\verified{Monotonicity}}%
2023-08-09 13:39:41 +00:00
\hyperlabel{sub:monotonicity}
2023-08-09 13:39:41 +00:00
For any sets $A$, $B$, and $C$,
\begin{enumerate}[(i)]
\item $A \subseteq B \Rightarrow A \cup C \subseteq B \cup C$
\item $A \subseteq B \Rightarrow A \cap C \subseteq B \cap C$
\item $A \subseteq B \Rightarrow \bigcup A \subseteq \bigcup B$
\end{enumerate}
\code{Bookshelf/Enderton/Set/Chapter\_2}
{Enderton.Set.Chapter\_2.monotonicity\_i}
2023-08-09 18:59:23 +00:00
\lean{Mathlib/Data/Set/Basic}
{Set.union\_subset\_union\_left}
2023-08-09 18:59:23 +00:00
\code{Bookshelf/Enderton/Set/Chapter\_2}
{Enderton.Set.Chapter\_2.monotonicity\_ii}
2023-08-09 18:59:23 +00:00
\lean{Mathlib/Data/Set/Basic}
{Set.inter\_subset\_inter\_left}
2023-08-09 18:59:23 +00:00
\code{Bookshelf/Enderton/Set/Chapter\_2}
{Enderton.Set.Chapter\_2.monotonicity\_iii}
2023-08-09 18:59:23 +00:00
\lean{Mathlib/Data/Set/Lattice}
{Set.sUnion\_mono}
2023-08-09 13:39:41 +00:00
\begin{proof}
Let $A$, $B$, and $C$ be arbitrary sets.
2023-08-09 13:39:41 +00:00
\paragraph{(i)}%
2023-08-09 13:39:41 +00:00
Suppose $A \subseteq B$.
Let $x \in A \cup C$.
There are two cases to consider.
2023-08-09 13:39:41 +00:00
\subparagraph{Case 1}%
2023-08-09 13:39:41 +00:00
Suppose $x \in A$.
Then, by definition of the subset, $x \in B$.
Therefore $x \in B \cup C$.
2023-08-09 13:39:41 +00:00
\subparagraph{Case 2}%
2023-08-09 13:39:41 +00:00
Suppose $x \in C$.
Then $x$ is trivially a member of $B \cup C$.
2023-08-09 13:39:41 +00:00
\subparagraph{Conclusion}%
2023-08-09 13:39:41 +00:00
Since these cases are exhaustive and both imply $x \in B \cup C$, it
follows $A \cup C \subseteq B \cup C$.
2023-08-09 13:39:41 +00:00
\paragraph{(ii)}%
2023-08-09 13:39:41 +00:00
Suppose $A \subseteq B$.
Let $x \in A \cap C$.
Then, by definition of the intersection of sets, $x \in A$ and $x \in C$.
By definition of the subset, $x \in A$ implies $x \in B$.
Therefore $x \in B$ and $x \in C$.
That is, $x \in B \cap C$.
Since this holds for arbitrary $x \in A \cap C$, it follows
$A \cap C \subseteq B \cap C$.
2023-08-09 13:39:41 +00:00
\paragraph{(iii)}%
2023-08-09 13:39:41 +00:00
Suppose $A \subseteq B$.
Let $x \in \bigcup A$.
Then, by definition of the union of sets, there exists some $b \in A$ such
that $x \in b$.
By definition of the subset, $b \in B$ as well.
Another application of the definition of the union of sets immediately
implies that $x$ is a member of $\bigcup B$.
2023-06-29 20:53:36 +00:00
2023-08-09 13:39:41 +00:00
\end{proof}
2023-06-29 20:53:36 +00:00
2023-08-09 18:59:23 +00:00
\subsection{\verified{Anti-monotonicity}}%
\hyperlabel{sub:anti-monotonicity}
2023-06-29 20:53:36 +00:00
2023-08-09 13:39:41 +00:00
For any sets $A$, $B$, and $C$,
\begin{enumerate}[(i)]
\item $A \subseteq B \Rightarrow C - B \subseteq C - A$
\item $\emptyset \neq A \subseteq B \Rightarrow
\bigcap B \subseteq \bigcap A$.
\end{enumerate}
\code{Bookshelf/Enderton/Set/Chapter\_2}
{Enderton.Set.Chapter\_2.anti\_monotonicity\_i}
2023-08-09 18:59:23 +00:00
\lean{Mathlib/Data/Set/Basic}
{Set.diff\_subset\_diff\_right}
\code{Bookshelf/Enderton/Set/Chapter\_2}
{Enderton.Set.Chapter\_2.anti\_monotonicity\_ii}
2023-08-09 18:59:23 +00:00
\lean{Mathlib/Data/Set/Lattice}
{Set.sInter\_subset\_sInter}
2023-08-09 13:39:41 +00:00
\begin{proof}
Let $A$, $B$, and $C$ be arbitrary sets.
2023-08-09 13:39:41 +00:00
\paragraph{(i)}%
2023-08-09 13:39:41 +00:00
Suppose $A \subseteq B$.
Let $x \in C - B$.
By definition of the relative complement, $x \in C$ and $x \not\in B$.
Then $x$ cannot be a member of $A$, since otherwise this would contradict
our subset hypothesis.
That is, $x \in C$ and $x \not\in A$.
Therefore $x \in C - A$.
Since this holds for arbitrary $x \in C - B$, it follows that
$C - B \subseteq C - A$.
2023-08-09 13:39:41 +00:00
\paragraph{(ii)}%
2023-08-09 13:39:41 +00:00
Suppose $A \neq \emptyset$ and $A \subseteq B$.
Then $B \neq \emptyset$.
Let $x \in \bigcap B$.
By definition of the intersection of sets, for all $b \in B$, $x \in b$.
But then, by definition of the subset, for all $a \in A$, $x \in a$.
Therefore $x \in \bigcap A$.
Since this holds for arbitrary $x \in \bigcap B$, it follows that
$\bigcap B \subseteq \bigcap A$.
2023-08-09 13:39:41 +00:00
\end{proof}
\subsection{\unverified{General Distributive Laws}}%
\hyperlabel{sub:general-distributive-laws}
2023-08-09 13:39:41 +00:00
For any sets $A$ and $\mathscr{B}$,
\begin{enumerate}[(i)]
\item $A \cup \bigcap \mathscr{B} =
2023-08-09 13:39:41 +00:00
\bigcap\; \{ A \cup X \mid X \in \mathscr{B} \}
\quad\text{for}\quad \mathscr{B} \neq \emptyset$
\item $A \cap \bigcup \mathscr{B} =
\bigcup\; \{ A \cap X \mid X \in \mathscr{B} \}$
\end{enumerate}
2023-08-09 13:39:41 +00:00
\begin{proof}
2023-08-09 13:39:41 +00:00
Let $A$ and $\mathscr{B}$ be sets.
2023-08-09 13:39:41 +00:00
\paragraph{(i)}%
2023-08-09 13:39:41 +00:00
Suppose $\mathscr{B}$ is nonempty.
Then $\bigcap \mathscr{B}$ is defined.
By definition of the union and intersection of sets,
\begin{align*}
A \cup \bigcap \mathscr{B}
& = \{ x \mid x \in A \lor x \in \bigcap \mathscr{B} \} \\
& = \{ x \mid x \in A \lor
x \in \{ y \mid (\forall b \in \mathscr{B}), y \in b \}\} \\
& = \{ x \mid x \in A \lor (\forall b \in \mathscr{B}), x \in b \} \\
& = \{ x \mid \forall b \in \mathscr{B}, x \in A \lor x \in b \} \\
& = \{ x \mid \forall b \in \mathscr{B}, x \in A \cup b \} \\
& = \{ x \mid
x \in \bigcap\; \{ A \cup X \mid X \in \mathscr{B} \}\} \\
& = \bigcap\; \{ A \cup X \mid X \in \mathscr{B} \}.
\end{align*}
2023-08-09 13:39:41 +00:00
\paragraph{(ii)}%
2023-08-09 13:39:41 +00:00
By definition of the intersection and union of sets,
\begin{align*}
A \cap \bigcup \mathscr{B}
& = \{ x \mid x \in A \land x \in \bigcup \mathscr{B} \} \\
& = \{ x \mid x \in A \land
x \in \{ y \mid (\exists b \in \mathscr{B}), y \in b \}\} \\
& = \{ x \mid x \in A \land (\exists b \in \mathscr{B}), x \in b \} \\
& = \{ x \mid \exists b \in \mathscr{B}, x \in A \land x \in b \} \\
& = \{ x \mid \exists b \in \mathscr{B} x \in A \cap b \} \\
& = \{ x \mid
x \in \bigcup\; \{ A \cap X \mid X \in \mathscr{B} \}\} \\
& = \bigcup\; \{ A \cap X \mid X \in \mathscr{B} \}.
\end{align*}
2023-08-09 13:39:41 +00:00
\end{proof}
\subsection{\unverified{General De Morgan's Laws}}%
\hyperlabel{sub:general-de-morgans-laws}
2023-08-09 13:39:41 +00:00
For any set $C$ and $\mathscr{A} \neq \emptyset$,
\begin{enumerate}[(i)]
\item $C - \bigcup \mathscr{A} =
\bigcap\; \{ C - X \mid X \in \mathscr{A} \}$
\item $C - \bigcap \mathscr{A} =
\bigcup\; \{ C - X \mid X \in \mathscr{A} \}$
\end{enumerate}
2023-08-09 13:39:41 +00:00
\begin{proof}
2023-08-09 13:39:41 +00:00
Let $C$ and $\mathscr{A}$ be sets such that $\mathscr{A} \neq \emptyset$.
2023-08-09 13:39:41 +00:00
\paragraph{(i)}%
2023-08-09 13:39:41 +00:00
By definition of the relative complement, union, and intersection of sets,
\begin{align*}
C - \bigcup \mathscr{A}
& = \{ x \mid x \in C \land x \not\in \bigcup \mathscr{A} \} \\
& = \{ x \mid x \in C \land
x \not\in \{ y \mid (\exists b \in \mathscr{A}) y \in b \}\} \\
& = \{ x \mid x \in C \land
\neg(\exists b \in \mathscr{A}, x \in b) \} \\
& = \{ x \mid x \in C \land
(\forall b \in \mathscr{A}, x \not\in b) \} \\
& = \{ x \mid
\forall b \in \mathscr{A}, x \in C \land x \not\in b \} \\
& = \{ x \mid \forall b \in \mathscr{A}, x \in C - b \} \\
& = \{ x \mid x \in \bigcap\; \{ C - X \mid X \in \mathscr{A} \} \\
& = \bigcap\; \{ C - X \mid X \in \mathscr{A} \}.
\end{align*}
2023-08-09 13:39:41 +00:00
\paragraph{(ii)}%
2023-08-09 13:39:41 +00:00
By definition of the relative complement, union, and intersection of sets,
\begin{align*}
C - \bigcap \mathscr{A}
& = \{ x \mid x \in C \land x \not\in \bigcap \mathscr{A} \} \\
& = \{ x \mid x \in C \land
x \not\in \{ y \mid (\forall b \in \mathscr{A}) y \in b \}\} \\
& = \{ x \mid x \in C \land
\neg(\forall b \in \mathscr{A}, x \in b) \} \\
& = \{ x \mid x \in C \land
\exists b \in \mathscr{A}, x \not\in b \} \\
& = \{ x \mid
\exists b \in \mathscr{A}, x \in C \land x \not\in b \} \\
& = \{ x \mid \exists b \in \mathscr{A}, x \in C - b \} \\
& = \{ x \mid x \in \bigcup\; \{ C - X \mid X \in \mathscr{A} \} \} \\
& = \bigcup\; \{ C - X \mid X \in \mathscr{A} \}.
\end{align*}
2023-08-09 13:39:41 +00:00
\end{proof}
2023-08-09 18:59:23 +00:00
\subsection{\verified{%
2023-06-29 20:53:36 +00:00
\texorpdfstring{$\cap$/$-$}{Intersection/Difference} Associativity}}%
\hyperlabel{sub:intersection-difference-associativity}
2023-06-29 20:53:36 +00:00
Let $A$, $B$, and $C$ be sets.
2023-08-09 13:39:41 +00:00
Then $A \cap (B - C) = (A \cap B) - C$.
\code*{Bookshelf/Enderton/Set/Chapter\_2}
2023-08-09 18:59:23 +00:00
{Enderton.Set.Chapter\_2.inter\_diff\_assoc}
\lean{Mathlib/Data/Set/Basic}
{Set.inter\_diff\_assoc}
2023-08-09 13:39:41 +00:00
\begin{proof}
Let $A$, $B$, and $C$ be sets.
By definition of the intersection and relative complement of sets,
\begin{align*}
A \cap (B - C)
& = \{ x \mid x \in A \land x \in B - C \} \\
& = \{ x \mid x \in A \land (x \in B \land x \not\in C) \} \\
& = \{ x \mid (x \in A \land x \in B) \land x \not\in C \} \\
& = \{ x \mid x \in A \cap B \land x \not \in C \} \\
& = (A \cap B) - C.
\end{align*}
\end{proof}
2023-06-29 20:53:36 +00:00
\subsection{\verified{Nonmembership of Symmetric Difference}}
\hyperlabel{sub:nonmembership-symmetric-difference}
2023-08-09 13:39:41 +00:00
Let $A$ and $B$ be sets. $x \not\in A + B$ if and only if either
$x \in A \cap B$ or $x \not\in A \cup B$.
\code*{Common/Set/Basic}
2023-07-06 21:31:09 +00:00
{Set.not\_mem\_symm\_diff\_inter\_or\_not\_union}
2023-08-09 13:39:41 +00:00
\begin{proof}
By definition of the \nameref{ref:symmetric-difference},
\begin{align*}
x \not\in A + B
& = \neg(x \in A + B) \\
& = \neg[x \in (A - B) \cup (B - A)] \\
& = \neg[x \in (A - B) \lor x \in (B - A)] \\
& = \neg[(x \in A \land x \not\in B) \lor
(x \in B \land x \not\in A)] \\
& = \neg(x \in A \land x \not\in B) \land
\neg(x \in B \land x \not\in A) \\
& = (x \not\in A \lor x \in B) \land (x \not\in B \lor x \in A) \\
& = ((x \not\in A \lor x \in B) \land x \not\in B) \lor
((x \not\in A \lor x \in B) \land x \in A) \\
& = (x \not\in A \land x \not\in B) \lor (x \in B \land x \in A) \\
& = \neg(x \in A \lor x \in B) \lor (x \in B \land x \in A) \\
& = x \not\in A \cup B \text{ or } x \in A \cap B.
\end{align*}
\end{proof}
2023-06-29 20:53:36 +00:00
\section{Exercises 2}%
\hyperlabel{sec:exercises-2}
2023-06-29 20:53:36 +00:00
\subsection{\verified{Exercise 2.1}}%
\hyperlabel{sub:exercise-2.1}
2023-08-09 13:39:41 +00:00
Assume that $A$ is the set of integers divisible by $4$.
Similarly assume that $B$ and $C$ are the sets of integers divisible by $9$ and
$10$, respectively.
What is in $A \cap B \cap C$?
2023-08-09 13:39:41 +00:00
\code*{Bookshelf/Enderton/Set/Chapter\_2}
2023-06-29 20:53:36 +00:00
{Enderton.Set.Chapter\_2.exercise\_2\_1}
2023-08-09 13:39:41 +00:00
\begin{answer}
The set of integers divisible by $4$, $9$, and $10$.
\end{answer}
2023-06-29 20:53:36 +00:00
\subsection{\verified{Exercise 2.2}}%
\hyperlabel{sub:exercise-2.2}
2023-08-09 13:39:41 +00:00
Give an example of sets $A$ and $B$ for which $\bigcup A = \bigcup B$ but
$A \neq B$.
2023-08-09 13:39:41 +00:00
\code*{Bookshelf/Enderton/Set/Chapter\_2}
2023-06-29 20:53:36 +00:00
{Enderton.Set.Chapter\_2.exercise\_2\_2}
2023-08-09 13:39:41 +00:00
\begin{answer}
Let $A = \{\{1\}, \{2\}\}$ and $B = \{\{1, 2\}\}$.
\end{answer}
2023-06-29 20:53:36 +00:00
\subsection{\verified{Exercise 2.3}}%
\hyperlabel{sub:exercise-2.3}
2023-08-09 13:39:41 +00:00
Show that every member of a set $A$ is a subset of $\bigcup A$.
(This was stated as an example in this section.)
2023-08-09 13:39:41 +00:00
\code*{Bookshelf/Enderton/Set/Chapter\_2}
2023-06-29 20:53:36 +00:00
{Enderton.Set.Chapter\_2.exercise\_2\_3}
2023-08-09 13:39:41 +00:00
\begin{proof}
Let $x \in A$.
By definition, $$\bigcup A = \{ y \mid (\exists b \in A) y \in b\}.$$
Then $\{ y \mid y \in x\} \subseteq \bigcup A$.
But $\{ y \mid y \in x\} = x$.
Thus $x \subseteq \bigcup A$.
\end{proof}
2023-06-29 20:53:36 +00:00
\subsection{\verified{Exercise 2.4}}%
\hyperlabel{sub:exercise-2.4}
2023-08-09 13:39:41 +00:00
Show that if $A \subseteq B$, then $\bigcup A \subseteq \bigcup B$.
2023-08-09 13:39:41 +00:00
\code*{Bookshelf/Enderton/Set/Chapter\_2}
2023-06-29 20:53:36 +00:00
{Enderton.Set.Chapter\_2.exercise\_2\_4}
2023-08-09 13:39:41 +00:00
\begin{proof}
Let $A$ and $B$ be sets such that $A \subseteq B$.
Let $x \in \bigcup A$.
By definition of the union, there exists some $b \in A$ such that $x \in b$.
By definition of the subset, $b \in B$.
This immediatley implies $x \in \bigcup B$.
Since this holds for all $x \in \bigcup A$, it follows
$\bigcup A \subseteq \bigcup B$.
\end{proof}
2023-06-29 20:53:36 +00:00
\subsection{\verified{Exercise 2.5}}%
\hyperlabel{sub:exercise-2.5}
2023-08-09 13:39:41 +00:00
Assume that every member of $\mathscr{A}$ is a subset of $B$.
Show that $\bigcup \mathscr{A} \subseteq B$.
2023-08-09 13:39:41 +00:00
\code*{Bookshelf/Enderton/Set/Chapter\_2}
2023-06-29 20:53:36 +00:00
{Enderton.Set.Chapter\_2.exercise\_2\_5}
2023-08-09 13:39:41 +00:00
\begin{proof}
Let $x \in \bigcup \mathscr{A}$.
By definition,
$$\bigcup \mathscr{A} = \{ y \mid (\exists b \in A)y \in b \}.$$
Then there exists some $b \in A$ such that $x \in b$.
By hypothesis, $b \subseteq B$.
Thus $x$ must also be a member of $B$.
Since this holds for all $x \in \bigcup \mathscr{A}$, it follows
$\bigcup \mathscr{A} \subseteq B$.
\end{proof}
2023-06-29 20:53:36 +00:00
\subsection{\verified{Exercise 2.6a}}%
\hyperlabel{sub:exercise-2.6a}
2023-08-09 13:39:41 +00:00
Show that for any set $A$, $\bigcup \powerset{A} = A$.
2023-08-09 13:39:41 +00:00
\code*{Bookshelf/Enderton/Set/Chapter\_2}
2023-06-29 20:53:36 +00:00
{Enderton.Set.Chapter\_2.exercise\_2\_6a}
2023-08-09 13:39:41 +00:00
\begin{proof}
2023-08-09 13:39:41 +00:00
We prove that (i) $\bigcup \powerset{A} \subseteq A$ and (ii)
$A \subseteq \bigcup \powerset{A}$.
2023-08-09 13:39:41 +00:00
\paragraph{(i)}%
\hyperlabel{par:exercise-2.6a-i}
2023-08-09 13:39:41 +00:00
By definition, the \nameref{ref:power-set} of $A$ is the set of all
subsets of $A$.
In other words, every member of $\powerset{A}$ is a subset of $A$.
By \nameref{sub:exercise-2.5}, $\bigcup \powerset{A} \subseteq A$.
2023-08-09 13:39:41 +00:00
\paragraph{(ii)}%
\hyperlabel{par:exercise-2.6a-ii}
2023-06-29 20:53:36 +00:00
2023-08-09 13:39:41 +00:00
Let $x \in A$.
By definition of the power set of $A$, $\{x\} \in \powerset{A}$.
By definition of the union,
$$\bigcup \powerset{A} =
\{ y \mid (\exists b \in \powerset{A}), y \in b).$$
Since $x \in \{x\}$ and $\{x\} \in \powerset{A}$, it follows
$x \in \bigcup \powerset{A}$.
Thus $A \subseteq \bigcup \powerset{A}$.
2023-06-29 20:53:36 +00:00
2023-08-09 13:39:41 +00:00
\paragraph{Conclusion}%
By \nameref{par:exercise-2.6a-i} and \nameref{par:exercise-2.6a-ii},
$\bigcup \powerset{A} = A$.
2023-08-09 13:39:41 +00:00
\end{proof}
2023-06-29 20:53:36 +00:00
\subsection{\verified{Exercise 2.6b}}%
\hyperlabel{sub:exercise-2.6b}
2023-08-09 13:39:41 +00:00
Show that $A \subseteq \powerset{\bigcup A}$.
Under what conditions does equality hold?
2023-08-09 13:39:41 +00:00
\code*{Bookshelf/Enderton/Set/Chapter\_2}
2023-06-29 20:53:36 +00:00
{Enderton.Set.Chapter\_2.exercise\_2\_6b}
2023-08-09 13:39:41 +00:00
\begin{proof}
2023-08-09 13:39:41 +00:00
Let $x \in A$.
By \nameref{sub:exercise-2.3}, $x$ is a subset of $\bigcup A$.
By the definition of the \nameref{ref:power-set},
$$\powerset{\bigcup A} = \{ y \mid y \subseteq \bigcup A \}.$$
Therefore $x \in \powerset{\bigcup A}$.
Since this holds for all $x \in A$, $A \subseteq \powerset{\bigcup A}$.
2023-08-09 13:39:41 +00:00
\suitdivider
2023-06-29 20:53:36 +00:00
2023-08-09 13:39:41 +00:00
We show equality holds if and only if there exists some set $B$ such that
$A = \powerset{B}$.
2023-06-29 20:53:36 +00:00
2023-08-09 13:39:41 +00:00
\paragraph{($\Rightarrow$)}%
\hyperlabel{par:exercise-2.6b-right}
2023-06-29 20:53:36 +00:00
2023-08-09 13:39:41 +00:00
Suppose $A = \powerset{\bigcup A}$.
Then our statement immediately follows by settings $B = \bigcup A$.
2023-06-29 20:53:36 +00:00
2023-08-09 13:39:41 +00:00
\paragraph{($\Leftarrow$)}%
\hyperlabel{par:exercise-2.6b-left}
Suppose there exists some set $B$ such that $A = \powerset{B}$.
Therefore
\begin{align*}
\powerset{\bigcup A}
& = \powerset{\left(\bigcup {\powerset {B}}\right)} \\
& = \powerset{B} & \textref{sub:exercise-2.6a} \\
& = A.
\end{align*}
2023-08-09 13:39:41 +00:00
\paragraph{Conclusion}%
2023-06-29 20:53:36 +00:00
2023-08-09 13:39:41 +00:00
By \nameref{par:exercise-2.6b-right} and \nameref{par:exercise-2.6b-left},
$A = \powerset{\bigcup A}$ if and only if there exists some set $B$ such
that $A = \powerset{B}$.
2023-06-29 20:53:36 +00:00
2023-08-09 13:39:41 +00:00
\end{proof}
2023-06-29 20:53:36 +00:00
\subsection{\verified{Exercise 2.7a}}%
\hyperlabel{sub:exercise-2.7a}
2023-08-09 13:39:41 +00:00
Show that for any sets $A$ and $B$,
$$\powerset{A} \cap \powerset{B} = \powerset{(A \cap B)}.$$
2023-08-09 13:39:41 +00:00
\code*{Bookshelf/Enderton/Set/Chapter\_2}
2023-06-29 20:53:36 +00:00
{Enderton.Set.Chapter\_2.exercise\_2\_7a}
2023-08-09 13:39:41 +00:00
\begin{proof}
2023-08-09 13:39:41 +00:00
Let $A$ and $B$ be arbitrary sets. We show that
$\powerset{A} \cap \powerset{B} \subseteq \powerset{(A \cap B)}$ and then
show that
$\powerset{A} \cap \powerset{B} \supseteq \powerset{(A \cap B)}$.
2023-08-09 13:39:41 +00:00
\paragraph{($\subseteq$)}%
2023-08-09 13:39:41 +00:00
Let $x \in \powerset{A} \cap \powerset{B}$.
That is, $x \in \powerset{A}$ and $x \in \powerset{B}$.
By the definition of the \nameref{ref:power-set},
\begin{align*}
\powerset{A} & = \{ y \mid y \subseteq A \} \\
\powerset{B} & = \{ y \mid y \subseteq B \}
\end{align*}
Thus $x \subseteq A$ and $x \subseteq B$, meaning $x \subseteq A \cap B$.
But then $x \in \powerset{(A \cap B)}$, the set of all subsets of
$A \cap B$.
Since this holds for all $x \in \powerset{A} \cap \powerset{B}$, it
follows
$$\powerset{A} \cap \powerset{B} \subseteq \powerset{(A \cap B)}.$$
\paragraph{($\supseteq$)}%
Let $x \in \powerset{(A \cap B)}$.
By the definition of the \nameref{ref:power-set},
$$\powerset{(A \cap B)} = \{ y \mid y \subseteq A \cap B \}.$$
Thus $x \subseteq A \cap B$, meaning $x \subseteq A$ and $x \subseteq B$.
But this implies $x \in \powerset{A}$, the set of all subsets of $A$.
Likewise $x \in \powerset{B}$, the set of all subsets of $B$.
Thus $x \in \powerset{A} \cap \powerset{B}$.
Since this holds for all $x \in \powerset{(A \cap B)}$, it follows
$$\powerset{(A \cap B)} \subseteq \powerset{A} \cap \powerset{B}.$$
2023-06-29 20:53:36 +00:00
2023-08-09 13:39:41 +00:00
\paragraph{Conclusion}%
2023-06-29 20:53:36 +00:00
2023-08-09 13:39:41 +00:00
Since each side of our identity is a subset of the other,
$$\powerset{(A \cap B)} = \powerset{A} \cap \powerset{B}.$$
2023-08-09 13:39:41 +00:00
\end{proof}
2023-06-29 20:53:36 +00:00
\subsection{\verified{Exercise 2.7b}}%
\hyperlabel{sub:exercise-2.7b}
2023-08-09 13:39:41 +00:00
Show that $\powerset{A} \cup \powerset{B} \subseteq \powerset{(A \cup B)}$.
Under what conditions does equality hold?
\code*{Bookshelf/Enderton/Set/Chapter\_2}
2023-06-29 20:53:36 +00:00
{Enderton.Set.Chapter\_2.exercise\_2\_7b\_i}
\code{Bookshelf/Enderton/Set/Chapter\_2}
2023-06-29 20:53:36 +00:00
{Enderton.Set.Chapter\_2.exercise\_2\_7b\_ii}
2023-08-09 13:39:41 +00:00
\begin{proof}
2023-08-09 13:39:41 +00:00
Let $x \in \powerset{A} \cup \powerset{B}$.
By definition, $x \in \powerset{A}$ or $x \in \powerset{B}$ (or both).
2023-06-29 20:53:36 +00:00
By the definition of the \nameref{ref:power-set},
\begin{align*}
2023-08-09 13:39:41 +00:00
\powerset{A} &= \{ y \mid y \subseteq A \} \\
\powerset{B} &= \{ y \mid y \subseteq B \}.
2023-06-29 20:53:36 +00:00
\end{align*}
2023-08-09 13:39:41 +00:00
Thus $x \subseteq A$ or $x \subseteq B$.
Therefore $x \subseteq A \cup B$.
But then $x \in \powerset{(A \cup B)}$, the set of all subsets of
$A \cup B$.
\suitdivider
We show equality holds if and only if one of $A$ or $B$ is a subset of the
other.
\paragraph{($\Rightarrow$)}%
\hyperlabel{par:exercise-2.7b-right}
Suppose
\begin{equation}
\hyperlabel{sub:exercise-2.7b-eq1}
\powerset{A} \cup \powerset{B} = \powerset{(A \cup B)}.
\end{equation}
By the definition of the \nameref{ref:power-set},
$A \cup B \in \powerset{(A \cup B)}$.
Then \eqref{sub:exercise-2.7b-eq1} implies
$A \cup B \in \powerset{A} \cup \powerset{B}$.
That is, $A \cup B \in \powerset{A}$ or $A \cup B \in \powerset{B}$ (or
both).
For the sake of contradiction, suppose $A \not\subseteq B$ and
$B \not\subseteq A$.
Then there exists an element $x \in A$ such that $x \not\in B$ and there
exists an element $y \in B$ such that $y \not\in A$.
But then $A \cup B \not\in \powerset{A}$ since $y$ cannot be a member of a
member of $\powerset{A}$.
Likewise, $A \cup B \not\in \powerset{B}$ since $x$ cannot be a member of
a member of $\powerset{B}$.
Therefore our assumption is incorrect.
In other words, $A \subseteq B$ or $B \subseteq A$.
\paragraph{($\Leftarrow$)}%
\hyperlabel{par:exercise-2.7b-left}
WLOG, suppose $A \subseteq B$.
Then, by \nameref{sub:exercise-1.3},
$\powerset{A} \subseteq \powerset{B}$.
Thus
\begin{align*}
\powerset{A} \cup \powerset{B}
& = \powerset{B} \\
& = \powerset{A \cup B}.
\end{align*}
2023-08-09 13:39:41 +00:00
\paragraph{Conclusion}%
2023-08-09 13:39:41 +00:00
By \nameref{par:exercise-2.7b-right} and \nameref{par:exercise-2.7b-left},
it follows
$\powerset{A} \cup \powerset{B} \subseteq \powerset{(A \cup B)}$ if and
only if $A \subseteq B$ or $B \subseteq A$.
2023-08-09 13:39:41 +00:00
\end{proof}
2023-06-29 20:53:36 +00:00
\subsection{\unverified{Exercise 2.8}}%
\hyperlabel{sub:exercise-2.8}
2023-06-29 20:53:36 +00:00
2023-08-09 13:39:41 +00:00
Show that there is no set to which every singleton (that is, every set of the
form $\{x\}$) belongs.
[\textit{Suggestion}: Show that from such a set, we could construct a set to
which every set belonged.]
2023-08-09 13:39:41 +00:00
\begin{proof}
We proceed by contradiction.
Suppose there existed a set $A$ consisting of every singleton.
Then the \nameref{ref:union-axiom} suggests $\bigcup A$ is a set.
But this "set" is precisely the class of all sets, which is \textit{not} a
2023-08-09 13:39:41 +00:00
set.
Thus our original assumption was incorrect.
That is, there is no set to which every singleton belongs.
\end{proof}
2023-06-29 20:53:36 +00:00
\subsection{\verified{Exercise 2.9}}%
\hyperlabel{sub:exercise-2.9}
2023-08-09 13:39:41 +00:00
Give an example of sets $a$ and $B$ for which $a \in B$ but
$\powerset{a} \not\in \powerset{B}$.
2023-08-09 13:39:41 +00:00
\code*{Bookshelf/Enderton/Set/Chapter\_2}
2023-06-29 20:53:36 +00:00
{Enderton.Set.Chapter\_2.exercise\_2\_9}
2023-08-09 13:39:41 +00:00
\begin{answer}
Let $a = \{1\}$ and $B = \{\{1\}\}$.
Then
\begin{align*}
\powerset{a} & = \{\emptyset, \{1\}\} \\
\powerset{B} & = \{\emptyset, \{\{1\}\}\}.
\end{align*}
It immediately follows that $\powerset{a} \not\in \powerset{B}$.
\end{answer}
2023-06-29 20:53:36 +00:00
\subsection{\verified{Exercise 2.10}}%
\hyperlabel{sub:exercise-2.10}
2023-06-29 20:53:36 +00:00
2023-08-09 13:39:41 +00:00
Show that if $a \in B$, then
$\powerset{a} \in \powerset{\powerset{\bigcup B}}$.
[\textit{Suggestion}: If you need help, look in the Appendix.]
2023-06-29 20:53:36 +00:00
2023-08-09 13:39:41 +00:00
\code*{Bookshelf/Enderton/Set/Chapter\_2}
2023-06-29 20:53:36 +00:00
{Enderton.Set.Chapter\_2.exercise\_2\_10}
2023-08-09 13:39:41 +00:00
\begin{proof}
Suppose $a \in B$.
By \nameref{sub:exercise-2.3}, $a \subseteq \bigcup B$.
By \nameref{sub:exercise-1.3},
$\powerset{a} \subseteq \powerset{\bigcup B}$.
By the definition of the \nameref{ref:power-set},
$$\powerset{\powerset{\bigcup B}} =
\{ y \mid y \subseteq \powerset{\bigcup B} \}.$$
Therefore $\powerset{a} \in \powerset{\powerset{\bigcup B}}$.
\end{proof}
2023-06-29 20:53:36 +00:00
\subsection{\verified{Exercise 2.11}}%
\hyperlabel{sub:exercise-2.11}
2023-06-29 20:53:36 +00:00
2023-08-09 13:39:41 +00:00
Show that for any sets $A$ and $B$,
$$A = (A \cap B) \cup (A - B) \quad\text{and}\quad
A \cup (B - A) = A \cup B.$$
2023-06-29 20:53:36 +00:00
2023-08-09 13:39:41 +00:00
\code{Bookshelf/Enderton/Set/Chapter\_2}
2023-06-29 20:53:36 +00:00
{Enderton.Set.Chapter\_2.exercise\_2\_11\_i}
\code{Bookshelf/Enderton/Set/Chapter\_2}
2023-06-29 20:53:36 +00:00
{Enderton.Set.Chapter\_2.exercise\_2\_11\_ii}
2023-08-09 13:39:41 +00:00
\begin{proof}
Let $A$ and $B$ be sets.
We prove that
\begin{enumerate}[(i)]
\item $A = (A \cap B) \cup (A - B)$
\item $A \cup (B - A) = A \cup B$
\end{enumerate}
2023-06-29 20:53:36 +00:00
2023-08-09 13:39:41 +00:00
\paragraph{(i)}%
2023-06-29 20:53:36 +00:00
2023-08-09 13:39:41 +00:00
By definition of the intersection, union, and relative complements of
sets,
\begin{align*}
(A \cap B) \cup (A - B)
& = \{ x \mid x \in A \cap B \lor x \in A - B \} \\
& = \{ x \mid x \in \{ y \mid y \in A \land y \in B \} \lor
x \in A - B \} \\
& = \{ x \mid (x \in A \land x \in B) \lor x \in A - B \} \\
& = \{ x \mid (x \in A \land x \in B) \lor
x \in \{ y \mid y \in A \land y \not\in B \} \} \\
& = \{ x \mid (x \in A \land x \in B) \lor
(x \in A \land x \not\in B) \} \\
& = \{ x \mid x \in A \lor (x \in B \land x \not\in B) \} \\
& = \{ x \mid x \in A \lor F \} \\
& = \{ x \mid x \in A \} \\
& = A.
\end{align*}
2023-08-09 13:39:41 +00:00
\paragraph{(ii)}%
2023-08-09 13:39:41 +00:00
By definition of the union and relative complements of sets,
\begin{align*}
A \cup (B - A)
& = \{ x \mid x \in A \lor x \in B - A \} \\
& = \{ x \mid x \in A \lor
x \in \{ y \mid y \in B \land y \not\in A \} \} \\
& = \{ x \mid x \in A \lor (x \in B \land x \not\in A) \} \\
& = \{ x \mid (x \in A \lor x \in B) \land
(x \in A \lor x \not\in A) \} \\
& = \{ x \mid (x \in A \lor x \in B) \land T \} \\
& = \{ x \mid x \in A \lor x \in B \} \\
& = \{ x \mid x \in A \cup B \} \\
& = A \cup B.
\end{align*}
2023-08-09 13:39:41 +00:00
\end{proof}
2023-06-29 20:53:36 +00:00
\subsection{\verified{Exercise 2.12}}%
\hyperlabel{sub:exercise-2.12}
2023-08-09 13:39:41 +00:00
Verify the following identity (one of De Morgan's laws):
$$C - (A \cap B) = (C - A) \cup (C - B).$$
2023-08-09 13:39:41 +00:00
\begin{proof}
Refer to \nameref{sub:de-morgans-laws}.
\end{proof}
2023-06-29 20:53:36 +00:00
\subsection{\verified{Exercise 2.13}}%
\hyperlabel{sub:exercise-2.13}
2023-08-09 13:39:41 +00:00
Show that if $A \subseteq B$, then $C - B \subseteq C - A$.
2023-08-09 13:39:41 +00:00
\begin{proof}
Refer to \nameref{sub:anti-monotonicity}.
\end{proof}
2023-06-29 20:53:36 +00:00
\subsection{\verified{Exercise 2.14}}%
\hyperlabel{sub:exercise-2.14}
2023-08-09 13:39:41 +00:00
Show by example that for some sets $A$, $B$, and $C$, the set $A - (B - C)$ is
different from $(A - B) - C$.
2023-08-09 13:39:41 +00:00
\code*{Bookshelf/Enderton/Set/Chapter\_2}
2023-06-29 20:53:36 +00:00
{Enderton.Set.Chapter\_2.exercise\_2\_14}
2023-08-09 13:39:41 +00:00
\begin{proof}
Let $A = \{1, 2, 3\}$, $B = \{2, 3, 4\}$, and $C = \{3, 4, 5\}$.
Then
\begin{align*}
A - (B - C)
& = \{1, 2, 3\} - (\{2, 3, 4\} - \{3, 4, 5\}) \\
& = \{1, 2, 3\} - \{2\} \\
& = \{1, 3\}
\end{align*}
but
\begin{align*}
(A - B) - C
& = (\{1, 2, 3\} - \{2, 3, 4\}) - \{3, 4, 5\} \\
& = \{1\} - \{3, 4, 5\} \\
& = \{1\}.
\end{align*}
\end{proof}
\subsection{\verified{Exercise 2.15a}}%
\hyperlabel{sub:exercise-2.15a}
2023-08-09 13:39:41 +00:00
Show that $A \cap (B + C) = (A \cap B) + (A \cap C)$.
\code*{Bookshelf/Enderton/Set/Chapter\_2}
{Enderton.Set.Chapter\_2.exercise\_2\_15a}
\lean{Mathlib/Data/Set/Basic}
{Set.inter\_symmDiff\_distrib\_left}
2023-08-09 13:39:41 +00:00
\begin{proof}
By definition of the intersection, \nameref{ref:symmetric-difference}, and
relative complement of sets,
\begin{align*}
(A & \cap B) + (A \cap C) \\
& = [(A \cap B) - (A \cap C)] \cup [(A \cap C) - (A \cap B)] \\
& = [(A \cap B) - A] \\
& \qquad \cup [(A \cap B) - C] \\
& \qquad \cup [(A \cap C) - A] \\
& \qquad \cup [(A \cap C) - B]
& \textref{sub:de-morgans-laws} \\
& = [A \cap (B - A)] \\
& \qquad \cup [A \cap (B - C)] \\
& \qquad \cup [A \cap (C - A)] \\
& \qquad \cup [A \cap (C - B)]
& \textref{sub:intersection-difference-associativity} \\
& = \emptyset \\
& \qquad \cup [A \cap (B - C)] \\
& \qquad \cup \emptyset \\
& \qquad \cup [A \cap (C - B)]
& \textref{sub:identitives-involving-empty-set} \\
& = [A \cap (B - C)] \cup [A \cap (C - B)] \\
& = A \cap [(B - C) \cup (C - B)]
& \textref{sub:distributive-laws} \\
& = A \cap (B + C).
\end{align*}
\end{proof}
\subsection{\verified{Exercise 2.15b}}%
\hyperlabel{sub:exercise-2.15b}
2023-08-09 13:39:41 +00:00
Show that $A + (B + C) = (A + B) + C$.
\code*{Bookshelf/Enderton/Set/Chapter\_2}
{Enderton.Set.Chapter\_2.exercise\_2\_15b}
\lean{Mathlib/Order/SymmDiff}{symmDiff\_assoc}
2023-08-09 13:39:41 +00:00
\begin{proof}
Let $A$, $B$, and $C$ be sets.
We prove that
\begin{enumerate}[(i)]
\item $A + (B + C) \subseteq (A + B) + C$
\item $(A + B) + C \subseteq A + (B + C)$
\end{enumerate}
2023-08-09 13:39:41 +00:00
\paragraph{(i)}%
\hyperlabel{par:exercise-2.15b-i}
Let $x \in A + (B + C)$.
Then $x$ is in $A$ or in $B + C$, but not both.
There are two cases to consider:
\subparagraph{Case 1}%
Suppose $x \in A$ and $x \not\in B + C$.
Then, by \nameref{sub:nonmembership-symmetric-difference},
(a) $x \in B \cap C$ or (b) $x \not\in B \cup C$.
Suppose (a) was true.
That is, $x \in B$ and $x \in C$.
Since $x$ is a member of $A$ and $B$, $x \not\in (A + B)$.
Since $x$ is not a member of $A + B$ but is a member of $C$,
$x \in (A + B) + C$.
Now suppose (b) was true.
That is, $x \not\in B$ and $x \not\in C$.
Since $x$ is a member of $A$ but not $B$, $x \in (A + B)$.
Since $x$ is a member of $A + B$ but not $C$, $x \in (A + B) + C$.
\subparagraph{Case 2}%
Suppose $x \in B + C$ and $x \not\in A$.
Then (a) $x \in B$ or (b) $x \in C$ but not both.
Suppose (a) was true.
That is, $x \in B$ and $x \not\in C$.
Since $x$ is not a member of $A$ and is a member of $B$, $x \in A + B$.
Since $x$ is a member of $A + B$ but not $C$, $x \in (A + B) + C$.
Now suppose (b) was true.
That is, $x \not\in B$ and $x \in C$.
Since $x$ is not a member of $A$ nor a member of $B$, $x \not\in A + B$.
Since $x$ is not a member of $A + B$ but is a member of $C$,
$x \in (A + B) + C$.
\paragraph{(ii)}%
\hyperlabel{par:exercise-2.15b-ii}
Let $x \in (A + B) + C$.
Then $x$ is in $A + B$ or in $C$, but not both.
There are two cases to consider:
\subparagraph{Case 1}%
Suppose $x \in A + B$ and $x \not\in C$.
Then (a) $x \in A$ or (b) $x \in B$ but not both.
Suppose (a) was true.
That is, $x \in A$ and $x \not\in B$.
Since $x$ is not a member of $B$ nor $C$, $x \not\in B + C$.
Since $x$ is not a member of $B + C$ but is a member of $A$,
$x \in A + (B + C)$.
Now Suppose (b) was true.
That is, $x \not\in A$ and $x \in B$.
Since $x$ is a member of $B$ and not of $C$, then $x \in B + C$.
Since $x$ is a member of $B + C$ and not of $A$, $x \in A + (B + C)$.
\subparagraph{Case 2}%
Suppose $x \not\in A + B$ and $x \in C$.
Then, by \nameref{sub:nonmembership-symmetric-difference},
(a) $x \in A \cap B$ or (b) $x \not\in A \cup B$.
Suppose (a) was true.
That is, $x \in A \land x \in B$.
Since $x$ is a member of $B$ and $C$, $x \not\in B + C$.
Since $x$ is not a member of $B + C$ but is a member of $A$,
$x \in A + (B + C)$.
Now suppose (b) was true.
That is, $x \not\in A$ and $x \not\in B$.
Since $x$ is not a member of $B$ but is a member of $C$, $x \in B + C$.
Since $x$ is a member of $B + C$ but not of $A$, $x \in A + (B + C)$.
2023-08-09 13:39:41 +00:00
\paragraph{Conclusion}%
2023-08-09 13:39:41 +00:00
In both \nameref{par:exercise-2.15b-i} and
\nameref{par:exercise-2.15b-ii}, the subcases are exhaustive and prove
the desired subset relation.
Therefore $A + (B + C) = (A + B) + C$.
2023-08-09 13:39:41 +00:00
\end{proof}
2023-06-29 20:53:36 +00:00
\subsection{\verified{Exercise 2.16}}%
\hyperlabel{sub:exercise-2.16}
2023-08-09 13:39:41 +00:00
Simplify: $$[(A \cup B \cup C) \cap (A \cup B)] - [(A \cup (B - C)) \cap A].$$
2023-08-09 13:39:41 +00:00
\code*{Bookshelf/Enderton/Set/Chapter\_2}
2023-06-29 20:53:36 +00:00
{Enderton.Set.Chapter\_2.exercise\_2\_16}
2023-06-04 23:34:42 +00:00
2023-08-09 13:39:41 +00:00
\begin{proof}
Let $A$, $B$, and $C$ be arbitrary sets.
Then
\begin{align*}
[(A \cup B \cup C) \cap (A \cup B)] & - [(A \cup (B - C)) \cap A] \\
& = [A \cup B] - [A] \\
& = \{ x \mid x \in (A \cup B) \land x \not\in A \} \\
& = \{ x \mid x \in \{ y \mid y \in A \lor y \in B \} \land x \not\in A \} \\
& = \{ x \mid (x \in A \lor x \in B) \land x \not\in A \} \\\
& = \{ x \mid (x \in A \land x \not\in A) \lor (x \in B \land x \not\in A) \} \\
& = \{ x \mid F \lor (X \in B \land x \not\in A) \} \\
& = \{ x \mid x \in B \land x \not\in A \} \\
& = B - A.
\end{align*}
\end{proof}
2023-06-29 20:53:36 +00:00
\subsection{\verified{Exercise 2.17}}%
\hyperlabel{sub:exercise-2.17}
2023-08-09 13:39:41 +00:00
Show that the following four conditions are equivalent.
\begin{enumerate}[(a)]
\item $A \subseteq B$,
\item $A - B = \emptyset$,
\item $A \cup B = B$,
\item $A \cap B = A$.
\end{enumerate}
2023-06-04 23:34:42 +00:00
2023-08-09 13:39:41 +00:00
\code{Bookshelf/Enderton/Set/Chapter\_2}
2023-06-29 20:53:36 +00:00
{Enderton.Set.Chapter\_2.exercise\_2\_17\_i}
2023-06-04 23:34:42 +00:00
2023-08-09 13:39:41 +00:00
\code{Bookshelf/Enderton/Set/Chapter\_2}
2023-06-29 20:53:36 +00:00
{Enderton.Set.Chapter\_2.exercise\_2\_17\_ii}
2023-06-04 23:34:42 +00:00
2023-08-09 13:39:41 +00:00
\code{Bookshelf/Enderton/Set/Chapter\_2}
2023-06-29 20:53:36 +00:00
{Enderton.Set.Chapter\_2.exercise\_2\_17\_iii}
2023-06-04 23:34:42 +00:00
\code{Bookshelf/Enderton/Set/Chapter\_2}
2023-06-29 20:53:36 +00:00
{Enderton.Set.Chapter\_2.exercise\_2\_17\_iv}
2023-06-04 23:34:42 +00:00
2023-08-09 13:39:41 +00:00
\begin{proof}
Let $A$ and $B$ be arbitrary sets.
We show that (i) $(a) \Rightarrow (b)$, (ii) $(b) \Rightarrow (c)$, (iii)
$(c) \Rightarrow (d)$, and (iv) $(d) \Rightarrow (a)$.
2023-06-04 23:34:42 +00:00
2023-08-09 13:39:41 +00:00
\paragraph{(i)}%
2023-06-04 23:34:42 +00:00
2023-08-09 13:39:41 +00:00
Suppose $A \subseteq B$.
That is, $\forall t, t \in A \Rightarrow t \in B$.
Then there is no element such that $t \in A$ and $t \not\in B$.
By definition of the relative complement, this immediately implies
$A - B = \emptyset$.
2023-06-04 23:34:42 +00:00
2023-08-09 13:39:41 +00:00
\paragraph{(ii)}%
2023-06-04 23:34:42 +00:00
2023-08-09 13:39:41 +00:00
Suppose $A - B = \emptyset$.
By definition of the relative complement,
$$A - B = \emptyset = \{ x \mid x \in A \land x \not\in B \}.$$
Then, for all $t$,
$\neg(t \in A \land t \not\in B) = t \not\in A \lor t \in B$.
This implies, by definition of the subset, that $A \subseteq B$.
It then immediately follows that $A \cup B = B$.
2023-06-04 23:34:42 +00:00
2023-08-09 13:39:41 +00:00
\paragraph{(iii)}%
2023-06-04 23:34:42 +00:00
2023-08-09 13:39:41 +00:00
Suppose $A \cup B = B$.
Then there is no member of $A$ that is not a member of $B$.
In other words, $A \subseteq B$.
This immediately implies $A \cap B = A$.
2023-06-04 23:34:42 +00:00
2023-08-09 13:39:41 +00:00
\paragraph{(iv)}%
2023-06-04 23:34:42 +00:00
2023-08-09 13:39:41 +00:00
Suppose $A \cap B = A$.
Then every member of $A$ is a member of $B$.
This immediately implies $A \subseteq B$.
2023-08-09 13:39:41 +00:00
\end{proof}
\subsection{\unverified{Exercise 2.18}}%
\hyperlabel{sub:exercise-2.18}
2023-08-09 13:39:41 +00:00
Assume that $A$ and $B$ are subsets of $S$.
List all of the different sets that can be made from these three by use of the
binary operations $\cup$, $\cap$, and $-$.
\begin{proof}
We can reason about this diagrammatically:
\begin{figure}[ht]
\includegraphics[width=0.6\textwidth]{venn-diagram}
\centering
\end{figure}
In the above diagram, we assume the left circle corresponds to set $A$ and the
right circle corresponds to $B$.
The the possible sets we can make via the specified operators are:
\begin{itemize}
\item $A - B$, the left circle excluding the overlapping region.
\item $A \cap B$, the overlapping region.
\item $B - A$, the right circle excluding the overlapping region.
\item $(A \cup B) \cap A$, the left circle.
\item $(A \cup B) \cap B$, the right circle.
\item $(A - B) \cup (B - A)$, the symmetric difference.
\item $A \cup B$, the entire diagram.
\end{itemize}
\end{proof}
2023-06-29 20:53:36 +00:00
\subsection{\verified{Exercise 2.19}}%
\hyperlabel{sub:exercise-2.19}
2023-08-09 13:39:41 +00:00
Is $\powerset{(A - B)}$ always equal to $\powerset{A} - \powerset{B}$?
Is it ever equal to $\powerset{A} - \powerset{B}$?
2023-08-09 13:39:41 +00:00
\code*{Bookshelf/Enderton/Set/Chapter\_2}
2023-06-29 20:53:36 +00:00
{Enderton.Set.Chapter\_2.exercise\_2\_19}
2023-06-04 23:34:42 +00:00
2023-08-09 13:39:41 +00:00
\begin{proof}
Let $A$ and $B$ be arbitrary sets.
We show (i) that $\emptyset \in \powerset{(A - B})$ and (ii)
$\emptyset \not\in \powerset{A} - \powerset{B}$.
2023-06-04 23:34:42 +00:00
2023-08-09 13:39:41 +00:00
\paragraph{(i)}%
\hyperlabel{par:exercise-2.19-i}
2023-06-04 23:34:42 +00:00
2023-08-09 13:39:41 +00:00
By definition of the \nameref{ref:power-set},
$$\powerset{(A - B)} = \{ x \mid x \subseteq A - B \}.$$
But $\emptyset$ is a subset of \textit{every} set.
Thus $\emptyset \in \powerset{(A - B)}$.
2023-06-04 23:34:42 +00:00
2023-08-09 13:39:41 +00:00
\paragraph{(ii)}%
2023-06-04 23:34:42 +00:00
2023-08-09 13:39:41 +00:00
By the same reasoning found in \nameref{par:exercise-2.19-i},
$\emptyset \in \powerset{A}$ and $\emptyset \in \powerset{B}$.
But then, by definition of the relative complement,
$\emptyset \not\in \powerset{A} - \powerset{B}$.
2023-06-04 23:34:42 +00:00
2023-08-09 13:39:41 +00:00
\paragraph{Conclusion}%
2023-06-04 23:34:42 +00:00
2023-08-09 13:39:41 +00:00
By the \nameref{ref:extensionality-axiom}, the two sets are never equal.
2023-08-09 13:39:41 +00:00
\end{proof}
2023-06-29 20:53:36 +00:00
\subsection{\verified{Exercise 2.20}}%
\hyperlabel{sub:exercise-2.20}
2023-08-09 13:39:41 +00:00
Let $A$, $B$, and $C$ be sets such that $A \cup B = A \cup C$ and
$A \cap B = A \cap C$.
Show that $B = C$.
2023-08-09 13:39:41 +00:00
\code*{Bookshelf/Enderton/Set/Chapter\_2}
2023-06-29 20:53:36 +00:00
{Enderton.Set.Chapter\_2.exercise\_2\_20}
2023-06-04 23:34:42 +00:00
2023-08-09 13:39:41 +00:00
\begin{proof}
Let $A$, $B$, and $C$ be arbitrary sets.
By the \nameref{ref:extensionality-axiom}, $B = C$ if and only if for all
sets $x$, $x \in B \iff x \in C$.
We prove both directions of this biconditional.
2023-06-04 23:34:42 +00:00
2023-08-09 13:39:41 +00:00
\paragraph{($\Rightarrow$)}%
2023-06-04 23:34:42 +00:00
2023-08-09 13:39:41 +00:00
Suppose $x \in B$.
Then there are two cases to consider:
2023-06-04 23:34:42 +00:00
2023-08-09 13:39:41 +00:00
\subparagraph{Case 1}%
2023-06-04 23:34:42 +00:00
2023-08-09 13:39:41 +00:00
Assume $x \in A$.
Then $x \in A \cap B$.
By hypothesis, $A \cap B = A \cap C$.
Thus $x \in A \cap C$ immediately implying $x \in C$.
2023-06-04 23:34:42 +00:00
2023-08-09 13:39:41 +00:00
\subparagraph{Case 2}%
2023-06-04 23:34:42 +00:00
2023-08-09 13:39:41 +00:00
Assume $x \not\in A$.
Then $x \in A \cup B$.
By hypothesis, $A \cup B = A \cup C$.
Thus $x \in A \cup C$.
Since $x \not\in A$, it follows $x \in C$.
2023-06-04 23:34:42 +00:00
2023-08-09 13:39:41 +00:00
\paragraph{($\Leftarrow$)}%
2023-06-04 23:34:42 +00:00
2023-08-09 13:39:41 +00:00
Suppose $x \in C$.
Then there are two cases to consider:
2023-06-04 23:34:42 +00:00
2023-08-09 13:39:41 +00:00
\subparagraph{Case 1}%
2023-06-04 23:34:42 +00:00
2023-08-09 13:39:41 +00:00
Assume $x \in A$.
Then $x \in A \cap C$.
By hypothesis, $A \cap B = A \cap C$.
Thus $x \in A \cap B$, immediately implying $x \in B$.
2023-06-04 23:34:42 +00:00
2023-08-09 13:39:41 +00:00
\subparagraph{Case 2}%
2023-06-04 23:34:42 +00:00
2023-08-09 13:39:41 +00:00
Assume $x \not\in A$.
Then $x \in A \cup C$.
By hypothesis, $A \cup B = A \cup C$.
Thus $x \in A \cup B$.
Since $x \not\in A$, it follows $x \in B$.
2023-08-09 13:39:41 +00:00
\end{proof}
2023-06-29 20:53:36 +00:00
\subsection{\verified{Exercise 2.21}}%
\hyperlabel{sub:exercise-2.21}
2023-08-09 13:39:41 +00:00
Show that $\bigcup\; (A \cup B) = \bigcup A \cup \bigcup B$.
2023-08-09 13:39:41 +00:00
\code*{Bookshelf/Enderton/Set/Chapter\_2}
2023-06-29 20:53:36 +00:00
{Enderton.Set.Chapter\_2.exercise\_2\_21}
2023-06-04 23:34:42 +00:00
2023-08-09 13:39:41 +00:00
\begin{proof}
Let $A$ and $B$ be arbitrary sets.
By the \nameref{ref:extensionality-axiom}, the specified equality holds if
and only if for all sets $x$,
$$x \in \bigcup (A \cup B) \iff x \in \bigcup A \cup \bigcup B.$$
We prove both directions of this biconditional.
2023-06-04 23:34:42 +00:00
2023-08-09 13:39:41 +00:00
\paragraph{($\Rightarrow$)}%
2023-06-04 23:34:42 +00:00
2023-08-09 13:39:41 +00:00
Suppose $x \in \bigcup (A \cup B)$.
By definition of the union of sets, there exists some $b \in A \cup B$
such that $x \in b$.
If $b \in A$, then $x \in \bigcup A$ and $x \in \bigcup A \cup \bigcup B$.
Alternatively, if $b \in B$, then $x \in \bigcup B$ and
$x \in \bigcup A \cup \bigcup B$.
Regardless, $x$ is in the target set.
2023-06-04 23:34:42 +00:00
2023-08-09 13:39:41 +00:00
\paragraph{($\Leftarrow$)}%
2023-06-04 23:34:42 +00:00
2023-08-09 13:39:41 +00:00
Suppose $x \in \bigcup A \cup \bigcup B$.
Then $x \in \bigcup A$ or $x \in \bigcup B$.
WLOG, suppose $x \in \bigcup A$.
By definition of the union of sets, there exists some $b \in A$ such that
$x \in b$.
But then $b \in A \cup B$ meaning $x$ is also a member of
$\bigcup (A \cup B)$.
2023-08-09 13:39:41 +00:00
\end{proof}
2023-06-29 20:53:36 +00:00
\subsection{\verified{Exercise 2.22}}%
\hyperlabel{sub:exercise-2.22}
2023-08-09 13:39:41 +00:00
Show that if $A$ and $B$ are nonempty sets, then
$\bigcap (A \cup B) = \bigcap A \cap \bigcap B$.
2023-08-09 13:39:41 +00:00
\code*{Bookshelf/Enderton/Set/Chapter\_2}
2023-06-29 20:53:36 +00:00
{Enderton.Set.Chapter\_2.exercise\_2\_22}
2023-06-04 23:34:42 +00:00
2023-08-09 13:39:41 +00:00
\begin{proof}
Let $A$ and $B$ be arbitrary, nonempty sets.
By the \nameref{ref:extensionality-axiom}, the specified equality holds if
and only if for all sets $x$,
\begin{equation}
\hyperlabel{sub:exercise-2.22-eq1}
x \in \bigcap (A \cup B) \iff x \in \bigcap A \cap \bigcap B.
\end{equation}
We prove both directions of this biconditional.
2023-06-04 23:34:42 +00:00
2023-08-09 13:39:41 +00:00
\paragraph{($\Rightarrow$)}%
2023-06-04 23:34:42 +00:00
2023-08-09 13:39:41 +00:00
Suppose $x \in \bigcap (A \cup B)$.
Then for all $b \in A \cup B$, $x \in B$.
In other words, for every member $b_1$ of $A$ and every member $b_2$ of
$B$, $x$ is a member of both $b_1$ and $b_2$.
But that implies $x \in \bigcap A$ and $x \in \bigcap B$.
2023-06-04 23:34:42 +00:00
2023-08-09 13:39:41 +00:00
\paragraph{($\Leftarrow$)}%
2023-06-04 23:34:42 +00:00
2023-08-09 13:39:41 +00:00
Suppose $x \in \bigcap A \cap \bigcap B$.
That is, $x \in \bigcap A$ and $x \in \bigcap B$.
By definition of the intersection of sets, forall sets $b$, if $b \in A$,
then $x \in b$.
Likewise, if $b \in B$, then $x \in b$.
In other words, if $b$ is a member of either $A$ or $B$, $x \in b$.
That immediately implies $x \in \bigcap (A \cup B$.
2023-08-09 13:39:41 +00:00
\end{proof}
\subsection{\unverified{Exercise 2.23}}%
\hyperlabel{sub:exercise-2.23}
2023-08-09 13:39:41 +00:00
Show that if $\mathscr{B}$ is nonempty, then
$A \cup \bigcap \mathscr{B} =
\bigcap\; \{A \cup X \mid X \in \mathscr{B} \}$.
2023-08-09 13:39:41 +00:00
\begin{proof}
Refer to \nameref{sub:general-distributive-laws}.
\end{proof}
2023-06-29 20:53:36 +00:00
\subsection{\verified{Exercise 2.24a}}%
\hyperlabel{sub:exercise-2.24a}
2023-08-09 13:39:41 +00:00
Show that if $\mathscr{A}$ is nonempty, then
$\powerset{\bigcap\mathscr{A}} =
\bigcap\; \{\powerset{X} \mid X \in \mathscr{A} \}$.
2023-08-09 13:39:41 +00:00
\code*{Bookshelf/Enderton/Set/Chapter\_2}
2023-06-29 20:53:36 +00:00
{Enderton.Set.Chapter\_2.exercise\_2\_24a}
2023-06-04 23:34:42 +00:00
2023-08-09 13:39:41 +00:00
\begin{proof}
Suppose $\mathscr{A}$ is a nonempty set.
Then $\bigcap \mathscr{A}$ is well-defined.
Therefore
\begin{align*}
\powerset{\bigcap\mathscr{A}}
& = \{ x \mid x \subseteq \bigcap \mathscr{A} \}
& \textref{ref:power-set} \\
& = \{ x \mid x \subseteq
\{ y \mid \forall X \in \mathscr{A}, y \in X \} \}
& \text{def'n intersection} \\
& = \{ x \mid \forall t \in x,
t \in \{ y \mid \forall X \in \mathscr{A}, y \in X \} \}
& \text{def'n subset} \\
& = \{ x \mid \forall t \in x,
(\forall X \in \mathscr{A}, t \in X) \} \\
& = \{ x \mid \forall X \in \mathscr{A},
(\forall t \in x, t \in X) \} \\
& = \{ x \mid \forall X \in \mathscr{A}, x \subseteq X \} \\
& = \{ x \mid \forall X \in \mathscr{A}, x \in \powerset{X} \}
& \textref{ref:power-set-axiom} \\
& = \{ x \mid
\forall t \in \{ \powerset{X} \mid X \in \mathscr{A} \}, x \in t \} \\
& = \bigcap\; \{\powerset{X} \mid X \in \mathscr{A}\}.
\end{align*}
\end{proof}
2023-06-29 20:53:36 +00:00
\subsection{\verified{Exercise 2.24b}}%
\hyperlabel{sub:exercise-2.24b}
2023-08-09 13:39:41 +00:00
Show that
\begin{equation}
\hyperlabel{sub:exercise-2.24b-eq1}
\bigcup\; \{ \powerset{X} \mid X \in \mathscr{A} \} \subseteq
\powerset{\bigcup\mathscr{A}}.
\end{equation}
Under what conditions does equality hold?
2023-08-09 13:39:41 +00:00
\code*{Bookshelf/Enderton/Set/Chapter\_2}
2023-06-29 20:53:36 +00:00
{Enderton.Set.Chapter\_2.exercise\_2\_24b}
2023-06-04 23:34:42 +00:00
2023-08-09 13:39:41 +00:00
\begin{proof}
We first prove \eqref{sub:exercise-2.24b-eq1}.
Let $x \in \bigcup\; \{ \powerset{X} \mid X \in \mathscr{A} \}$.
By definition of the union of sets,
$(\exists X \in \mathscr{A}), x \in \powerset{X}$.
By definition of the \nameref{ref:power-set}, $x \subseteq X$.
By \nameref{sub:exercise-2.3}, $X \subseteq \bigcup \mathscr{A}$.
Therefore $x \subseteq \bigcup \mathscr{A}$, proving
$x \in \powerset{\mathscr{A}}$ as expected.
\suitdivider
\noindent
We show $\powerset{\bigcup A} \subseteq
\bigcup\;\{ \powerset{X} \mid X \in \mathscr{A} \}$ if and only if
$\bigcup\mathscr{A} \in \mathscr{A}$.
\paragraph{($\Rightarrow$)}%
Suppose $\powerset{\bigcup\mathscr{A}} \subseteq
\bigcup\;\{ \powerset{X} \mid X \in \mathscr{A} \}$.
By definition of the \nameref{ref:power-set},
$\bigcup\mathscr{A} \in \powerset{\bigcup\mathscr{A}}$.
By hypothesis, $\bigcup\mathscr{A} \in
\bigcup\;\{ \powerset{X} \mid X \in \mathscr{A} \}$.
By definition of the union of sets, there exists some $X \in \mathscr{A}$
such that $\bigcup\mathscr{A} \in \powerset{X}$.
That is, $\bigcup\mathscr{A} \subseteq X$.
But $\bigcup\mathscr{A}$ cannot be a proper subset of $X$ since
$X \in \mathscr{A}$.
Thus $\bigcup\mathscr{A} = X$.
This proves $\bigcup\mathscr{A} \in
\bigcup\;\{ \powerset{X} \mid X \in \mathscr{A} \}$.
\paragraph{($\Leftarrow$)}%
Suppose $\bigcup\mathscr{A} \in A$.
Let $x \in \powerset{\bigcup\mathscr{A}}$.
Since $\bigcup\mathscr{A} \in \mathscr{A}$, it immediately follows that
$x \in \{\powerset{X} \mid X \in \mathscr{A}\}$.
\paragraph{Conclusion}%
Equality follows immediately from this fact in conjunction with the proof
of \eqref{sub:exercise-2.24b-eq1}.
\end{proof}
2023-06-29 20:53:36 +00:00
\subsection{\verified{Exercise 2.25}}%
\hyperlabel{sub:exercise-2.25}
2023-08-09 13:39:41 +00:00
Is $A \cup \bigcup \mathscr{B}$ always the same as
$\bigcup\;\{ A \cup X \mid X \in \mathscr{B} \}$?
If not, then under what conditions does equality hold?
2023-08-09 13:39:41 +00:00
\code*{Bookshelf/Enderton/Set/Chapter\_2}
2023-06-29 20:53:36 +00:00
{Enderton.Set.Chapter\_2.exercise\_2\_25}
2023-06-04 23:34:42 +00:00
2023-08-09 13:39:41 +00:00
\begin{proof}
We prove that
\begin{equation}
\hyperlabel{sub:exercise-2.25-eq1}
A \cup \bigcup \mathscr{B} =
\bigcup\;\{ A \cup X \mid X \in \mathscr{B} \}
\end{equation}
if and only if $A = \emptyset$ or $\mathscr{B} \neq \emptyset$.
We prove both directions of this biconditional.
2023-06-04 23:34:42 +00:00
2023-08-09 13:39:41 +00:00
\paragraph{($\Rightarrow$)}%
2023-06-04 23:34:42 +00:00
2023-08-09 13:39:41 +00:00
Suppose \eqref{sub:exercise-2.25-eq1} holds true.
There are two cases to consider:
2023-06-04 23:34:42 +00:00
2023-08-09 13:39:41 +00:00
\subparagraph{Case 1}%
2023-06-04 23:34:42 +00:00
2023-08-09 13:39:41 +00:00
Suppose $B \neq \emptyset$.
Then $A = \emptyset \lor \mathscr{B} \neq \emptyset$ holds trivially.
2023-06-04 23:34:42 +00:00
2023-08-09 13:39:41 +00:00
\subparagraph{Case 2}%
2023-06-04 23:34:42 +00:00
2023-08-09 13:39:41 +00:00
Suppose $B = \emptyset$.
Then $$A \cup \bigcup \mathscr{B} = A \cup \bigcup \emptyset = A$$ and
$$
\bigcup\;\{ A \cup X \mid X \in \mathscr{B} \}
= \bigcup \emptyset \\
= \emptyset.
$$
Then by hypothesis \eqref{sub:exercise-2.25-eq1}, $A = \emptyset$.
Then $A = \emptyset \lor \mathscr{B} \neq \emptyset$ holds trivially.
2023-06-04 23:34:42 +00:00
2023-08-09 13:39:41 +00:00
\paragraph{($\Leftarrow$)}%
2023-06-04 23:34:42 +00:00
2023-08-09 13:39:41 +00:00
Suppose $A = \emptyset$ or $\mathscr{B} \neq \emptyset$.
There are two cases to consider:
2023-06-04 23:34:42 +00:00
2023-08-09 13:39:41 +00:00
\paragraph{Case 1}%
2023-06-04 23:34:42 +00:00
2023-08-09 13:39:41 +00:00
Suppose $A = \emptyset$.
Then $A \cup \bigcup \mathscr{B} = \bigcup{\mathscr{B}}$.
Likewise,
$$
\bigcup \{ A \cup X \mid X \in \mathscr{B} \}
= \bigcup \{ X \mid X \in \mathscr{B} \} \\
= \bigcup \mathscr{B}.
$$
Therefore \eqref{sub:exercise-2.25-eq1} holds.
2023-06-04 23:34:42 +00:00
2023-08-09 13:39:41 +00:00
\paragraph{Case 2}%
2023-06-04 23:34:42 +00:00
2023-08-09 13:39:41 +00:00
Suppose $B \neq \emptyset$.
Then
\begin{align*}
A \cup \bigcup\mathscr{B}
& = \{ x \mid x \in A \lor x \in \bigcup\mathscr{B} \} \\
& = \{ x \mid x \in A \lor (\exists b \in \mathscr{B}) x \in b \} \\
& = \{ x \mid (\exists b \in \mathscr{B}) x \in A \lor x \in b \} \\
& = \{ x \mid (\exists b \in \mathscr{B}) x \in A \cup b \} \\
& = \{ x \mid x \in \bigcup \{ A \cup X \mid X \in \mathscr{B} \} \\
& = \bigcup \{ A \cup X \mid X \in \mathscr{B} \}.
\end{align*}
Therefore \eqref{sub:exercise-2.25-eq1} holds.
2023-08-09 13:39:41 +00:00
\end{proof}
2023-06-07 02:16:06 +00:00
\chapter{Relations and Functions}%
\hyperlabel{chap:relations-functions}
2023-06-07 02:16:06 +00:00
\section{Ordered Pairs}%
\hyperlabel{sec:ordered-pairs}
2023-06-07 02:16:06 +00:00
\subsection{\verified{Theorem 3A}}%
\hyperlabel{sub:theorem-3a}
2023-06-07 02:16:06 +00:00
2023-08-09 13:39:41 +00:00
\begin{theorem}[3A]
For any sets $x$, $y$, $u$, and $v$,
\begin{equation}
\hyperlabel{sub:theorem-3a-eq1}
\tuple{u, v} = \tuple{x, y} \iff u = x \land v = y.
2023-08-09 13:39:41 +00:00
\end{equation}
\end{theorem}
2023-06-07 02:16:06 +00:00
\code{Bookshelf/Enderton/Set/Chapter\_3}
{Enderton.Set.Chapter\_3.OrderedPair.ext\_iff}
2023-06-07 02:16:06 +00:00
2023-08-09 13:39:41 +00:00
\begin{proof}
Let $x$, $y$, $u$, and $v$ be arbitrary sets.
2023-06-07 02:16:06 +00:00
2023-08-09 13:39:41 +00:00
\paragraph{($\Leftarrow$)}%
2023-06-07 02:16:06 +00:00
2023-08-09 13:39:41 +00:00
This follows trivially.
2023-06-07 02:16:06 +00:00
2023-08-09 13:39:41 +00:00
\paragraph{($\Rightarrow$)}%
2023-06-07 02:16:06 +00:00
Suppose $\tuple{u, v} = \tuple{x, y}$.
2023-08-09 13:39:41 +00:00
Then, by definition of an \nameref{ref:ordered-pair},
\begin{equation}
\hyperlabel{sub:theorem-3a-eq2}
\{\{u\}, \{u, v\}\} = \{\{x\}, \{x, y\}\}.
\end{equation}
By the \nameref{ref:extensionality-axiom}, it follows
$\{u\} \in \{\{x\}, \{x, y\}\}$ and
$\{u, v\} \in \{\{x\}, \{x, y\}\}$.
That is,
$$\{u\} = \{x\} \quad\text{or}\quad \{u\} = \{x, y\}$$
and
$$\{u, v\} = \{x\} \quad\text{or}\quad \{u, v\} = \{x, y\}.$$
There are 4 cases to consider:
2023-06-07 02:16:06 +00:00
2023-08-09 13:39:41 +00:00
\paragraph{Case 1}%
2023-06-07 02:16:06 +00:00
2023-08-09 13:39:41 +00:00
Suppose $\{u\} = \{x\}$ and $\{u, v\} = \{x\}$.
The former identity implies $u = x$.
The latter identity implies $u = v = x$.
Then \eqref{sub:theorem-3a-eq2} simplifies to
$$\{\{u\}\} = \{\{x\}, \{x, y\}\},$$ meaning $x = y$.
Thus $v = y$ as well.
2023-06-07 02:16:06 +00:00
2023-08-09 13:39:41 +00:00
\paragraph{Case 2}%
2023-06-07 02:16:06 +00:00
2023-08-09 13:39:41 +00:00
Suppose $\{u\} = \{x\}$ and $\{u, v\} = \{x, y\}$.
The former identity implies $u = x$.
Substituting into the latter identity yields $\{u, v\} = \{u, y\}$.
This holds if and only if $v = y$.
2023-06-07 02:16:06 +00:00
2023-08-09 13:39:41 +00:00
\paragraph{Case 3}%
2023-06-07 02:16:06 +00:00
2023-08-09 13:39:41 +00:00
Suppose $\{u\} = \{x, y\}$ and $\{u, v\} = \{x\}$.
The former identity implies $x = y = u$.
Substituting into the latter yields $\{u, v\} = \{u\}$.
Thus $u = v$ which in turn implies $v = y$.
2023-06-07 02:16:06 +00:00
2023-08-09 13:39:41 +00:00
\paragraph{Case 4}%
Suppose $\{u\} = \{x, y\}$ and $\{u, v\} = \{x, y\}$.
The former identity implies $x = y = u$.
Substituting into the latter yields $\{u, v\} = \{u\}$.
This implies $v = u$ which in turn implies $v = y$.
2023-06-07 02:16:06 +00:00
2023-08-09 13:39:41 +00:00
\paragraph{Conclusion}%
2023-06-07 02:16:06 +00:00
2023-08-09 13:39:41 +00:00
These cases are exhaustive and each implies that $u = x$ and $v = y$.
2023-06-07 02:16:06 +00:00
2023-08-09 13:39:41 +00:00
\end{proof}
2023-06-07 02:16:06 +00:00
2023-06-08 23:57:02 +00:00
\subsection{\verified{Lemma 3B}}%
\hyperlabel{sub:lemma-3b}
2023-06-08 23:57:02 +00:00
2023-08-09 13:39:41 +00:00
\begin{lemma}[3B]
If $x \in C$ and $y \in C$, then $\tuple{x, y} \in \powerset{\powerset{C}}$.
2023-08-09 13:39:41 +00:00
\end{lemma}
2023-06-08 23:57:02 +00:00
\code{Bookshelf/Enderton/Set/Chapter\_3}
2023-08-09 13:39:41 +00:00
{Enderton.Set.Chapter\_3.lemma\_3b}
\begin{proof}
Let $C$ be an arbitrary set and $x, y \in C$.
Then, by definition of the \nameref{ref:power-set},
$\{x\}$ and $\{x, y\}$ are members of $\powerset{C}$.
Likewise, $\{\{x\}, \{x, y\}\}$ is a member of $\powerset{\powerset{C}}$.
By definition of an \nameref{ref:ordered-pair},
$\tuple{x, y} = \{\{x\}, \{x, y\}\}$.
2023-08-09 13:39:41 +00:00
This concludes our proof.
\end{proof}
2023-06-08 23:57:02 +00:00
2023-08-09 18:59:23 +00:00
\subsection{\unverified{Corollary 3C}}%
\hyperlabel{sub:corollary-3c}
2023-06-08 23:57:02 +00:00
2023-08-09 13:39:41 +00:00
\begin{theorem}[3C]
For any sets $A$ and $B$, there is a set whose members are exactly the
pairs $\tuple{x, y}$ with $x \in A$ and $y \in B$.
2023-08-09 13:39:41 +00:00
\end{theorem}
\lean{Mathlib/SetTheory/ZFC/Basic}{Set.prod}
\begin{proof}
Define $C = A \cup B$.
Then for all $x \in A$ and for all $y \in B$, $x$ and $y$ are both in $C$.
By \nameref{sub:lemma-3b}, it follows that
$\tuple{x, y} \in \powerset{\powerset{C}}$.
2023-08-09 13:39:41 +00:00
The \nameref{ref:power-set-axiom} indicates $\powerset{\powerset{C}}$ is
indeed a set.
Therefore the \nameref{ref:subset-axioms} are applicable.
This implies the existence of a set $D$ such that
$$\forall z, (z \in D \iff z \in \powerset{\powerset{C}} \land
(\exists x, \exists y, x \in A \land y \in B \land
z = \tuple{x, y})).$$
2023-08-09 13:39:41 +00:00
By construction $D$ is the set whose members are exactly the pairs
$\tuple{x, y}$ with $x \in A$ and $y \in B$.
2023-08-09 13:39:41 +00:00
\end{proof}
2023-06-08 23:57:02 +00:00
2023-06-29 20:53:36 +00:00
\section{Relations}%
\hyperlabel{sec:relations}
2023-06-10 03:03:06 +00:00
2023-06-29 20:53:36 +00:00
\subsection{\verified{Theorem 3D}}%
\hyperlabel{sub:theorem-3d}
2023-06-10 03:03:06 +00:00
2023-08-09 13:39:41 +00:00
\begin{theorem}[3D]
If $\tuple{x, y} \in A$, then $x$ and $y$ belong to $\bigcup\bigcup A$.
2023-08-09 13:39:41 +00:00
\end{theorem}
2023-06-10 03:03:06 +00:00
\code{Bookshelf/Enderton/Set/Chapter\_3}
2023-06-29 20:53:36 +00:00
{Enderton.Set.Chapter\_3.theorem\_3d}
2023-06-10 03:03:06 +00:00
2023-08-09 13:39:41 +00:00
\begin{proof}
Let $A$ be a set and $\tuple{x, y} \in A$.
2023-08-09 13:39:41 +00:00
By definition of an \nameref{ref:ordered-pair},
$$\tuple{x, y} = \{\{x\}, \{x, y\}\}.$$
2023-08-09 13:39:41 +00:00
By \nameref{sub:exercise-2.3}, $\{\{x\}, \{x, y\}\} \subseteq \bigcup A$.
Then $\{x, y\} \in \bigcup A$.
Another application of \nameref{sub:exercise-2.3} implies
$\{x, y\} \subseteq \bigcup\bigcup A$.
Therefore $x, y \in \bigcup\bigcup A$.
\end{proof}
2023-06-10 03:03:06 +00:00
2023-06-29 20:53:36 +00:00
\section{Functions}%
\hyperlabel{sec:functions}
2023-06-10 03:03:06 +00:00
2023-06-29 20:53:36 +00:00
\subsection{\verified{Theorem 3E}}%
\hyperlabel{sub:theorem-3e}
2023-06-10 03:03:06 +00:00
2023-08-09 13:39:41 +00:00
\begin{theorem}[3E]
For a set $F$, $\dom{(F^{-1})} = \ran{F}$ and $\ran{(F^{-1})} = \dom{F}$.
For a relation $F$, $(F^{-1})^{-1} = F$.
\end{theorem}
2023-06-10 03:03:06 +00:00
2023-08-09 13:39:41 +00:00
\code{Bookshelf/Enderton/Set/Relation}
2023-06-29 20:53:36 +00:00
{Set.Relation.dom\_inv\_eq\_ran\_self}
2023-06-10 03:03:06 +00:00
2023-08-09 13:39:41 +00:00
\code{Bookshelf/Enderton/Set/Relation}
2023-06-29 20:53:36 +00:00
{Set.Relation.ran\_inv\_eq\_dom\_self}
2023-06-10 03:03:06 +00:00
\code{Bookshelf/Enderton/Set/Relation}
2023-06-29 20:53:36 +00:00
{Set.Relation.inv\_inv\_eq\_self}
2023-06-10 03:03:06 +00:00
2023-08-09 13:39:41 +00:00
\begin{proof}
We prove that (i) $\dom{(F^{-1})} = \ran{F}$, (ii)
$\ran{(F^{-1})} = \dom{F}$, and (iii) $(F^{-1})^{-1} = F$.
2023-06-10 03:03:06 +00:00
2023-08-09 13:39:41 +00:00
\paragraph{(i)}%
2023-06-10 03:03:06 +00:00
2023-08-09 13:39:41 +00:00
By definition of the \nameref{ref:domain}, $x \in \dom{(F^{-1})}$ if and
only if there exists some $y$ such that $\tuple{x, y} \in F^{-1}$.
2023-08-09 13:39:41 +00:00
By definition of the \nameref{ref:inverse} of a set,
$\tuple{y, x} \in F$.
2023-08-09 13:39:41 +00:00
By definition of the \nameref{ref:range}, $x \in \ran{F}$.
Since each step holds biconditionally, it follows
$\dom{(F^{-1})} = \ran{F}$ as expected.
2023-06-10 11:51:42 +00:00
2023-08-09 13:39:41 +00:00
\paragraph{(ii)}%
2023-06-10 11:51:42 +00:00
2023-08-09 13:39:41 +00:00
By definition of the \nameref{ref:range}, $x \in \ran{(F^{-1})}$ if and
only if there exists some $t$ such that $\tuple{t, x} \in F^{-1}$.
2023-08-09 13:39:41 +00:00
By definition of the \nameref{ref:inverse} of a set,
$\tuple{x, t} \in F$.
2023-08-09 13:39:41 +00:00
By definition of the \nameref{ref:domain}, $x \in \dom{F}$.
Since each step holds biconditionally, it follows
$\ran{(F^{-1})} = \dom{F}$.
2023-06-10 11:51:42 +00:00
2023-08-09 13:39:41 +00:00
\paragraph{(iii)}%
2023-06-10 11:51:42 +00:00
2023-08-09 13:39:41 +00:00
By definition of the \nameref{ref:inverse} of a set,
\begin{align*}
(F^{-1})^{-1}
& = \{\tuple{u, v} \mid \tuple{v, u} \in F^{-1}\} \\
& = \{\tuple{u, v} \mid \tuple{u, v} \in F\} \\
2023-08-09 13:39:41 +00:00
& = F.
\end{align*}
2023-06-10 11:51:42 +00:00
2023-08-09 13:39:41 +00:00
\end{proof}
2023-06-10 11:51:42 +00:00
2023-06-29 20:53:36 +00:00
\subsection{\verified{Theorem 3F}}%
\hyperlabel{sub:theorem-3f}
2023-06-10 11:51:42 +00:00
2023-08-09 13:39:41 +00:00
\begin{theorem}[3F]
For a set $F$, $F^{-1}$ is a function iff $F$ is single-rooted.
A relation $F$ is a function iff $F^{-1}$ is single-rooted.
\end{theorem}
2023-06-10 11:51:42 +00:00
2023-08-09 13:39:41 +00:00
\code{Bookshelf/Enderton/Set/Relation}
2023-06-29 20:53:36 +00:00
{Set.Relation.single\_valued\_inv\_iff\_single\_rooted\_self}
2023-06-10 11:51:42 +00:00
\code{Bookshelf/Enderton/Set/Relation}
2023-06-29 20:53:36 +00:00
{Set.Relation.single\_valued\_self\_iff\_single\_rooted\_inv}
2023-06-10 11:51:42 +00:00
2023-08-09 13:39:41 +00:00
\begin{proof}
We prove that (i) any set $F$, $F^{-1}$ is a function iff $F$ is
single-rooted and (ii) any relation $F$ is a function iff $F^{-1}$ is
single-rooted.
2023-08-09 13:39:41 +00:00
\paragraph{(i)}%
\hyperlabel{par:theorem-3f-i}
2023-08-09 13:39:41 +00:00
Let $F$ be any set.
2023-06-10 11:51:42 +00:00
2023-08-09 13:39:41 +00:00
\subparagraph{($\Rightarrow$)}%
2023-06-10 11:51:42 +00:00
2023-08-09 13:39:41 +00:00
Suppose $F^{-1}$ is a \nameref{ref:function}.
By definition, for each $x \in \dom{(F^{-1})}$, there is only one $y$
such that $\tuple{x, y} \in F^{-1}$.
2023-08-09 13:39:41 +00:00
By definition of the \nameref{ref:inverse} of $F$,
$F^{-1} = \{\tuple{u, v} \mid vFu\}$.
2023-08-09 13:39:41 +00:00
Then for each $x \in \ran{F}$, there exists exactly one $y$ such that
$\tuple{y, x} \in F$.
2023-08-09 13:39:41 +00:00
This definitionally means $F$ is single-rooted.
2023-06-10 11:51:42 +00:00
2023-08-09 13:39:41 +00:00
\subparagraph{($\Leftarrow$)}%
2023-06-10 11:51:42 +00:00
2023-08-09 13:39:41 +00:00
Suppose $F$ is single-rooted.
By definition, for each $x \in \ran{F}$, there is only one $t$ such that
$\tuple{t, x} \in F$.
2023-08-09 13:39:41 +00:00
By definition of the \nameref{ref:inverse} of $F$,
$F^{-1} = \{\tuple{u, v} \mid vFu\}$.
2023-08-09 13:39:41 +00:00
Then for each $x \in \dom{(F^{-1})}$ there exists exactly one $t$ such
that $\tuple{x, t} \in F^{-1}$.
2023-08-09 13:39:41 +00:00
This definitionally means $F^{-1}$ is a function.
2023-06-10 11:51:42 +00:00
2023-08-09 13:39:41 +00:00
\paragraph{(ii)}%
2023-06-10 11:51:42 +00:00
2023-08-09 13:39:41 +00:00
Let $F$ be a \nameref{ref:relation}.
2023-06-10 11:51:42 +00:00
2023-08-09 13:39:41 +00:00
\subparagraph{($\Rightarrow$)}%
2023-06-10 11:51:42 +00:00
2023-08-09 13:39:41 +00:00
Suppose $F$ is a function.
By \nameref{sub:theorem-3e}, $F = (F^{-1})^{-1}$.
Then by \nameref{par:theorem-3f-i}, $F^{-1}$ is single-rooted.
2023-06-10 11:51:42 +00:00
2023-08-09 13:39:41 +00:00
\subparagraph{($\Leftarrow$)}%
2023-08-09 13:39:41 +00:00
Suppose $F^{-1}$ is single-rooted.
Then by \nameref{par:theorem-3f-i}, $(F^{-1})^{-1}$ is a function.
By \nameref{sub:theorem-3e}, $(F^{-1})^{-1} = F$.
Thus $F$ is a function.
2023-08-09 13:39:41 +00:00
\end{proof}
\subsection{\verified{One-to-One Inverse}}%
\hyperlabel{sub:one-to-one-inverse}
\begin{lemma}
For any one-to-one function $F$, $F^{-1}$ is also a one-to-one function.
2023-08-09 13:39:41 +00:00
\end{lemma}
\code{Bookshelf/Enderton/Set/Relation}
2023-06-29 20:53:36 +00:00
{Set.Relation.one\_to\_one\_self\_iff\_one\_to\_one\_inv}
2023-08-09 13:39:41 +00:00
\begin{proof}
We prove that (i) $F^{-1}$ is a function and (ii) $F^{-1}$ is single-rooted.
2023-06-10 11:51:42 +00:00
2023-08-09 13:39:41 +00:00
\paragraph{(i)}%
\hyperlabel{par:lemma-1-i}
2023-06-10 11:51:42 +00:00
2023-08-09 13:39:41 +00:00
By hypothesis, $F$ is one-to-one.
This means it is single-rooted, i.e. for all $x \in \ran{F}$, there exists
exactly one $t$ such that $\tuple{t, x} \in F$.
2023-08-09 13:39:41 +00:00
By definition of the \nameref{ref:inverse} of $F$,
$\tuple{x, t} \in F^{-1}$.
2023-08-09 13:39:41 +00:00
But then for all $x \in \dom{(F^{-1})}$, there exists exactly one $t$ such
that $\tuple{x, t} \in F^{-1}$.
2023-08-09 13:39:41 +00:00
Thus $F^{-1}$ is a function.
2023-08-09 13:39:41 +00:00
\paragraph{(ii)}%
\hyperlabel{par:lemma-1-ii}
2023-08-09 13:39:41 +00:00
By hypothesis, $F$ is single-valued.
That is, for all $x \in \dom{F}$, there exists exactly one $y$ such that
$\tuple{x, y} \in F$.
2023-08-09 13:39:41 +00:00
By definition of the \nameref{ref:inverse} of $F$,
$\tuple{y, x} \in F^{-1}$.
2023-08-09 13:39:41 +00:00
But then for all $x \in \ran{(F^{-1})}$, there exists exactly one $y$ such
that $\tuple{y, x} \in F^{-1}$.
2023-08-09 13:39:41 +00:00
Thus $F^{-1}$ is single-rooted.
2023-08-09 13:39:41 +00:00
\paragraph{Conclusion}%
2023-08-09 13:39:41 +00:00
By \nameref{par:lemma-1-i} and \nameref{par:lemma-1-ii}, $F^{-1}$ is
a one-to-one function.
2023-08-09 13:39:41 +00:00
\end{proof}
2023-06-29 20:53:36 +00:00
\subsection{\verified{Theorem 3G}}%
\hyperlabel{sub:theorem-3g}
2023-08-09 13:39:41 +00:00
\begin{theorem}[3G]
Assume that $F$ is a one-to-one function.
If $x \in \dom{F}$, then $F^{-1}(F(x)) = x$.
If $y \in \ran{F}$, then $F(F^{-1}(y)) = y$.
\end{theorem}
2023-08-09 13:39:41 +00:00
\code{Bookshelf/Enderton/Set/Chapter\_3}
2023-06-29 20:53:36 +00:00
{Enderton.Set.Chapter\_3.theorem\_3g\_i}
\code{Bookshelf/Enderton/Set/Chapter\_3}
2023-06-29 20:53:36 +00:00
{Enderton.Set.Chapter\_3.theorem\_3g\_ii}
2023-08-09 13:39:41 +00:00
\begin{proof}
Suppose $F$ is a one-to-one \nameref{ref:function}.
Then \nameref{sub:one-to-one-inverse} indicates $F^{-1}$ is a one-to-one
function with domain $\ran{F}$ and range $\dom{F}$.
For all $x \in \dom{F}$, $\tuple{x, F(x)} \in F$.
Then $\tuple{F(x), x} \in F^{-1}$.
2023-08-09 13:39:41 +00:00
Since $F^{-1}$ is single-valued, $F^{-1}(F(x)) = x$.
For all $y \in \ran{F}$, $\tuple{y, F^{-1}(y)} \in F^{-1}$.
Then $\tuple{F^{-1}(y), y} \in F$.
2023-08-09 13:39:41 +00:00
Since $F$ is single-valued, $F(F^{-1}(y)) = y$.
\end{proof}
2023-06-29 20:53:36 +00:00
\subsection{\verified{Theorem 3H}}%
\hyperlabel{sub:theorem-3h}
2023-08-09 13:39:41 +00:00
\begin{theorem}[3H]
Assume that $F$ and $G$ are functions.
Then $F \circ G$ is a function, its domain is
\begin{equation}
\hyperlabel{sub:theorem-3h-eq1}
\{x \in \dom{G} \mid G(x) \in \dom{F}\},
\end{equation}
and for $x$ in its domain, $(F \circ G)(x) = F(G(x))$.
\end{theorem}
2023-06-29 20:53:36 +00:00
2023-08-09 13:39:41 +00:00
\code{Bookshelf/Enderton/Set/Relation}
2023-06-29 20:53:36 +00:00
{Set.Relation.single\_valued\_comp\_is\_single\_valued}
\code{Bookshelf/Enderton/Set/Chapter\_3}
2023-06-29 20:53:36 +00:00
{Enderton.Set.Chapter\_3.theorem\_3h\_dom}
2023-08-09 13:39:41 +00:00
\begin{proof}
Let $F$ and $G$ be \nameref{ref:function}s.
By definition of the \nameref{ref:composition} of $F$ and $G$,
\begin{equation}
\hyperlabel{sub:theorem-3h-eq2}
F \circ G = \{\tuple{u, v} \mid \exists t(uGt \land tFv)\}.
2023-08-09 13:39:41 +00:00
\end{equation}
By construction, $F \circ G$ is a relation.
By the definition of the \nameref{ref:domain} of a relation,
$x \in \dom{(F \circ G)}$ if and only if there exists some $y$ such that
$\tuple{x, y} \in F \circ G$.
2023-08-09 13:39:41 +00:00
We prove that (i) $F \circ G$ is a function with domain satisfying
\eqref{sub:theorem-3h-eq1}, and (ii) $(F \circ G)(x) = F(G(x))$.
2023-08-09 13:39:41 +00:00
\paragraph{(i)}%
\hyperlabel{par:theorem-3h-i}
2023-08-09 13:39:41 +00:00
By \eqref{sub:theorem-3h-eq2}, there exists some $t$ such that
$\tuple{x, t} \in G$ and $\tuple{t, y} \in F$.
2023-08-09 13:39:41 +00:00
Since $G$ is single-valued, $t$ is uniquely determined by $x$.
Since $F$ is single-valued, $y$ is uniquely determined by $t$.
Therefore, by transitivity, $y$ is uniquely determined by $x$.
Thus $F \circ G$ is single-valued, i.e. $F \circ G$ is a function.
2023-08-09 13:39:41 +00:00
Furthermore, by definition of function application, $t = G(x)$.
Thus $$\tuple{x, G(x)} \in G \quad\text{and}\quad \tuple{G(x), y} \in F.$$
2023-08-09 13:39:41 +00:00
This immediately implies \eqref{sub:theorem-3h-eq1} holds true.
2023-08-09 13:39:41 +00:00
\paragraph{(ii)}%
2023-08-09 13:39:41 +00:00
Let $x \in \dom{(F \circ G)}$.
By definition, $\tuple{x, (F \circ G)(x)} \in F \circ G$.
2023-08-09 13:39:41 +00:00
Then \eqref{sub:theorem-3h-eq2} implies $(F \circ G)(x)$ satisfies
$\tuple{G(x), (F \circ G)(x)} \in F$.
2023-08-09 13:39:41 +00:00
This is equivalent to saying $F(G(x)) = (F \circ G)(x)$ as expected.
2023-08-09 13:39:41 +00:00
\end{proof}
\subsection{\verified{One-to-One Composition}}%
\hyperlabel{sub:one-to-one-composition}
\begin{lemma}
Let $F$ and $G$ be one-to-one functions.
Then $F \circ G$ is also a one-to-one function.
\end{lemma}
\code{Bookshelf/Enderton/Set/Relation}
{Set.Relation.one\_to\_one\_comp\_is\_one\_to\_one}
\lean{Mathlib/Data/Set/Function}
{Set.InjOn.comp}
\begin{proof}
Let $F \colon B \rightarrow C$ and $G \colon A \rightarrow B$ be
one-to-one \nameref{ref:function}s from sets $A$, $B$, and $C$.
By definition of the \nameref{ref:composition} of functions,
\begin{equation}
\hyperlabel{sub:one-to-one-composition-eq1}
F \circ G = \{\tuple{u, v} \mid \exists t(uGt \land tFv)\}.
\end{equation}
By \nameref{sub:theorem-3h}, $F \circ G$ is a function.
All that remains is proving $F \circ G$ is one-to-one.
Let $(F \circ G)(x_1) = (F \circ G)(x_2) = y$.
By \eqref{sub:one-to-one-composition-eq1}, there exists some $t_1$ such that
$x_1Gt_1$ and $t_1Fy$.
Likewise, there exists some $t_2$ such that $x_2Gt_2$ and $t_2Fy$.
Since $F$ is one-to-one, it follows $t_1 = t_2$.
Then, since $G$ is also one-to-one, it follows $x_1 = x_2$.
Hence $F \circ G$ is one-to-one.
\end{proof}
2023-06-29 20:53:36 +00:00
\subsection{\verified{Theorem 3I}}%
\hyperlabel{sub:theorem-3i}
2023-08-09 13:39:41 +00:00
\begin{theorem}[3I]
For any sets $F$ and $G$, $$(F \circ G)^{-1} = G^{-1} \circ F^{-1}.$$
\end{theorem}
2023-06-22 20:34:50 +00:00
\code{Bookshelf/Enderton/Set/Relation}
2023-06-29 20:53:36 +00:00
{Set.Relation.comp\_inv\_eq\_inv\_comp\_inv}
2023-06-22 20:34:50 +00:00
2023-08-09 13:39:41 +00:00
\begin{proof}
By definition of the \nameref{ref:composition} of $F$ and $G$,
$$F \circ G = \{\tuple{u, v} \mid \exists t(uGt \land tFv)\}.$$
2023-08-09 13:39:41 +00:00
By definition of the \nameref{ref:inverse} of a function,
\begin{align*}
(F \circ G)^{-1}
& = \{\tuple{u, v} \mid \exists t (vGt \land tFu)\} \\
& = \{\tuple{u, v} \mid \exists t (tFu \land vGt)\} \\
& = \{\tuple{u, v} \mid
2023-08-09 13:39:41 +00:00
\exists t \left[ u(F^{-1})t \land t(G^{-1})v \right]\} \\
& = G^{-1} \circ F^{-1}.
\end{align*}
\end{proof}
\subsection{\verified{Theorem 3J (a)}}%
\hyperlabel{sub:theorem-3j-a}
\begin{theorem}[3J(a)]
2023-08-09 13:39:41 +00:00
Assume that $F \colon A \rightarrow B$, and that $A$ is nonempty.
There exists a function $G \colon B \rightarrow A$ (a "left inverse") such
that $G \circ F$ is the identity function $I_A$ on $A$ iff $F$ is
one-to-one.
2023-08-09 13:39:41 +00:00
\end{theorem}
\code{Bookshelf/Enderton/Set/Chapter\_3}
{Enderton.Set.Chapter\_3.theorem\_3j\_a}
2023-08-09 13:39:41 +00:00
\begin{proof}
2023-08-09 13:39:41 +00:00
Let $F$ be a \nameref{ref:function} from nonempty set $A$ to set $B$.
\subparagraph{($\Rightarrow$)}%
2023-08-09 13:39:41 +00:00
Let $G \colon B \rightarrow A$ such that $G \circ F = I_A$.
All that remains is to prove $F$ is single-rooted.
Let $y \in \ran{F}$.
By definition of the \nameref{ref:range} of a function, there exists
some $x_1$ such that $\tuple{x_1, y} \in F$.
Suppose there exists a set $x_2$ such that $\tuple{x_2, y} \in F$.
By hypothesis, $G(F(x_1)) = G(F(x_2))$ implies $I_A(x_1) = I_A(x_2)$.
Thus $x_1 = x_2$.
Therefore $F$ must be single-rooted.
2023-08-09 13:39:41 +00:00
\subparagraph{($\Leftarrow$)}%
2023-08-09 13:39:41 +00:00
Let $F$ be one-to-one.
Since $A$ is nonempty, there exists some $a \in A$.
Let $G \colon B \rightarrow A$ be given by
$$G(y) = \begin{cases}
F^{-1}(y) & \text{if } y \in \ran{F} \\
a & \text{otherwise}.
\end{cases}$$
$G$ is a function by virtue of \nameref{sub:one-to-one-inverse} and
choice of mapping for all values $y \not\in \ran{F}$.
Furthermore, for all $x \in A$, $F(x) \in \ran{F}$.
Thus $(G \circ F)(x) = G(F(x)) = F^{-1}(F(x)) = x$ by
\nameref{sub:theorem-3g}.
2023-08-09 13:39:41 +00:00
\end{proof}
2023-08-09 13:39:41 +00:00
\subsection{\unverified{Theorem 3J (b)}}%
\hyperlabel{sub:theorem-3j-b}
2023-08-09 13:39:41 +00:00
\begin{theorem}[3J(b)]
Assume that $F \colon A \rightarrow B$, and that $A$ is nonempty.
There exists a function $H \colon B \rightarrow A$ (a "right inverse") such
that $F \circ H$ is the identity function $I_B$ on $B$ iff $F$ maps $A$
\textit{onto} $B$.
\end{theorem}
2023-08-09 13:39:41 +00:00
\code{Bookshelf/Enderton/Set/Chapter\_3}
{Enderton.Set.Chapter\_3.theorem\_3j\_b}
2023-08-09 13:39:41 +00:00
\begin{proof}
2023-08-09 13:39:41 +00:00
Let $F$ be a \nameref{ref:function} from nonempty set $A$ to set $B$.
2023-08-09 13:39:41 +00:00
\subparagraph{($\Rightarrow$)}%
Suppose $H \colon B \rightarrow A$ such that $F \circ H = I_A$.
All that remains is to prove $\ran{F} = B$.
Note that $\ran{F} \subseteq B$ by hypothesis.
Let $y \in B$.
But $F(H(y)) = y$ meaning $y \in \ran{F}$.
Thus $B \subseteq \ran{F}$.
Since $\ran{F} \subseteq B$ and $B \subseteq \ran{F}$, $\ran{F} = B$.
2023-08-09 13:39:41 +00:00
\subparagraph{($\Leftarrow$)}%
Suppose $F$ maps $A$ \textit{onto} $B$.
By definition of maps onto, $\ran{F} = B$.
Then for all $y \in B$, there exists some $x \in A$ such that
$\tuple{x, y} \in F$.
Notice though that $F^{-1}[\{y\}]$ may not be a singleton set.
Then there is no obvious way to \textit{choose} an element from each
preimage to form a function.
By the \nameref{ref:axiom-of-choice-1}, there exists a function
$H \subseteq F^{-1}$ such that $\dom{H} = \dom{F^{-1}} = B$.
For all $y \in B$, $\tuple{y, H(y)} \in H \subseteq F^{-1}$
meaning $\tuple{H(y), y} \in F$.
Thus $F(H(y)) = y$ as expected.
2023-08-09 13:39:41 +00:00
\end{proof}
\subsection{\unverified{Bijections and Inverses}}%
\hyperlabel{sub:bijections-inverses}
\begin{corollary}
A function $f$ is a one-to-one correspondence if and only if it has a left
and right inverse.
\end{corollary}
\begin{proof}
By definition, a one-to-one correspondence $f$ between sets $A$ and $B$ must
be both one-to-one and onto.
By \nameref{sub:theorem-3j-a}, $f$ is one-to-one if and only if it has a
left inverse.
By \nameref{sub:theorem-3j-b}, $f$ is onto $B$ if and only if it has a right
inverse.
\end{proof}
\subsection{\unverified{Left and Right Inverses and Two-Sided Inverses}}%
\hyperlabel{sub:left-right-inverse-two-sided-inverse}
\begin{lemma}
Let $f$ be a function with left inverse $g_1$ and right inverse $g_2$.
Then $g_1 = g_2 = f^{-1}$.
\end{lemma}
\begin{proof}
Let $I$ denote the identity map with appropriate domain and codomain
depending on placement in the following:
\begin{align*}
g_1
& = g_1 \circ I \\
& = g_1 \circ (f \circ g_2) \\
& = (g_1 \circ f) \circ g_2 \\
& = I \circ g_2 \\
& = g_2.
\end{align*}
By \nameref{sub:bijections-inverses}, $f$ is a bijection meaning $f^{-1}$
is both a left and right inverse.
Hence $g_1 = g_2 = f^{-1}$.
\end{proof}
\subsection{\verified{Theorem 3K(a)}}%
\hyperlabel{sub:theorem-3k-a}
2023-08-09 13:39:41 +00:00
\begin{theorem}[3K(a)]
The following hold for any sets. ($F$ need not be a function.)
The image of a union is the union of the images:
\begin{equation}
\hyperlabel{sub:theorem-3k-a-eq1}
\img{F}{A \cup B} = \img{F}{A} \cup \img{F}{B}
\end{equation}
and
\begin{equation}
\hyperlabel{sub:theorem-3k-a-eq2}
\img{F}{\bigcup{\mathscr{A}}} =
\bigcup\;\{\img{F}{A} \mid A \in \mathscr{A}\}.
\end{equation}
\end{theorem}
\code{Bookshelf/Enderton/Set/Chapter\_3}
{Enderton.Set.Chapter\_3.theorem\_3k\_a}
2023-08-09 13:39:41 +00:00
\begin{proof}
Let $F$, $A$, $B$, and $\mathscr{A}$ be arbitrary sets.
We prove (i) \eqref{sub:theorem-3k-a-eq1} and (ii)
\eqref{sub:theorem-3k-a-eq2}.
2023-06-23 20:47:25 +00:00
2023-08-09 13:39:41 +00:00
\paragraph{(i)}%
2023-06-23 20:47:25 +00:00
2023-08-09 13:39:41 +00:00
By definition of the \nameref{ref:image} of a set:
\begin{align*}
\img{F}{A \cup B}
& = \{v \mid \exists u, u \in A \cup B \land uFv\} \\
& = \{v \mid \exists u,
(u \in A \land uFv) \lor (u \in B \land uFv)\} \\
& = \{v \mid (\exists u \in A) uFv\} \cup
\{v \mid (\exists u \in B) uFv\} \\
& = \img{F}{A} \cup \img{F}{B}.
\end{align*}
2023-06-23 20:47:25 +00:00
2023-08-09 13:39:41 +00:00
\paragraph{(ii)}%
2023-06-23 20:47:25 +00:00
2023-08-09 13:39:41 +00:00
We prove that both sides of \eqref{sub:theorem-3k-a-eq2} is a subset of
the other.
2023-06-23 20:47:25 +00:00
2023-08-09 13:39:41 +00:00
\subparagraph{($\subseteq$)}%
2023-06-23 20:47:25 +00:00
2023-08-09 13:39:41 +00:00
Let $v \in \img{F}{\bigcup{\mathscr{A}}}$.
By definition of the \nameref{ref:image} of a set, there exists a set
$u$ such that $u \in \bigcup{\mathscr{A}} \land uFv$.
Then, by definition of the union of sets, there exists some
$A \in \mathscr{A}$ such that $u \in A$.
Therefore $v \in \img{F}{A}$ meaning
$v \in \bigcup\{\img{F}{A} \mid A \in \mathscr{A}\}$.
2023-06-29 20:53:36 +00:00
2023-08-09 13:39:41 +00:00
\subparagraph{($\supseteq$)}%
2023-06-29 20:53:36 +00:00
2023-08-09 13:39:41 +00:00
Let $v \in \bigcup\{\img{F}{A} \mid A \in \mathscr{A}\}$.
Then there exists some $b \in \{\img{F}{A} \mid A \in \mathscr{A}\}$
such that $v \in b$.
In other words, there exists some $A \in \mathscr{A}$ such that
$v \in b = \img{F}{A}$.
By definition of the \nameref{ref:image} of a set, there exists a set
$u$ such that $u \in A \land uFv$.
But this implies that $u \in \bigcup{\mathscr{A}} \land uFv$.
Therefore $v \in \img{F}{\bigcup{\mathscr{A}}}$.
2023-08-09 13:39:41 +00:00
\end{proof}
\subsection{\verified{Theorem 3K(b)}}%
\hyperlabel{sub:theorem-3k-b}
2023-08-09 13:39:41 +00:00
\begin{theorem}[3K(b)]
The following hold for any sets. ($F$ need not be a function.)
The image of an intersection is included in the intersection of the images:
\begin{equation}
\hyperlabel{sub:theorem-3k-b-eq1}
\img{F}{A \cap B} \subseteq \img{F}{A} \cap \img{F}{B}
\end{equation}
and
\begin{equation}
\hyperlabel{sub:theorem-3k-b-eq2}
\img{F}{\bigcap\mathscr{A}} \subseteq
\bigcap\;\{\img{F}{A} \mid A \in \mathscr{A}\}.
\end{equation}
for nonempty $\mathscr{A}$.
Equality holds if $F$ is single-rooted.
\end{theorem}
2023-08-09 13:39:41 +00:00
\code{Bookshelf/Enderton/Set/Chapter\_3}
{Enderton.Set.Chapter\_3.theorem\_3k\_b\_i}
\code{Bookshelf/Enderton/Set/Chapter\_3}
{Enderton.Set.Chapter\_3.theorem\_3k\_b\_ii}
2023-08-09 13:39:41 +00:00
\begin{proof}
Let $F$, $A$, $B$ be arbitrary sets.
Let $\mathscr{A}$ be a nonempty set.
We first prove (i) \eqref{sub:theorem-3k-b-eq1} and (ii)
\eqref{sub:theorem-3k-b-eq2}.
Then, assuming $F$ is single-rooted, we prove both (iii)
\eqref{sub:theorem-3k-b-eq1} and (iv) \eqref{sub:theorem-3k-b-eq2} hold
under equality.
\paragraph{(i)}%
\hyperlabel{par:theorem-3k-b-i}
Let $v \in \img{F}{A \cap B}$.
By definition of the \nameref{ref:image} of a set,
$\exists u \in A \cap B, uFv$.
Then $u \in A \land uFv$ and $u \in B \land uFv$.
Therefore $v \in \img{F}{A} \cap \img{F}{B}$.
\paragraph{(ii)}%
\hyperlabel{par:theorem-3k-b-ii}
Let $v \in \img{F}{\bigcap{\mathscr{A}}}$.
By definition of the \nameref{ref:image} of a set,
$\exists u \in \bigcap{\mathscr{A}}, uFv$.
Then $\exists u, (\forall A \in \mathscr{A}, u \in A) \land uFv$.
This implies that $\forall A \in \mathscr{A}, \exists u \in A, uFv$.
Then $\forall A \in \mathscr{A}, v \in \img{F}{A}$.
Thus $v \in \bigcap\{\img{F}{A} \mid A \in \mathscr{A}\}$.
\paragraph{(iii)}%
2023-08-09 13:39:41 +00:00
Suppose $F$ is single-rooted.
By \nameref{par:theorem-3k-b-i},
$$\img{F}{A \cap B} \subseteq \img{F}{A} \cap \img{F}{B}.$$
All that remains is showing
$$\img{F}{A} \cap \img{F}{B} \subseteq \img{F}{A \cap B}.$$
Let $v \in \img{F}{A} \cap \img{F}{B}$.
Then $v \in \img{F}{A}$ and $v \in \img{F}{B}$.
That is, $\exists u \in A, uFv$ and $\exists w \in B, wFv$.
Since $F$ is single rooted, it follows $u = w$.
Thus $u \in A \cap B \land uFv$ meaning $v \in \img{F}{A \cap B}$.
\paragraph{(iv)}%
2023-06-24 15:52:45 +00:00
2023-08-09 13:39:41 +00:00
Suppose $F$ is single-rooted.
By \nameref{par:theorem-3k-b-ii},
$$\img{F}{\bigcap\mathscr{A}} \subseteq
\bigcap\;\{\img{F}{A} \mid A \in \mathscr{A}\}.$$
All that remains is showing
$$\bigcap\;\{\img{F}{A} \mid A \in \mathscr{A}\} \subseteq
\img{F}{\bigcap\mathscr{A}}.$$
Let $v \in \bigcap\;\{\img{F}{A} \mid A \in \mathscr{A}\}$.
Then $\forall A \in \mathscr{A}, v \in \img{F}{A}$.
By definition of the \nameref{ref:image} of a set,
$\forall A \in \mathscr{A}, \exists u \in A, uFv$.
Since $F$ is single-rooted and $\mathscr{A}$ is nonempty, it follows that
$\exists u, (\forall A \in \mathscr{A}, u \in A) \land uFv$.
Equivalently, $\exists u \in \bigcap{A}, uFv$.
Thus $v \in \img{F}{\bigcap{A}}$.
\end{proof}
2023-08-09 13:39:41 +00:00
\subsection{\verified{Theorem 3K(c)}}%
\hyperlabel{sub:theorem-3k-c}
2023-06-29 20:53:36 +00:00
2023-08-09 13:39:41 +00:00
\begin{theorem}[3K(c)]
The following hold for any sets. ($F$ need not be a function.)
The image of a difference includes the difference of the images:
\begin{equation}
\hyperlabel{sub:theorem-3k-c-eq1}
\img{F}{A} - \img{F}{B} \subseteq \img{F}{A - B}.
\end{equation}
Equality holds if $F$ is single-rooted.
\end{theorem}
2023-08-09 13:39:41 +00:00
\code{Bookshelf/Enderton/Set/Chapter\_3}
{Enderton.Set.Chapter\_3.theorem\_3k\_c\_i}
\code{Bookshelf/Enderton/Set/Chapter\_3}
{Enderton.Set.Chapter\_3.theorem\_3k\_c\_ii}
2023-08-09 13:39:41 +00:00
\begin{proof}
We prove that (i) \eqref{sub:theorem-3k-c-eq1} holds and (ii) equality holds
if $F$ is single-rooted.
2023-06-24 15:52:45 +00:00
2023-08-09 13:39:41 +00:00
\paragraph{(i)}%
\hyperlabel{par:theorem-3k-c-i}
2023-08-09 13:39:41 +00:00
Let $v \in \img{F}{A} - \img{F}{B}$.
By definition of the difference of two sets,
$v \in \img{F}{A}$ and $v \not\in \img{F}{B}$.
By definition of the \nameref{ref:image} of a set, there exists a set
$u \in A$ such that $\tuple{u, v} \in F$.
Likewise, $\forall w \in B, \tuple{w, v} \not\in F$.
2023-08-09 13:39:41 +00:00
Thus $u \not\in B$, since otherwise we get an immediate contradiction.
Therefore $u \in A - B$ meaning $v \in \img{F}{A - B}$.
2023-08-09 13:39:41 +00:00
\paragraph{(ii)}%
2023-08-09 13:39:41 +00:00
Suppose $F$ is single-rooted.
By \nameref{par:theorem-3k-c-i},
$$\img{F}{A} - \img{F}{B} \subseteq \img{F}{A - B}.$$
All that remains is showing
$$\img{F}{A - B} \subseteq \img{F}{A} - \img{F}{B}.$$
Let $v \in \img{F}{A - B}$.
By definition of the \nameref{ref:image} of a set, there exists a set
$u \in A - B$ such that $uFv$.
Then $u \in A$ and $u \not\in B$.
The former membership relation implies $v \in \img{F}{A}$.
The latter implies $v \not\in \img{F}{B}$ since $F$ being single-rooted
would otherwise invoke an immediate contradiction.
Thus $v \in \img{F}{A} - \img{F}{B}$.
\end{proof}
2023-08-09 13:39:41 +00:00
\subsection{\verified{Corollary 3L}}%
\hyperlabel{sub:corollary-3l}
2023-08-09 13:39:41 +00:00
\begin{theorem}[3L]
For any function $G$ and sets $A$, $B$, and $\mathscr{A}$:
\begin{align}
\img{G^{-1}}{\bigcup{\mathscr{A}}}
& = \bigcup\;\{\img{G^{-1}}{A} \mid A \in \mathscr{A}\},
\hyperlabel{sub:corollary-3l-eq1} \\
\img{G^{-1}}{\bigcap{\mathscr{A}}}
& = \bigcap\;\{\img{G^{-1}}{A} \mid A \in \mathscr{A}\}
\text{ for } \mathscr{A} \neq \emptyset,
\hyperlabel{sub:corollary-3l-eq2} \\
\img{G^{-1}}{A - B} & = \img{G^{-1}}{A} - \img{G^{-1}}{B}.
\hyperlabel{sub:corollary-3l-eq3}
\end{align}
\end{theorem}
2023-08-09 13:39:41 +00:00
\code{Bookshelf/Enderton/Set/Chapter\_3}
{Enderton.Set.Chapter\_3.corollary\_3l\_i}
2023-08-09 13:39:41 +00:00
\code{Bookshelf/Enderton/Set/Chapter\_3}
{Enderton.Set.Chapter\_3.corollary\_3l\_ii}
\code{Bookshelf/Enderton/Set/Chapter\_3}
{Enderton.Set.Chapter\_3.corollary\_3l\_iii}
2023-08-09 13:39:41 +00:00
\begin{proof}
\nameref{sub:theorem-3k-a} implies \eqref{sub:corollary-3l-eq1}.
Because the inverse of a function is always single-rooted,
\nameref{sub:theorem-3k-b} implies \eqref{sub:corollary-3l-eq2}.
Likewise \nameref{sub:theorem-3k-c} implies \eqref{sub:corollary-3l-eq3}.
\end{proof}
2023-06-26 13:08:25 +00:00
\section{Equivalence Relations}%
\hyperlabel{sec:equivalence-relations}
\subsection{\verified{Theorem 3M}}%
\hyperlabel{sub:theorem-3m}
2023-08-09 13:39:41 +00:00
\begin{theorem}[3M]
If $R$ is a symmetric and transitive relation, then $R$ is an equivalence
relation on $\fld{R}$.
\end{theorem}
\code{Bookshelf/Enderton/Set/Chapter\_3}
{Enderton.Set.Chapter\_3.theorem\_3m}
2023-08-09 13:39:41 +00:00
\begin{proof}
Suppose $R$ is a \nameref{ref:symmetric} and \nameref{ref:transitive}
\nameref{ref:relation}.
By definition, the \nameref{ref:field} of $R$ is given by
$\fld{R} = \dom{R} \cup \ran{R}$.
An \nameref{ref:equivalence-relation} on $\fld{R}$ is, by definition, a
binary relation \nameref{ref:reflexive} on $\fld{R}$, symmetric, and
transitive.
All that remains is to show $R$ is reflexive on $\fld{R}$.
Let $x \in \fld{R}$.
Then $x \in \dom{R}$ or $x \in \ran{R}$.
If $x \in \dom{R}$, there exists some $y$ such that $xRy$.
Since $R$ is symmetric, it follows $yRx$.
Since $R$ is transitive, it follows $xRx$.
If instead $x \in \ran{R}$, there exists some $t$ such that $tRx$.
Since $R$ is symmetric, it follows $xRt$.
Since $R$ is transitive, it follows $xRx$.
Thus $R$ is reflexive on $\fld{R}$.
\end{proof}
\subsection{\verified{Lemma 3N}}%
\hyperlabel{sub:lemma-3n}
2023-08-09 13:39:41 +00:00
\begin{lemma}[3N]
Assume that $R$ is an equivalence relation on $A$ and that $x$ and $y$
belong to $A$.
Then $$[x]_R = [y]_R \iff xRy.$$
\end{lemma}
\code{Bookshelf/Enderton/Set/Relation}
{Set.Relation.neighborhood\_iff\_mem}
2023-08-09 13:39:41 +00:00
\begin{proof}
Suppose $R$ is an \nameref{ref:equivalence-relation} on set $A$.
Let $x, y \in A$.
2023-08-09 13:39:41 +00:00
\paragraph{($\Rightarrow$)}%
2023-08-09 13:39:41 +00:00
Suppose $[x]_R = [y]_R$.
Since $R$ is an equivalence relation, it is reflexive on $A$.
Thus $yRy$ meaning $y \in [y]_R = \{t \mid yRt\}$.
Since $[x]_R = [y]_R$, $y \in \{t \mid xRt\}$ as well.
That is, $xRy$.
2023-08-09 13:39:41 +00:00
\paragraph{($\Leftarrow$)}%
2023-08-09 13:39:41 +00:00
Suppose $xRy$.
We show $[x]_R \subseteq [y]_R$ and $[y]_R \subseteq [x]_R$.
2023-08-09 13:39:41 +00:00
\subparagraph{($\subseteq$)}%
2023-08-09 13:39:41 +00:00
Let $t \in [x]_R$.
Then $xRt$.
Since $R$ is symmetric, $xRy$ implies $yRx$.
Since $R$ is transitive, $yRx$ and $xRt$ implies $yRt$.
Thus $t \in [y]_R$.
2023-08-09 13:39:41 +00:00
\subparagraph{($\supseteq$)}%
2023-08-09 13:39:41 +00:00
Let $t \in [y]_R$.
Then $yRt$.
Since $R$ is transitive, $xRy$ and $yRt$ implies $xRt$.
Thus $t \in [x]_R$.
2023-08-09 13:39:41 +00:00
\end{proof}
\subsection{\verified{Theorem 3P}}%
\hyperlabel{sub:theorem-3p}
2023-08-09 13:39:41 +00:00
\begin{theorem}[3P]
Assume that $R$ is an equivalence relation on $A$.
Then the set $\{[x]_R \mid x \in A\}$ of all equivalence classes is a
partition of $A$.
\end{theorem}
\code{Bookshelf/Enderton/Set/Relation}
{Set.Relation.modEquiv\_partition}
2023-08-09 13:39:41 +00:00
\begin{proof}
Let $\Pi = \{[x]_R \mid x \in A\}$.
We show that (i) there are no empty sets in $\Pi$, (ii) no two different
sets in $\Pi$ have any common elements and (iii) that each element of $A$
is in some set in $\Pi$.
2023-08-09 13:39:41 +00:00
\paragraph{(i)}%
2023-08-09 13:39:41 +00:00
By construction, every element of $\Pi$ is of form $[x]_R$ for some
$x \in A$.
At the very least, $x \in A$ is also in $[x]_R$.
Thus every element of $\Pi$ must be nonempty.
2023-08-09 13:39:41 +00:00
\paragraph{(ii)}%
2023-08-09 13:39:41 +00:00
Let $[x]_R, [y]_R \in \Pi$ be two different sets.
We must show that $[x]_R \cap [y]_R = \emptyset$.
For the sake of contradiction, suppose $[x]_R \cap [y]_R \neq \emptyset$.
Let $z \in [x]_R \cap [y]_R$.
Then $xRz$ and $yRz$.
Since $R$ is an \nameref{ref:equivalence-relation} on $A$, it is
\nameref{ref:symmetric} and \nameref{ref:transitive}.
Then $zRy$ and $xRy$.
By \nameref{sub:lemma-3n}, $xRy$ if and only if $[x]_R = [y]_R$,
contradicting the distinctness of $[x]_R$ and $[y]_R$.
Thus it follows $[x]_R \cap [y]_R] = \emptyset$.
2023-08-09 13:39:41 +00:00
\paragraph{(iii)}%
2023-08-09 13:39:41 +00:00
Let $x \in A$.
Since $R$ is an \nameref{ref:equivalence-relation} on $A$, it follows
$xRx$.
Thus $x$ is a member of some set in $\Pi$, namely $[x]_R$.
2023-08-09 13:39:41 +00:00
\end{proof}
\subsection{\unverified{Theorem 3Q}}%
\hyperlabel{sub:theorem-3q}
2023-08-09 13:39:41 +00:00
\begin{theorem}[3Q]
Assume that $R$ is an equivalence relation on $A$ and that
$F \colon A \rightarrow A$.
If $F$ is compatible with $R$, then there exists a unique
$\hat{F} \colon A / R \rightarrow A / R$ such that
\begin{equation}
\hyperlabel{sub:theorem-3q-eq1}
\hat{F}([x]_R) = [F(x)]_R \quad\text{for all } x \text{ in } A.
\end{equation}
If $F$ is not compatible with $R$, then no such $\hat{F}$ exists.
\end{theorem}
\begin{proof}
Let $R$ be an \nameref{ref:equivalence-relation} on $A$ and
$F \colon A \rightarrow A$.
Suppose $F$ is \nameref{ref:compatible} with $R$.
Next define \nameref{ref:relation} $\hat{F}$ to be
$$\hat{F} = \{\tuple{[x]_R, [F(x)]_R} \mid x \in A\}.$$
2023-08-09 13:39:41 +00:00
By construction $\hat{F}$ has domain $A / R$ and
$\ran{\hat{F}} \subseteq A / R$.
All that remains is proving (i) $\hat{F}$ is single-valued and (ii)
$\hat{F}$ is unique.
\paragraph{(i)}%
Let $[x_1]_R, [x_2]_R \in \dom{\hat{F}}$ such that $[x_1]_R = [x_2]_R$.
By definition of $\hat{F}$, $\tuple{[x_1]_R, [F(x_1)]_R} \in \hat{F}$
and $\tuple{[x_2]_R, [F(x_2)]_R} \in \hat{F}$.
By \nameref{sub:lemma-3n}, $[x_1]_R = [x_2]_R$ implies $x_1Rx_2$.
Since $F$ is compatible, $F(x_1)RF(x_2)$.
Another application of \nameref{sub:lemma-3n} implies that
$[F(x_1)]_R = [F(x_2)]_R$.
Thus $\hat{F}$ is single-valued, i.e. a function.
\paragraph{(ii)}%
Suppose there exists another function, say $\hat{G}$, that satisfies
\eqref{sub:theorem-3q-eq1}.
That is,
$$\hat{G}([x]_R) = [F(x)]_R \quad\text{for all } x \text{ in } A.$$
Let $x \in A$.
Then $\hat{G}([x]_R) = [F(x)]_R$ and $\hat{F}([x]_R) = [F(x)]_R$.
Since this holds for all $x \in A$, $\hat{F}$ and $\hat{G}$ agree on all
equivalence classes in $A / R$.
Hence, by the \nameref{ref:extensionality-axiom}, $\hat{F} = \hat{G}$.
2023-08-09 13:39:41 +00:00
\suitdivider
\noindent
2023-08-09 13:39:41 +00:00
Suppose $F$ is not compatible with $R$.
Then there exists some $x, y \in A$ such that $xRy$ and $\neg F(x)RF(y)$.
By \nameref{sub:lemma-3n}, $[x]_R = [y]_R$.
For the sake of contradiction, suppose a function $\hat{F}$ exists
satisfying \eqref{sub:theorem-3q-eq1}.
Then $\hat{F}([x]_R) = \hat{F}([y]_R)$ meaning $[F(x)]_R = [F(y)]_R$.
Then \nameref{sub:lemma-3n} implies $F(x)RF(y)$, a contradiction.
Therefore our original hypothesis must be incorrect.
That is, there is no incompatible function $\hat{F}$ satisfying
2023-08-09 13:39:41 +00:00
\eqref{sub:theorem-3q-eq1}.
\end{proof}
\section{Ordering Relations}%
\hyperlabel{sec:ordering-relations}
\subsection{\verified{Theorem 3R}}%
\hyperlabel{sub:theorem-3r}
2023-08-09 13:39:41 +00:00
\begin{theorem}[3R]
Let $R$ be a linear ordering on $A$.
\begin{enumerate}[(i)]
\item There is no $x$ for which $xRx$.
\item For distinct $x$ and $y$ in $A$, either $xRy$ or $yRx$ (but not both).
\end{enumerate}
\end{theorem}
\code{Bookshelf/Enderton/Set/Chapter\_3}
{Enderton.Set.Chapter\_3.theorem\_3r}
2023-08-09 13:39:41 +00:00
\begin{proof}
Suppose $R$ is a \nameref{ref:linear-ordering} on $A$.
2023-08-09 13:39:41 +00:00
\paragraph{(i)}%
2023-08-09 13:39:41 +00:00
Let $x \in A$.
By definition, $R$ is \nameref{ref:trichotomous}.
Then only one of $xRx$ and $x = x$ can hold.
Since $x = x$ obviously holds, it follows $\tuple{x, x} \not\in R$.
2023-08-09 13:39:41 +00:00
\paragraph{(ii)}%
2023-08-09 13:39:41 +00:00
Let $x, y \in A$ such that $x \neq y$.
By definition, $R$ is \nameref{ref:trichotomous}.
Thus only one of $$xRy, \quad x = y, \quad yRx$$ hold.
By hypothesis $x \neq y$ meaning either $xRy$ or $yRx$ (but not both).
2023-08-09 13:39:41 +00:00
\end{proof}
2023-06-29 20:53:36 +00:00
\section{Exercises 3}%
\hyperlabel{sec:exercises-3}
2023-06-26 13:08:25 +00:00
2023-06-29 20:53:36 +00:00
\subsection{\verified{Exercise 3.1}}%
\hyperlabel{sub:exercise-3.1}
2023-06-24 15:52:45 +00:00
2023-08-09 13:39:41 +00:00
Suppose that we attempted to generalize the Kuratowski definitions of ordered
pairs to ordered triples by defining
$$\tuple{x, y, z}^* = \{\{x\}, \{x, y\}, \{x, y, z\}\}.$$
2023-08-09 13:39:41 +00:00
Show that this definition is unsuccessful by giving examples of objects
$u$, $v$, $w$, $x$, $y$, $z$ with
$\tuple{x, y, z}^* = \tuple{u, v, w}^*$ but with either
2023-08-09 13:39:41 +00:00
$y \neq v$ or $z \neq w$ (or both).
2023-08-09 13:39:41 +00:00
\code*{Bookshelf/Enderton/Set/Chapter\_3}
2023-06-29 20:53:36 +00:00
{Enderton.Set.Chapter\_3.exercise\_3\_1}
2023-08-09 13:39:41 +00:00
\begin{proof}
Let $x = 1$, $y = 1$, and $z = 2$.
Let $u = 1$, $v = 2$, and $w = 2$.
Then
\begin{align*}
\tuple{x, y, z}^*
2023-08-09 13:39:41 +00:00
& = \{\{x\}, \{x, y\}, \{x, y, z\}\} \\
& = \{\{1\}, \{1, 1\}, \{1, 1, 2\}\} \\
& = \{\{1\}, \{1, 2\}\}.
\end{align*}
Likewise
\begin{align*}
\tuple{u, v, w}^*
2023-08-09 13:39:41 +00:00
& = \{\{u\}, \{u, v\}, \{u, v, w\}\} \\
& = \{\{1\}, \{1, 2\}, \{1, 2, 2\}\} \\
& = \{\{1\}, \{1, 2\}\}.
\end{align*}
Thus $\tuple{x, y, z}^* = \tuple{u, v, w}^*$ but $y \neq v$.
2023-08-09 13:39:41 +00:00
\end{proof}
2023-06-29 20:53:36 +00:00
\subsection{\verified{Exercise 3.2a}}%
\hyperlabel{sub:exercise-3.2a}
2023-08-09 13:39:41 +00:00
Show that $A \times (B \cup C) = (A \times B) \cup (A \times C)$.
2023-08-09 13:39:41 +00:00
\code*{Bookshelf/Enderton/Set/Chapter\_3}
2023-06-29 20:53:36 +00:00
{Enderton.Set.Chapter\_3.exercise\_3\_2a}
2023-08-09 13:39:41 +00:00
\begin{proof}
Let $A$, $B$, and $C$ be arbitrary sets.
Then by \nameref{sub:corollary-3c} and the definition of the union of sets,
\begin{align*}
A \times (B \cup C)
& = \{ \tuple{x, y} \mid x \in A \land y \in (B \cup C) \} \\
& = \{ \tuple{x, y} \mid
2023-08-09 13:39:41 +00:00
x \in A \land (y \in B \lor y \in C) \} \\
& = \{ \tuple{x, y} \mid
2023-08-09 13:39:41 +00:00
(x \in A \land y \in B) \lor (x \in A \land y \in C) \} \\
& = \{ \tuple{x, y} \mid (x \in A \land y \in B) \} \cup
\{ \tuple{x, y} \mid (x \in A \land y \in C) \} \\
2023-08-09 13:39:41 +00:00
& = (A \times B) \cup (A \times C).
\end{align*}
\end{proof}
2023-06-29 20:53:36 +00:00
\subsection{\verified{Exercise 3.2b}}%
\hyperlabel{sub:exercise-3.2b}
2023-08-09 13:39:41 +00:00
Show that if $A \times B = A \times C$ and $A \neq \emptyset$, then $B = C$.
2023-08-09 13:39:41 +00:00
\code*{Bookshelf/Enderton/Set/Chapter\_3}
2023-06-29 20:53:36 +00:00
{Enderton.Set.Chapter\_3.exercise\_3\_2b}
2023-08-09 13:39:41 +00:00
\begin{proof}
Let $A$, $B$, and $C$ be arbitrary sets such that $A \neq \emptyset$.
By \nameref{sub:corollary-3c},
\begin{align}
A \times B & = \{ \tuple{x, y} \mid x \in A \land y \in B \}
2023-08-09 13:39:41 +00:00
& \hyperlabel{sub:exercise-3.2b-eq1} \\
A \times C & = \{ \tuple{x, y} \mid x \in A \land y \in C \}.
2023-08-09 13:39:41 +00:00
& \hyperlabel{sub:exercise-3.2b-eq2}
\end{align}
There are two cases to consider:
2023-08-09 13:39:41 +00:00
\paragraph{Case 1}%
2023-08-09 13:39:41 +00:00
Suppose $B \neq \emptyset$.
Then $A \times B \neq \emptyset$ and $A \times C \neq \emptyset$.
Let $\tuple{x, y} \in A \times B$.
2023-08-09 13:39:41 +00:00
By \eqref{sub:exercise-3.2b-eq1}, $x \in A$ and $y \in B$.
By the \nameref{ref:extensionality-axiom},
$$\tuple{x, y} \in A \times B \iff \tuple{x, y} \in A \times C.$$
Therefore $\tuple{x, y} \in A \times C$.
2023-08-09 13:39:41 +00:00
By \eqref{sub:exercise-3.2b-eq2}, $x \in A$ and $y \in C$.
Since membership of $y$ in $B$ and in $C$ holds biconditionally, the
\nameref{ref:extensionality-axiom} indicates $B = C$.
2023-08-09 13:39:41 +00:00
\paragraph{Case 2}%
2023-08-09 13:39:41 +00:00
Suppose $B = \emptyset$.
Then there is no $\tuple{x, y}$ such that $x \in A$ and $y \in B$.
2023-08-09 13:39:41 +00:00
Thus $A \times B = \emptyset$ and $A \times C = \emptyset$.
But then there cannot exist an $\tuple{x, y}$ such that $x \in A$
2023-08-09 13:39:41 +00:00
and $y \in C$ either.
Since $A \neq \emptyset$, it must be the case that $C = \emptyset$.
Thus $B = C$.
2023-06-29 20:53:36 +00:00
2023-08-09 13:39:41 +00:00
\end{proof}
2023-06-29 20:53:36 +00:00
\subsection{\verified{Exercise 3.3}}%
\hyperlabel{sub:exercise-3.3}
2023-08-09 13:39:41 +00:00
Show that $A \times \bigcup \mathscr{B} =
\bigcup\;\{ A \times X \mid X \in \mathscr{B} \}$.
2023-08-09 13:39:41 +00:00
\code*{Bookshelf/Enderton/Set/Chapter\_3}
2023-06-29 20:53:36 +00:00
{Enderton.Set.Chapter\_3.exercise\_3\_3}
2023-06-26 21:11:19 +00:00
2023-08-09 13:39:41 +00:00
\begin{proof}
Let $A$ and $\mathscr{B}$ be arbitrary sets.
By \nameref{sub:corollary-3c} and the definition of the union of sets,
\begin{align*}
A \times \bigcup\mathscr{B}
& = \{ \tuple{x, y} \mid
2023-08-09 13:39:41 +00:00
x \in A \land y \in \bigcup\mathscr{B} \} \\
& = \{ \tuple{x, y} \mid
2023-08-09 13:39:41 +00:00
x \in A \land (\exists b \in \mathscr{B}), y \in b \} \\
& = \{ \tuple{x, y} \mid
2023-08-09 13:39:41 +00:00
(\exists b \in \mathscr{B}), x \in A \land y \in b \} \\
& = \bigcup\; \{ A \times X \mid X \in \mathscr{B} \}.
\end{align*}
\end{proof}
\subsection{\unverified{Exercise 3.4}}%
\hyperlabel{sub:exercise-3.4}
2023-08-09 13:39:41 +00:00
Show that there is no set to which every ordered pair belongs.
2023-08-09 13:39:41 +00:00
\begin{proof}
For the sake of contradiction, suppose there exists a set $A$ to which every
ordered pair belongs.
That is, for all sets $x$ and $y$, $\tuple{x, y} = \{\{x\}, \{x, y\}\}$
2023-08-09 13:39:41 +00:00
is a member of $A$.
By the \nameref{ref:union-axiom}, it follows that $\bigcup\bigcup A$ is the
set to which every set belongs.
But \nameref{sub:theorem-2a} shows this is impossible.
Thus our original assumption was wrong; there exists no set to which every
ordered pair belongs.
\end{proof}
2023-06-29 20:53:36 +00:00
\subsection{\verified{Exercise 3.5a}}%
\hyperlabel{sub:exercise-3.5a}
2023-08-09 13:39:41 +00:00
Assume that $A$ and $B$ are given sets, and show that there exists a set $C$
such that for any $y$,
\begin{equation}
\hyperlabel{sub:exercise-3.5a-eq1}
y \in C \iff y = \{x\} \times B \text{ for some } x \text{ in } A.
\end{equation}
In other words, show that $\{\{x\} \times B \mid x \in A\}$ is a set.
2023-08-09 13:39:41 +00:00
\code*{Bookshelf/Enderton/Set/Chapter\_3}
2023-06-29 20:53:36 +00:00
{Enderton.Set.Chapter\_3.exercise\_3\_5a}
2023-08-09 13:39:41 +00:00
\begin{proof}
Let $a \in A$.
By the \nameref{ref:pairing-axiom}, $\{a\}$ is a set.
By \nameref{sub:corollary-3c}, $\{a\} \times B$ is a set.
Again by the \nameref{ref:pairing-axiom}, $\{\{a\} \times B\}$ is a set.
Next, by another application of \nameref{sub:corollary-3c}, $A \times B$
is a set.
By the \nameref{ref:power-set-axiom}, $\powerset{(A \times B)}$ is a set.
Thus, by the \nameref{ref:subset-axioms}, the following is also a set:
$$C = \{ y \in \powerset{(A \times B)} \mid
\exists a \in A, \forall x, \left[ x \in y \iff
\exists b \in B, x = \tuple{a, b} \right] \}.$$
2023-08-09 13:39:41 +00:00
We now show that $C$ satisfies \eqref{sub:exercise-3.5a-eq1}.
\paragraph{($\Rightarrow$)}%
Suppose $y \in C$.
Then there exists some $a \in A$ such that
$$\forall x, \left[ x \in y \iff
\exists b \in B, x = \tuple{a, b} \right].$$
2023-08-09 13:39:41 +00:00
By the \nameref{ref:extensionality-axiom},
\begin{align*}
y
& = \{ \tuple{a, b} \mid b \in B \} \\
& = \{ \tuple{x, b} \mid x \in \{a\} \land b \in B \} \\
2023-08-09 13:39:41 +00:00
& = \{ \{a\} \times B \}.
\end{align*}
2023-08-09 13:39:41 +00:00
\paragraph{($\Leftarrow$)}%
2023-08-09 13:39:41 +00:00
Suppose $y = \{a\} \times B$ for some $a \in A$.
By \nameref{sub:corollary-3c}, $x \in \{a\} \times B$ if and only if
$\exists b \in B$ such that $x = \tuple{a, b}$.
2023-08-09 13:39:41 +00:00
But then $x \in y$ if and only if $\exists b \in B$ such that
$x = \tuple{a, b}$.
2023-08-09 13:39:41 +00:00
This immediately proves $y \in C$.
2023-08-09 13:39:41 +00:00
\end{proof}
2023-06-29 20:53:36 +00:00
\subsection{\verified{Exercise 3.5b}}%
\hyperlabel{sub:exercise-3.5b}
2023-08-09 13:39:41 +00:00
With $A$, $B$, and $C$ as above, show that $A \times B = \bigcup C$.
2023-06-29 20:53:36 +00:00
2023-08-09 13:39:41 +00:00
\code*{Bookshelf/Enderton/Set/Chapter\_3}
2023-06-29 20:53:36 +00:00
{Enderton.Set.Chapter\_3.exercise\_3\_5b}
2023-08-09 13:39:41 +00:00
\begin{proof}
Let $A$ and $B$ be arbitrary sets.
We want to show that
\begin{equation}
\hyperlabel{sub:exercise-3.5b-eq1}
A \times B = \bigcup\; \{\{x\} \times B \mid x \in A\}.
\end{equation}
The left-hand side of \eqref{sub:exercise-3.5b-eq1} is a set by virtue of
\nameref{sub:corollary-3c}.
The right-hand side of \eqref{sub:exercise-3.5b-eq1} is a set by virtue of
\nameref{sub:exercise-3.5a}.
We prove the set on each side is a subset of the other.
\paragraph{($\subseteq$)}%
Let $c \in A \times B$.
Then there exists some $a \in A$ and $b \in B$ such that
$c = \tuple{a, b}$.
2023-08-09 13:39:41 +00:00
Thus $c \in \{a\} \times B$.
We also note $\{a\} \times B \in \{\{x\} \times B \mid x \in A\}$,
specifically when $x = a$.
Therefore, by the \nameref{ref:union-axiom},
$c \in \bigcup\;\{\{x\} \times B \mid x \in A\}$.
\paragraph{($\supseteq$)}%
Let $c \in \bigcup\; \{\{x\} \times B \mid x \in A\}$.
By the \nameref{ref:union-axiom}, there exists some
$b \in \{\{x\} \times B \mid x \in A\}$ such that $c \in b$.
Then there exists some $x \in A$ such that $b = \{x\} \times B$.
Therefore $c \in \{x\} \times B$.
But $x \in A$ meaning $c \in A \times B$ as well.
2023-06-28 19:19:59 +00:00
2023-08-09 13:39:41 +00:00
\paragraph{Conclusion}%
2023-06-28 19:19:59 +00:00
2023-08-09 13:39:41 +00:00
Since we have shown
$A \times B \subseteq \bigcup\; \{\{x\} \times B \mid x \in A\}$ and
$A \times B \supseteq \bigcup\; \{\{x\} \times B \mid x \in A\}$, it
follows \eqref{sub:exercise-3.5b-eq1} is a true identity.
2023-06-28 19:19:59 +00:00
2023-08-09 13:39:41 +00:00
\end{proof}
2023-06-28 19:19:59 +00:00
2023-06-29 20:53:36 +00:00
\subsection{\verified{Exercise 3.6}}%
\hyperlabel{sub:exercise-3.6}
2023-06-28 19:19:59 +00:00
2023-08-09 13:39:41 +00:00
Show that a set $A$ is a relation iff $A \subseteq \dom{A} \times \ran{A}$.
2023-06-28 19:19:59 +00:00
2023-08-09 13:39:41 +00:00
\code*{Bookshelf/Enderton/Set/Chapter\_3}
2023-06-29 20:53:36 +00:00
{Enderton.Set.Chapter\_3.exercise\_3\_6}
2023-06-28 19:19:59 +00:00
2023-08-09 13:39:41 +00:00
\begin{proof}
Let $A$ be a set.
We prove the forward and reverse direction of the bidirectional.
2023-06-28 19:19:59 +00:00
2023-08-09 13:39:41 +00:00
\paragraph{($\Rightarrow$)}%
2023-06-29 20:53:36 +00:00
2023-08-09 13:39:41 +00:00
Suppose $A$ is a \nameref{ref:relation}.
We show for all $a \in A$, $a \in \dom{A} \times \ran{A}$.
Let $a \in A$.
Since $A$ is a relation, $a$ is an ordered pair.
Then there exists some sets $x$ and $y$ such that $a = \tuple{x, y}$.
2023-08-09 13:39:41 +00:00
By the definition of the \nameref{ref:domain} and \nameref{ref:range} of
$A$, $x \in \dom{A}$ and $y \in \ran{A}$.
Thus $a = \tuple{x, y} \in \dom{A} \times \ran{A}$ as well.
2023-08-09 13:39:41 +00:00
This proves $A \subseteq \dom{A} \times \ran{A}$.
2023-06-29 20:53:36 +00:00
2023-08-09 13:39:41 +00:00
\paragraph{($\Leftarrow$)}%
2023-06-29 20:53:36 +00:00
2023-08-09 13:39:41 +00:00
Suppose $A \subseteq \dom{A} \times \ran{A}$.
Then for all $a \in A$, $a \in \dom{A} \times \ran{A}$.
Therefore $a$ is an ordered pair.
Since this holds for all $a \in A$, it follows $A$ is a relation.
2023-06-28 19:19:59 +00:00
2023-08-09 13:39:41 +00:00
\end{proof}
2023-06-28 19:19:59 +00:00
2023-06-29 20:53:36 +00:00
\subsection{\verified{Exercise 3.7}}%
\hyperlabel{sub:exercise-3.7}
2023-08-09 13:39:41 +00:00
Show that if $R$ is a relation, then $\fld{R} = \bigcup\bigcup R$.
2023-08-09 13:39:41 +00:00
\code*{Bookshelf/Enderton/Set/Chapter\_3}
2023-06-29 20:53:36 +00:00
{Enderton.Set.Chapter\_3.exercise\_3\_7}
2023-08-09 13:39:41 +00:00
\begin{proof}
Let $R$ be a \nameref{ref:relation}.
We show that (i) $\fld{R} \subseteq \bigcup\bigcup R$ and (ii) that
$\bigcup\bigcup R \subseteq \fld{R}$.
2023-06-28 19:19:59 +00:00
2023-08-09 13:39:41 +00:00
\paragraph{(i)}%
\hyperlabel{par:exercise-3.7-i}
2023-06-28 19:19:59 +00:00
2023-08-09 13:39:41 +00:00
Let $x \in \fld{R} = \dom{R} \cup \ran{R}$.
That is, $x \in \dom{R}$ or $x \in \ran{R}$.
2023-06-28 19:19:59 +00:00
2023-08-09 13:39:41 +00:00
If $x \in \dom{R}$, then there exists some $y$ such that
$\tuple{x, y} = \{\{x\}, \{x, y\}\} \in R$.
2023-08-09 13:39:41 +00:00
Then $\{x\} \in \bigcup R$ and $x \in \bigcup\bigcup R$.
2023-06-28 19:19:59 +00:00
2023-08-09 13:39:41 +00:00
On the other hand, if $x \in \ran{R}$, then there exists some $t$ such that
$\tuple{t, x} = \{\{t\}, \{t, x\}\} \in R$.
2023-08-09 13:39:41 +00:00
Then $\{t, x\} \in \bigcup R$ and $x \in \bigcup\bigcup R$.
2023-06-28 19:19:59 +00:00
2023-08-09 13:39:41 +00:00
\paragraph{(ii)}%
\hyperlabel{par:exercise-3.7-ii}
2023-06-28 19:19:59 +00:00
2023-08-09 13:39:41 +00:00
Let $t \in \bigcup\bigcup R$.
Then there exists some member $T \in \bigcup R$ such that $t \in T$.
Likewise there exists some member $T' \in R$ such that $T \in T'$.
By definition of a relation, $T' = \tuple{x, y} = \{\{x\}, \{x, y\}\}$ for
2023-08-09 13:39:41 +00:00
some sets $x$ and $y$.
Thus $t = x$ or $t = y$.
By \nameref{sub:exercise-3.6}, $t \in \dom{R}$ or $t \in \ran{R}$.
In other words, $t \in \fld{R}$.
2023-06-28 19:19:59 +00:00
2023-08-09 13:39:41 +00:00
\paragraph{Conclusion}%
2023-06-28 19:19:59 +00:00
2023-08-09 13:39:41 +00:00
Since \nameref{par:exercise-3.7-i} and \nameref{par:exercise-3.7-ii} hold,
$\fld{R} = \bigcup\bigcup{R}$.
2023-08-09 13:39:41 +00:00
\end{proof}
2023-06-29 20:53:36 +00:00
\subsection{\verified{Exercise 3.8}}%
\hyperlabel{sub:exercise-3.8}
2023-06-28 19:19:59 +00:00
2023-08-09 13:39:41 +00:00
Show that for any set $\mathscr{A}$:
\begin{align}
\dom{\bigcup{\mathscr{A}}}
& = \bigcup\;\{ \dom{R} \mid R \in \mathscr{A} \},
& \hyperlabel{sub:exercise-3.8-eq1} \\
\ran{\bigcup{\mathscr{A}}}
& = \bigcup\;\{ \ran{R} \mid R \in \mathscr{A} \}.
& \hyperlabel{sub:exercise-3.8-eq2}
\end{align}
2023-06-28 19:19:59 +00:00
2023-08-09 13:39:41 +00:00
\code{Bookshelf/Enderton/Set/Chapter\_3}
2023-06-29 20:53:36 +00:00
{Enderton.Set.Chapter\_3.exercise\_3\_8\_i}
2023-06-28 19:19:59 +00:00
\code{Bookshelf/Enderton/Set/Chapter\_3}
2023-06-29 20:53:36 +00:00
{Enderton.Set.Chapter\_3.exercise\_3\_8\_ii}
2023-08-09 13:39:41 +00:00
\begin{proof}
We prove (i) \eqref{sub:exercise-3.8-eq1} and then (ii)
\eqref{sub:exercise-3.8-eq2}.
\paragraph{(i)}%
Let $x \in \dom{\bigcup{\mathscr{A}}}$.
By definition of a domain, there exists some $y$ such that
$\tuple{x, y} \in \bigcup{\mathscr{A}}$.
2023-08-09 13:39:41 +00:00
By definition of the union of sets,
$\exists y, \exists R \in \mathscr{A}, \tuple{x, y} \in R$.
2023-08-09 13:39:41 +00:00
Equivalently,
$\exists R \in \mathscr{A}, \exists y, \tuple{x, y} \in R$.
2023-08-09 13:39:41 +00:00
By another application of the definition of a domain,
$\exists R \in \mathscr{A}, x \in \dom{R}$.
By another application of the definition of the union of sets,
$x \in \bigcup\;\{ \dom{R} \mid R \in \mathscr{A} \}$.
Since membership of these two sets holds biconditionally, it follows
\eqref{sub:exercise-3.8-eq1} holds.
\paragraph{(ii)}%
Let $x \in \ran{\bigcup{\mathscr{A}}}$.
By definition of a range, there exists some $t$ such that
$\tuple{t, x} \in \bigcup{\mathscr{A}}$.
2023-08-09 13:39:41 +00:00
By definition of the union of sets,
$\exists t, \exists R \in \mathscr{A}, \tuple{t, x} \in R$.
2023-08-09 13:39:41 +00:00
Equivalently,
$\exists R \in \mathscr{A}, \exists t, \tuple{t, x} \in R$.
2023-08-09 13:39:41 +00:00
By another application of the definition of a range,
$\exists R \in \mathscr{A}, x \in \ran{R}$.
By another application of the definition of the union of sets,
$x \in \bigcup\;\{ \ran{R} \mid R \in \mathscr{A} \}$.
Since membership of these two sets holds biconditionally, it follows
\eqref{sub:exercise-3.8-eq2} holds.
\end{proof}
2023-06-28 19:19:59 +00:00
2023-06-29 20:53:36 +00:00
\subsection{\verified{Exercise 3.9}}%
\hyperlabel{sub:exercise-3.9}
2023-08-09 13:39:41 +00:00
Discuss the result of replacing the union operation by the intersection
operation in the preceding problem.
\code*{Bookshelf/Enderton/Set/Chapter\_3}
2023-06-29 20:53:36 +00:00
{Enderton.Set.Chapter\_3.exercise\_3\_9\_i}
\code{Bookshelf/Enderton/Set/Chapter\_3}
2023-06-29 20:53:36 +00:00
{Enderton.Set.Chapter\_3.exercise\_3\_9\_ii}
2023-08-09 13:39:41 +00:00
\begin{answer}
Replacing the union operation with the intersection problem produces the
following relationships
\begin{align}
\dom{\bigcap{\mathscr{A}}}
& \subseteq \bigcap\;\{ \dom{R} \mid R \in \mathscr{A} \},
& \hyperlabel{sub:exercise-3.9-eq1} \\
\ran{\bigcap{\mathscr{A}}}
& \subseteq \bigcap\;\{ \ran{R} \mid R \in \mathscr{A} \}.
& \hyperlabel{sub:exercise-3.9-eq2}
\end{align}
2023-06-29 20:53:36 +00:00
2023-08-09 13:39:41 +00:00
We prove (i) \eqref{sub:exercise-3.9-eq1} and then (ii)
\eqref{sub:exercise-3.9-eq2}.
\paragraph{(i)}%
Let $x \in \dom{\bigcap{\mathscr{A}}}$.
By definition of the \nameref{ref:domain} of a set,
$\exists y, \tuple{x, y} \in \bigcap{\mathscr{A}}$.
2023-08-09 13:39:41 +00:00
By definition of the intersection of sets,
$\exists y, \forall R \in \mathscr{A}, \tuple{x, y} \in R$.
2023-08-09 13:39:41 +00:00
But this implies that
$\forall R \in \mathscr{A}, \exists y, \tuple{x, y} \in R$.
2023-08-09 13:39:41 +00:00
By another application of the definition of the \nameref{ref:domain} of a
set, $\forall R \in \mathscr{A}, x \in \dom{R}$.
By another application of the intersection of sets,
$x \in \bigcap\;\{ \dom{R} \mid R \in \mathscr{A} \}$.
Thus \eqref{sub:exercise-3.9-eq1} holds.
\paragraph{(ii)}%
Let $x \in \ran{\bigcap{\mathscr{A}}}$.
By definition of the \nameref{ref:range} of a set,
$\exists t, \tuple{t, x} \in \bigcap{\mathscr{A}}$.
2023-08-09 13:39:41 +00:00
By definition of the intersection of sets,
$\exists t, \forall R \in \mathscr{A}, \tuple{t, x} \in R$.
2023-08-09 13:39:41 +00:00
But this implies that
$\forall R \in \mathscr{A}, \exists t, \tuple{t, x} \in R$.
2023-08-09 13:39:41 +00:00
By another application of the definition of the \nameref{ref:range} of a
set, $\forall R \in \mathscr{A}, x \in \ran{R}$.
By another application of the intersection of sets,
$x \in \bigcap\;\{ \ran{R} \mid R \in \mathscr{A} \}$.
Thus \eqref{sub:exercise-3.9-eq2} holds.
\end{answer}
2023-06-29 20:53:36 +00:00
\subsection{\unverified{Exercise 3.10}}%
\hyperlabel{sub:exercise-3.10}
2023-06-29 20:53:36 +00:00
2023-08-09 13:39:41 +00:00
Show that an ordered $4$-tuple is also an ordered $m$-tuple for every positive
integer $m$ less than $4$.
2023-06-29 20:53:36 +00:00
2023-08-09 13:39:41 +00:00
\begin{answer}
Let $\tuple{x_1, x_2, x_3, x_4}$ denote an arbitrary $4$-tuple.
2023-08-09 13:39:41 +00:00
Then
\begin{align}
\tuple{x_1, x_2, x_3, x_4}
& = \tuple{\tuple{x_1, x_2, x_3}, x_4}
2023-08-09 13:39:41 +00:00
& \hyperlabel{sub:exercise-7.10-eq1} \\
& = \tuple{\tuple{\tuple{x_1, x_2}, x_3}, x_4}
2023-08-09 13:39:41 +00:00
& \hyperlabel{sub:exercise-7.10-eq2}
\end{align}
Here \eqref{sub:exercise-7.10-eq1} is an equivalent ordered $2$-tuple and
\eqref{sub:exercise-7.10-eq2} is an equivalent ordered $3$-tuple.
Furthermore,
$\tuple{x_1, x_2, x_3, x_4} = \tuple{\tuple{x_1, x_2, x_3, x_4}}$,
2023-08-09 13:39:41 +00:00
showing it can be represented as an ordered $1$-tuple as well.
\end{answer}
2023-06-28 21:31:37 +00:00
\subsection{\unverified{Exercise 3.11}}%
\hyperlabel{sub:exercise-3.11}
2023-06-28 21:31:37 +00:00
2023-08-09 13:39:41 +00:00
Prove the following version (for functions) of the extensionality principle:
Assume that $F$ and $G$ are functions, $\dom{F} = \dom{G}$, and
$F(x) = G(x)$ for all $x$ in the common domain.
Then $F = G$.
2023-08-09 13:39:41 +00:00
\lean*{Init/Core}{funext}
2023-06-28 21:31:37 +00:00
2023-08-09 13:39:41 +00:00
\begin{proof}
Let $F$ and $G$ be functions such that $\dom{F} = \dom{G}$ and $F(x) = G(x)$
for all $x$ in the common domain.
We prove that $\tuple{x, y} \in F$ if and only if $\tuple{x, y} \in G$.
2023-08-09 13:39:41 +00:00
But this follows immediately:
\begin{align*}
\tuple{x, y} \in F
& \iff y = F(x) \land \tuple{x, F(x)} \in F \\
& \iff y = G(x) \land \tuple{x, G(x)} \in G \\
& \iff \tuple{x, y} \in G.
2023-08-09 13:39:41 +00:00
\end{align*}
By the \nameref{ref:extensionality-axiom}, $F = G$.
\end{proof}
2023-06-28 21:31:37 +00:00
\subsection{\verified{Exercise 3.12}}%
\hyperlabel{sub:exercise-3.12}
2023-06-28 21:31:37 +00:00
2023-08-09 13:39:41 +00:00
Assume that $f$ and $g$ are functions and show that
$$f \subseteq g \iff \dom{f} \subseteq \dom{g} \land
(\forall x \in \dom{f}) f(x) = g(x).$$
2023-06-28 21:31:37 +00:00
\code{Bookshelf/Enderton/Set/Chapter\_3}
{Enderton.Set.Chapter\_3.exercise\_3\_12}
2023-08-09 13:39:41 +00:00
\begin{proof}
Let $f$ and $g$ be \nameref{ref:function}s.
2023-08-09 13:39:41 +00:00
\paragraph{($\Rightarrow$)}%
2023-08-09 13:39:41 +00:00
Suppose $f \subseteq g$.
Then for all \nameref{ref:ordered-pair}s $\tuple{x, y}$,
$\tuple{x, y} \in f$ implies $\tuple{x, y} \in g$.
2023-08-09 13:39:41 +00:00
Thus every $x \in \dom{f}$ must be a member of $\dom{g}$.
Likewise, by definition of a function, $f$ and $g$ are single-valued.
Thus $f(x) = y$ and $g(x) = y$.
Since $x$ is an arbitrary element in the domain of $f$, it follows
$(\forall x \in \dom{f}) f(x) = y = g(x)$.
2023-08-09 13:39:41 +00:00
\paragraph{($\Leftarrow$)}%
2023-08-09 13:39:41 +00:00
Suppose $\dom{f} \subseteq \dom{g}$ and
$(\forall x \in \dom{f}) f(x) = g(x)$.
Let $\tuple{x, y} \in f$.
2023-08-09 13:39:41 +00:00
By hypothesis, $x \in \dom{g}$ and $y = f(x) = g(x)$.
Thus $\tuple{x, y} \in g$ as well.
2023-08-09 13:39:41 +00:00
Therefore $f \subseteq g$.
2023-06-28 21:31:37 +00:00
2023-08-09 13:39:41 +00:00
\end{proof}
2023-06-28 21:31:37 +00:00
\subsection{\verified{Exercise 3.13}}%
\hyperlabel{sub:exercise-3.13}
2023-06-28 21:31:37 +00:00
2023-08-09 13:39:41 +00:00
Assume that $f$ and $g$ are functions with $f \subseteq g$ and
$\dom{g} \subseteq \dom{f}$.
Show that $f = g$.
2023-06-28 21:31:37 +00:00
2023-08-09 13:39:41 +00:00
\code*{Bookshelf/Enderton/Set/Chapter\_3}
{Enderton.Set.Chapter\_3.exercise\_3\_13}
2023-08-09 13:39:41 +00:00
\begin{proof}
Let $f$ and $g$ be functions such that $f \subseteq g$ and
$\dom{g} \subseteq \dom{f}$.
By \nameref{sub:exercise-3.12}, it follows that $\dom{f} \subseteq \dom{g}$
and $(\forall x \in \dom{f}) f(x) = g(x)$.
Since $\dom{g} \subseteq \dom{f}$ and $\dom{f} \subseteq \dom{g}$, it
follows that $\dom{g} = \dom{f}$.
By \nameref{sub:exercise-3.11}, $f = g$.
\end{proof}
2023-06-28 21:31:37 +00:00
\subsection{\verified{Exercise 3.14}}%
\hyperlabel{sub:exercise-3.14}
2023-06-28 21:31:37 +00:00
2023-08-09 13:39:41 +00:00
Assume that $f$ and $g$ are functions.
\begin{enumerate}[(a)]
\item Show that $f \cap g$ is a function.
\item Show that $f \cup g$ is a function iff $f(x) = g(x)$ for every $x$ in
$(\dom{f}) \cap (\dom{g})$.
\end{enumerate}
2023-08-09 13:39:41 +00:00
\code{Bookshelf/Enderton/Set/Chapter\_3}
{Enderton.Set.Chapter\_3.exercise\_3\_14\_a}
\code{Bookshelf/Enderton/Set/Chapter\_3}
{Enderton.Set.Chapter\_3.exercise\_3\_14\_b}
2023-08-09 13:39:41 +00:00
\begin{proof}
Assume $f$ and $g$ are \nameref{ref:function}s.
2023-08-09 13:39:41 +00:00
\paragraph{(a)}%
2023-08-09 13:39:41 +00:00
Consider $f \cap g$.
By definition of the intersection of sets, $f \cap g \subseteq f$.
Since $f$ is single-valued, it trivially follows that so must $f \cap g$.
Therefore $f \cap g$ is a function.
2023-08-09 13:39:41 +00:00
\paragraph{(b)}%
2023-08-09 13:39:41 +00:00
\subparagraph{($\Rightarrow$)}%
2023-08-09 13:39:41 +00:00
Suppose $f \cup g$ is a function.
Let $x \in (\dom{f}) \cap (\dom{g})$.
That is, $x \in \dom{f}$ and $x \in \dom{g}$.
Then there exists only one $y_1$ such that $\tuple{x, y_1} \in f$.
2023-08-09 13:39:41 +00:00
Likewise there exists only one $y_2$ such that
$\tuple{x, y_2} \in g$.
But $\tuple{x, y_1} \in f \cup g$ and $\tuple{x, y_2} \in f \cup g$.
2023-08-09 13:39:41 +00:00
Since $f \cup g$ is single-valued, it follows $y_1 = y_2$.
That is, $f(x) = g(x)$.
2023-08-09 13:39:41 +00:00
\subparagraph{($\Leftarrow$)}%
2023-08-09 13:39:41 +00:00
Suppose $f(x) = g(x)$ for every $x \in (\dom{f}) \cap (\dom{g})$.
Let $x \in \dom{(f \cup g)}$.
There are three cases to consider:
2023-08-09 13:39:41 +00:00
\begin{enumerate}[(i)]
\item Suppose $x \in \dom{f}$ but not in $\dom{g}$.
Since $f$ is a function, it follows $f \cup g$ has only one value $y$
such that $\tuple{x, y} \in f \cup g$.
2023-08-09 13:39:41 +00:00
\item Suppose $x \in \dom{g}$ but not in $\dom{f}$.
Again, since $g$ is a function, it follows $f \cup g$ has only one
value $y$ such that $\tuple{x, y} \in f \cup g$.
2023-08-09 13:39:41 +00:00
\item Suppose $x \in \dom{f}$ and $x \in \dom{g}$.
By hypothesis, $f(x) = g(x)$ meaning there is only one value $y$ such
that $\tuple{x, y} \in f \cup g$.
2023-08-09 13:39:41 +00:00
\end{enumerate}
2023-08-09 13:39:41 +00:00
The above cases are exhaustive.
Together they imply that $f \cup g$ is single-valued, i.e. a function.
2023-06-28 21:31:37 +00:00
2023-08-09 13:39:41 +00:00
\end{proof}
2023-06-28 21:31:37 +00:00
\subsection{\verified{Exercise 3.15}}%
\hyperlabel{sub:exercise-3.15}
2023-06-28 21:31:37 +00:00
2023-08-09 13:39:41 +00:00
Let $\mathscr{A}$ be a set of functions such that for any $f$ and $g$ in
$\mathscr{A}$, either $f \subseteq g$ or $g \subseteq f$.
Show that $\bigcup{\mathscr{A}}$ is a function.
2023-06-28 21:31:37 +00:00
2023-08-09 13:39:41 +00:00
\code*{Bookshelf/Enderton/Set/Chapter\_3}
{Enderton.Set.Chapter\_3.exercise\_3\_15}
2023-08-09 13:39:41 +00:00
\begin{proof}
Let $\mathscr{A}$ be a set of \nameref{ref:function}s such that for any $f$
and $g$ in $\mathscr{A}$, either $f \subseteq g$ or $g \subseteq f$.
Let $x \in \dom{\bigcup{\mathscr{A}}}$.
Then there exists some $y_1$ such that
$\tuple{x, y_1} \in \bigcup{\mathscr{A}}$.
2023-08-09 13:39:41 +00:00
Suppose there also exists some $y_2$ such that
$\tuple{x, y_2} \in \bigcup{\mathscr{A}}$.
2023-08-09 13:39:41 +00:00
By definition of the union of sets, there exists some function
$f \in \mathscr{A}$ such that $\tuple{x, y_1} \in f$.
2023-08-09 13:39:41 +00:00
Likewise there exists some function $g \in \mathscr{A}$ such that
$\tuple{x, y_2} \in g$.
2023-08-09 13:39:41 +00:00
There are two cases to consider:
2023-08-09 13:39:41 +00:00
\paragraph{Case 1}%
2023-08-09 13:39:41 +00:00
Suppose $f \subseteq g$.
Then $\tuple{x, y_1}, \tuple{x, y_2} \in g$.
2023-08-09 13:39:41 +00:00
Since $g$ is a function, i.e. single-valued, $y_1 = y_2$.
2023-08-09 13:39:41 +00:00
\paragraph{Case 2}%
2023-08-09 13:39:41 +00:00
Suppose $g \subseteq f$.
Then $\tuple{x, y_1}, \tuple{x, y_2} \in f$.
2023-08-09 13:39:41 +00:00
Since $f$ is a function, i.e. single-valued, $y_1 = y_2$.
2023-08-09 13:39:41 +00:00
\paragraph{Conclusion}%
2023-08-09 13:39:41 +00:00
Since the above two cases applies for all
$x \in \dom{\bigcup{\mathscr{A}}}$ and appropriate choices of $f$ and $g$,
it follows $\bigcup{\mathscr{A}}$ is indeed a function.
2023-06-28 21:31:37 +00:00
2023-08-09 13:39:41 +00:00
\end{proof}
2023-06-28 21:31:37 +00:00
\subsection{\unverified{Exercise 3.16}}%
\hyperlabel{sub:exercise-3.16}
2023-06-28 21:31:37 +00:00
2023-08-09 13:39:41 +00:00
Show that there is no set to which every function belongs.
2023-06-28 21:31:37 +00:00
2023-08-09 13:39:41 +00:00
\begin{proof}
Every \nameref{ref:relation} consisting of a single
\nameref{ref:ordered-pair} is, by definition, a \nameref{ref:function}.
By \nameref{sub:exercise-3.4}, there is no set to which every ordered pair
belongs.
Thus there is no set to which every function of the described type belongs
either, let alone a set to which \textit{every} function belongs.
\end{proof}
2023-06-28 21:31:37 +00:00
2023-07-05 22:04:43 +00:00
\subsection{\verified{Exercise 3.17}}%
\hyperlabel{sub:exercise-3.17}
2023-06-28 21:31:37 +00:00
2023-08-09 13:39:41 +00:00
Show that the composition of two single-rooted sets is again single-rooted.
Conclude that the composition of two one-to-one functions is again one-to-one.
2023-07-05 22:04:43 +00:00
\code*{Bookshelf/Enderton/Set/Chapter\_3}
2023-07-05 22:04:43 +00:00
{Enderton.Set.Chapter\_3.exercise\_3\_17\_i}
\code{Bookshelf/Enderton/Set/Chapter\_3}
2023-07-05 22:04:43 +00:00
{Enderton.Set.Chapter\_3.exercise\_3\_17\_ii}
2023-08-09 13:39:41 +00:00
\begin{proof}
Let $F$ and $G$ be two single-rooted sets.
Consider $F \circ G$.
By definition of the \nameref{ref:composition} of sets,
\begin{equation}
\hyperlabel{sub:exercise-3.17-eq1}
F \circ G = \{\tuple{u, v} \mid \exists t(uGt \land tFv)\}.
2023-08-09 13:39:41 +00:00
\end{equation}
Consider any $v \in \ran{(F \circ G)}$.
By definition of the \nameref{ref:range} of a \nameref{ref:relation}, there
exists some $u_1$ such that $\tuple{u_1, v} \in F \circ G$.
Let $u_2$ be a set such that $\tuple{u_2, v} \in F \circ G$.
2023-08-09 13:39:41 +00:00
By \eqref{sub:exercise-3.17-eq1}, there exists a set $t_1$ such that
$\tuple{u_1, t_1} \in G$ and $\tuple{t_1, v} \in F$.
2023-08-09 13:39:41 +00:00
Likewise, there exists a set $t_2$ such that
$\tuple{u_2, t_2} \in G$ and $\tuple{t_2, v} \in F$.
2023-08-09 13:39:41 +00:00
But $F$ is single-rooted, meaning $t_1 = t_2$.
Likewise, because $G$ is single-rooted, $u_1 = u_2$.
Thus $F \circ G$ must also be single-rooted.
\suitdivider
Let $f$ and $g$ be one-to-one functions.
By \nameref{sub:theorem-3h}, $f \circ g$ is single-valued.
By the above, $f \circ g$ is single-rooted.
Thus $f \circ g$ is one-to-one.
\end{proof}
2023-06-28 21:31:37 +00:00
2023-07-05 22:04:43 +00:00
\subsection{\verified{Exercise 3.18}}%
\hyperlabel{sub:exercise-3.18}
2023-06-28 21:31:37 +00:00
2023-08-09 13:39:41 +00:00
Let $R$ be the set
$$\{ \tuple{0, 1}, \tuple{0, 2}, \tuple{0, 3},
\tuple{1, 2}, \tuple{1, 3}, \tuple{2, 3}\}.$$
2023-08-09 13:39:41 +00:00
Evaluate the following: $R \circ R$, $R \restriction \{1\}$,
$R^{-1} \restriction \{1\}$, $\img{R}{\{1\}}$, and $\img{R^{-1}}{\{1\}}$.
2023-07-05 22:04:43 +00:00
\code*{Bookshelf/Enderton/Set/Chapter\_3}
2023-07-05 22:04:43 +00:00
{Enderton.Set.Chapter\_3.exercise\_3\_18\_i}
2023-08-09 13:39:41 +00:00
\code{Bookshelf/Enderton/Set/Chapter\_3}
2023-07-05 22:04:43 +00:00
{Enderton.Set.Chapter\_3.exercise\_3\_18\_ii}
2023-08-09 13:39:41 +00:00
\code{Bookshelf/Enderton/Set/Chapter\_3}
2023-07-05 22:04:43 +00:00
{Enderton.Set.Chapter\_3.exercise\_3\_18\_iii}
2023-08-09 13:39:41 +00:00
\code{Bookshelf/Enderton/Set/Chapter\_3}
2023-07-05 22:04:43 +00:00
{Enderton.Set.Chapter\_3.exercise\_3\_18\_iv}
\code{Bookshelf/Enderton/Set/Chapter\_3}
2023-07-05 22:04:43 +00:00
{Enderton.Set.Chapter\_3.exercise\_3\_18\_v}
2023-08-09 13:39:41 +00:00
\begin{proof}
\begin{enumerate}[(i)]
\item $R \circ R = \{ \tuple{0, 2}, \tuple{0, 3}, \tuple{1, 3} \}$.
\item $R \restriction \{1\} = \{ \tuple{1, 2}, \tuple{1, 3} \}$.
\item $R^{-1} \restriction \{1\} = \{\tuple{1, 0}\}$.
2023-08-09 13:39:41 +00:00
\item $\img{R}{\{1\}} = \{2, 3\}$.
\item $\img{R^{-1}}{\{1\}} = \{0\}$.
\end{enumerate}
\end{proof}
2023-06-28 21:31:37 +00:00
2023-07-06 13:31:13 +00:00
\subsection{\verified{Exercise 3.19}}%
\hyperlabel{sub:exercise-3.19}
2023-06-28 21:31:37 +00:00
2023-08-09 13:39:41 +00:00
Let $$A = \{
\tuple{\emptyset, \{\emptyset, \{\emptyset\}\}},
\tuple{\{\emptyset\}, \emptyset}
2023-08-09 13:39:41 +00:00
\}.$$
Evaluate each of the following: $A(\emptyset)$, $\img{A}{\emptyset}$,
$\img{A}{\{\emptyset\}}$, $\img{A}{\{\emptyset, \{\emptyset\}\}}$,
$A^{-1}$, $A \circ A$, $A \restriction \emptyset$,
$A \restriction \{\emptyset\}$,
$A \restriction \{\emptyset, \{\emptyset\}\}$,
$\bigcup\bigcup A$.
2023-07-06 13:31:13 +00:00
2023-08-09 13:39:41 +00:00
\code{Bookshelf/Enderton/Set/Chapter\_3}
2023-07-06 13:31:13 +00:00
{Enderton.Set.Chapter\_3.exercise\_3\_19\_i}
2023-08-09 13:39:41 +00:00
\code{Bookshelf/Enderton/Set/Chapter\_3}
2023-07-06 13:31:13 +00:00
{Enderton.Set.Chapter\_3.exercise\_3\_19\_ii}
2023-08-09 13:39:41 +00:00
\code{Bookshelf/Enderton/Set/Chapter\_3}
2023-07-06 13:31:13 +00:00
{Enderton.Set.Chapter\_3.exercise\_3\_19\_iii}
2023-08-09 13:39:41 +00:00
\code{Bookshelf/Enderton/Set/Chapter\_3}
2023-07-06 13:31:13 +00:00
{Enderton.Set.Chapter\_3.exercise\_3\_19\_iv}
2023-08-09 13:39:41 +00:00
\code{Bookshelf/Enderton/Set/Chapter\_3}
2023-07-06 13:31:13 +00:00
{Enderton.Set.Chapter\_3.exercise\_3\_19\_v}
2023-08-09 13:39:41 +00:00
\code{Bookshelf/Enderton/Set/Chapter\_3}
2023-07-06 13:31:13 +00:00
{Enderton.Set.Chapter\_3.exercise\_3\_19\_vi}
2023-08-09 13:39:41 +00:00
\code{Bookshelf/Enderton/Set/Chapter\_3}
2023-07-06 13:31:13 +00:00
{Enderton.Set.Chapter\_3.exercise\_3\_19\_vii}
2023-08-09 13:39:41 +00:00
\code{Bookshelf/Enderton/Set/Chapter\_3}
2023-07-06 13:31:13 +00:00
{Enderton.Set.Chapter\_3.exercise\_3\_19\_viii}
2023-08-09 13:39:41 +00:00
\code{Bookshelf/Enderton/Set/Chapter\_3}
2023-07-06 13:31:13 +00:00
{Enderton.Set.Chapter\_3.exercise\_3\_19\_ix}
\code{Bookshelf/Enderton/Set/Chapter\_3}
2023-07-06 13:31:13 +00:00
{Enderton.Set.Chapter\_3.exercise\_3\_19\_x}
2023-08-09 13:39:41 +00:00
\begin{proof}
\begin{enumerate}[(i)]
\item $A(\emptyset) = \{\emptyset, \{\emptyset\}\}$.
\item $\img{A}{\emptyset} = \emptyset$.
\item $\img{A}{\{\emptyset\}} = \{\{\emptyset, \{\emptyset\}\}\}$.
\item $\img{A}{\{\emptyset, \{\emptyset\}\}} =
\{\{\emptyset, \{\emptyset\}\}, \emptyset\}$.
\item $A^{-1} = \{
\tuple{\{\emptyset, \{\emptyset\}\}, \emptyset},
\tuple{\emptyset, \{\emptyset\}}
2023-08-09 13:39:41 +00:00
\}$.
\item $A \circ A =
\{\tuple{\{\emptyset\}, \{\emptyset, \{\emptyset\}\}}\}$.
2023-08-09 13:39:41 +00:00
\item $A \restriction \emptyset = \emptyset$
\item $A \restriction \{\emptyset\} =
\{\tuple{\emptyset, \{\emptyset, \{\emptyset\}\}}\}$.
2023-08-09 13:39:41 +00:00
\item $A \restriction \{\emptyset, \{\emptyset\}\} = A$.
\item $\bigcup\bigcup A =
\{\emptyset, \{\emptyset\}, \{\emptyset, \{\emptyset\}\}\}$.
\end{enumerate}
\end{proof}
2023-06-28 21:31:37 +00:00
2023-07-06 21:31:09 +00:00
\subsection{\verified{Exercise 3.20}}%
\hyperlabel{sub:exercise-3.20}
2023-06-28 21:31:37 +00:00
2023-08-09 13:39:41 +00:00
Show that $F \restriction A = F \cap (A \times \ran{F})$.
2023-06-28 21:31:37 +00:00
2023-08-09 13:39:41 +00:00
\code*{Bookshelf/Enderton/Set/Chapter\_3}
2023-07-06 21:31:09 +00:00
{Enderton.Set.Chapter\_3.exercise\_3\_20}
2023-08-09 13:39:41 +00:00
\begin{proof}
Let $F$ and $A$ be arbitrary sets.
By \nameref{sub:corollary-3c} and definition of the
\nameref{ref:restriction}, intersection, and \nameref{ref:range} of sets,
\begin{align*}
F \restriction A
& = \{\tuple{u, v} \mid uFv \land u \in A\} \\
& = \{\tuple{u, v} \mid
2023-08-09 13:39:41 +00:00
uFv \land u \in A \land v \in \ran{F}\} \\
& = \{\tuple{u, v} \mid uFv\} \cap
\{\tuple{u, v} \mid u \in A \land v \in \ran{F}\} \\
& = F \cap \{\tuple{u, v} \mid u \in A \land v \in \ran{F}\} \\
2023-08-09 13:39:41 +00:00
& = F \cap (A \times \ran{F}).
\end{align*}
\end{proof}
2023-06-28 21:31:37 +00:00
2023-07-06 21:31:09 +00:00
\subsection{\verified{Exercise 3.21}}%
\hyperlabel{sub:exercise-3.21}
2023-06-28 21:31:37 +00:00
2023-08-09 13:39:41 +00:00
Show that $(R \circ S) \circ T = R \circ (S \circ T)$.
2023-06-28 21:31:37 +00:00
2023-08-09 13:39:41 +00:00
\code*{Bookshelf/Enderton/Set/Relation}
2023-07-06 21:31:09 +00:00
{Set.Relation.comp\_assoc}
2023-08-09 13:39:41 +00:00
\begin{proof}
Let $R$, $S$, and $T$ be arbitrary sets.
By definition of the \nameref{ref:composition} of sets,
\begin{align*}
(R \circ S) \circ T
& = \{\tuple{u, v} \mid
2023-08-09 13:39:41 +00:00
\exists t(uTt \land t(R \circ S)v)\} \\
& = \{\tuple{u, v} \mid
2023-08-09 13:39:41 +00:00
\exists t(uTt \land (\exists a(tSa \land aRv))\} \\
& = \{\tuple{u, v} \mid
2023-08-09 13:39:41 +00:00
\exists t, \exists a, (uTt \land tSa) \land aRv)\} \\
& = \{\tuple{u, v} \mid
2023-08-09 13:39:41 +00:00
\exists a, \exists t, (uTt \land tSa) \land aRv)\} \\
& = \{\tuple{u, v} \mid
2023-08-09 13:39:41 +00:00
\exists a, (\exists t(uTt \land tSa)) \land aRv)\} \\
& = \{\tuple{u, v} \mid
2023-08-09 13:39:41 +00:00
\exists a, u(S \circ T)a \land aRv)\} \\
& = R \circ (S \circ T).
\end{align*}
\end{proof}
2023-06-28 21:31:37 +00:00
2023-07-06 21:31:09 +00:00
\subsection{\verified{Exercise 3.22}}%
\hyperlabel{sub:exercise-3.22}
2023-06-28 21:31:37 +00:00
2023-08-09 13:39:41 +00:00
Show that the following are correct for any sets.
\begin{enumerate}[(a)]
\item $A \subseteq B \Rightarrow \img{F}{A} \subseteq \img{F}{B}$.
\item $\img{(F \circ G)}{A} = \img{F}{\img{G}{A}}$.
\item $Q \restriction (A \cup B) =
(Q \restriction A) \cup (Q \restriction B)$.
\end{enumerate}
2023-07-06 21:31:09 +00:00
2023-08-09 13:39:41 +00:00
\code{Bookshelf/Enderton/Set/Chapter\_3}
2023-07-06 21:31:09 +00:00
{Enderton.Set.Chapter\_3.exercise\_3\_22\_a}
2023-08-09 13:39:41 +00:00
\code{Bookshelf/Enderton/Set/Chapter\_3}
2023-07-06 21:31:09 +00:00
{Enderton.Set.Chapter\_3.exercise\_3\_22\_b}
\code{Bookshelf/Enderton/Set/Chapter\_3}
2023-07-06 21:31:09 +00:00
{Enderton.Set.Chapter\_3.exercise\_3\_22\_c}
2023-08-09 13:39:41 +00:00
\begin{proof}
Let $A$, $B$, $F$, $G$, and $Q$ be arbitrary sets.
2023-08-09 13:39:41 +00:00
\paragraph{(a)}%
2023-08-09 13:39:41 +00:00
Suppose $A \subseteq B$.
Let $x \in \img{F}{A}$.
By definition of the \nameref{ref:image} of a set,
$\img{F}{A} = \{v \mid (\exists u \in A) uFv\}$.
Thus there exists some $u \in A$ such that $uFx$.
But $A \subseteq B$ meaning $u \in B$.
That is, $(\exists u \in B)uFx$.
Thus $$x \in \{v \mid (\exists u \in B)uFv\} = \img{F}{B}.$$
2023-08-09 13:39:41 +00:00
\paragraph{(b)}%
2023-08-09 13:39:41 +00:00
By definition of the \nameref{ref:composition} and \nameref{ref:image} of a
set,
\begin{align*}
\img{(F \circ G)}{A}
& = \{v \mid (\exists u \in A) u(F \circ G)v\} \\
& = \{v \mid (\exists u \in A) \tuple{u, v} \in F \circ G\} \\
2023-08-09 13:39:41 +00:00
& = \{v \mid (\exists u \in A)
\tuple{u, v} \in \{\tuple{b, c} \mid
2023-08-09 13:39:41 +00:00
\exists a(bGa \land aFc)\}\} \\
& = \{v \mid \exists u \in A, \exists a, uGa \land aFv\} \\
& = \{v \mid \exists a, \exists u \in A, uGa \land aFv\} \\
& = \{v \mid \exists a, (\exists u \in A, uGa) \land aFv\} \\
& = \{v \mid \exists a \in \{w \mid (\exists u \in A)uGw\}, aFv\} \\
& = \{v \mid (\exists a \in \img{G}{A}) aFv\} \\
& = \img{F}{\img{G}{A}}.
\end{align*}
2023-08-09 13:39:41 +00:00
\paragraph{(c)}%
2023-08-09 13:39:41 +00:00
By definition of the \nameref{ref:restriction} of a set,
\begin{align*}
Q \restriction (A \cup B)
& = \{\tuple{u, v} \mid uQv \land u \in A \cup B\} \\
& = \{\tuple{u, v} \mid uQv \land (u \in A \lor u \in B)\} \\
& = \{\tuple{u, v} \mid
2023-08-09 13:39:41 +00:00
(uQv \land u \in A) \lor (uQv \land u \in B)\} \\
& = \{\tuple{u, v} \mid uQv \land u \in A\} \cup
\{\tuple{u, v} \mid uQv \land u \in B\} \\
2023-08-09 13:39:41 +00:00
& = (Q \restriction A) \cup (Q \restriction B).
\end{align*}
2023-06-28 21:31:37 +00:00
2023-08-09 13:39:41 +00:00
\end{proof}
2023-06-28 21:31:37 +00:00
2023-07-06 21:31:09 +00:00
\subsection{\verified{Exercise 3.23}}%
\hyperlabel{sub:exercise-3.23}
2023-06-28 21:31:37 +00:00
2023-08-09 13:39:41 +00:00
Let $I_A$ be the identity function on the set $A$.
Show that for any sets $B$ and $C$,
$$B \circ I_A = B \restriction A \quad\text{and}\quad
\img{I_A}{C} = A \cap C.$$
2023-07-06 21:31:09 +00:00
2023-08-09 13:39:41 +00:00
\code{Bookshelf/Enderton/Set/Chapter\_3}
2023-07-06 21:31:09 +00:00
{Enderton.Set.Chapter\_3.exercise\_3\_23\_i}
\code{Bookshelf/Enderton/Set/Chapter\_3}
2023-07-06 21:31:09 +00:00
{Enderton.Set.Chapter\_3.exercise\_3\_23\_ii}
2023-08-09 13:39:41 +00:00
\begin{proof}
Let $I_A$ be the identity function on the set $A$.
That is, $I_A = \{\tuple{u, u} \mid u \in A\}$.
2023-08-09 13:39:41 +00:00
Let $B$ and $C$ be any sets.
We show that (i) $B \circ I_A = B \restriction A$ and (ii)
$\img{I_A}{C} = A \cap C$.
2023-08-09 13:39:41 +00:00
\paragraph{(i)}%
2023-08-09 13:39:41 +00:00
We show that $B \circ I_A \subseteq B \restriction A$ and
$B \restriction A \subseteq B \circ I_A$.
2023-08-09 13:39:41 +00:00
\subparagraph{($\subseteq$)}%
Let $\tuple{x, y} \in B \circ I_A$.
2023-08-09 13:39:41 +00:00
By definition of the \nameref{ref:composition} of sets,
there exists some $t$ such that $x(I_A)t$ and $tBy$.
By definition of the identity function, $I_A(x) = t$ implies $x = t$.
Thus $xBy$.
By hypothesis, $x \in \dom{(B \circ I_A)}$.
Therefore $x \in \dom{I_A} = A$.
Thus $$\tuple{x, y} \in \{\tuple{u, v} \mid u \in A \land uBv\}
2023-08-09 13:39:41 +00:00
= B \restriction A.$$
2023-08-09 13:39:41 +00:00
\subparagraph{($\supseteq$)}%
Let $\tuple{x, y} \in B \restriction A$.
2023-08-09 13:39:41 +00:00
By definition of the \nameref{ref:restriction} of sets,
$x \in A$ and $xBy$.
But $I_A(x) = x$ meaning $\tuple{I_A(x), y} \in B$.
In other words, $\tuple{x, y} \in B \circ I_A$.
2023-08-09 13:39:41 +00:00
\paragraph{(ii)}%
2023-08-09 13:39:41 +00:00
By definition of the \nameref{ref:image} of sets,
\begin{align*}
\img{I_A}{C}
& = \{v \mid (\exists u \in C) \tuple{u, v} \in I_A\} \\
2023-08-09 13:39:41 +00:00
& = \{v \mid \exists u \in C, u \in A \land u = v\} \\
& = \{v \mid v \in C \land v \in A\} \\
& = C \cap A.
\end{align*}
2023-06-28 21:31:37 +00:00
2023-08-09 13:39:41 +00:00
\end{proof}
2023-06-28 21:31:37 +00:00
\subsection{\verified{Exercise 3.24}}%
\hyperlabel{sub:exercise-3.24}
2023-06-28 21:31:37 +00:00
2023-08-09 13:39:41 +00:00
Show that for a function $F$,
$\img{F^{-1}}{A} = \{x \in \dom{F} \mid F(x) \in A\}$.
2023-06-28 21:31:37 +00:00
2023-08-09 13:39:41 +00:00
\code*{Bookshelf/Enderton/Set/Chapter\_3}
{Enderton.Set.Chapter\_3.exercise\_3\_24}
2023-08-09 13:39:41 +00:00
\begin{proof}
Let $F$ be a function.
By definition of the \nameref{ref:inverse} of a set,
\begin{align*}
\img{F^{-1}}{A}
& = \{x \mid (\exists y \in A) yF^{-1}x\} \\
& = \{x \mid (\exists y \in A) xFy\} \\
& = \{x \mid (\exists y \in A) \tuple{x, y} \in F\} \\
2023-08-09 13:39:41 +00:00
& = \{x \mid x \in \dom{F} \land F(x) \in A\} \\
& = \{x \in \dom{F} \mid F(x) \in A\}.
\end{align*}
\end{proof}
2023-06-28 21:31:37 +00:00
\subsection{\verified{Exercise 3.25}}%
\hyperlabel{sub:exercise-3.25}
2023-06-28 21:31:37 +00:00
2023-08-09 13:39:41 +00:00
\begin{enumerate}[(a)]
\item Assume that $G$ is a one-to-one function.
Show that $G \circ G^{-1}$ is $I_{\ran{G}}$, the identity function on
$\ran{G}$.
\item Show that the result of part (a) holds for any function $G$, not
necessarily one-to-one.
\end{enumerate}
2023-08-09 13:39:41 +00:00
\code{Bookshelf/Enderton/Set/Chapter\_3}
{Enderton.Set.Chapter\_3.exercise\_3\_25\_b}
\code{Bookshelf/Enderton/Set/Chapter\_3}
{Enderton.Set.Chapter\_3.exercise\_3\_25\_a}
2023-08-09 13:39:41 +00:00
\begin{proof}
2023-08-09 13:39:41 +00:00
\paragraph{(b)}%
\hyperlabel{par:exercise-3.25-b}
2023-08-09 13:39:41 +00:00
Let $G$ be an arbitrary function.
We show that $G \circ G^{-1} \subseteq I_{\ran{G}}$ and that
$I_{\ran{G}} \subseteq G \circ G^{-1}$.
2023-08-09 13:39:41 +00:00
\subparagraph{($\subseteq$)}%
Let $\tuple{x, y} \in G \circ G^{-1}$.
2023-08-09 13:39:41 +00:00
By definition of the \nameref{ref:composition} of sets, there exists some
set $t$ such that $x(G^{-1})t$ and $tGy$.
By definition of the \nameref{ref:inverse} of a set,
$$x(G^{-1})t \iff tGx.$$
The right hand side of the above biconditional indicates $x \in \ran{G}$.
Since $G$ is single-valued, $tGy \land tGx$ implies $x = y$.
Thus $\tuple{x, y} \in I_{\ran{G}}$.
2023-08-09 13:39:41 +00:00
\subparagraph{($\supseteq$)}%
Let $\tuple{x, x} \in I_{\ran{G}}$ where $x \in \ran{G}$.
2023-08-09 13:39:41 +00:00
By definition of the \nameref{ref:range} of a function, there exists some
$t$ such that $\tuple{t, x} \in G$.
2023-08-09 13:39:41 +00:00
By definition of the \nameref{ref:inverse} of a set, it follows
$\tuple{x, t} \in G^{-1}$.
Thus $\tuple{x, x} \in G \circ G^{-1}$.
2023-08-09 13:39:41 +00:00
\subparagraph{Conclusion}%
2023-08-09 13:39:41 +00:00
Since $G \circ G^{-1}$ is a subset of $I_{\ran{G}}$ and vice versa, it
follows that these two sets are equal.
2023-08-09 13:39:41 +00:00
\paragraph{(a)}%
2023-06-28 21:31:37 +00:00
2023-08-09 13:39:41 +00:00
This immediately follows from part \nameref{par:exercise-3.25-b}.
\end{proof}
2023-06-28 21:31:37 +00:00
\subsection{\verified{Exercise 3.26}}%
\hyperlabel{sub:exercise-3.26}
2023-06-28 21:31:37 +00:00
2023-08-09 13:39:41 +00:00
Prove the second halves of parts (a) and (b) of Theorem 3K.
2023-06-28 21:31:37 +00:00
2023-08-09 13:39:41 +00:00
\begin{proof}
Refer to \nameref{sub:theorem-3k-a}, \nameref{sub:theorem-3k-b}, and
\nameref{sub:theorem-3k-c}.
\end{proof}
2023-06-28 21:31:37 +00:00
\subsection{\verified{Exercise 3.27}}%
\hyperlabel{sub:exercise-3.27}
2023-06-28 21:31:37 +00:00
2023-08-09 13:39:41 +00:00
Show that $\dom{(F \circ G)} = \img{G^{-1}}{\dom{F}}$ for any sets $F$ and
$G$.
($F$ and $G$ need not be functions.)
2023-06-28 21:31:37 +00:00
2023-08-09 13:39:41 +00:00
\code*{Bookshelf/Enderton/Set/Chapter\_3}
{Enderton.Set.Chapter\_3.exercise\_3\_27}
2023-08-09 13:39:41 +00:00
\begin{proof}
Let $F$ and $G$ be arbitrary sets.
We show that each side of our desired equality is a subset of the other.
2023-08-09 13:39:41 +00:00
\paragraph{($\subseteq$)}%
2023-08-09 13:39:41 +00:00
Let $x \in \dom{(F \circ G)}$.
Then there exists a set $y$ such that $\tuple{x, y} \in F \circ G$.
2023-08-09 13:39:41 +00:00
By definition of the \nameref{ref:composition} of sets, there exists a set
$t$ such that $xGt$ and $tFy$.
Thus $t \in \dom{F}$.
Therefore
\begin{align*}
x
& \in \{v \mid (\exists t \in \dom{F}) vGt\} \\
& = \{v \mid (\exists t \in \dom{F}) t(G^{-1})v\} \\
& = \img{G^{-1}}{\dom{F}}.
\end{align*}
2023-08-09 13:39:41 +00:00
\paragraph{($\supseteq$)}%
2023-08-09 13:39:41 +00:00
Let $x \in \img{G^{-1}}{\dom{F}}$.
Then, by definition of the \nameref{ref:image} of a set, there exists some
$u \in \dom{F}$ such that $u(G^{-1})x$.
By definition of the \nameref{ref:inverse} of a set, $xGu$.
By definition of the \nameref{ref:domain} of a set, there exists some $t$
such that $uFt$.
Thus $xGu \land uFt$.
By definition of the \nameref{ref:composition} of sets,
$\tuple{x, t} \in F \circ G$.
2023-08-09 13:39:41 +00:00
Therefore $x \in \dom{(F \circ G)}$.
2023-06-28 21:31:37 +00:00
2023-08-09 13:39:41 +00:00
\end{proof}
2023-06-28 21:31:37 +00:00
\subsection{\verified{Exercise 3.28}}%
\hyperlabel{sub:exercise-3.28}
2023-06-28 21:31:37 +00:00
2023-08-09 13:39:41 +00:00
Assume that $f$ is a one-to-one function from $A$ into $B$, and that $G$ is
the function with $\dom{G} = \powerset{A}$ defined by the equation
$G(X) = \img{f}{X}$.
Show that $G$ maps $\powerset{A}$ one-to-one into $\powerset{B}$.
2023-06-28 21:31:37 +00:00
2023-08-09 13:39:41 +00:00
\code*{Bookshelf/Enderton/Set/Chapter\_3}
{Enderton.Set.Chapter\_3.exercise\_3\_28}
2023-08-09 13:39:41 +00:00
\begin{proof}
By construction, $\dom{G} = \powerset{A}$.
Likewise, $\ran{G} \subseteq \powerset{B}$ by definition of the
\nameref{ref:image} of sets.
Thus $G$ maps $\powerset{A}$ into $\powerset{B}$.
Let $y \in \ran{G}$.
Then there exists an $X_1 \in \powerset{A}$ such that $\img{f}{X_1} = y$.
To prove $G$ is one-to-one into $\powerset{B}$, assume there exists an
$X_2 \in \powerset{A}$ such that $\img{f}{X_2} = y$.
All that remains is showing $X_1 = X_2$.
Let $t \in X_1$.
By definition of the \nameref{ref:image} of a set, $f(t) \in \img{f}{X_1}$.
Since $\img{f}{X_1} = \img{f}{X_2}$, it follows $f(t) \in \img{f}{X_2}$.
Because $f$ is one-to-one, $f(t) \in \img{f}{X_2}$ if and only if
$t \in X_2$.
Thus $t \in X_1$ if and only if $t \in X_2$.
By the \nameref{ref:extensionality-axiom}, it follows $X_1 = X_2$.
\end{proof}
2023-06-28 21:31:37 +00:00
2023-07-08 02:30:53 +00:00
\subsection{\verified{Exercise 3.29}}%
\hyperlabel{sub:exercise-3.29}
2023-06-28 21:31:37 +00:00
2023-08-09 13:39:41 +00:00
Assume that $f \colon A \rightarrow B$ and define a function
$G \colon B \rightarrow \powerset{A}$ by
\begin{equation}
\hyperlabel{sub:exercise-3.29-eq1}
G(b) = \{x \in A \mid f(x) = b\}.
\end{equation}
Show that if $f$ maps $A$ \textit{onto} $B$, then $G$ is one-to-one.
Does the converse hold?
2023-06-28 21:31:37 +00:00
2023-08-09 13:39:41 +00:00
\code*{Bookshelf/Enderton/Set/Chapter\_3}
{Enderton.Set.Chapter\_3.exercise\_3\_29}
2023-07-08 02:30:53 +00:00
2023-08-09 13:39:41 +00:00
\begin{proof}
Let $f \colon A \rightarrow B$ such that $f$ maps $A$ onto $B$.
Define $G \colon B \rightarrow \powerset{A}$ by
\eqref{sub:exercise-3.29-eq1}.
Let $y \in \ran{G}$.
By definition of the \nameref{ref:range} of a set, there exists an
$x_1 \in B$ such that $G(x_1) = y$.
To prove $G$ is one-to-one, suppose there exists an $x_2 \in B$ such
that $G(x_2) = y$.
All that remains is proving $x_1 = x_2$.
By \eqref{sub:exercise-3.29-eq1}, it follows
\begin{align*}
G(x_1) & = \{x \in A \mid f(x) = x_1\} \\
G(x_2) & = \{x \in A \mid f(x) = x_2\}.
\end{align*}
Since $f$ maps $A$ onto $B$, $\ran{f} = B$.
Thus $x_1, x_2 \in \ran{f}$.
By definition of the \nameref{ref:range} of a set, there exist some
$t \in A$ such that $f(t) = x_1$.
Therefore $t \in G(x_1)$.
By the \nameref{ref:extensionality-axiom}, $t \in G(x_2)$.
Then $f(t) = x_2$.
But $f$ is a \nameref{ref:function}, i.e. single-valued.
Thus $x_1 = x_2$.
\suitdivider
If $G$ is one-to-one, it does not follow that $f$ maps $A$ onto $B$.
As a counterexample, let $f \colon \{1\} \rightarrow \{1, 2\}$ given by
$f(x) = x$.
Define $G \colon \{1, 2\} \rightarrow \powerset{\{1\}}$ by
$$G(b) = \{x \in \{1\} \mid f(x) = b\}.$$
$G$ is trivially one-to-one since $G(1) = \{1\}$ and $G(2) = \emptyset$.
But $f$ does not map onto $\{1, 2\}$; there is no element in its domain that
corresponds to value $2$.
\end{proof}
2023-06-28 21:31:37 +00:00
\subsection{\verified{Exercise 3.30}}%
\hyperlabel{sub:exercise-3.30}
2023-06-28 21:31:37 +00:00
2023-08-09 13:39:41 +00:00
Assume that $F \colon \powerset{A} \rightarrow \powerset{A}$ and that $F$ has
the monotonicity property:
$$X \subseteq Y \subseteq A \Rightarrow F(X) \subseteq F(Y).$$
Define
$$B = \bigcap\{X \subseteq A \mid F(X) \subseteq X\} \quad\text{and}\quad
C = \bigcup\{X \subseteq A \mid X \subseteq F(X)\}.$$
\subsubsection{\verified{Exercise 3.30 (a)}}%
\hyperlabel{ssub:exercise-3.30-a}
2023-08-09 13:39:41 +00:00
Show that $F(B) = B$ and $F(C) = C$.
\code*{Bookshelf/Enderton/Set/Chapter\_3}
{Enderton.Set.Chapter\_3.exercise\_3\_30\_a}
2023-08-09 13:39:41 +00:00
\begin{proof}
We prove that (i) $F(B) = B$ and (ii) $F(C) = C$.
\paragraph{(i)}%
\hyperlabel{par:exercise-3.30-a-i}
To prove $F(B) = B$, we show $F(B) \subseteq B$ and $F(B) \supseteq B$.
\subparagraph{($\subseteq$)}%
\hyperlabel{spar:exercise-3.30-i-sub}
Let $x \in F(B)$ and $X \subseteq A$ such that $F(X) \subseteq X$.
By definition of $B$, $B \subseteq X$.
Then monotonicity implies $F(B) \subseteq F(X) \subseteq X$.
Therefore $x \in X$.
Since $X$ was arbitrarily chosen, it follows that
$$\forall X, X \subseteq A \land F(X) \subseteq X \Rightarrow
x \in X.$$
By definition of the intersection of sets,
$$x \in \bigcap\{X \subseteq A \mid F(X) \subseteq X\} = B.$$
Hence $F(B) \subseteq B$.
\subparagraph{($\supseteq$)}%
By \nameref{spar:exercise-3.30-i-sub}, $F(B) \subseteq B$.
Then monotonicity implies $F(F(B)) \subseteq F(B)$.
Thus $F(B) \in \{X \subseteq A \mid F(X) \subseteq X\}$.
Hence, by definition of $B$, $B \subseteq F(B)$.
\paragraph{(ii)}%
To prove $F(C) = C$, we show $F(C) \supseteq C$ and $F(C) \subseteq C$.
\subparagraph{($\supseteq$)}%
\hyperlabel{spar:exercise-3.30-i-sup}
Let $x \in C$.
By definition of $C$, there exists some $X \subseteq A$ such that
$X \subseteq F(X)$ and $x \in X$.
Since $C$ contains $X$, $X \subseteq C$.
Then monotonicity implies $X \subseteq F(X) \subseteq F(C)$.
Therefore $x \in F(C)$.
\subparagraph{($\subseteq$)}%
By \nameref{spar:exercise-3.30-i-sup}, $C \subseteq F(C)$.
Then monotonicity implies $F(C) \subseteq F(F(C))$.
Thus $F(C) \in \{X \subseteq A \mid X \subseteq F(X)\}$.
Hence, by definition of $C$, $F(C) \subseteq C$.
2023-08-09 13:39:41 +00:00
\end{proof}
\subsubsection{\verified{Exercise 3.30 (b)}}%
\hyperlabel{ssub:exercise-3.30-b}
2023-08-09 13:39:41 +00:00
Show that if $F(X) = X$, then $B \subseteq X \subseteq C$.
2023-06-28 21:31:37 +00:00
\code*{Bookshelf/Enderton/Set/Chapter\_3}
{Enderton.Set.Chapter\_3.exercise\_3\_30\_b}
2023-08-09 13:39:41 +00:00
\begin{proof}
Suppose $F(X) = X$ for some $X \subseteq A$.
By the \nameref{ref:extensionality-axiom}, $F(X) \subseteq X$ and
$X \subseteq F(X)$.
Therefore
\begin{align}
X & \in \{X \subseteq A \mid F(X) \subseteq X\}
& \hyperlabel{ssub:exercise-3.30-b-eq1} \\
X & \in \{X \subseteq A \mid X \subseteq F(X)\}.
& \hyperlabel{ssub:exercise-3.30-b-eq2}
\end{align}
\eqref{ssub:exercise-3.30-b-eq1} immediately implies $B \subseteq X$ and
\eqref{ssub:exercise-3.30-b-eq2} immediately implies $X \subseteq C$.
Thus $B \subseteq X \subseteq C$.
2023-08-09 13:39:41 +00:00
\end{proof}
2023-06-28 21:31:37 +00:00
\subsection{\unverified{Exercise 3.31}}%
\hyperlabel{sub:exercise-3.31}
2023-06-29 20:05:08 +00:00
2023-08-09 13:39:41 +00:00
Show that from the first form of the axiom of choice we can prove the second
form, and conversely.
2023-06-29 20:05:08 +00:00
2023-08-09 13:39:41 +00:00
\begin{proof}
We prove the first form holds if and only if the second form holds.
2023-08-09 13:39:41 +00:00
\paragraph{($\Rightarrow$)}%
2023-08-09 13:39:41 +00:00
We assume the first form of the axiom of choice.
Let $I$ be a set and $H$ be a function with $\dom{H} = I$.
Furthermore, suppose $H(i) \neq \emptyset$ for all $i \in I$.
By definition of the \nameref{ref:cartesian-product},
$$\bigtimes_{i \in I} H(i) = \{f \mid
f \text{ is a function with } \dom{f} = I \text{ and }
(\forall i \in I) f(i) \in H(i)\}.$$
Consider the relation $R$ formed by
$$R = \bigcup_{i \in I} \{i\} \times H(i).$$
By the \nameref{ref:axiom-of-choice-1}, there exists a function
$f \subseteq R$ with $\dom{f} = I$.
Furthermore, for all $i \in I$, it must be $f(i) \in H(i)$ by
construction.
Then $f$ is a member of $\bigtimes_{i \in I} H(i)$.
That is, $\bigtimes_{i \in I} H(i) \neq \emptyset$.
2023-08-09 13:39:41 +00:00
\paragraph{($\Leftarrow$)}%
2023-08-09 13:39:41 +00:00
We assume the second form of the axiom of choice.
Let $R$ be an arbitrary relation.
There are two cases to consider:
2023-08-09 13:39:41 +00:00
\subparagraph{Case 1}%
2023-08-09 13:39:41 +00:00
Suppose $\ran{R} = \emptyset$.
Then $R = \emptyset$.
Thus the function $\emptyset \subseteq R$ satisfies
$\dom{\emptyset} = \dom{R}$.
2023-08-09 13:39:41 +00:00
\subparagraph{Case 2}%
2023-08-09 13:39:41 +00:00
Suppose $\ran{R} \neq \emptyset$.
Let $I = \dom{R}$ and define $H \colon I \rightarrow \{\ran{R}\}$ as
$H(i) = \ran{R}$ for all $i \in I$.
By the \nameref{ref:axiom-of-choice-2},
$\bigtimes_{i \in I} H(i) \neq \emptyset$.
By definition of the \nameref{ref:cartesian-product}, there exists some
function $f$ such that $\dom{f} = I$ and
$(\forall i \in I) f(i) \in H(i) = \ran{R}$.
Thus $\dom{f} = \dom{R}$ and $f \subseteq R$ as desired.
2023-08-09 13:39:41 +00:00
\paragraph{Conclusion}%
2023-08-09 13:39:41 +00:00
The above cases are exhaustive and yield the same conclusion: for any
relation $R$ there exists a function $f \subseteq R$ such that
$\dom{f} = \dom{R}$.
2023-06-29 20:05:08 +00:00
2023-08-09 13:39:41 +00:00
\end{proof}
2023-06-29 20:05:08 +00:00
\subsection{\verified{Exercise 3.32a}}%
\hyperlabel{sub:exercise-3.32-a}
2023-08-09 13:39:41 +00:00
Show that $R$ is symmetric iff $R^{-1} \subseteq R$.
2023-08-09 13:39:41 +00:00
\code*{Bookshelf/Enderton/Set/Chapter\_3}
{Enderton.Set.Chapter\_3.exercise\_3\_32\_a}
2023-08-09 13:39:41 +00:00
\begin{proof}
2023-08-09 13:39:41 +00:00
\paragraph{($\Rightarrow$)}%
2023-08-09 13:39:41 +00:00
Suppose $R$ is \nameref{ref:symmetric}.
Let $\tuple{x, y} \in R^{-1}$.
2023-08-09 13:39:41 +00:00
By definition of the \nameref{ref:inverse} of a set,
$\tuple{y, x} \in R$.
By symmetry, $\tuple{x, y} \in R$.
2023-08-09 13:39:41 +00:00
Thus $R^{-1} \subseteq R$.
2023-08-09 13:39:41 +00:00
\paragraph{($\Leftarrow$)}%
2023-08-09 13:39:41 +00:00
Suppose $R^{-1} \subseteq R$.
Let $\tuple{x, y} \in R$.
2023-08-09 13:39:41 +00:00
By definition of the \nameref{ref:inverse} of a set,
$\tuple{y, x} \in R^{-1}$.
Since $R^{-1} \subseteq R$, $\tuple{y, x} \in R$.
Therefore $\tuple{x, y}$ and $\tuple{y, x}$ are both in $R$.
2023-08-09 13:39:41 +00:00
In other words, $R$ is symmetric.
\end{proof}
\subsection{\verified{Exercise 3.32b}}%
\hyperlabel{sub:exercise-3.32-b}
2023-08-09 13:39:41 +00:00
Show that $R$ is transitive iff $R \circ R \subseteq R$.
2023-08-09 13:39:41 +00:00
\code*{Bookshelf/Enderton/Set/Chapter\_3}
{Enderton.Set.Chapter\_3.exercise\_3\_32\_b}
2023-08-09 13:39:41 +00:00
\begin{proof}
\paragraph{($\Rightarrow$)}%
2023-08-09 13:39:41 +00:00
Suppose $R$ is \nameref{ref:transitive}.
Let $\tuple{x, y} \in R \circ R$.
2023-08-09 13:39:41 +00:00
By definition of the \nameref{ref:composition} of a set,
there exists some $t$ such that $xRt \land tRy$.
That is, $\tuple{x, t} \in R$ and $\tuple{t, y} \in R$.
Since $R$ is transitive, it follows $\tuple{x, y} \in R$.
2023-08-09 13:39:41 +00:00
\paragraph{($\Leftarrow$)}%
2023-08-09 13:39:41 +00:00
Suppose $R \circ R \subseteq R$.
Let $\tuple{x, y} \tuple{y, z} \in R$.
2023-08-09 13:39:41 +00:00
By definition of the \nameref{ref:composition} of a set,
$$R \circ R = \{\tuple{u, v} \mid \exists t(uRt \land tRv)\}.$$
Then $\tuple{x, z} \in R \circ R$.
Since $R \circ R \subseteq R$, it follows $\tuple{x, z} \in R$.
2023-08-09 13:39:41 +00:00
Thus $R$ is transitive.
2023-08-09 13:39:41 +00:00
\end{proof}
\subsection{\verified{Exercise 3.33}}%
\hyperlabel{sub:exercise-3.33}
2023-08-09 13:39:41 +00:00
Show that $R$ is a symmetric and transitive relation iff $R = R^{-1} \circ R$.
2023-08-09 13:39:41 +00:00
\code*{Bookshelf/Enderton/Set/Chapter\_3}
{Enderton.Set.Chapter\_3.exercise\_3\_33}
2023-08-09 13:39:41 +00:00
\begin{proof}
By definition of the \nameref{ref:inverse} and \nameref{ref:composition}
of sets,
\begin{align}
R^{-1} \circ R
& = \{ (u, v) \mid \exists t(uRt \land tR^{-1}v) \}
\nonumber \\
& = \{ (u, v) \mid \exists t(uRt \land vRt) \}.
\hyperlabel{sub:exercise-3.33-eq1}
\end{align}
2023-08-09 13:39:41 +00:00
\paragraph{($\Rightarrow$)}%
Suppose $R$ is symmetric and transitive.
We now show that $R \subseteq R^{-1} \circ R$ and
$R^{-1} \circ R \subseteq R$.
\subparagraph{($\subseteq$)}%
Let $\tuple{x, y} \in R$.
Since $R$ is symmetric, $\tuple{y, x} \in R$.
Since $R$ is transitive, $\tuple{x, x} \in R$.
Then there exists a $t$ such that $\tuple{x, t} \in R$ and
$\tuple{y, t} \in R$, namely $t = x$.
2023-08-09 13:39:41 +00:00
By \eqref{sub:exercise-3.33-eq1},
$\tuple{x, y} \in R^{-1} \circ R$.
2023-08-09 13:39:41 +00:00
\subparagraph{($\supseteq$)}%
Let $\tuple{x, y} \in R^{-1} \circ R$.
2023-08-09 13:39:41 +00:00
By \eqref{sub:exercise-3.33-eq1}, there exists some $t$ such that
$\tuple{x, t} \in R$ and $\tuple{y, t} \in R$.
But $R$ is symmetric meaning $\tuple{t, y} \in R$.
Since $R$ is transitive, it follows $\tuple{x, y} \in R$.
2023-08-09 13:39:41 +00:00
\paragraph{($\Leftarrow$)}%
Suppose $R = R^{-1} \circ R$.
We prove that (i) $R$ is symmetric and (ii) $R$ is transitive.
\subparagraph{(i)}%
\hyperlabel{spar:exercise-3.33-i}
First we note that $R$ is equal to its inverse:
\begin{align}
R^{-1}
& = (R^{-1} \circ R)^{-1} \nonumber \\
& = R^{-1} \circ (R^{-1})^{-1}
& \textref{sub:theorem-3i} \nonumber \\
& = R^{-1} \circ R
& \textref{sub:theorem-3e} \nonumber \\
& = R \hyperlabel{sub:exercise-3.33-eq2}.
\end{align}
Now let $\tuple{x, y} \in R$.
By \eqref{sub:exercise-3.33-eq2} $\tuple{x, y} \in R^{-1}$.
2023-08-09 13:39:41 +00:00
By definition of the \nameref{ref:inverse} of a set,
$\tuple{y, x} \in R$.
2023-08-09 13:39:41 +00:00
Thus $R$ is symmetric.
\subparagraph{(ii)}%
Let $\tuple{x, y}, \tuple{y, z} \in R$.
2023-08-09 13:39:41 +00:00
By \nameref{spar:exercise-3.33-i}, $R$ is symmetric.
Thus $\tuple{z, y} \in R$.
2023-08-09 13:39:41 +00:00
By \eqref{sub:exercise-3.33-eq1}, it follows
$\tuple{x, z} \in R^{-1} \circ R$.
Since $R^{-1} \circ R = R$, it follows $\tuple{x, z} \in R$.
2023-08-09 13:39:41 +00:00
Thus $R$ is transitive.
\end{proof}
\subsection{\verified{Exercise 3.34}}%
\hyperlabel{sub:exercise-3.34}
2023-08-09 13:39:41 +00:00
Assume that $\mathscr{A}$ is a nonempty set, every member of which is a
transitive relation.
\begin{enumerate}[(a)]
\item Is the set $\bigcap{\mathscr{A}}$ a transitive relation?
\item Is $\bigcup{\mathscr{A}}$ a transitive relation?
\end{enumerate}
2023-08-09 13:39:41 +00:00
\code{Bookshelf/Enderton/Set/Chapter\_3}
{Enderton.Set.Chapter\_3.exercise\_3\_34\_a}
\code{Bookshelf/Enderton/Set/Chapter\_3}
{Enderton.Set.Chapter\_3.exercise\_3\_34\_b}
2023-08-09 13:39:41 +00:00
\begin{proof}
\paragraph{(a)}%
Because $\mathscr{A} \neq \emptyset$, $\bigcap{\mathscr{A}}$ is
well-defined.
We prove that $\bigcap{\mathscr{A}}$ is a transitive relation.
Let $\tuple{x, y}, \tuple{y, z} \in \bigcap{\mathscr{A}}$.
2023-08-09 13:39:41 +00:00
Then forall $A$ in $\mathscr{A}$, it follows
$\tuple{x, y}, \tuple{y, z} \in A$.
Since $A$ is transitive, it follows $\tuple{x, z} \in A$.
2023-08-09 13:39:41 +00:00
Since this holds for all $A \in \mathscr{A}$, it follows that
$\tuple{x, z} \in A$ as well.
2023-08-09 13:39:41 +00:00
Thus $\bigcap{\mathscr{A}}$ is transitive.
\paragraph{(b)}%
We show that $\bigcup{\mathscr{A}}$ is not necessarily transitive with a
counterexample.
Suppose $$\mathscr{A} = \{
\{\tuple{1, 2}, \tuple{2, 3}, \tuple{1, 3}\}, \{\tuple{2, 1}\}
2023-08-09 13:39:41 +00:00
\}.$$
Notice that the two members of $\mathscr{A}$ are transitive relations.
Now $$\bigcup{\mathscr{A}} = \{
\tuple{1, 2}, \tuple{2, 3}, \tuple{1, 3}, \tuple{2, 1},
2023-08-09 13:39:41 +00:00
\}.$$
But the above cannot be transitive, for $\tuple{1, 2}$ and $\tuple{2, 1}$
are members of the set, but $\tuple{1, 1}$ is not.
2023-08-09 13:39:41 +00:00
\end{proof}
\subsection{\verified{Exercise 3.35}}%
\hyperlabel{sub:exercise-3.35}
2023-08-09 13:39:41 +00:00
Show that for any $R$ and $x$, we have $[x]_R = \img{R}{\{x\}}$.
2023-08-09 13:39:41 +00:00
\code*{Bookshelf/Enderton/Set/Chapter\_3}
{Enderton.Set.Chapter\_3.exercise\_3\_35}
2023-08-09 13:39:41 +00:00
\begin{proof}
Let $R$ and $x$ be arbitrary sets.
Then
\begin{align*}
[x]_R
& = \{t \mid xRt\} \\
& = \{t \mid (\exists u \in \{x\})uRt\} \\
& = \img{R}{\{x\}}.
\end{align*}
\end{proof}
\subsection{\verified{Exercise 3.36}}%
\hyperlabel{sub:exercise-3.36}
2023-08-09 13:39:41 +00:00
Assume that $f \colon A \rightarrow B$ and that $R$ is an equivalence relation
on $B$.
Define $Q$ to be the set
\begin{equation}
\hyperlabel{sub:exercise-3.36-eq1}
\{\tuple{x, y} \in A \times A \mid \tuple{f(x), f(y)} \in R\}.
2023-08-09 13:39:41 +00:00
\end{equation}
Show that $Q$ is an equivalence relation on $A$.
2023-08-09 13:39:41 +00:00
\code*{Bookshelf/Enderton/Set/Chapter\_3}
{Enderton.Set.Chapter\_3.exercise\_3\_36}
2023-08-09 13:39:41 +00:00
\begin{proof}
We prove that (i) $Q$ is \nameref{ref:reflexive} on $A$, (ii) $Q$ is
\nameref{ref:symmetric}, and (iii) $Q$ is \nameref{ref:transitive}.
2023-08-09 13:39:41 +00:00
\paragraph{(i)}%
2023-08-09 13:39:41 +00:00
Let $x \in A$.
By hypothesis, $f(x) \in B$.
Since $R$ is an equivalence relation on $B$, $R$ is reflexive on $B$.
Thus $\tuple{f(x), f(x)} \in R$.
But then \eqref{sub:exercise-3.36-eq1} implies $\tuple{x, x} \in Q$.
2023-08-09 13:39:41 +00:00
Thus $Q$ is reflexive on $A$.
2023-08-09 13:39:41 +00:00
\paragraph{(ii)}%
Let $\tuple{x, y} \in Q$.
By \eqref{sub:exercise-3.36-eq1}, $\tuple{f(x), f(y)} \in R$.
2023-08-09 13:39:41 +00:00
Since $R$ is an equivalence relation on $B$, $R$ is symmetric.
Thus $\tuple{f(y), f(x)} \in R$.
But then \eqref{sub:exercise-3.36-eq1} implies $\tuple{y, x} \in Q$.
2023-08-09 13:39:41 +00:00
Thus $Q$ is symmetric.
2023-08-09 13:39:41 +00:00
\paragraph{(iii)}%
Let $\tuple{x, y}, \tuple{y, z} \in Q$.
2023-08-09 13:39:41 +00:00
By \eqref{sub:exercise-3.36-eq1},
$\tuple{f(x), f(y)}, \tuple{f(y), f(z)} \in R$.
2023-08-09 13:39:41 +00:00
Since $R$ is an equivalence relation on $B$, $R$ is transitive.
Thus $\tuple{f(x), f(z)} \in R$.
But then \eqref{sub:exercise-3.36-eq1} implies $\tuple{x, z} \in Q$.
2023-08-09 13:39:41 +00:00
Thus $Q$ is transitive.
2023-08-09 13:39:41 +00:00
\end{proof}
\subsection{\verified{Exercise 3.37}}%
\hyperlabel{sub:exercise-3.37}
2023-08-09 13:39:41 +00:00
Assume that $\Pi$ is a partition of a set $A$.
Define the relation $R_\Pi$ as follows:
\begin{equation}
\hyperlabel{sub:exercise-3.37-eq1}
xR_{\Pi}y \iff (\exists B \in \Pi)(x \in B \land y \in B).
\end{equation}
Show that $R_\Pi$ is an equivalence relation on $A$.
(This is a formalized version of the discussion at the beginning of this
section.)
2023-08-09 13:39:41 +00:00
\code*{Bookshelf/Enderton/Set/Chapter\_3}
{Enderton.Set.Chapter\_3.exercise\_3\_37}
2023-08-09 13:39:41 +00:00
\begin{proof}
We prove that (i) $R_\Pi$ is \nameref{ref:reflexive} on $B$, (ii) $R_\Pi$ is
\nameref{ref:symmetric}, and (iii) $R_\Pi$ is \nameref{ref:transitive}.
2023-08-09 13:39:41 +00:00
\paragraph{(i)}%
2023-08-09 13:39:41 +00:00
Let $x \in A$.
By definition of a \nameref{ref:partition}, there exists some nonempty set
$B \in \Pi$ such that $x \in B$.
Thus $(\exists B \in \Pi)(x \in B \land x \in B)$.
By \eqref{sub:exercise-3.37-eq1}, $\tuple{x, x} \in R_\Pi$.
2023-08-09 13:39:41 +00:00
Therefore $R_\Pi$ is reflexive on $A$.
2023-08-09 13:39:41 +00:00
\paragraph{(ii)}%
Let $\tuple{x, y} \in R_\Pi$.
2023-08-09 13:39:41 +00:00
By \eqref{sub:exercise-3.37-eq1}, there exists some $B \in \Pi$ such that
$x \in B \land y \in B$.
But then $y \in B \land x \in B$.
Thus $\tuple{y, x} \in R_\Pi$.
2023-08-09 13:39:41 +00:00
In other words, $R_\Pi$ is symmetric.
2023-08-09 13:39:41 +00:00
\paragraph{(iii)}%
Let $\tuple{x, y}, \tuple{y, z} \in R_\Pi$.
2023-08-09 13:39:41 +00:00
By \eqref{sub:exercise-3.37-eq1}, there exists some $B_1 \in \Pi$ such that
$x \in B_1 \land y \in B_1$.
Likewise there exists some $B_2 \in \Pi$ such that
$y \in B_2 \land z \in B_2$.
But $\Pi$ is a \nameref{ref:partition} meaning $y \in B_1$ and $y \in B_2$
if $B_1 = B_2$.
Therefore $x \in B_1 \land z \in B_1$ and $\tuple{x, z} \in R_\Pi$.
2023-08-09 13:39:41 +00:00
In other words, $R_\Pi$ is transitive.
2023-07-11 22:48:46 +00:00
2023-08-09 13:39:41 +00:00
\end{proof}
2023-07-11 22:48:46 +00:00
2023-08-09 13:39:41 +00:00
\subsection{\verified{Exercise 3.38}}%
\hyperlabel{sub:exercise-3.38}
2023-07-11 22:48:46 +00:00
2023-08-09 13:39:41 +00:00
\nameref{sub:theorem-3p} shows that $A / R$ is a partition of $A$ whenever $R$
is an equivalence relation on $A$.
Show that if we start with the equivalence relation $R_\Pi$ of the preceding
exercise, then the partition $A / R_\Pi$ is just $\Pi$.
2023-07-11 22:48:46 +00:00
2023-08-09 13:39:41 +00:00
\code*{Bookshelf/Enderton/Set/Chapter\_3}
{Enderton.Set.Chapter\_3.exercise\_3\_38}
2023-07-11 22:48:46 +00:00
2023-08-09 13:39:41 +00:00
\begin{proof}
By definition,
\begin{equation}
\hyperlabel{sub:exercise-3.38-eq1}
R_\Pi = \{ (x, y) \mid (\exists B \in \Pi)(x \in B \land y \in B) \}.
\end{equation}
We prove that $A / R_\Pi = \Pi$.
By the \nameref{ref:extensionality-axiom}, these two sets are equal when
$$B \in A / R_\Pi \iff B \in \Pi.$$
We prove both directions of this biconditional.
\paragraph{($\Rightarrow$)}%
Suppose $B \in A / R_\Pi$.
By \nameref{sub:exercise-3.37}, $R_\Pi$ is an equivalence class.
Then, by definition of a \nameref{ref:quotient-set},
$$A / R_\Pi = \{[x]_{R_\Pi} \mid x \in A\},$$
whose members are the \nameref{ref:equivalence-class}es.
Thus there exists some $x \in A$ such that $B = [x]_{R_\Pi}$.
By definition of a \nameref{ref:partition}, there exists a unique set
$B' \in \Pi$ containing $x$.
Thus it suffices to prove that $B = B'$, for then $B = [x]_{R_\Pi} = B'$
is a member of $\Pi$ as desired.
We proceed by extensionality again; that is, we show
$$y \in B \iff y \in B'.$$
\subparagraph{($\rightarrow$)}%
Suppose $y \in B$.
Then
\begin{align*}
y
& \in B = [x]_{R_\Pi} \\
& = \{t \mid \tuple{x, t} \in R_\Pi\} \\
2023-08-09 13:39:41 +00:00
& = \{t \mid (\exists B_1 \in \Pi)(x \in B_1 \land t \in B_1)\}.
& \eqref{sub:exercise-3.38-eq1}
\end{align*}
Thus there exists some $B_1 \in \Pi$ such that $x \in B_1$ and
$y \in B_1$.
By definition of a \nameref{ref:partition}, $B_1 \in \Pi$ is the unique
member of $\Pi$ containing $y$.
Thus $B_1 = B'$ meaning $y \in B'$ as desired.
\subparagraph{($\leftarrow$)}%
Suppose $y \in B'$.
By construction, $x \in B'$.
Then there exists a set $B_1$ such that $x \in B_1$ and $y \in B_1$,
namely $B'$.
Therefore
\begin{align*}
y
& \in \{t \mid (\exists B_1 \in \Pi)(x \in B_1 \land t \in B_1)\} \\
& = \{t \mid \tuple{x, t} \in R_\Pi\} \\
2023-08-09 13:39:41 +00:00
& = [x]_{R_\Pi} = B.
\end{align*}
\subparagraph{Conclusion}%
By the \nameref{ref:extensionality-axiom}, it follows $B = B'$.
Since $B' \in P$, it also follows $B \in P$.
\paragraph{($\Leftarrow$)}%
Let $B \in \Pi$.
By definition of a \nameref{ref:partition}, $B$ is nonempty.
Let $x \in B$.
By definition of a set, $B = \{t \mid x \in B \land t \in B\}$.
By definition of a \nameref{ref:partition}, every member of $B$ must
belong to only $B$ (i.e. no other sets in the partition).
Thus we can equivalently write
\begin{align*}
B
& = \{t \mid (\exists B_1 \in \Pi)(x \in B_1 \land t \in B_1)\} \\
& = \{ t \mid \tuple{x, t} \in R_\Pi \} \\
2023-08-09 13:39:41 +00:00
& = [x]_{R_\Pi}.
\end{align*}
Therefore $B \in A / R_{\Pi}$.
2023-08-09 13:39:41 +00:00
\end{proof}
\subsection{\verified{Exercise 3.39}}%
\hyperlabel{sub:exercise-3.39}
2023-08-09 13:39:41 +00:00
Assume that we start with an equivalence relation $R$ on $A$ and define $\Pi$
to be the partition $A / R$.
Show that $R_\Pi$, as defined in Exercise 37, is just $R$.
2023-08-09 13:39:41 +00:00
\code*{Bookshelf/Enderton/Set/Chapter\_3}
{Enderton.Set.Chapter\_3.exercise\_3\_39}
2023-08-09 13:39:41 +00:00
\begin{proof}
By definition,
\begin{equation}
\hyperlabel{sub:exercise-3.39-eq1}
R_\Pi = \{ (x, y) \mid (\exists B \in \Pi)(x \in B \land y \in B) \}.
\end{equation}
We prove that $R_\Pi = R$.
By the \nameref{ref:extensionality-axiom}, these two sets are equal when
$$(x, y) \in R_\Pi \iff (x, y) \in R.$$
We prove both directions of this biconditional.
\paragraph{($\Rightarrow$)}%
Let $(x, y) \in R_\Pi$.
By \eqref{sub:exercise-3.39-eq1}, there exists some $B \in \Pi$ such that
$x \in B$ and $y \in B$.
Since $\Pi = A / R = \{[x]_R \mid x \in A\}$, there must exist some
$z \in A$ such that $B = [z]_R$.
By definition of an \nameref{ref:equivalence-class}, $x \in [z]_R$ implies
that $zRx$ and $y \in [z]_R$ implies $zRy$.
Since $R$ is \nameref{ref:symmetric}, $xRz$.
Since $R$ is \nameref{ref:transitive}, $xRy$.
\paragraph{($\Leftarrow$)}%
Let $(x, y) \in R$.
By definition of an \nameref{ref:equivalence-class}, $x \in [x]_R$ and
$y \in [x]_R$.
Note also that $[x]_R \in A / R = \Pi$.
Thus there exists some $B \in \Pi$ such that $x \in B$ and $y \in B$,
namely $B = [x]_R$.
By \eqref{sub:exercise-3.39-eq1}, $(x, y) \in R_\Pi$.
\end{proof}
\subsection{\unverified{Exercise 3.40}}%
\hyperlabel{sub:exercise-3.40}
2023-08-09 13:39:41 +00:00
Define an equivalence relation $R$ on the set $P$ of positive integers by
$$mRn \iff m \text{ and } n \text{ have the same number of prime factors}.$$
Is there a function $f \colon P / R \rightarrow P / R$ such that
$f([n]_R) = [3n]_R$ for each $n$?
\begin{proof}
Define $g \colon P \rightarrow P$ as $g(x) = 3x$ for all $x \in P$.
We first show that $g$ is \nameref{ref:compatible} with $R$.
Let $m, n \in P$ such that $mRn$.
Then $m$ and $n$ have the same prime factors.
Then $3m$ has one additional prime factor than $m$, namely $3$.
Likewise $3n$ has one additional prime factor than $n$, also $3$.
Thus $(3m)R(3n)$, i.e. $g$ is compatible with $R$.
By \nameref{sub:theorem-3q}, it follows there exists a unique function
$f \colon P / R \rightarrow P / R$ such that
$f([n]_R) = [g(n)]_R = [3n]_R$ as expected.
\end{proof}
\subsection{\unverified{Exercise 3.41}}%
\hyperlabel{sub:exercise-3.41}
2023-08-09 13:39:41 +00:00
Let $\mathbb{R}$ be the set of real numbers and define the relation $Q$ on
$\mathbb{R} \times \mathbb{R}$ by $\tuple{u, v}Q\tuple{x, y}$ iff
2023-08-09 13:39:41 +00:00
$u + y = x + v$.
\subsubsection{\verified{Exercise 3.41a}}%
\hyperlabel{ssub:exercise-3.41-a}
2023-08-09 13:39:41 +00:00
Show that $Q$ is an equivalence relation on $\mathbb{R} \times \mathbb{R}$.
2023-08-09 13:39:41 +00:00
\code*{Bookshelf/Enderton/Set/Chapter\_3}
{Enderton.Set.Chapter\_3.exercise\_3\_41\_a}
2023-08-09 13:39:41 +00:00
\begin{proof}
We show (i) $Q$ is \nameref{ref:reflexive} on
$\mathbb{R} \times \mathbb{R}$, (ii) $Q$ is \nameref{ref:symmetric}, and
(iii) $Q$ is \nameref{ref:transitive}.
2023-08-09 13:39:41 +00:00
\paragraph{(i)}%
Let $\tuple{x, y} \in R \times R$.
Since $x + y = x + y$, it immediately follows $\tuple{x, y}Q\tuple{x, y}$.
2023-08-09 13:39:41 +00:00
Thus $Q$ is reflexive on $\mathbb{R}$.
2023-08-09 13:39:41 +00:00
\paragraph{(ii)}%
Let $\tuple{\tuple{u, v}, \tuple{x, y}} \in Q$.
2023-08-09 13:39:41 +00:00
Then $u + y = x + v$.
Likewise, $x + v = u + y$.
This immediately implies that $\tuple{\tuple{x, y}, \tuple{u, v}} \in Q$.
2023-08-09 13:39:41 +00:00
Thus $Q$ is symmetric.
2023-08-09 13:39:41 +00:00
\paragraph{(iii)}%
Let $\tuple{\tuple{u, v}, \tuple{x, y}} \in Q$ and
$\tuple{\tuple{x, y}, \tuple{a, b}} \in Q$.
2023-08-09 13:39:41 +00:00
Then $u + y = x + v$ and $x + b = a + y$.
Rearranging terms, we have $u - v = x - y$ and $x - y = a - b$.
Thus $u - v = a - b$.
Rearranging terms once more yields $u + b = a + v$.
Thus $\tuple{\tuple{u, v}, \tuple{a, b}} \in Q$.
2023-08-09 13:39:41 +00:00
Therefore $Q$ is transitive.
2023-08-09 13:39:41 +00:00
\end{proof}
\subsubsection{\unverified{Exercise 3.41b}}%
\hyperlabel{ssub:exercise-3.41-b}
2023-08-09 13:39:41 +00:00
Is there a function $G \colon (\mathbb{R} \times \mathbb{R}) / Q
\rightarrow (\mathbb{R} \times \mathbb{R}) / Q$ satisfying the equation
\begin{equation}
\hyperlabel{ssub:exercise-3.41-b-eq1}
G([\tuple{x, y}]_Q) = [\tuple{x + 2y, y + 2x}]_Q?
2023-08-09 13:39:41 +00:00
\end{equation}
2023-08-09 13:39:41 +00:00
\begin{proof}
Let $f \colon \mathbb{R} \times \mathbb{R}
\rightarrow \mathbb{R} \times \mathbb{R}$ be given by
$f(\tuple{x, y}) = \tuple{x + 2y, y + 2x}$.
2023-08-09 13:39:41 +00:00
We show (i) that $f$ is \nameref{ref:compatible} with $Q$ and then (ii)
there exists such a function satisfying \eqref{ssub:exercise-3.41-b-eq1}.
2023-08-09 13:39:41 +00:00
\paragraph{(i)}%
\hyperlabel{par:exercise-3.41-b-i}
Let $\tuple{u, v}, \tuple{x, y} \in \mathbb{R} \times \mathbb{R}$ such
that $\tuple{u, v} Q \tuple{x, y}$.
2023-08-09 13:39:41 +00:00
Thus
\begin{equation}
\hyperlabel{ssub:exercise-3.41-b-eq2}
u + y = x + v
\end{equation}
Next consider
\begin{align*}
f(\tuple{u, v}) & = \tuple{u + 2v, v + 2u}, \\
f(\tuple{x, y}) & = \tuple{x + 2y, y + 2x}.
2023-08-09 13:39:41 +00:00
\end{align*}
Then
\begin{align*}
u + y & = x + v \\
\iff 3u + 3y & = 3x + 3v \\
\iff (u + y) + (2u + 2y) & = (x + v) + (2x + 2v) \\
\iff (x + v) + (2u + 2y) & = (u + y) + (2x + 2v)
& \eqref{ssub:exercise-3.41-b-eq2} \\
\iff (x + 2y) + (v + 2u) & = (u + 2v) + (y + 2x) \\
\iff (u + 2v) + (y + 2x) & = (x + 2y) + (v + 2u).
\end{align*}
This last equality shows $f(\tuple{u, v}) \,Q\, f(\tuple{x, y})$.
2023-08-09 13:39:41 +00:00
Thus $f$ is compatible with $Q$.
2023-08-09 13:39:41 +00:00
\paragraph{(ii)}%
2023-08-09 13:39:41 +00:00
By \nameref{par:exercise-3.41-b-i} and \nameref{sub:theorem-3q}, there
exists a unique $G \colon (\mathbb{R} \times \mathbb{R}) / Q \rightarrow
(\mathbb{R} \times \mathbb{R}) / Q$ satisfying
\eqref{ssub:exercise-3.41-b-eq1}.
2023-08-09 13:39:41 +00:00
\end{proof}
\subsection{\unverified{Exercise 3.42}}%
\hyperlabel{sub:exercise-3.42}
State precisely the "analogous results" mentioned in \nameref{sub:theorem-3q}.
2023-08-09 13:39:41 +00:00
(This will require extending the concept of compatibility in a suitable way.)
\linedivider
\begin{theorem}[3.42]
A function $F \colon A \times A \rightarrow A$ is said to be
\textbf{compatible} with relation $R$ if and only if for all
$x_1, y_1, x_2, y_2 \in A$,
$$x_1Rx_2 \land y_1Ry_2 \Rightarrow F(x_1, y_1)RF(x_2, y_2).$$
\noindent
Assume that $R$ is an equivalence relation on $A$ and that
$F \colon A \times A \rightarrow A$.
If $F$ is compatible with $R$, then there exists a unique function
$\hat{F} \colon A / R \times A / R \rightarrow A / R$ such that
\begin{equation}
\hyperlabel{sub:exercise-3.42-eq1}
\hat{F}([x]_R, [y]_R) = [F(x, y)]_R
\end{equation}
for all $x, y \in A$.
If $F$ is not compatible with $R$, then no such $\hat{F}$ exists.
\end{theorem}
2023-08-09 13:39:41 +00:00
\begin{proof}
Let $R$ be an equivalence relation on $A$ and
$F \colon A \times A \rightarrow A$ be compatible with $R$.
Define relation $\hat{F}$ to be
$$\hat{F} = \{\tuple{[x]_R, [y]_R, [F(x, y)]_R} \mid x, y \in A\}.$$
By construction, $\dom{\hat{F}} = A / R \times A / R$ and
$\ran{\hat{F}} \subseteq A / R$.
All that remains is proving (i) $\hat{F}$ is single-valued and (ii)
$\hat{F}$ is unique.
\paragraph{(i)}%
Let $[x_1]_R, [y_1]_R, [x_2]_R, [y_2]_R \in \dom{\hat{F}}$ such that
\begin{equation}
\hyperlabel{par:theorem-3q-i-eq1}
\tuple{[x_1]_R, [y_1]_R} = \tuple{[x_2]_R, [y_2]_R}.
\end{equation}
By definition of $\hat{F}$,
\begin{align*}
\tuple{[x_1]_R, [y_1]_R, [F(x_1, y_1)]_R} & \in \hat{F} \\
\tuple{[x_2]_R, [y_2]_R, [F(x_2, y_2)]_R} & \in \hat{F}.
\end{align*}
By \eqref{par:theorem-3q-i-eq1}, $[x_1]_R = [x_2]_R$ and
$[y_1]_R = [y_2]_R$.
Then \nameref{sub:lemma-3n} implies $x_1Rx_2$ and $y_1Ry_2$ respectively.
Since $F$ is compatible, it follows $F(x_1, y_1)RF(x_2, y_2)$.
Another application of \nameref{sub:lemma-3n} implies that
$[F(x_1, y_1)]_R = [F(x_2, y_2)]_R$.
Thus $\hat{F}$ is single-valued, i.e. a function.
\paragraph{(ii)}%
Suppose there exists another function, say $\hat{G}$, that satisfies
\eqref{sub:exercise-3.42-eq1}.
That is,
$$\hat{G}([x]_R, [y]_R) = [F(x, y)]_R \quad\text{for all } x, y \in A.$$
Let $x, y \in A$.
Then $\hat{G}([x]_R, [y]_R) = [F(x, y)]_R$ and
$\hat{F}([x]_R, [y]_R) = [F(x, y)]_R$.
Since this holds for all $x, y \in A$, $\hat{F}$ and $\hat{G}$ agree on
all members of $A / R \times A / R$.
Hence, by the \nameref{ref:extensionality-axiom}, $\hat{F} = \hat{G}$.
\suitdivider
\noindent
Suppose $F$ is not compatible with $R$.
Then there exists some $x_1, y_1, x_2, y_2 \in A$ such that $x_1Rx_2$ and
$y_1Ry_2$ but $\neg F(x_1, y_1)RF(x_2, y_2)$.
By \nameref{sub:lemma-3n}, $[x_1]_R = [x_2]_R$ and $[y_1]_R = [y_2]_R$.
For the sake of contradiction, suppose a function $\hat{F}$ exists
satisfying \eqref{sub:exercise-3.42-eq1}.
Then $\hat{F}([x_1]_R, [y_1]_R) = \hat{F}([x_2]_R, [y_2]_R)$ meaning
$[F(x_1, y_1)]_R = [F(x_2, y_2)]_R$.
Then \nameref{sub:lemma-3n} implies $F(x_1, y_1)RF(x_2, y_2)$, a
contradiction.
Therefore our original hypothesis must be incorrect.
That is, there is no incompatible function $\hat{F}$ satisfying
\eqref{sub:exercise-3.42-eq1}.
2023-08-09 13:39:41 +00:00
\end{proof}
\subsection{\verified{Exercise 3.43}}%
\hyperlabel{sub:exercise-3.43}
2023-08-09 13:39:41 +00:00
Assume that $R$ is a linear ordering on a set $A$.
Show that $R^{-1}$ is also a linear ordering on $A$.
2023-08-09 13:39:41 +00:00
\code*{Bookshelf/Enderton/Set/Chapter\_3}
{Enderton.Set.Chapter\_3.exercise\_3\_43}
2023-08-09 13:39:41 +00:00
\begin{proof}
2023-08-09 13:39:41 +00:00
Assume that $R$ is a \nameref{ref:linear-ordering} on a set $A$.
Then $R$ is \nameref{ref:transitive} and \nameref{ref:trichotomous}.
We show that (i) $R^{-1}$ is transitive and (ii) $R^{-1}$ is trichotomous.
2023-08-09 13:39:41 +00:00
\paragraph{(i)}%
\hyperlabel{par:exercise-3.43-i}
Let $\tuple{x, y}, \tuple{y, z} \in R^{-1}$.
2023-08-09 13:39:41 +00:00
By definition of the \nameref{ref:inverse} of a set,
$\tuple{y, x}$, $\tuple{z, y} \in R$.
Since $R$ is transitive, it must be that $\tuple{z, x} \in R$.
Then $\tuple{x, z} \in R^{-1}$.
2023-08-09 13:39:41 +00:00
Thus $R^{-1}$ is transitive.
\paragraph{(ii)}%
\hyperlabel{par:exercise-3.43-ii}
Let $x, y \in A$.
Since $R$ is trichotomous on $A$, it follows that exactly one of the
following conditions hold: $$xRy, \quad x = y, \quad yRx.$$
By definition of the \nameref{ref:inverse} of a set, the above
possibilities are equivalently expressed as
$$yR^{-1}x, \quad x = y, \quad xR^{-1}y.$$
Thus $R^{-1}$ is trichotomous.
2023-08-09 13:39:41 +00:00
\paragraph{Conclusion}%
2023-08-09 13:39:41 +00:00
Since $R^{-1}$ is transitive by \nameref{par:exercise-3.43-i} and
trichotomous by \nameref{par:exercise-3.43-ii}, it follows $R^{-1}$ is a
linear ordering on $A$.
2023-08-09 13:39:41 +00:00
\end{proof}
\subsection{\verified{Exercise 3.44}}%
\hyperlabel{sub:exercise-3.44}
2023-08-09 13:39:41 +00:00
Assume that $<$ is a linear ordering on a set $A$.
Assume that $f \colon A \rightarrow A$ and that $f$ has the property that
whenever $x < y$, then $f(x) < f(y)$.
Show that $f$ is one-to-one and that whenever $f(x) < f(y)$, then $x < y$.
\code*{Bookshelf/Enderton/Set/Chapter\_3}
{Enderton.Set.Chapter\_3.exercise\_3\_44\_i}
\code{Bookshelf/Enderton/Set/Chapter\_3}
{Enderton.Set.Chapter\_3.exercise\_3\_44\_ii}
2023-08-09 13:39:41 +00:00
\begin{proof}
2023-08-09 13:39:41 +00:00
We show that (i) $f$ is one-to-one and (ii) whenever $f(x) < f(y)$, then
$x < y$.
2023-08-09 13:39:41 +00:00
\paragraph{(i)}%
2023-08-09 13:39:41 +00:00
Let $y \in \ran{f}$.
By definition of the \nameref{ref:range} of a set, there exists some
$x_1 \in A$ such that $f(x_1) = y$.
Suppose there exists some $x_2 \in A$ such that $f(x_2) = y$.
We prove $f$ is one-to-one by showing $x_1 = x_2$.
Because $<$ is a linear ordering on $A$, there exist three cases to
consider:
2023-08-09 13:39:41 +00:00
\subparagraph{Case 1}%
2023-08-09 13:39:41 +00:00
Assume $x_1 < x_2$.
By hypothesis, $f$ is monotonic.
Thus $f(x_1) < f(x_2)$.
But $<$ is a trichotomous relation meaning it is not possible for
\textit{both} $f(x) = f(y)$ and $f(x) < f(y)$.
Thus our original assumption must be wrong.
2023-08-09 13:39:41 +00:00
\subparagraph{Case 2}%
2023-08-09 13:39:41 +00:00
Assume $x_1 = x_2$.
Then we are immediately finished.
2023-08-09 13:39:41 +00:00
\subparagraph{Case 3}%
2023-08-09 13:39:41 +00:00
Assume $x_1 > x_2$.
By hypothesis, $f$ is monotonic.
Thus $f(x_1) > f(x_2)$.
But $<$ is a trichotomous relation meaning it is not possible for
\textit{both} $f(x) = f(y)$ and $f(x) > f(y)$.
Thus our original assumption must be wrong.
2023-08-09 13:39:41 +00:00
\subparagraph{Conclusion}%
2023-08-09 13:39:41 +00:00
Since the above cases are exhaustive, the only possibility is
$x_1 = x_2$.
Thus $f$ is one-to-one.
2023-08-09 13:39:41 +00:00
\paragraph{(ii)}%
2023-08-09 13:39:41 +00:00
Suppose $f(x) < f(y)$.
There are three cases to consider:
\subparagraph{Case 1}%
2023-08-09 13:39:41 +00:00
Assume $x < y$.
Then we are immediately finished.
2023-08-09 13:39:41 +00:00
\subparagraph{Case 2}%
2023-08-09 13:39:41 +00:00
Assume $x = y$.
Then $f(x) = f(y)$.
But $<$ is a trichotomous relation meaning it is not possible for
\textit{both} $f(x) = f(y)$ and $f(x) < f(y)$.
Thus our original assumption must be wrong.
2023-08-09 13:39:41 +00:00
\subparagraph{Case 3}%
2023-08-09 13:39:41 +00:00
Assume $x > y$.
By hypothesis, $f$ is monotonic.
Thus $f(x) > f(y)$.
But $<$ is a trichotomous relation meaning it is not possible for
\textit{both} $f(x) < f(y)$ and $f(x) > f(y)$.
Thus our original assumption must be wrong.
2023-08-09 13:39:41 +00:00
\subparagraph{Conclusion}%
2023-08-09 13:39:41 +00:00
Since the above cases are exhaustive, the only possibility is $x < y$.
2023-08-09 13:39:41 +00:00
\end{proof}
\subsection{\verified{Exercise 3.45}}%
\hyperlabel{sub:exercise-3.45}
2023-08-09 13:39:41 +00:00
Assume that $<_A$ and $<_B$ are linear orderings on $A$ and $B$, respectively.
Define the binary relation $<_L$ on the Cartesian product $A \times B$ by:
$$\tuple{a_1, b_1} <_L \tuple{a_2, b_2} \quad\text{iff}\quad
2023-08-09 13:39:41 +00:00
\text{either } a_1 <_A a_2 \text{ or } (a_1 = a_2 \land b_1 <_B b_2).$$
Show that $<_L$ is a linear ordering on $A \times B$.
(The relation $<_L$ is called \textit{lexicographic} ordering, being the
ordering used in making dictionaries.)
2023-08-09 13:39:41 +00:00
\code*{Bookshelf/Enderton/Set/Chapter\_3}
{Enderton.Set.Chapter\_3.exercise\_3\_45}
2023-08-09 13:39:41 +00:00
\begin{proof}
We show that $<_L$ is (i) \nameref{ref:transitive} and (ii)
\nameref{ref:trichotomous} on $A \times B$.
2023-08-09 13:39:41 +00:00
\paragraph{(i)}%
Let $\tuple{a_1, b_1} <_L \tuple{a_2, b_2}$ and
$\tuple{a_2, b_2} <_L \tuple{a_3, b_3}$.
2023-08-09 13:39:41 +00:00
Then either $a_1 <_A a_2$ or $a_1 = a_2 \land b_1 <_B b_2$.
Likewise, either $a_2 <_A a_3$ or $a_2 = a_3 \land b_2 <_B b_3$.
We consider each combination of cases in turn:
2023-08-09 13:39:41 +00:00
\subparagraph{Case 1}%
2023-08-09 13:39:41 +00:00
Suppose $a_1 <_A a_2$ and $a_2 <_A a_3$.
Since $<_A$ is a linear ordering, it follows $<_A$ is transitive.
Thus $a_1 <_A a_3$.
Therefore $\tuple{a_1, b_2} <_L \tuple{a_3, b_3}$.
2023-08-09 13:39:41 +00:00
\subparagraph{Case 2}%
2023-08-09 13:39:41 +00:00
Suppose $a_1 <_A a_2$, $a_2 = a_3$, and $b_2 <_B b_3$.
Then $a_1 < a_3$.
Therefore $\tuple{a_1, b_2} <_L \tuple{a_3, b_3}$.
2023-08-09 13:39:41 +00:00
\subparagraph{Case 3}%
2023-08-09 13:39:41 +00:00
Suppose $a_1 = a_2$, $b_1 <_B b_2$, and $a_2 <_A a_3$.
Then $a_1 <_A a_3$.
Therefore $\tuple{a_1, b_2} <_L \tuple{a_3, b_3}$.
2023-08-09 13:39:41 +00:00
\subparagraph{Case 4}%
2023-08-09 13:39:41 +00:00
Suppose $a_1 = a_2$, $b_1 <_B b_2$, $a_2 = a_3$, and $b_2 <_B b_3$.
Then $a_1 = a_3$.
Since $<_B$ is a linear ordering, it follows $<_B$ is transitive.
Thus $b_1 <_B b_3$.
Therefore $\tuple{a_1, b_2} <_L \tuple{a_3, b_3}$.
2023-08-09 13:39:41 +00:00
\subparagraph{Conclusion}%
2023-08-09 13:39:41 +00:00
These four cases are exhaustive and each conclude that $<_L$ is
transitive.
2023-08-09 13:39:41 +00:00
\paragraph{(ii)}%
Let $\tuple{a_1, b_1}, \tuple{a_2, b_2} \in A \times B$.
2023-08-09 13:39:41 +00:00
Because $<_A$ and $<_B$ are linear orderings on $A$ and $B$ respectively,
it follows $<_A$ and $<_B$ are both trichotomous on their respective
sets.
Thus exactly one of $$a_1 <_A a_2, \quad a_1 = a_2, \quad a_2 <_A a_1$$
and $$b_1 <_B b_2, \quad b_1 = b_2, \quad b_2 <_B b_1$$ holds.
There are three cases we examine:
2023-08-09 13:39:41 +00:00
\subparagraph{Case 1}%
\hyperlabel{spar:exercise-3.45-ii-case-1}
2023-08-09 13:39:41 +00:00
Suppose $a_1 <_A a_2$.
Then $\tuple{a_1, b_1} <_L \tuple{a_2, b_2}$.
2023-08-09 13:39:41 +00:00
This is trivially the only possible relationship between the ordered
pairs.
2023-08-09 13:39:41 +00:00
\subparagraph{Case 2}%
2023-08-09 13:39:41 +00:00
Suppose $a_1 = a_2$.
If $b_1 <_B b_2$, then $\tuple{a_1, b_1} <_L \tuple{a_2, b_2}$ is the
only possibility.
If $b_1 = b_2$, then $\tuple{a_1, b_1} = \tuple{a_2, b_2}$ is the only
2023-08-09 13:39:41 +00:00
possibility.
If $b_2 <_B b_1$, then $\tuple{a_2, b_2} <_L \tuple{a_1, b_1}$ is the
only possibility.
2023-08-09 13:39:41 +00:00
\subparagraph{Case 3}%
2023-08-09 13:39:41 +00:00
Suppose $a_2 <_A a_1$.
This case is analagous to \eqref{spar:exercise-3.45-ii-case-1}.
2023-08-09 13:39:41 +00:00
\subparagraph{Conclusion}%
2023-08-09 13:39:41 +00:00
In each of the above cases, we are always left with exactly one of
$$\tuple{a_1, b_1} <_L \tuple{a_2, b_2}, \quad
\tuple{a_1, b_1} = \tuple{a_2, b_2}, \quad
\tuple{a_2, b_2} <_L \tuple{a_1, b_1}.$$
2023-08-09 13:39:41 +00:00
Thus $<_L$ is trichotomous.
2023-08-09 13:39:41 +00:00
\end{proof}
\chapter{Natural Numbers}%
\hyperlabel{chap:natural-numbers}
\section{Inductive Sets}%
\hyperlabel{sec:inductive-sets}
\subsection{\unverified{Theorem 4A}}%
\hyperlabel{sub:theorem-4a}
2023-08-09 13:39:41 +00:00
\begin{theorem}[4A]
There is a set whose members are exactly the natural numbers.
\end{theorem}
\begin{proof}
By the \nameref{ref:infinity-axiom}, there exists an
\nameref{ref:inductive-set} $A$.
By the \nameref{ref:subset-axioms}, there exists a set $B$ such that
$$x \in B \iff x \in A \land \left[\forall C,
(\emptyset \in C \land
(\forall c \in C) c^+ \in C) \Rightarrow x \in C\right].$$
In other words, $x \in B$ if and only if $x \in A$ and $x$ is a natural
number.
Thus $B$ is the set whose members are exactly the natural numbers.
\end{proof}
\subsection{\unverified{Theorem 4B}}%
\hyperlabel{sub:theorem-4b}
2023-08-09 13:39:41 +00:00
\begin{theorem}[4B]
$\omega$ is inductive, and is a subset of every other inductive set.
\end{theorem}
2023-08-09 13:39:41 +00:00
\begin{proof}
2023-08-09 13:39:41 +00:00
$\omega$ denotes the set of \nameref{ref:natural-number}s.
We show $\omega$ is an \nameref{ref:inductive-set} by proving (i)
$\emptyset \in \omega$ and (ii) $\omega$ is closed under
\nameref{ref:successor}.
2023-08-09 13:39:41 +00:00
\paragraph{(i)}%
\hyperlabel{par:theorem-4b-i}
2023-08-09 13:39:41 +00:00
By definition, $\emptyset$ is a member of every inductive set.
Thus $\emptyset$ is a natural number, i.e. a member of $\omega$.
2023-08-09 13:39:41 +00:00
\paragraph{(ii)}%
\hyperlabel{par:theorem-4b-ii}
2023-08-09 13:39:41 +00:00
Let $n \in \omega$.
That is, let $n$ be a natural number.
By definition, $n$ is a member of every inductive set.
By definition of an inductive set, $n^+$ is then a member of every
inductive set as well.
Thus $n^+$ is a natural number, i.e. $n^+ \in \omega$.
2023-08-09 13:39:41 +00:00
\paragraph{Conclusion}%
2023-08-09 13:39:41 +00:00
By \nameref{par:theorem-4b-i} and \nameref{par:theorem-4b-ii}, it follows
$\omega$ is inductive.
It follows immediately from the definition of a natural number that
$\omega$ is a subset of every other inductive set.
2023-08-09 13:39:41 +00:00
\end{proof}
\subsection{\verified{Theorem 4C}}%
\hyperlabel{sub:theorem-4c}
2023-08-09 13:39:41 +00:00
\begin{theorem}[4C]
Every natural number except $0$ is the successor of some natural number.
\end{theorem}
\code{Bookshelf/Enderton/Set/Chapter\_4}
{Enderton.Set.Chapter\_4.theorem\_4c}
2023-08-09 13:39:41 +00:00
\begin{proof}
Let $T = \{n \mid n = 0 \lor (\exists m) n = m^+\}$.
It trivially follows that $\emptyset \in T$.
Let $x \in T$.
Then $x^+ \in T$ since $(\exists m) x^+ = m^+$, namely $m = x$.
Therefore $T$ is inductive.
By \nameref{sub:theorem-4b}, $\omega$ is a subset of $T$.
Thus every natural number satisfies the condition written in $T$'s
definition.
In other words, every natural number except $0$ is the successor of some
natural number.
\end{proof}
2023-07-21 19:40:38 +00:00
\section{Peano's Postulates}%
\hyperlabel{sec:peanos-postulates}
\subsection{\unverified{Theorem 4E}}%
\hyperlabel{sub:theorem-4e}
2023-08-09 13:39:41 +00:00
\begin{theorem}[4E]
For a transitive set $a$, $$\bigcup \left(a^+\right) = a.$$
\end{theorem}
2023-08-09 13:39:41 +00:00
\begin{proof}
2023-08-09 13:39:41 +00:00
Let $a$ be a \nameref{ref:transitive-set}.
We show that
\begin{equation}
\hyperlabel{sub:theorem-4e-eq1}
x \in \bigcup \left(a^+\right) \iff x \in a.
\end{equation}
2023-08-09 13:39:41 +00:00
\paragraph{($\Rightarrow$)}%
2023-08-09 13:39:41 +00:00
Suppose $x \in \bigcup \left(a^+\right)$.
By definition of \nameref{ref:successor},
$x \in \bigcup \left(a \cup \{a\}\right)$.
Then there exists some $b \in a \cup \{a\}$ such that $x \in b$.
There are two cases to consider:
2023-08-09 13:39:41 +00:00
\subparagraph{Case 1}%
2023-08-09 13:39:41 +00:00
Suppose $b \in a$.
By definition of a transitive set, $x \in b \in a$ means $x \in a$.
2023-08-09 13:39:41 +00:00
\subparagraph{Case 2}%
2023-08-09 13:39:41 +00:00
Suppose $b \in \{a\}$.
Then $b = a$ and immediately $x \in b = a$.
2023-08-09 13:39:41 +00:00
\paragraph{($\Leftarrow$)}%
2023-08-09 13:39:41 +00:00
Suppose $x \in a$.
Then immediately $x \in a \cup \{a\}$.
Thus there exists some $b$ such that $b \in a \cup \{a\}$ and $x \in b$,
namely $b = \{a\}$.
Thus $x \in \bigcup \left(a^+\right)$.
2023-08-09 13:39:41 +00:00
\paragraph{Conclusion}%
2023-08-09 13:39:41 +00:00
We have shown both sides of \eqref{sub:theorem-4e-eq1} holds.
By the \nameref{ref:extensionality-axiom}, $\bigcup \left(a^+\right) = a$.
2023-08-09 13:39:41 +00:00
\end{proof}
\subsection{\unverified{Theorem 4F}}%
\hyperlabel{sub:theorem-4f}
2023-08-09 13:39:41 +00:00
\begin{theorem}[4F]
Every natural number is a transitive set.
\end{theorem}
2023-08-09 13:39:41 +00:00
\begin{proof}
2023-08-09 13:39:41 +00:00
Let $T = \{n \in \omega \mid n \text{ is a transitive set}\}$.
We (i) prove that $T$ is an \nameref{ref:inductive-set} and then (ii) every
natural number is a transitive set.
2023-08-09 13:39:41 +00:00
\paragraph{(i)}%
\hyperlabel{par:theorem-4f-i}
2023-08-09 13:39:41 +00:00
First, $\emptyset \in T$ since it vacuously holds that a member of a
member of $\emptyset$ is itself a member of $\emptyset$.
Next, let $n \in T$ and consider whether $n^+ \in T$.
Since $n$ is a transitive set, \nameref{sub:theorem-4e} implies
$\bigcup \left(n^+\right) = n$.
But $n \subseteq n^+ = n \cup \{n\}$.
Thus $\bigcup \left(n^+\right) \subseteq n+$, i.e. $n^+$ is a transitive
set.
Therefore $n^+ \in T$.
Hence $T$ is inductive.
2023-08-09 13:39:41 +00:00
\paragraph{(ii)}%
2023-08-09 13:39:41 +00:00
Notice $T \subseteq \omega$.
By \nameref{par:theorem-4f-i} and \nameref{sub:theorem-4b}, $T = \omega$.
Thus every natural number is a transitive set.
2023-08-09 13:39:41 +00:00
\end{proof}
\subsection{\verified{Theorem 4D}}%
\hyperlabel{sub:theorem-4d}
2023-08-09 13:39:41 +00:00
\begin{theorem}[4D]
$\langle \omega, \sigma, 0 \rangle$ is a Peano system.
\end{theorem}
\code{Common/Set/Peano}{Peano.instSystemNatUnivSuccOfNatInstOfNatNat}
2023-08-09 13:39:41 +00:00
\begin{note}
This theorem depends on \nameref{sub:theorem-4e} and
\nameref{sub:theorem-4f}.
\end{note}
2023-08-09 13:39:41 +00:00
\begin{proof}
Note $\sigma$ is defined as $\sigma = \{\tuple{n, n^+} \mid n \in \omega\}$.
2023-08-09 13:39:41 +00:00
To prove $\langle \omega, \sigma, 0 \rangle$ is a
\nameref{ref:peano-system}, we must show that (i) $0 \not\in \ran{S}$,
(ii) $\sigma$ is one-to-one, and (iii) every subset $A$ of $\omega$
containing $0$ and closed under $\sigma$ is $\omega$ itself.
2023-08-09 13:39:41 +00:00
\paragraph{(i)}%
2023-08-09 13:39:41 +00:00
This follows immediately from \nameref{sub:theorem-4c}.
2023-08-09 13:39:41 +00:00
\paragraph{(ii)}%
2023-08-09 13:39:41 +00:00
Let $m, n \in \omega$ and suppose $m^+ = n^+$.
Then $\bigcup \left(m^+\right) = \bigcup \left(n^+\right)$.
By \nameref{sub:theorem-4f}, every natural number is a
\nameref{ref:transitive-set}.
Therefore, by \nameref{sub:theorem-4e},
$$\bigcup \left(m^+\right) = m = \bigcup \left(n^+\right) = n.$$
2023-08-09 13:39:41 +00:00
\paragraph{(iii)}%
2023-08-09 13:39:41 +00:00
This follows immediately from \nameref{sub:theorem-4b}.
2023-08-09 13:39:41 +00:00
\end{proof}
\subsection{\unverified{Theorem 4G}}%
\hyperlabel{sub:theorem-4g}
2023-08-09 13:39:41 +00:00
\begin{theorem}[4G]
The set $\omega$ is a transitive set.
\end{theorem}
2023-08-09 13:39:41 +00:00
\begin{proof}
2023-08-09 13:39:41 +00:00
Let $T = \{n \in \omega \mid \forall t \in n, t \in \omega\}$.
We prove that (i) $T$ is an \nameref{ref:inductive-set} and then (ii) every
member of a natural number is itself a natural number.
2023-08-09 13:39:41 +00:00
\paragraph{(i)}%
\hyperlabel{par:theorem-4g-i}
2023-08-09 13:39:41 +00:00
First, it vacuously holds that $\emptyset \in T$.
Next, let $n \in T$.
We must prove that $n^+ \in T$ as well.
By definition of the \nameref{ref:successor}, $n^+ = n \cup \{n\}$.
That is, either $n^+ = n$ or $n^+ = \{n\}$.
If the former, then every member of $n^+$ must be a natural number since
this already holds for $n$.
If the latter, the only member of $n^+$ is $n$ which is, by definition of
$T$, a natural number.
Thus $n^+ \in T$.
We conclude that $T$ is an inductive set.
2023-08-09 13:39:41 +00:00
\paragraph{(ii)}%
2023-08-09 13:39:41 +00:00
Since $T \subseteq \omega$, \nameref{par:theorem-4g-i} and
\nameref{sub:theorem-4b} implies $T = \omega$.
Thus the member of every natural number is itself a natural number.
In other words, $\bigcup \omega \subseteq \omega$.
Therefore $\omega$ is indeed a \nameref{ref:transitive-set}.
2023-08-09 13:39:41 +00:00
\end{proof}
\section{Recursion on \texorpdfstring{$\omega$}{the Natural Numbers}}%
\hyperlabel{sec:recursion-natural-numbers}
\subsection{\unverified{%
Recursion Theorem on \texorpdfstring{$\omega$}{the Natural Numbers}}}%
\hyperlabel{sub:recursion-theorem-natural-numbers}
2023-08-09 13:39:41 +00:00
\begin{theorem}
Let $A$ be a set, $a \in A$, and $F \colon A \rightarrow A$.
Then there exists a unique function $h \colon \omega \rightarrow A$ such
that $$h(0) = a,$$ and for every $n \in \omega$, $$h(n^+) = F(h(n)).$$
\end{theorem}
2023-08-09 13:39:41 +00:00
\begin{note}
This proof was written a few days after reading Enderton's proof as a means
of ensuring I remember the main arguments.
\end{note}
2023-08-09 13:39:41 +00:00
\begin{proof}
2023-08-09 13:39:41 +00:00
Define set
\begin{align*}
H = \{ v \mid & v \text{ is a function with } \\
& \text{(a) } \dom{v} \subseteq \omega, \\
& \text{(b) } \ran{v} \subseteq A, \\
& \text{(c) if } 0 \in \dom{v}, \text{ then } v(0) = a, \text{ and} \\
& \text{(d) if } n^+ \in \dom{v},
\text{ then } n \in \dom{v} \text{ and } v(n^+) = F(v(n))
\}.
\end{align*}
Define a function satisfying properties (a)-(d) above as
\textit{acceptable}.
That is, $H$ is the set of all acceptable functions.
Define $h = \bigcup H$.
We prove that (i) $h$ is a \nameref{ref:function}, (ii) $h \in H$, (iii)
$\dom{h} = \omega$, and (iv) $h$ is unique.
\paragraph{(i)}%
\hyperlabel{par:recursion-theorem-natural-numbers-i}
We prove that $h$ is a function.
Consider set
$$S = \{x \in \omega \mid h(x) = y \text{ for at most one value } y\}.$$
We show (1) that $0 \in S$ and (2) if $n \in S$ then $n^+ \in S$.
\subparagraph{(1)}%
\hyperlabel{spar:recursion-theorem-natural-numbers-i-1}
Suppose $0 \in \dom{h}$.
By construction, there must exist some $y_1 \in A$ and acceptable
function $v_1$ such that $v_1(0) = y_1$.
Suppose there also exists a $y_2 \in A$ and acceptable function $v_2$
such that $v_2(0) = y_2$.
By property (c), $v_1(0) = a$ and $v_2(0) = a$.
Thus $y_1 = a = y_2$ and $h(0) = a$.
Therefore $0 \in S$.
\subparagraph{(2)}%
\hyperlabel{spar:recursion-theorem-natural-numbers-i-2}
Suppose $n$ and $n^+$ are members of $\dom{h}$.
By construction, there must exist some $y_1 \in A$ and acceptable
function $v_1$ such that $v_1(n^+) = y_1$.
Suppose there also exists a $y_2 \in A$ and acceptable function $v_2$
such that $v_2(n^+) = y_2$.
By property (d), it follows $n \in \dom{v_1}$, $n \in \dom{v_2}$,
$v_1(n^+) = F(v_1(n))$, and $v_2(n^+) = F(v_2(n))$.
But $n \in S$ meaning there is at most one value $y$ such that
$v_1(n) = y = v_2(n)$.
Thus $F(v_1(n)) = F(y) = F(v_2(n))$ and $h(n^+) = F(y)$.
Therefore $n^+ \in S$.
\subparagraph{Subconclusion}%
2023-08-09 13:39:41 +00:00
By \nameref{spar:recursion-theorem-natural-numbers-i-1} and
\nameref{spar:recursion-theorem-natural-numbers-i-2}, $S$ is an
\nameref{ref:inductive-set}.
By \nameref{sub:theorem-4b}, $S = \omega$.
Since $S = \omega$, it follows $h$ has at most one value for every
$x \in \omega$.
In other words, $h$ is a function.
\paragraph{(ii)}%
\hyperlabel{par:recursion-theorem-natural-numbers-ii}
We now prove $h \in H$, i.e. $h$ is an acceptable function.
It trivially holds that $\dom{h} \subseteq \omega$ and
$\ran{h} \subseteq A$.
Thus we are left with proving properties (c) and (d).
\subparagraph{(c)}%
Note $\{\tuple{0, a}\}$ is an acceptable function.
Thus $\tuple{0, a} \in h$.
2023-08-09 13:39:41 +00:00
By \nameref{par:recursion-theorem-natural-numbers-i}, $h$ is a function.
Therefore $a$ is the only value $h(0)$ takes on.
\subparagraph{(d)}%
Suppose $n^+ \in \dom{h}$.
Then there exists some acceptable function $v$ such that
$v(n^+) = h(n^+)$.
By definition of acceptable, $\tuple{n, v(n)} \in v$.
2023-08-09 13:39:41 +00:00
Since \nameref{par:recursion-theorem-natural-numbers-i} indicates $h$ is
a function, $n \in \dom{h}$ and $h(n) = v(n)$.
Also by definition of acceptable, $v(n^+) = F(v(n))$.
Therefore $$h(n^+) = v(n^+) = F(v(n)) = F(h(n)).$$
Hence $h \in H$.
\paragraph{(iii)}%
\hyperlabel{par:recursion-theorem-natural-numbers-iii}
We now prove that $\dom{h} = \omega$.
We show that (1) $0 \in \dom{h}$ and (2) if $n \in \dom{h}$ then
$n^+ \in \dom{h}$.
\subparagraph{(1)}%
\hyperlabel{spar:recursion-theorem-natural-numbers-iii-1}
We note that $\{\tuple{0, a}\}$ is an acceptable function.
2023-08-09 13:39:41 +00:00
By construction of $h$, $0 \in \dom{h}$.
\subparagraph{(2)}%
\hyperlabel{spar:recursion-theorem-natural-numbers-iii-2}
Suppose $n \in \dom{h}$.
Since $n \in \dom{h}$ there exists an acceptable function $v$ with
$n \in \dom{v}$.
Define $$v' = v \cup \{\tuple{n^+, F(v(n))}\}.$$
2023-08-09 13:39:41 +00:00
We prove that $v'$ is acceptable:
\begin{enumerate}[(a)]
\item It trivially holds that $\dom{v'} \subseteq \omega$.
\item It trivially holds that $\ran{v'} \subseteq A$.
\item If $0 \in \dom{v'}$, then $v'(0) = v(0) = a$.
\item
Suppose $m^+ \in \dom{v'}$ for some $m \in \omega$.
If $m^+ = n^+$, then $m = n$ and $v'(m^+) = F(v(m))$ by
construction.
If $m^+ \neq n^+$, then $m^+ \in \dom{v}$.
Since $v$ is an acceptable function,
$v'(m^+) = v(m^+) = F(v(m)) = F(v(m^+))$.
\end{enumerate}
Since $v'$ is acceptable, $n^+ \in \dom{h}$.
\subparagraph{Subconclusion}%
2023-08-09 13:39:41 +00:00
By \nameref{spar:recursion-theorem-natural-numbers-iii-1} and
\nameref{spar:recursion-theorem-natural-numbers-iii-2},
$\dom{h}$ is an inductive set.
\nameref{sub:theorem-4b} implies $\dom{h} = \omega$.
\paragraph{(iv)}%
\hyperlabel{par:recursion-theorem-natural-numbers-iv}
We now prove $h$ is a unique function.
Let $h_1$ and $h_2$ both satisfy the conclusion of the theorem.
Define $$S = \{n \in \omega \mid h_1(n) = h_2(n)\}.$$
It suffices to prove $S$ is an inductive set, for \nameref{sub:theorem-4b}
would then imply $S = \omega$, i.e. $h_1$ and $h_2$ agree on all of
$\omega$.
By definition of an acceptable function, $h_1(0) = a = h_2(0)$ meaning
$0 \in S$.
Next, suppose $n \in S$.
By \nameref{par:recursion-theorem-natural-numbers-iii}, it follows $n^+$
in $\dom{h_1}$ and $\dom{h_2}$.
Since both $h_1$ and $h_2$ are acceptable, $h_1(n^+) = F(h_1(n))$ and
$h_2(n^+) = F(h_2(n))$.
Since $n \in S$, $h_1(n) = h_2(n)$.
Therefore $h_1$ and $h_2$ coincide with input $n^+$.
Thus $n^+ \in S$.
Hence $S$ is an inductive set.
2023-08-09 13:39:41 +00:00
\paragraph{Conclusion}%
2023-08-09 13:39:41 +00:00
By \nameref{par:recursion-theorem-natural-numbers-i},
\nameref{par:recursion-theorem-natural-numbers-iii}, and
\nameref{par:recursion-theorem-natural-numbers-iv}, it follows $h$ is a
unique function mapping $\omega$ into $A$.
\nameref{par:recursion-theorem-natural-numbers-ii} shows $h$ satisfies the
desired conditions.
\end{proof}
\subsection{\verified{Theorem 4H}}%
\hyperlabel{sub:theorem-4h}
2023-08-09 13:39:41 +00:00
\begin{theorem}[4H]
Let $\langle N, S, e \rangle$ be a Peano system.
Then $\langle \omega, \sigma, 0 \rangle$ is isomorphic to
$\langle N, S, e \rangle$, i.e., there is a function $h$ mapping $\omega$
one-to-one onto $N$ in a way that preserves the successor operation
$$h(\sigma(n)) = S(h(n))$$ and the zero element $$h(0) = e.$$
\end{theorem}
\code{Common/Set/Peano}
{Peano.nat\_isomorphism}
2023-08-09 13:39:41 +00:00
\begin{proof}
2023-08-27 19:48:10 +00:00
Let $\langle N, S, e \rangle$ be a \nameref{ref:peano-system}.
By the \nameref{sub:recursion-theorem-natural-numbers}, there exists a
unique function $h \colon \omega \rightarrow N$ such that $h(0) = e$ and
for every $n \in \omega$, $h(n^+) = h(\sigma(n)) = S(h(n))$.
All that remains is proving $h$ is one-to-one and onto.
\suitdivider
\noindent
We first show $h$ is one-to-one by induction.
Define
$$S = \{n \in \omega \mid \forall m, h(m) = h(n) \Rightarrow m = n\}.$$
We show that (i) $0 \in S$ and (ii) if $n \in S$, then $n^+ \in S$.
Afterward we show (iii) that $h$ is one-to-one.
\paragraph{(i)}%
\hyperlabel{par:theorem-4h-i}
Let $m \in \omega$ such that $h(m) = h(0) = e$.
Suppose $m \neq 0$.
Then \nameref{sub:theorem-4c} indicates there exists some natural number
$p$ such that $p^+ = m$.
But then $h(m) = h(p^+) = S(h(p)) = e$.
By definition of a Peano system, $e \not\in \ran{S}$.
This is a contradiction.
Hence $m = 0$, i.e. $0 \in S$.
\paragraph{(ii)}%
\hyperlabel{par:theorem-4h-ii}
Suppose $n \in S$.
Let $m \in \omega$ such that $h(m) = h(n^+) = S(h(n))$.
There are two cases to consider:
\subparagraph{Case 1}%
Suppose $m = 0$.
Then $h(0) = e = S(h(n))$.
But, by definition of a Peano system, $e \not\in \ran{S}$.
Thus we have a contradiction.
\subparagraph{Case 2}%
Suppose $m \neq 0$.
Then \nameref{sub:theorem-4c} indicates there exists some natural number
$p$ such that $p^+ = m$.
Then $$S(h(n)) = h(m) = h(p^+) = S(h(p)).$$
By definition of a Peano system, $S$ is one-to-one.
Therefore $h(n) = h(p)$.
Since $n \in S$, it follows $n = p$.
Therefore $n^+ = p^+ = m$.
\subparagraph{Subconclusion}%
The above two cases are exhaustive.
Hence $n^+ \in S$.
\paragraph{(iii)}%
By \nameref{par:theorem-4h-i} and \nameref{par:theorem-4h-ii}, $S$ is an
\nameref{ref:inductive-set}.
By \nameref{sub:theorem-4b}, $S = \omega$.
Thus for all natural numbers $m, n \in \omega$, if $h(m) = h(n)$, then
$m = n$.
In other words, $f$ is one-to-one.
\suitdivider
\noindent
We next show that $\ran{h} = N$.
By the Peano posulate, every subset $A$ of $N$ containing $e$ and closed under $S$ is $N$ itself.
Thus it suffices to prove that (i) $e \in \ran{h}$ and (ii) $\ran{h}$ is closed under $S$.
\paragraph{(i)}%
That is, $h(0) = e$ by definition.
Thus $e \in \ran{h}$.
\paragraph{(ii)}%
Let $y \in \ran{h}$.
Then there exists some $n \in \omega$ such that $h(n) = y$.
By definition of $h$, $h(n^+) = S(h(n)) = S(y)$.
Therefore $S(y) \in \ran{h}$.
Since this holds for any $y \in \ran{h}$, $\ran{h}$ is closed under $S$.
2023-08-09 13:39:41 +00:00
\end{proof}
\section{Arithmetic}%
\hyperlabel{sec:arithmetic}
\subsection{\verified{Theorem 4I}}
\hyperlabel{sub:theorem-4i}
2023-08-09 13:39:41 +00:00
\begin{theorem}[4I]
For \nameref{ref:natural-number}s $m$ and $n$,
2023-08-09 13:39:41 +00:00
\begin{align}
m + 0 & = m, \hyperlabel{sub:theorem-4i-eq1} \\
m + n^+ & = (m + n)^+. \hyperlabel{sub:theorem-4i-eq2}
\end{align}
\end{theorem}
\code{Bookshelf/Enderton/Set/Chapter\_4}
{Enderton.Set.Chapter\_4.theorem\_4i}
2023-08-09 13:39:41 +00:00
\lean{Init/Data/Nat/Basic}{Nat.add\_zero}
\lean{Init/Prelude}{Nat.add}
2023-08-09 13:39:41 +00:00
\begin{proof}
\paragraph{\eqref{sub:theorem-4i-eq1}}%
2023-08-09 13:39:41 +00:00
Let $m$ be a \nameref{ref:natural-number}.
By definition of \nameref{ref:addition}, $m + 0 = A_m(0)$.
By definition of $A_m$, $A_m(0) = m$.
Thus $m + 0 = m$.
2023-08-09 13:39:41 +00:00
\paragraph{\eqref{sub:theorem-4i-eq2}}%
2023-08-09 13:39:41 +00:00
Let $m$ and $n$ be natural numbers.
By definition of \nameref{ref:addition},
$$m + n^+ = A_m(n^+) = A_m(n)^+ = (m + n)^+.$$
2023-08-09 13:39:41 +00:00
\end{proof}
\subsection{\verified{Theorem 4J}}
\hyperlabel{sub:theorem-4j}
2023-08-09 13:39:41 +00:00
\begin{theorem}[4J]
For natural numbers $m$ and $n$,
\begin{align}
m \cdot 0 & = 0, \hyperlabel{sub:theorem-4j-eq1} \\
m \cdot n^+ & = m \cdot n + m. \hyperlabel{sub:theorem-4j-eq2}
\end{align}
\end{theorem}
\code{Bookshelf/Enderton/Set/Chapter\_4}
{Enderton.Set.Chapter\_4.theorem\_4j}
2023-08-09 13:39:41 +00:00
\lean{Init/Data/Nat/Basic}{Nat.mul\_zero}
\lean{Init/Prelude}{Nat.mul}
2023-08-09 13:39:41 +00:00
\begin{proof}
2023-08-09 13:39:41 +00:00
\paragraph{\eqref{sub:theorem-4j-eq1}}%
2023-08-09 13:39:41 +00:00
Let $m$ be a \nameref{ref:natural-number}.
By definition of \nameref{ref:multiplication}, $$m \cdot 0 = M_m(0) = 0.$$
2023-08-09 13:39:41 +00:00
\paragraph{\eqref{sub:theorem-4j-eq2}}%
2023-08-09 13:39:41 +00:00
Let $m$ and $n$ be natural numbers.
By definition of \nameref{ref:multiplication},
$$m \cdot n^+ = M_m(n^+) = M_m(n) + m = m \cdot n + m.$$
\end{proof}
\subsection{\verified{Left Additive Identity}}%
\hyperlabel{sub:left-additive-identity}
2023-08-09 13:39:41 +00:00
\begin{lemma}
For all $n \in \omega$, $A_0(n) = n$.
In other words, $$0 + n = n.$$
\end{lemma}
\code{Bookshelf/Enderton/Set/Chapter\_4}
{Enderton.Set.Chapter\_4.left\_additive\_identity}
\lean{Init/Data/Nat/Basic}{Nat.zero\_add}
2023-08-09 13:39:41 +00:00
\begin{proof}
2023-08-09 13:39:41 +00:00
Let $S = \{n \in \omega \mid 0 + n = n\}$.
We prove that (i) $0 \in S$ and (ii) if $n \in S$ then $n^+ \in S$.
Afterwards we show that (iii) our theorem holds.
\paragraph{(i)}%
\hyperlabel{par:left-additive-identity-i}
2023-08-09 13:39:41 +00:00
By \nameref{sub:theorem-4i}, $0 + 0 = 0$.
Thus $0 \in S$.
2023-08-09 13:39:41 +00:00
\paragraph{(ii)}%
\hyperlabel{par:left-additive-identity-ii}
2023-08-09 13:39:41 +00:00
Suppose $n \in S$.
By \nameref{sub:theorem-4i}, $0 + n^+ = (0 + n)^+$.
Since $n \in S$, $0 + n = n$ which in turn implies that $(0 + n)^+ = n^+$.
Thus $n^+ \in S$.
2023-08-09 13:39:41 +00:00
\paragraph{(iii)}%
2023-08-09 13:39:41 +00:00
By \nameref{par:left-additive-identity-i} and
\nameref{par:left-additive-identity-ii}, $S$ is an
\nameref{ref:inductive-set}.
Hence \nameref{sub:theorem-4b} implies $S = \omega$.
Thus for all $n \in \omega$, $0 + n = n$.
2023-08-09 13:39:41 +00:00
\end{proof}
\subsection{\verified{Successor Commutativity}}%
\hyperlabel{sub:successor-commutativity}
\begin{lemma}
2023-08-09 13:39:41 +00:00
For all $m, n \in \omega$, $A_{m^+}(n) = A_m(n^+)$.
In other words, $$m^+ + n = m + n^+.$$
\end{lemma}
\code{Bookshelf/Enderton/Set/Chapter\_4}
{Enderton.Set.Chapter\_4.lemma\_2}
\lean{Std/Data/Nat/Lemmas}{Nat.succ\_add\_eq\_succ\_add}
2023-08-09 13:39:41 +00:00
\begin{proof}
2023-08-09 13:39:41 +00:00
Let $m \in \omega$ and define
$$S = \{n \in \omega \mid m^+ + n = m + n^+\}.$$
We prove that (i) $0 \in S$ and (ii) if $n \in S$ then $n^+ \in S$.
Afterwards we show that (iii) our theorem holds.
2023-08-09 13:39:41 +00:00
\paragraph{(i)}%
\hyperlabel{par:lemma-2-i}
2023-08-09 13:39:41 +00:00
By \nameref{sub:theorem-4i}, $m^+ + 0 = m^+$.
Likewise, $m + 0^+ = (m + 0)^+ = m^+$.
Thus $0 \in S$.
2023-08-09 13:39:41 +00:00
\paragraph{(ii)}%
\hyperlabel{par:lemma-2-ii}
2023-08-09 13:39:41 +00:00
Suppose $n \in S$.
By \nameref{sub:theorem-4i}, $m^+ + n^+ = (m^+ + n)^+$.
Since $n \in S$, $m^+ + n = m + n^+$.
Therefore $(m^+ + n)^+ = (m + n^+)^+ = m + n^{++}$.
Thus $n^+ \in S$.
2023-08-09 13:39:41 +00:00
\paragraph{(iii)}%
By \nameref{par:lemma-2-i} and \nameref{par:lemma-2-ii}, $S$ is inductive.
2023-08-09 13:39:41 +00:00
Hence \nameref{sub:theorem-4b} implies $S = \omega$.
Thus for all $n \in \omega$, $m^+ + n = m + n^+$.
2023-08-09 13:39:41 +00:00
\end{proof}
\subsection{\verified{Theorem 4K-1}}%
2023-08-09 13:39:41 +00:00
\hyperlabel{sub:theorem-4k-1}
2023-08-09 13:39:41 +00:00
\begin{theorem}[4K-1]
Associative law for \nameref{ref:addition}.
2023-08-09 13:39:41 +00:00
For $m, n, p \in \omega$, $$m + (n + p) = (m + n) + p.$$
\end{theorem}
\code{Bookshelf/Enderton/Set/Chapter\_4}
{Enderton.Set.Chapter\_4.theorem\_4k\_1}
\lean{Init/Data/Nat/Basic}{Nat.add\_assoc}
2023-08-09 13:39:41 +00:00
\begin{proof}
2023-08-09 13:39:41 +00:00
Fix $n, p \in \omega$ and define
\begin{equation}
\hyperlabel{sub:theorem-4k-1-eq1}
S = \{m \in \omega \mid m + (n + p) = (m + n) + p\}.
\end{equation}
We show that (i) $0 \in S$ and (ii) if $m \in S$ then $m^+ \in S$.
Afterward we show that (iii) the associative law for addition holds.
2023-08-09 13:39:41 +00:00
\paragraph{(i)}%
\hyperlabel{par:theorem-4k-1-i}
2023-08-09 13:39:41 +00:00
By \nameref{sub:left-additive-identity},
$$0 + (n + p) = n + p = (0 + n) + p.$$
Thus $0 \in S$.
2023-08-09 13:39:41 +00:00
\paragraph{(ii)}%
\hyperlabel{par:theorem-4k-1-ii}
2023-08-09 13:39:41 +00:00
Suppose $m \in S$.
Then
\begin{align*}
m^+ + (n + p)
& = m + (n + p)^+ & \textref{sub:successor-commutativity} \\
2023-08-09 13:39:41 +00:00
& = (m + (n + p))^+ & \textref{sub:theorem-4i} \\
& = ((m + n) + p)^+ & \eqref{sub:theorem-4k-1-eq1} \\
& = (m + n) + p^+ & \textref{sub:theorem-4i} \\
& = (m + n)^+ + p & \textref{sub:successor-commutativity} \\
2023-08-09 13:39:41 +00:00
& = (m + n^+) + p & \textref{sub:theorem-4i} \\
& = (m^+ + n) + p. & \textref{sub:successor-commutativity}
2023-08-09 13:39:41 +00:00
\end{align*}
Thus $m^+ \in S$.
2023-08-09 13:39:41 +00:00
\paragraph{(iii)}%
\hyperlabel{par:theorem-4k-1-iii}
2023-08-09 13:39:41 +00:00
By \nameref{par:theorem-4k-1-i} and \nameref{par:theorem-4k-1-ii}, $S$ is
an \nameref{ref:inductive-set}.
By \nameref{sub:theorem-4b}, $S = \omega$.
Thus for all $m, n, p \in \omega$, $m + (n + p) = (m + n) + p$.
2023-08-09 13:39:41 +00:00
\end{proof}
\subsection{\verified{Theorem 4K-2}}%
2023-08-09 13:39:41 +00:00
\hyperlabel{sub:theorem-4k-2}
2023-08-09 13:39:41 +00:00
\begin{theorem}[4K-2]
Commutative law for \nameref{ref:addition}.
2023-08-09 13:39:41 +00:00
For $m, n \in \omega$, $$m + n = n + m.$$
\end{theorem}
\code{Bookshelf/Enderton/Set/Chapter\_4}
{Enderton.Set.Chapter\_4.theorem\_4k\_2}
\lean{Init/Data/Nat/Basic}{Nat.add\_comm}
2023-08-09 13:39:41 +00:00
\begin{proof}
2023-08-09 13:39:41 +00:00
Fix $n \in \omega$ and define
\begin{equation}
\hyperlabel{sub:theorem-4k-2-eq1}
S = \{m \in \omega \mid m + n = n + m\}.
\end{equation}
We show that (i) $0 \in S$ and (ii) if $m \in S$ then $m^+ \in S$.
Afterward we show that (iii) the commutative law for addition holds.
2023-08-09 13:39:41 +00:00
\paragraph{(i)}%
\hyperlabel{par:theorem-4k-2-i}
2023-08-09 13:39:41 +00:00
By definition of \nameref{ref:addition} and
\nameref{sub:left-additive-identity}, $$0 + n = n = n + 0.$$
Thus $0 \in S$.
2023-08-09 13:39:41 +00:00
\paragraph{(ii)}%
\hyperlabel{par:theorem-4k-2-ii}
Suppose $m \in S$.
Then
\begin{align*}
m^+ + n
& = m + n^+ & \textref{sub:successor-commutativity} \\
2023-08-09 13:39:41 +00:00
& = (m + n)^+ & \textref{sub:theorem-4i} \\
& = (n + m)^+ & \eqref{sub:theorem-4k-2-eq1} \\
& = n + m^+. & \textref{sub:theorem-4i}
\end{align*}
Thus $m^+ \in S$.
2023-08-09 13:39:41 +00:00
\paragraph{(iii)}%
2023-08-09 13:39:41 +00:00
By \nameref{par:theorem-4k-2-i} and \nameref{par:theorem-4k-2-ii}, $S$
is an \nameref{ref:inductive-set}.
By \nameref{sub:theorem-4b}, $S = \omega$.
Thus for all $m, n \in \omega$, $m + n = n + m$.
2023-08-09 13:39:41 +00:00
\end{proof}
\subsection{\verified{Zero Multiplicand}}%
\hyperlabel{sub:zero-multiplicand}
2023-08-09 13:39:41 +00:00
\begin{lemma}
For all $n \in \omega$, $M_0(n) = 0$.
In other words, $$0 \cdot n = 0.$$
\end{lemma}
\code{Bookshelf/Enderton/Set/Chapter\_4}
{Enderton.Set.Chapter\_4.zero\_multiplicand}
\lean{Init/Data/Nat/Basic}{Nat.zero\_mul}
2023-08-09 13:39:41 +00:00
\begin{proof}
Define
\begin{equation}
\hyperlabel{sub:zero-multiplicand-eq1}
S = \{n \in \omega \mid 0 \cdot n = 0\}.
\end{equation}
We prove that (i) $0 \in S$ and (ii) that if $n \in S$, then $n^+ \in S$.
Afterwards we show that (iii) our theorem holds.
2023-08-09 13:39:41 +00:00
\paragraph{(i)}%
\hyperlabel{par:zero-multiplicand-i}
2023-08-09 13:39:41 +00:00
By \nameref{sub:theorem-4j}, $0 \cdot 0 = 0$.
Thus $0 \in S$.
2023-08-09 13:39:41 +00:00
\paragraph{(ii)}%
\hyperlabel{par:zero-multiplicand-ii}
2023-08-09 13:39:41 +00:00
Suppose $n \in S$.
Then
\begin{align*}
0 \cdot n^+
& = 0 \cdot n + 0 & \textref{sub:theorem-4j} \\
& = 0 \cdot n & \textref{sub:theorem-4i} \\
& = 0. & \eqref{sub:zero-multiplicand-eq1}
2023-08-09 13:39:41 +00:00
\end{align*}
Thus $n^+ \in S$.
2023-08-09 13:39:41 +00:00
\paragraph{(iii)}%
2023-08-09 13:39:41 +00:00
By \nameref{par:zero-multiplicand-i} and
\nameref{par:zero-multiplicand-ii}, $S$ is an
\nameref{ref:inductive-set}.
Hence \nameref{sub:theorem-4b} implies $S = \omega$.
Thus for all $n \in \omega$, $0 \cdot n = 0$.
2023-08-09 13:39:41 +00:00
\end{proof}
\subsection{\verified{Successor Distribution}}%
\hyperlabel{sub:successor-distribution}
2023-08-09 13:39:41 +00:00
\begin{lemma}
For all $m, n \in \omega$, $M_{m^+}(n) = M_m(n) + n$.
In other words, $$m^+ \cdot n = m \cdot n + n.$$
\end{lemma}
\code{Bookshelf/Enderton/Set/Chapter\_4}
{Enderton.Set.Chapter\_4.succ\_distrib}
\lean{Init/Data/Nat/Basic}{Nat.succ\_mul}
2023-08-09 13:39:41 +00:00
\begin{proof}
Let $m \in \omega$ and define
\begin{equation}
\hyperlabel{sub:successor-distribution-eq1}
S = \{n \in \omega \mid m^+ \cdot n = m \cdot n + n\}.
\end{equation}
We prove that (i) $0 \in S$ and (ii) that if $n \in S$, then $n^+ \in S$.
Afterwards we show that (iii) our theorem holds.
2023-08-09 13:39:41 +00:00
\paragraph{(i)}%
\hyperlabel{par:successor-distribution-i}
2023-08-09 13:39:41 +00:00
By \nameref{sub:theorem-4j}, $m^+ \cdot 0 = 0$.
Likewise, by \nameref{sub:theorem-4i}, $m \cdot 0 + 0 = 0$.
Thus $0 \in S$.
2023-08-09 13:39:41 +00:00
\paragraph{(ii)}%
\hyperlabel{par:successor-distribution-ii}
2023-08-09 13:39:41 +00:00
Suppose $n \in S$.
Then
\begin{align*}
m^+ \cdot n^+
& = m^+ \cdot n + m^+ & \textref{sub:theorem-4j} \\
& = (m \cdot n + n) + m^+ & \eqref{sub:successor-distribution-eq1} \\
& = m \cdot n + (n + m^+) & \textref{sub:theorem-4k-1} \\
& = m \cdot n + (n^+ + m) & \textref{sub:successor-commutativity} \\
2023-08-09 13:39:41 +00:00
& = m \cdot n + (m + n^+) & \textref{sub:theorem-4k-2} \\
& = (m \cdot n + m) + n^+ & \textref{sub:theorem-4k-1} \\
& = m \cdot n^+ + n^+. & \textref{sub:theorem-4j}
\end{align*}
Thus $n^+ \in S$.
2023-08-09 13:39:41 +00:00
\paragraph{(iii)}%
2023-08-09 13:39:41 +00:00
By \nameref{par:successor-distribution-i} and
\nameref{par:successor-distribution-ii}, $S$ is an
\nameref{ref:inductive-set}.
By \nameref{sub:theorem-4b}, $S = \omega$.
Thus for all $m, n \in \omega$, $m^+ \cdot n = m \cdot n + n$.
2023-08-09 13:39:41 +00:00
\end{proof}
\subsection{\verified{Theorem 4K-3}}
\hyperlabel{sub:theorem-4k-3}
2023-08-09 13:39:41 +00:00
\begin{theorem}[4K-3]
Distributive law.
For $m, n, p \in \omega$, $$m \cdot (n + p) = m \cdot n + m \cdot p.$$
\end{theorem}
\code{Bookshelf/Enderton/Set/Chapter\_4}
{Enderton.Set.Chapter\_4.theorem\_4k\_3}
\lean{Init/Data/Nat/Basic}{Nat.left\_distrib}
2023-08-09 13:39:41 +00:00
\begin{proof}
2023-08-09 13:39:41 +00:00
Fix $n, p \in \omega$ and define
\begin{equation}
\hyperlabel{sub:theorem-4k-3-eq1}
S = \{m \in \omega \mid m \cdot (n + p) = m \cdot n + m \cdot p\}.
\end{equation}
We show that (i) $0 \in S$ and (ii) if $m \in S$ then $m^+ \in S$.
Afterward we show that (iii) the distributive law holds.
2023-08-09 13:39:41 +00:00
\paragraph{(i)}%
\hyperlabel{par:theorem-4k-3-i}
2023-08-09 13:39:41 +00:00
By definition of \nameref{ref:multiplication} and \nameref{ref:addition},
\begin{align*}
0 \cdot (n + p)
& = 0 & \textref{sub:zero-multiplicand} \\
& = 0 + 0 & \textref{ref:addition} \\
& = 0 \cdot n + 0 \cdot p. & \textref{sub:zero-multiplicand}
\end{align*}
Thus $0 \in S$.
2023-08-09 13:39:41 +00:00
\paragraph{(ii)}%
\hyperlabel{par:theorem-4k-3-ii}
2023-08-09 13:39:41 +00:00
Suppose $m \in S$.
By definition of \nameref{ref:multiplication} and \nameref{ref:addition},
\begin{align*}
m^+ \cdot (n + p)
& = m \cdot (n + p) + (n + p)
& \textref{sub:successor-distribution} \\
& = m \cdot (n + p) + n + p & \textref{sub:theorem-4k-1} \\
& = m \cdot n + m \cdot p + n + p
& \eqref{sub:theorem-4k-3-eq1} \\
& = m \cdot n + n + m \cdot p + p & \textref{sub:theorem-4k-2} \\
& = m^+ \cdot n + m^+ \cdot p.
& \textref{sub:successor-distribution}
\end{align*}
Thus $m^+ \in S$.
\paragraph{(iii)}%
2023-08-09 13:39:41 +00:00
By \nameref{par:theorem-4k-3-i} and \nameref{par:theorem-4k-3-ii}, $S$
is an \nameref{ref:inductive-set}.
By \nameref{sub:theorem-4b}, $S = \omega$.
Thus for all $m, n, p \in \omega$,
$m \cdot (n + p) = m \cdot n + m \cdot p$.
2023-08-09 13:39:41 +00:00
\end{proof}
\subsection{\verified{Successor Identity}}%
\hyperlabel{sub:successor-identity}
2023-08-09 13:39:41 +00:00
\begin{lemma}
For all $m \in \omega$, $A_m(1) = m^+$.
In other words, $$m + 1 = m^+.$$
\end{lemma}
\code{Bookshelf/Enderton/Set/Chapter\_4}
{Enderton.Set.Chapter\_4.succ\_identity}
\lean{Std/Data/Nat/Lemmas}{Nat.succ\_eq\_one\_add}
2023-08-09 13:39:41 +00:00
\begin{proof}
Let
\begin{equation}
\hyperlabel{sub:successor-identity-eq1}
S = \{m \in \omega \mid m + 1 = m^+\}.
\end{equation}
We prove that (i) $0 \in S$ and (ii) if $m \in S$, then $m^+ \in S$.
Afterwards we show that (iii) our theorem holds.
2023-08-09 13:39:41 +00:00
\paragraph{(i)}%
\hyperlabel{par:successor-identity-i}
2023-08-09 13:39:41 +00:00
By \nameref{sub:left-additive-identity}, $0 + 1 = 1$.
By definition of the \nameref{ref:successor},
$0^+ = \emptyset \cup \{\emptyset\} = 1$.
Thus $0 \in S$.
2023-08-09 13:39:41 +00:00
\paragraph{(ii)}%
\hyperlabel{par:successor-identity-ii}
2023-08-09 13:39:41 +00:00
Let $m \in S$.
Then
\begin{align*}
m^+ + 1
& = m + 1^+ & \textref{sub:successor-commutativity} \\
2023-08-09 13:39:41 +00:00
& = (m + 1)^+ & \textref{sub:theorem-4i} \\
& = (m^+)^+. & \eqref{sub:successor-identity-eq1}
\end{align*}
Thus $m^+ \in S$.
2023-08-09 13:39:41 +00:00
\paragraph{(iii)}%
2023-08-09 13:39:41 +00:00
By \nameref{par:successor-identity-i} and
\nameref{par:successor-identity-ii}, $S$ is an
\nameref{ref:inductive-set}.
Hence \nameref{sub:theorem-4b} implies $S = \omega$.
Thus for all $m \in \omega$, $m + 1 = m^+$.
2023-08-09 13:39:41 +00:00
\end{proof}
\subsection{\verified{Right Multiplicative Identity}}%
\hyperlabel{sub:right-multiplicative-identity}
2023-08-09 13:39:41 +00:00
\begin{lemma}
For all $m \in \omega$, $M_m(1) = m$.
In other words, $$m \cdot 1 = m.$$
\end{lemma}
\code{Bookshelf/Enderton/Set/Chapter\_4}
{Enderton.Set.Chapter\_4.right\_mul\_id}
\lean{Init/Data/Nat/Basic}{Nat.mul\_one}
2023-08-09 13:39:41 +00:00
\begin{proof}
2023-08-09 13:39:41 +00:00
Let
\begin{equation}
\hyperlabel{sub:right-multiplicative-identity-eq1}
S = \{m \in \omega \mid m \cdot 1 = m\}.
\end{equation}
We prove that (i) $0 \in S$ and (ii) if $m \in S$, then $m^+ \in S$.
Afterwards we show that (iii) our theorem holds.
2023-08-09 13:39:41 +00:00
\paragraph{(i)}%
\hyperlabel{par:right-multiplicative-identity-i}
2023-08-09 13:39:41 +00:00
By \nameref{sub:zero-multiplicand}, $0 \cdot 1 = 0$.
Thus $0 \in S$.
2023-08-09 13:39:41 +00:00
\paragraph{(ii)}%
\hyperlabel{par:right-multiplicative-identity-ii}
2023-08-09 13:39:41 +00:00
Suppose $m \in S$.
Then
\begin{align*}
m^+ \cdot 1
& = m \cdot 1 + 1 & \textref{sub:successor-distribution} \\
& = m + 1 & \eqref{sub:right-multiplicative-identity-eq1} \\
& = m^+. & \textref{sub:successor-identity}
\end{align*}
Thus $m^+ \in S$.
2023-08-09 13:39:41 +00:00
\paragraph{(iii)}%
2023-08-09 13:39:41 +00:00
By \nameref{par:right-multiplicative-identity-i} and
\nameref{par:right-multiplicative-identity-ii}, $S$
is an \nameref{ref:inductive-set}.
By \nameref{sub:theorem-4b}, $S = \omega$.
Thus for all $m \in \omega$, $m \cdot 1 = m$.
\end{proof}
\subsection{\verified{Theorem 4K-5}}
\hyperlabel{sub:theorem-4k-5}
2023-08-09 13:39:41 +00:00
\begin{theorem}[4K-5]
Commutative law for \nameref{ref:multiplication}.
2023-08-09 13:39:41 +00:00
For $m, n \in \omega$, $$m \cdot n = n \cdot m.$$
\end{theorem}
\code{Bookshelf/Enderton/Set/Chapter\_4}
{Enderton.Set.Chapter\_4.theorem\_4k\_5}
\lean{Init/Data/Nat/Basic}{Nat.mul\_comm}
\begin{note}
We prove commutativity before associativity, though Enderton orders these
two properties in the opposite direction.
\end{note}
2023-08-09 13:39:41 +00:00
\begin{proof}
2023-08-09 13:39:41 +00:00
Fix $n \in \omega$ and define
\begin{equation}
\hyperlabel{sub:theorem-4k-5-eq1}
S = \{m \in \omega \mid m \cdot n = n \cdot m\}.
\end{equation}
We show that (i) $0 \in S$ and (ii) if $m \in S$ then $m^+ \in S$.
Afterward we show that (iii) the commutative law for multiplication holds.
2023-08-09 13:39:41 +00:00
\paragraph{(i)}%
\hyperlabel{par:theorem-4k-5-i}
2023-08-09 13:39:41 +00:00
By \nameref{sub:theorem-4j} and \nameref{sub:zero-multiplicand},
$$0 \cdot n = 0 = n \cdot 0.$$
Thus $0 \in S$.
2023-08-09 13:39:41 +00:00
\paragraph{(ii)}%
\hyperlabel{par:theorem-4k-5-ii}
Suppose $m \in S$.
Then
\begin{align*}
m^+ \cdot n
& = m \cdot n + n & \textref{sub:successor-distribution} \\
& = n \cdot m + n & \eqref{sub:theorem-4k-5-eq1} \\
& = n \cdot m + n \cdot 1
& \textref{sub:right-multiplicative-identity} \\
& = n \cdot (m + 1) & \textref{sub:theorem-4k-3} \\
& = n \cdot m^+. & \textref{sub:successor-identity}
\end{align*}
Thus $m^+ \in S$.
2023-08-09 13:39:41 +00:00
\paragraph{(iii)}%
\hyperlabel{par:theorem-4k-5-iii}
2023-08-09 13:39:41 +00:00
By \nameref{par:theorem-4k-5-i} and \nameref{par:theorem-4k-5-ii}, $S$
is an \nameref{ref:inductive-set}.
By \nameref{sub:theorem-4b}, $S = \omega$.
Thus for all $m, n \in \omega$, $m \cdot n = n \cdot m$.
2023-08-09 13:39:41 +00:00
\end{proof}
\subsection{\verified{Theorem 4K-4}}%
\hyperlabel{sub:theorem-4k-4}
2023-08-09 13:39:41 +00:00
\begin{theorem}[4K-4]
Associative law for \nameref{ref:multiplication}.
2023-08-09 13:39:41 +00:00
For $m, n, p \in \omega$, $$m \cdot (n \cdot p) = (m \cdot n) \cdot p.$$
\end{theorem}
\code{Bookshelf/Enderton/Set/Chapter\_4}
{Enderton.Set.Chapter\_4.theorem\_4k\_4}
\lean{Init/Data/Nat/Basic}{Nat.mul\_assoc}
2023-08-09 13:39:41 +00:00
\begin{proof}
2023-08-09 13:39:41 +00:00
Fix $m, n \in \omega$ and define
\begin{equation}
\hyperlabel{sub:theorem-4k-4-eq1}
S = \{p \in \omega \mid m \cdot (n \cdot p) = (m \cdot n) \cdot p\}.
\end{equation}
We show that (i) $0 \in S$ and (ii) if $p \in S$ then $p^+ \in S$.
Afterward we show that (iii) the associative law for multiplication holds.
2023-08-09 13:39:41 +00:00
\paragraph{(i)}%
\hyperlabel{par:theorem-4k-4-i}
2023-08-09 13:39:41 +00:00
By \nameref{sub:theorem-4j},
$$m \cdot (n \cdot 0) = 0 = (m \cdot n) \cdot 0.$$
Thus $0 \in S$.
2023-08-09 13:39:41 +00:00
\paragraph{(ii)}%
\hyperlabel{par:theorem-4k-4-ii}
Suppose $p \in S$.
Then
\begin{align*}
m \cdot (n \cdot p^+)
& = m \cdot (n \cdot p + n) & \textref{sub:theorem-4j} \\
& = m \cdot (n \cdot p) + m \cdot n & \textref{sub:theorem-4k-3} \\
& = (m \cdot n) \cdot p + m \cdot n & \eqref{sub:theorem-4k-4-eq1} \\
& = p \cdot (m \cdot n) + m \cdot n & \textref{sub:theorem-4k-5} \\
& = p^+ \cdot (m \cdot n) & \textref{sub:successor-distribution} \\
& = (m \cdot n) \cdot p^+ & \textref{sub:theorem-4k-5}
\end{align*}
Thus $p^+ \in S$.
2023-08-09 13:39:41 +00:00
\paragraph{(iii)}%
2023-08-09 13:39:41 +00:00
By \nameref{par:theorem-4k-4-i} and \nameref{par:theorem-4k-4-ii}, $S$
is an \nameref{ref:inductive-set}.
By \nameref{sub:theorem-4b}, $S = \omega$.
Thus for all $m, n, p \in \omega$,
$m \cdot (n \cdot p) = (m \cdot n) \cdot p$.
2023-08-09 13:39:41 +00:00
\end{proof}
\section{Ordering on \texorpdfstring{$\omega$}{Natural Numbers}}%
\hyperlabel{sec:ordering-natural-numbers}
\subsection{\unverified{Ordering on Successor}}%
\hyperlabel{sub:ordering-successor}
2023-08-09 13:39:41 +00:00
\begin{lemma}
Let $m, n \in \omega$.
Then $m < n^+ \iff m \leq n$.
\end{lemma}
\lean{Std/Data/Nat/Lemmas}{Nat.lt\_succ}
2023-08-09 13:39:41 +00:00
\begin{proof}
Let $m, n \in \omega$.
By \nameref{ref:ordering-natural-numbers},
\begin{align*}
m < n^+
& \iff m \in n^+ \\
& \iff m \in n \cup \{n\} & \textref{ref:successor} \\
& \iff m \in n \lor m \in \{n\} \\
& \iff m \in n \lor m = n \\
& \iff m \leq n.
\end{align*}
\end{proof}
\subsection{\unverified{Members of Natural Numbers}}%
\hyperlabel{sub:members-natural-numbers}
2023-08-09 13:39:41 +00:00
\begin{lemma}
Every \nameref{ref:natural-number} is the set of all smaller natural
numbers.
2023-08-09 13:39:41 +00:00
\end{lemma}
2023-08-09 13:39:41 +00:00
\begin{proof}
Let $n \in \omega$.
Consider $m \in n$.
By \nameref{sub:theorem-4b}, $\omega$ is a \nameref{ref:transitive-set}.
Thus $m \in n$ implies $m \in \omega$.
Thus $m \in n \iff m \in \omega \land m \in n$.
\end{proof}
\subsection{\unverified{Lemma 4L(a)}}%
2023-08-09 13:39:41 +00:00
\hyperlabel{sub:lemma-4l-a}
2023-08-09 13:39:41 +00:00
\begin{lemma}[4L(a)]
For any \nameref{ref:natural-number}s $m$ and $n$,
$$m \in n \iff m^+ \in n^+.$$
2023-08-09 13:39:41 +00:00
\end{lemma}
\lean{Std/Data/Nat/Lemmas}{Nat.succ\_lt\_succ\_iff}
2023-08-09 13:39:41 +00:00
\begin{note}
Here I referred to Enderton's proof in the forward direction.
\end{note}
2023-08-09 13:39:41 +00:00
\begin{proof}
2023-08-09 13:39:41 +00:00
Let $m$ and $n$ be \nameref{ref:natural-number}s.
2023-08-09 13:39:41 +00:00
\paragraph{($\Rightarrow$)}%
2023-08-09 13:39:41 +00:00
Define $$S = \{n \in \omega \mid (\forall m \in n) m^+ \in n^+\}.$$
We prove that (i) $0 \in S$ and (ii) if $n \in S$ then $n^+ \in S$.
Afterwards we show that (iii) the forward direction of the stated
biconditional holds.
2023-08-09 13:39:41 +00:00
\subparagraph{(i)}%
\hyperlabel{spar:lemma-4l-a-i}
2023-08-09 13:39:41 +00:00
$0 \in S$ vacuously.
That is, there are no members of $0 = \emptyset$ by definition.
2023-08-09 13:39:41 +00:00
\subparagraph{(ii)}%
\hyperlabel{spar:lemma-4l-a-ii}
2023-08-09 13:39:41 +00:00
Suppose $n \in S$.
We need to show for all $m \in n^+$, $m^+ \in n^{++}$.
Let $m \in n^+ = n \cup \{n\}$.
Then $m \in n$ or $m \in \{n\}$.
2023-08-09 13:39:41 +00:00
If $m \in n$, then $n \in S$ implies $m^+ \in n^+ \in n^{++}$.
By \nameref{sub:theorem-4f}, every natural number is a
\nameref{ref:transitive-set}.
Therefore $m^+ \in n^{++}$.
On the other hand, if $m \in \{n\}$, then $m = n$.
Since $n^+ \in n^{++}$, it immediately follows $m^+ \in n^{++}$.
Hence $n^+ \in S$.
2023-08-09 13:39:41 +00:00
\subparagraph{(iii)}%
2023-08-09 13:39:41 +00:00
By \nameref{spar:lemma-4l-a-i} and \nameref{spar:lemma-4l-a-ii}, $S$ is
an \nameref{ref:inductive-set}.
Hence \nameref{sub:theorem-4b} implies $S = \omega$.
Thus for all $n \in \omega$, $m \in n \Rightarrow m^+ \in n^+$.
2023-08-09 13:39:41 +00:00
\paragraph{($\Leftarrow$)}%
2023-08-09 13:39:41 +00:00
Suppose $m^+ \in n^+$.
The definition of \nameref{ref:successor} immediately implies that
$m \in m^+$.
Likewise, $m^+ \in n^+$ implies $m^+ \in n$ or $m^+ = n$.
2023-08-09 13:39:41 +00:00
If the latter, $m \in n$ immediately follows.
If the former, we note $n$ is a transitive set by
\nameref{sub:theorem-4f}.
Thus $m \in m^+ \in n$ implies $m \in n$.
\end{proof}
\subsection{\verified{Lemma 4L(b)}}%
\hyperlabel{sub:lemma-4l-b}
2023-08-09 13:39:41 +00:00
\begin{lemma}[4L(b)]
No \nameref{ref:natural-number} is a member of itself.
2023-08-09 13:39:41 +00:00
\end{lemma}
\code{Bookshelf/Enderton/Set/Chapter\_4}
{Enderton.Set.Chapter\_4.theorem\_4l\_b}
\lean{Init/Prelude}{Nat.lt\_irrefl}
2023-08-09 13:39:41 +00:00
\begin{proof}
Define
\begin{equation}
\hyperlabel{sub:lemma-4l-b-eq1}
S = \{n \in \omega \mid n \not\in n\}.
\end{equation}
We prove that (i) $0 \in S$ and (ii) if $n \in S$ then $n^+ \in S$.
Afterwards we show that (iii) our theorem holds.
2023-08-09 13:39:41 +00:00
\paragraph{(i)}%
\hyperlabel{par:lemma-4l-b-i}
2023-08-09 13:39:41 +00:00
By definition, $0 = \emptyset$.
It obviously holds that $\emptyset \not\in \emptyset$ since $\emptyset$,
by definition, has no members.
Thus $0 \in S$.
2023-08-09 13:39:41 +00:00
\paragraph{(ii)}%
\hyperlabel{par:lemma-4l-b-ii}
2023-08-09 13:39:41 +00:00
Suppose $n \in S$.
By \eqref{sub:lemma-4l-b-eq1}, $n \not\in n$.
By \nameref{sub:lemma-4l-a}, it follows $n^+ \not\in n^+$.
Thus $n^+ \in S$.
2023-08-09 13:39:41 +00:00
\paragraph{(iii)}%
2023-08-09 13:39:41 +00:00
By \nameref{par:lemma-4l-b-i} and \nameref{par:lemma-4l-b-ii}, $S$ is an
\nameref{ref:inductive-set}.
Hence \nameref{sub:theorem-4b} implies $S = \omega$.
Thus for all $n \in \omega$, $n \not\in n$.
2023-08-09 13:39:41 +00:00
\end{proof}
\subsection{\verified{\texorpdfstring{$0$}{Zero} is the Least Natural Number}}%
\hyperlabel{sub:zero-least-natural-number}
2023-08-09 13:39:41 +00:00
\begin{lemma}
For every \nameref{ref:natural-number} $n \neq 0$, $0 \in n$.
2023-08-09 13:39:41 +00:00
\end{lemma}
\code{Bookshelf/Enderton/Set/Chapter\_4}
{Enderton.Set.Chapter\_4.zero\_least\_nat}
\lean{Std/Data/Nat/Init/Lemmas}{Nat.pos\_of\_ne\_zero}
2023-08-09 13:39:41 +00:00
\begin{proof}
Let $$S = \{n \in \omega \mid n = 0 \lor 0 \in n\}.$$
We prove that (i) $0 \in S$ and (ii) if $n \in S$ then $n^+ \in S$.
Afterwards we show that (iii) our theorem holds.
2023-08-09 13:39:41 +00:00
\paragraph{(i)}%
\hyperlabel{par:zero-least-natural-number-i}
2023-08-09 13:39:41 +00:00
This trivially holds by definition of $S$.
2023-08-09 13:39:41 +00:00
\paragraph{(ii)}%
\hyperlabel{par:zero-least-natural-number-ii}
2023-08-09 13:39:41 +00:00
Suppose $n \in S$.
By definition of the \nameref{ref:successor} function,
$n^+ = n \cup \{n\}$.
Thus $n \in n^+$.
By \nameref{sub:theorem-4f}, $n^+$ is a \nameref{ref:transitive-set}.
Since $0 \in n$ and $n \in n^+$, it follows $0 \in n^+$.
Thus $n^+ \in S$.
2023-08-09 13:39:41 +00:00
\paragraph{(iii)}%
2023-08-09 13:39:41 +00:00
By \nameref{par:zero-least-natural-number-i} and
\nameref{par:zero-least-natural-number-ii}, $S$ is an
\nameref{ref:inductive-set}.
Hence \nameref{sub:theorem-4b} implies $S = \omega$.
Thus for all $n \in \omega$, either $n = 0$ or $0 \in n$.
2023-08-09 13:39:41 +00:00
\end{proof}
\subsection{\unverified{%
Trichotomy Law for \texorpdfstring{$\omega$}{Natural Numbers}}}%
\hyperlabel{sub:trichotomy-law-natural-numbers}
2023-08-09 13:39:41 +00:00
\begin{theorem}
For any \nameref{ref:natural-number}s $m$ and $n$, exactly one of the three
conditions $$m \in n, \quad m = n, \quad n \in m$$ holds.
2023-08-09 13:39:41 +00:00
\end{theorem}
\lean{Mathlib/Order/RelClasses}{IsAsymm}
\lean{Mathlib/Init/Algebra/Classes}{IsTrichotomous}
2023-08-09 13:39:41 +00:00
\begin{proof}
2023-08-09 13:39:41 +00:00
Let $n \in \omega$ and define
\begin{equation}
\hyperlabel{sub:trichotomy-law-natural-numbers-eq1}
S = \{m \in \omega \mid m \in n \lor m = n \lor n \in m\}.
\end{equation}
We prove that (i) $0 \in S$ and (ii) if $m \in S$ then $m^+ \in S$.
Afterwards we show that (iii) our theorem holds.
2023-08-09 13:39:41 +00:00
\paragraph{(i)}%
\hyperlabel{par:trichotomy-law-natural-numbers-i}
2023-08-09 13:39:41 +00:00
If $n = 0$, then it trivially follows $0 \in S$.
Otherwise \nameref{sub:zero-least-natural-number} implies $0 \in n$.
Thus $0 \in S$.
2023-08-09 13:39:41 +00:00
\paragraph{(ii)}%
\hyperlabel{par:trichotomy-law-natural-numbers-ii}
2023-08-09 13:39:41 +00:00
Suppose $m \in S$.
By \eqref{sub:trichotomy-law-natural-numbers-eq1}, there are three cases
to consider:
2023-08-09 13:39:41 +00:00
\subparagraph{Case 1}%
2023-08-09 13:39:41 +00:00
Suppose $m \in n$.
By \nameref{sub:lemma-4l-a}, $m^+ \in n^+ = n \cup \{n\}$.
By definition of the \nameref{ref:successor}, $m^+ \in n$ or $m^+ = n$.
Either way, $m^+ \in S$.
2023-08-09 13:39:41 +00:00
\subparagraph{Case 2}%
2023-08-09 13:39:41 +00:00
Suppose $m = n$.
Since $m \in m^+$, it follows $n \in m^+$.
Thus $m^+ \in S$.
2023-08-09 13:39:41 +00:00
\subparagraph{Case 3}%
2023-08-09 13:39:41 +00:00
Suppose $n \in m$.
Then $n \in m \cup \{m\} = m^+$.
Thus $m^+ \in S$.
2023-08-09 13:39:41 +00:00
\subparagraph{Conclusion}%
2023-08-09 13:39:41 +00:00
Since the above three cases are exhaustive, it follows $m^+ \in S$.
2023-08-09 13:39:41 +00:00
\paragraph{(iii)}%
2023-08-09 13:39:41 +00:00
By \nameref{par:trichotomy-law-natural-numbers-i} and
\nameref{par:trichotomy-law-natural-numbers-ii}, $S$ is an
\nameref{ref:inductive-set}.
Hence \nameref{sub:theorem-4b} implies $S = \omega$.
Thus for all $m, n \in \omega$, $$m \in n \lor m = n \lor n \in m.$$
We now prove that
$$\in_\omega = \{\tuple{m, n} \in \omega \times \omega \mid m \in n\}$$
2023-08-09 13:39:41 +00:00
is \nameref{ref:irreflexive} and \nameref{ref:connected}.
Irreflexivity immediately follows from \nameref{sub:lemma-4l-b}.
Connectivity follows immediately from the fact $S = \omega$.
Furthermore, it is not possible both $m \in n$ and $n \in m$ since, by
\nameref{sub:theorem-4f}, $m$ and $n$ are \nameref{ref:transitive-set}s.
This would otherwise imply $m \in m$, an immediate contradiction to
irreflexivity.
Thus $\in_\omega$ is a \nameref{ref:trichotomous} relation.
\end{proof}
\subsection{\unverified{%
Linear Ordering on \texorpdfstring{$\omega$}{Natural Numbers}}}%
\hyperlabel{sub:linear-ordering-natural-numbers}
2023-08-09 13:39:41 +00:00
\begin{theorem}
\nameref{ref:relation}
2023-08-09 13:39:41 +00:00
\begin{equation}
\hyperlabel{sub:linear-ordering-natural-numbers-eq1}
\in_\omega = \{\tuple{m, n} \in \omega \times \omega \mid m \in n\}
2023-08-09 13:39:41 +00:00
\end{equation}
is a \nameref{ref:linear-ordering} on $\omega$.
2023-08-09 13:39:41 +00:00
\end{theorem}
\lean{Mathlib/Init/Algebra/Order}
{LinearOrder.isStrictTotalOrder\_of\_linearOrder}
2023-08-09 13:39:41 +00:00
\begin{proof}
By definition, \eqref{sub:linear-ordering-natural-numbers-eq1} is a linear
ordering on $\omega$ if it is (i) transitive and (ii) trichotomous.
2023-08-09 13:39:41 +00:00
\paragraph{(i)}%
2023-08-09 13:39:41 +00:00
Suppose $p, q, r \in \omega$ such that $p \in q$ and $q \in r$.
By \nameref{sub:theorem-4f}, $p$, $q$, and $r$ are
\nameref{ref:transitive-set}s.
By definition of a transitive set, it follows $p \in r$.
Hence \eqref{sub:linear-ordering-natural-numbers-eq1} is
\nameref{ref:transitive}.
2023-08-09 13:39:41 +00:00
\paragraph{(ii)}%
2023-08-09 13:39:41 +00:00
By \nameref{sub:trichotomy-law-natural-numbers},
\eqref{sub:linear-ordering-natural-numbers-eq1} is trichotomous.
\end{proof}
\subsection{\unverified{Corollary 4M}}%
\hyperlabel{sub:corollary-4m}
2023-08-09 13:39:41 +00:00
\begin{corollary}[4M]
For any natural numbers $m$ and $n$,
\begin{equation}
\hyperlabel{sub:corollary-4m-eq1}
m \in n \iff m \subset n
\end{equation}
and
\begin{equation}
\hyperlabel{sub:corollary-4m-eq2}
m \ineq n \iff m \subseteq n.
\end{equation}
\end{corollary}
2023-08-09 13:39:41 +00:00
\begin{proof}
2023-08-09 13:39:41 +00:00
\paragraph{\eqref{sub:corollary-4m-eq1}}%
2023-08-09 13:39:41 +00:00
We prove both directions of the biconditional specified in
\eqref{sub:corollary-4m-eq1}:
2023-08-09 13:39:41 +00:00
\subparagraph{($\Rightarrow$)}%
2023-08-09 13:39:41 +00:00
Suppose $m \in n$ and $t \in m$.
By \nameref{sub:theorem-4f}, $n$ is a \nameref{ref:transitive-set}.
Therefore $t \in n$.
Hence $m \subseteq n$.
Since $m \in n$, \nameref{sub:linear-ordering-natural-numbers} implies
$m \neq n$.
Thus, by definition of \nameref{ref:proper-subset}, $m \subset n$.
2023-08-09 13:39:41 +00:00
\subparagraph{($\Leftarrow$)}%
2023-08-09 13:39:41 +00:00
Suppose $m \subset n$.
By \nameref{sub:linear-ordering-natural-numbers}, exactly one of
$$m \in n, \quad m = n, \quad n \in m$$ holds.
By definition of \nameref{ref:proper-subset}, $m \subseteq n$ and
$m \neq n$.
Furthermore, it cannot be that $n \in m$ since otherwise $n \in n$,
contradicting \nameref{sub:lemma-4l-b}.
Thus $m \in n$ is the only possibility.
2023-08-09 13:39:41 +00:00
\paragraph{\eqref{sub:corollary-4m-eq2}}%
2023-08-09 13:39:41 +00:00
We prove both directions of the biconditional specified in
\eqref{sub:corollary-4m-eq2}:
2023-08-09 13:39:41 +00:00
\subparagraph{($\Rightarrow$)}%
2023-08-09 13:39:41 +00:00
Suppose $m \ineq n$.
By definition, $m \in n$ or $m = n$.
Let $p \in m$.
Then $p \in m \in n$ or $p \in m = n$.
By \nameref{sub:theorem-4f}, $n$ is a \nameref{ref:transitive-set}.
Thus $p \in n$ in either case.
Hence $m \subseteq n$.
2023-08-09 13:39:41 +00:00
\subparagraph{($\Leftarrow$)}%
2023-08-09 13:39:41 +00:00
Suppose $m \subseteq n$.
By \nameref{sub:linear-ordering-natural-numbers}, exactly one of
$$m \in n, \quad m = n, \quad n \in m$$ holds.
But it cannot be that $n \in m$ since that would imply $n \in n$,
contradicting \nameref{sub:lemma-4l-b}.
Therefore $m \in n$ or $m = n$.
Hence $m \ineq n$.
2023-08-09 13:39:41 +00:00
\end{proof}
\subsection{\verified{Theorem 4N}}%
\hyperlabel{sub:theorem-4n}
2023-08-09 13:39:41 +00:00
\begin{theorem}[4N]
For any natural numbers $n$, $m$, and $p$,
\begin{equation}
m \in n \iff m + p \in n + p. \tag{i}
\end{equation}
If, in addition, $p \neq 0$, then
\begin{equation}
m \in n \iff m \cdot p \in n \cdot p. \tag{ii}
\end{equation}
2023-08-09 13:39:41 +00:00
\end{theorem}
\code{Enderton/Set/Chapter\_4}
{Enderton.Set.Chapter\_4.theorem\_4n\_i}
2023-08-09 13:39:41 +00:00
\lean{Std/Data/Nat/Lemmas}{Nat.add\_lt\_add\_iff\_right}
2023-08-08 14:13:39 +00:00
\code{Enderton/Set/Chapter\_4}
{Enderton.Set.Chapter\_4.theorem\_4n\_ii}
2023-08-08 14:13:39 +00:00
\lean{Init/Data/Nat/Basic}{Nat.mul\_lt\_mul\_of\_pos\_right}
2023-08-09 13:39:41 +00:00
\begin{proof}
\paragraph{(i)}%
\hyperlabel{par:theorem-4n-i}
Let $m$ and $n$ be \nameref{ref:natural-number}s.
\subparagraph{($\Rightarrow$)}%
\hyperlabel{spar:theorem-4n-i-right}
Suppose $m \in n$.
Let $$S = \{p \in \omega \mid m + p \in n + p\}.$$
It trivially follows that $0 \in S$.
Next, suppose $p \in S$.
That is, suppose $m + p \in n + p$.
By \nameref{sub:lemma-4l-a}, this holds if and only if
$(m + p)^+ \in (n + p)^+$.
\nameref{sub:theorem-4i} then implies that $m + p^+ \in n + p^+$ meaning
$p^+ \in S$.
Thus $S$ is an \nameref{ref:inductive-set}.
Hence \nameref{sub:theorem-4b} implies $S = \omega$.
Therefore, for all $p \in \omega$, $m \in n$ implies $m + p \in n + p$.
\subparagraph{($\Leftarrow$)}%
Let $p$ be a natural number and suppose $m + p \in n + p$.
By the \nameref{sub:trichotomy-law-natural-numbers}, there are two
cases to consider regarding how $m$ and $n$ relate to one another:
\vspace{8pt}
\textbf{Case 1}: Suppose $m = n$.
Then $m + p \in n + p = m + p$.
\nameref{sub:lemma-4l-b} shows this is impossible.
\vspace{8pt}
\textbf{Case 2}: Suppose $n \in m$.
Then \nameref{spar:theorem-4n-i-right} indicates $n + p \in m + p$.
But this contradicts \nameref{sub:trichotomy-law-natural-numbers} since,
by hypothesis, $m + p \in n + p$.
\vspace{8pt}
\textbf{Conclusion}: By trichotomy, it follows $m \in n$.
\paragraph{(ii)}%
\hyperlabel{par:theorem-4n-ii}
Let $m$ and $n$ be \nameref{ref:natural-number}s.
\subparagraph{($\Rightarrow$)}%
\hyperlabel{spar:theorem-4n-ii-right}
Suppose $m \in n$.
Let $$S = \{p \in \omega \mid m \cdot p^+ \in n \cdot p^+\}.$$
$0 \in S$ by \nameref{sub:right-multiplicative-identity}.
Next, suppose $p \in S$.
That is, $m \cdot p^+ \in n \cdot p^+$.
Then
\begin{align*}
m \cdot p^{++}
& = m \cdot p^+ + m & \textref{sub:theorem-4j} \\
& \in n \cdot p^+ + m & \textref{par:theorem-4n-i} \\
& = m + n \cdot p^+ & \textref{sub:theorem-4k-2} \\
& \in n + n \cdot p^+ & \textref{par:theorem-4n-i} \\
& = n \cdot p^+ + n & \textref{sub:theorem-4k-2} \\
& = n \cdot p^{++}. & \textref{sub:theorem-4j}
\end{align*}
Therefore $p^+ \in S$.
Thus $S$ is an \nameref{ref:inductive-set}.
Hence \nameref{sub:theorem-4b} implies $S = \omega$.
By \nameref{sub:theorem-4c}, every natural number except 0 is the
successor of some natural number.
Therefore, for all $p \in \omega$ such that $p \neq 0$, $m \in n$
implies $m \cdot p \in n \cdot p$.
\subparagraph{($\Leftarrow$)}%
Let $p \neq 0$ be a natural number and suppose $m \cdot p \in n \cdot p$.
By the \nameref{sub:trichotomy-law-natural-numbers}, there are two
cases to consider regarding how $m$ and $n$ relate to one another:
\vspace{8pt}
\textbf{Case 1}: Suppose $m = n$.
Then $m \cdot p \in n \cdot p = m \cdot p$.
\nameref{sub:lemma-4l-b} shows this is impossible.
\vspace{8pt}
\textbf{Case 2}: Suppose $n \in m$.
Then \nameref{spar:theorem-4n-ii-right} indicates
$n \cdot p \in m \cdot p$.
But this contradicts \nameref{sub:trichotomy-law-natural-numbers} since,
by hypothesis, $m \cdot p \in n \cdot p$.
\vspace{8pt}
\textbf{Conclusion}: By trichotomy, it follows $m \in n$.
\end{proof}
2023-08-08 14:13:39 +00:00
\subsection{\verified{Corollary 4P}}%
2023-08-09 13:39:41 +00:00
\hyperlabel{sub:corollary-4p}
2023-08-08 14:13:39 +00:00
2023-08-09 13:39:41 +00:00
\begin{corollary}[4P]
The following cancellation laws hold for $m$, $n$, and $p$ in $\omega$:
\begin{align}
m + p = n + p & \Rightarrow m = n,
& \hyperlabel{sub:corollary-4p-eq1} \\
m \cdot p = n \cdot p \land p \neq 0 & \Rightarrow m = n.
& \hyperlabel{sub:corollary-4p-eq2}
\end{align}
\end{corollary}
2023-08-08 14:13:39 +00:00
\code{Enderton/Set/Chapter\_4}
{Enderton.Set.Chapter\_4.corollary\_4p\_i}
2023-08-08 14:13:39 +00:00
\lean{Init/Data/Nat/Basic}{Nat.add\_right\_cancel}
\code{Common/Nat/Basic}{Nat.mul\_right\_cancel}
2023-08-09 13:39:41 +00:00
\begin{proof}
2023-08-08 14:13:39 +00:00
2023-08-09 13:39:41 +00:00
\paragraph{\eqref{sub:corollary-4p-eq1}}%
2023-08-08 14:13:39 +00:00
2023-08-09 13:39:41 +00:00
Suppose $m + p = n + p$.
2023-08-08 14:56:13 +00:00
By the \nameref{sub:trichotomy-law-natural-numbers}, there are two
2023-08-09 13:39:41 +00:00
cases to consider regarding how $m$ and $n$ relate to one another.
If $m \in n$, then \nameref{sub:theorem-4n} implies $m + p \in n + p$.
If $n \in m$, then \nameref{sub:theorem-4n} implies $n + p \in m + p$.
Both of these contradict the \nameref{sub:trichotomy-law-natural-numbers}
of $m + p$ and $n + p$.
Thus $m = n$ is the only remaining possibility.
2023-08-08 14:13:39 +00:00
2023-08-09 13:39:41 +00:00
\paragraph{\eqref{sub:corollary-4p-eq2}}%
2023-08-08 14:13:39 +00:00
2023-08-09 13:39:41 +00:00
Suppose $m \cdot p = n \cdot p$ and $p \neq 0$.
2023-08-08 14:56:13 +00:00
By the \nameref{sub:trichotomy-law-natural-numbers}, there are two
2023-08-09 13:39:41 +00:00
cases to consider regarding how $m$ and $n$ relate to one another.
If $m \in n$, then \nameref{sub:theorem-4n} implies
$m \cdot p \in n \cdot p$.
If $n \in m$, then \nameref{sub:theorem-4n} implies
2023-08-08 14:13:39 +00:00
$n \cdot p \in m \cdot p$.
2023-08-09 13:39:41 +00:00
Both of these contradict the \nameref{sub:trichotomy-law-natural-numbers}
of $m \cdot p$ and $n \cdot p$.
Thus $m = n$ is the only remaining possibility.
2023-08-08 14:56:13 +00:00
2023-08-09 13:39:41 +00:00
\end{proof}
\subsection{\verified{%
Well Ordering of \texorpdfstring{$\omega$}{Natural Numbers}}}%
\hyperlabel{sub:well-ordering-natural-numbers}
2023-08-09 13:39:41 +00:00
\begin{theorem}
Let $A$ be a nonempty subset of $\omega$.
Then there is some $m \in A$ such that $m \ineq n$ for all $n \in A$.
\end{theorem}
\code{Bookshelf/Enderton/Set/Chapter\_4}
{Enderton.Set.Chapter\_4.well\_ordering\_nat}
\lean{Mathlib/SetTheory/Ordinal/Basic}{WellOrder}
\begin{note}
This proof was written a few days after reading Enderton's proof as a means
of ensuring I remember the main arguments.
\end{note}
2023-08-09 13:39:41 +00:00
\begin{proof}
Let $A$ be a nonempty subset of $\omega$.
For the sake of contradiction, suppose $A$ does not have a least element.
It then suffices to prove that the complement of $A$ equals $\omega$.
If we do so, then $A = \emptyset$, a contradiction.
Define
\begin{equation}
\hyperlabel{sub:well-ordering-natural-numbers-eq1}
S = \{n \in \omega \mid (\forall m \in n) m \not\in A\}.
\end{equation}
We prove $S$ is an \nameref{ref:inductive-set} by showing that (i)
$0 \in S$ and (ii) if $n \in S$, then $n^+ \in S$.
Afterward we show that $\omega - A = \omega$, completing the proof.
\paragraph{(i)}%
\hyperlabel{par:well-ordering-natural-numbers-i}
It vacuously holds that $0 \in S$.
\paragraph{(ii)}%
\hyperlabel{par:well-ordering-natural-numbers-ii}
Suppose $n \in S$.
We want to prove that $$\forall m, m \in n^+ \Rightarrow m \not\in A.$$
To this end, let $m \in \omega$ such that $m \in n^+$.
By definition of the \nameref{ref:successor}, $m \in n$ or $m = n$.
If the former, $n \in S$ implies $m \not\in A$.
If the latter, it isn't possible for $n \in A$ since the
\nameref{sub:trichotomy-law-natural-numbers} would otherwise imply
$n$ is the least element of $A$, which is assumed to not exist.
Hence $n^+ \in S$.
\paragraph{Conclusion}%
By \nameref{par:well-ordering-natural-numbers-i} and
\nameref{par:well-ordering-natural-numbers-ii}, $S$ is an inductive set.
Since $S \subseteq \omega$, \nameref{sub:theorem-4b} implies $S = \omega$.
But this immediately implies $\omega = \omega - A$ meaning $A$ is the
empty set.
2023-08-09 13:39:41 +00:00
\end{proof}
\subsection{\unverified{Corollary 4Q}}%
\hyperlabel{sub:corollary-4q}
2023-08-09 13:39:41 +00:00
\begin{corollary}[4Q]
There is no function $f \colon \omega \rightarrow \omega$ such that
$f(n^+) \in f(n)$ for every natural number $n$.
\end{corollary}
2023-08-09 13:39:41 +00:00
\begin{proof}
Let $f \colon \omega \rightarrow \omega$.
Then $\emptyset \subset \ran{f} \subseteq \omega$.
By the \nameref{sub:well-ordering-natural-numbers}, $\ran{f}$ must have
a least element.
Therefore it isn't possible $f(n^+) \in f(n)$ for all $n \in \omega$.
2023-08-09 13:39:41 +00:00
\end{proof}
\subsection{\verified{%
Strong Induction Principle for \texorpdfstring{$\omega$}{Natural Numbers}}}%
\hyperlabel{sub:strong-induction-principle-natural-numbers}
2023-08-09 13:39:41 +00:00
\begin{theorem}
Let $A$ be a subset of $\omega$, and assume that for every $n \in \omega$,
\begin{equation}
\hyperlabel{sub:strong-induction-principle-natural-numbers-eq1}
\text{if every number less than } n \text{ is in } A,
\text{ then } n \in A.
\end{equation}
2023-08-09 13:39:41 +00:00
Then $A = \omega$.
\end{theorem}
\code{Bookshelf/Enderton/Set/Chapter\_4}
{Enderton.Set.Chapter\_4.strong\_induction\_principle\_nat}
2023-08-09 13:39:41 +00:00
\begin{proof}
For the sake of contradiction, suppose $\omega - A$ is a nonempty set.
By \nameref{sub:well-ordering-natural-numbers}, there exists a least element
$m \in \omega - A$.
Then every number less than $m$ is in $A$.
But then \eqref{sub:strong-induction-principle-natural-numbers-eq1} implies
$m \in A$, a contradiction.
Thus $\omega - A$ is an empty set meaning $A = \omega$.
2023-08-09 13:39:41 +00:00
\end{proof}
\section{Exercises 4}%
\hyperlabel{sec:exercises-4}
\subsection{\verified{Exercise 4.1}}%
2023-07-21 19:40:38 +00:00
\hyperlabel{sub:exercise-4.1}
2023-08-09 13:39:41 +00:00
Show that $1 \neq 3$ i.e., that $\emptyset^+ \neq \emptyset^{+++}$.
2023-08-09 13:39:41 +00:00
\code*{Bookshelf/Enderton/Set/Chapter\_4}
{Enderton.Set.Chapter\_4.exercise\_4\_1}
2023-08-09 13:39:41 +00:00
\begin{proof}
By definition,
\begin{align*}
1 & = \{\emptyset\} \\
3 & = \{\emptyset, \{\emptyset\}, \{\emptyset, \{\emptyset\}\}\}.
\end{align*}
By the \nameref{ref:extensionality-axiom}, these two sets are trivially not
equal to one another.
\end{proof}
\subsection{\unverified{Exercise 4.2}}%
\hyperlabel{sub:exercise-4.2}
2023-08-09 13:39:41 +00:00
Show that if $a$ is a transitive set, then $a^+$ is also a transitive set.
2023-08-09 13:39:41 +00:00
\begin{proof}
Suppose $a$ is a \nameref{ref:transitive-set}.
By \nameref{sub:theorem-4e}, it follows $\bigcup \left(a^+\right) = a$.
By definition of a \nameref{ref:successor}, $a^+ = a \cup \{a\}$.
Thus it immediately follows
$$\bigcup \left(a^+\right) = a \subseteq a \cup \{a\} = a^+.$$
Therefore $a^+$ is indeed a transitive set.
\end{proof}
\subsection{\unverified{Exercise 4.3}}%
\hyperlabel{sub:exercise-4.3}
2023-08-09 13:39:41 +00:00
\begin{enumerate}[(a)]
\item Show that if $a$ is a transitive set, then $\powerset{a}$ is also a
transitive set.
\item Show that if $\powerset{a}$ is a transitive set, then $a$ is also a
transitive set.
\end{enumerate}
2023-08-09 13:39:41 +00:00
\begin{proof}
2023-08-09 13:39:41 +00:00
\paragraph{(a)}%
2023-08-09 13:39:41 +00:00
Suppose $a$ is a \nameref{ref:transitive-set}.
We show that $\bigcup \powerset{a} \subseteq \powerset{a}$.
Let $t \in \bigcup \powerset{a}$.
By definition of the \nameref{ref:power-set}, there exists some
$X \subseteq a$ such that $t \in X$.
Thus $t \in a$.
Because $a$ is a transitive set, every member of $t$ is a member of $a$.
In other words, $t \subseteq a$.
Equivalently, $t \in \powerset{a}$.
2023-08-09 13:39:41 +00:00
\paragraph{(b)}%
2023-08-09 13:39:41 +00:00
Suppose $\powerset{a}$ is a transitive set.
We show that $\bigcup a \subseteq a$.
Let $t \in \bigcup a$.
Then there exists some $b \in a$ such that $t \in b$.
Since $\{b\}$ is a member of $\powerset{a}$ and $\powerset{a}$ is a
transitive set, $b \in \powerset{a}$.
That is, $b \subseteq a$.
Thus $t \in a$.
2023-08-09 13:39:41 +00:00
\end{proof}
\subsection{\unverified{Exercise 4.4}}%
\hyperlabel{sub:exercise-4.4}
2023-08-09 13:39:41 +00:00
Show that if $a$ is a transitive set, then $\bigcup a$ is also a transitive set.
2023-08-09 13:39:41 +00:00
\begin{proof}
Suppose $a$ is a transitive set.
We show that $\bigcup\bigcup{a} \subseteq \bigcup a$.
Let $t \in \bigcup\bigcup{a}$.
Then there exists some $b \in \bigcup{a}$ such that $t \in b$.
Since $a$ is transitive, $\bigcup{a} \subseteq a$.
Thus $b \in a$ and $t \in \bigcup a$.
\end{proof}
\subsection{\unverified{Exercise 4.5}}%
\hyperlabel{sub:exercise-4.5}
2023-08-09 13:39:41 +00:00
Assume that every member of $\mathscr{A}$ is a transitive set.
\begin{enumerate}[(a)]
\item Show that $\bigcup\mathscr{A}$ is a transitive set.
\item Show that $\bigcap\mathscr{A}$ is a transitive set (assume that
$\mathscr{A}$ is nonempty).
\end{enumerate}
2023-08-09 13:39:41 +00:00
\begin{proof}
2023-08-09 13:39:41 +00:00
\paragraph{(a)}%
2023-08-09 13:39:41 +00:00
Suppose every member of $\mathscr{A}$ is a transitive set.
We show that $\bigcup\bigcup{\mathscr{A}} \subseteq \bigcup{\mathscr{A}}$.
Let $t \in \bigcup\bigcup{\mathscr{A}}$.
Then there exists some $b_1 \in \bigcup{\mathscr{A}}$ such that
$t \in b_1$.
Likewise there exists some $b_2 \in \mathscr{A}$ such that $b_1 \in b_2$.
By hypothesis, $b_2$ is transitive meaning $t \in b_2$.
Thus $t \in \bigcup{\mathscr{A}}$.
2023-08-09 13:39:41 +00:00
\paragraph{(b)}%
2023-08-09 13:39:41 +00:00
Suppose every member of nonempty set $\mathscr{A}$ is a transitive set.
We show that $\bigcup\bigcap{\mathscr{A}} \subseteq \bigcap{\mathscr{A}}$.
Let $t \in \bigcup\bigcap{\mathscr{A}}$.
Then there exists some $b \in \bigcap{\mathscr{A}}$ such that $t \in b$.
Thus $b$ is a member of every member of $\mathscr{A}$.
By hypothesis, every member of $\mathscr{A}$ is a transitive set meaning
$t$ must be a member of every member of $\mathscr{A}$.
In other words, $t \in \bigcap{\mathscr{A}}$.
2023-08-09 13:39:41 +00:00
\end{proof}
\subsection{\unverified{Exercise 4.6}}%
\hyperlabel{sub:exercise-4.6}
2023-08-09 13:39:41 +00:00
Prove the converse to \nameref{sub:theorem-4e}: If
$\bigcup \left(a^+\right) = a$, then $a$ is a transitive set.
2023-08-09 13:39:41 +00:00
\begin{proof}
Let $a$ be a set such that $\bigcup \left(a^+\right) = a$.
Then
\begin{align*}
\bigcup \left(a^+\right)
& = \bigcup (a \cup \{a\}) & \textref{ref:successor} \\
& = \bigcup a \cup \bigcup \{a\} & \textref{sub:exercise-2.21} \\
& = a. & \text{by hypothesis}
\end{align*}
But $$\bigcup{a} \subseteq \bigcup a \cup \bigcup \{a\} = a.$$
Thus $a$ is indeed a \nameref{ref:transitive-set}.
\end{proof}
\subsection{\unverified{Exercise 4.7}}%
\hyperlabel{sub:exercise-4.7}
2023-08-09 13:39:41 +00:00
Complete part 4 of the proof of the
\nameref{sub:recursion-theorem-natural-numbers}.
2023-08-09 13:39:41 +00:00
\begin{proof}
Refer to \nameref{par:recursion-theorem-natural-numbers-iv}.
\end{proof}
\subsection{\unverified{Exercise 4.8}}%
\hyperlabel{sub:exercise-4.8}
2023-08-09 13:39:41 +00:00
Let $f$ be a one-to-one function from $A$ into $A$, and assume that
$c \in A - \ran{f}$.
Define $h \colon \omega \rightarrow A$ by recursion:
\begin{align*}
h(0) & = c, \\
h(n^+) & = f(h(n)).
\end{align*}
Show that $h$ is one-to-one.
\begin{proof}
Let
$$S = \{x \in \omega \mid \forall y,
\left[ h(x) = h(y) \Rightarrow x = y \right]\}.$$
We prove that (i) $S$ is an \nameref{ref:inductive-set} and (ii) that $h$ is
one-to-one.
\paragraph{(i)}%
\hyperlabel{par:exercise-4.8-i}
2023-08-09 13:39:41 +00:00
We first show that $0 \in S$.
Suppose there exists some $n \in \omega$ such that $h(0) = c = h(n)$.
For the sake of contradiction, suppose $n \neq 0$.
By \nameref{sub:theorem-4c}, there exists some $m$ such that $m^+ = n$.
Then $$h(n) = h(m^+) = f(h(m)) = c.$$
But $c \in A - \ran{f}$, meaning the previous identity is an
impossibility.
Thus $n = 0$, i.e. $0 \in S$.
2023-08-09 13:39:41 +00:00
Next, suppose $y, n \in \omega$ such that $n \in S$ and $h(n^+) = h(y)$.
We must show $n^+ \in S$.
There are two cases to consider:
2023-08-09 13:39:41 +00:00
\subparagraph{Case 1}%
2023-08-09 13:39:41 +00:00
Suppose $y = 0$.
Then $h(y) = h(0) = c = h(n^+) = f(h(n))$.
Since $c \in A - \ran{f}$, $f(h(n)) \neq c$.
Thus $y \neq 0$, a contradiction.
2023-08-09 13:39:41 +00:00
\subparagraph{Case 2}%
2023-08-09 13:39:41 +00:00
Suppose $y \neq 0$.
\nameref{sub:theorem-4c} implies there exists some $z \in \omega$ such
that $z^+ = y$.
Thus $$h(n^+) = f(h(n)) = f(h(z)) = h(z^+).$$
But $f$ is one-to-one meaning $h(n) = h(z)$.
Since $n \in S$, $h(n) = h(z)$ implies $n = z$ which in turn implies
$n^+ = z^+ = y$.
Thus $n^+$ is in $S$.
2023-08-09 13:39:41 +00:00
\paragraph{(ii)}%
By \nameref{par:exercise-4.8-i}, $S \subseteq \omega$ is an inductive set.
Then \nameref{sub:theorem-4b} states $S = \omega$.
Hence $h$ is one-to-one.
\end{proof}
\subsection{\unverified{Exercise 4.9}}%
\hyperlabel{sub:exercise-4.9}
2023-08-09 13:39:41 +00:00
Let $f$ be a function from $B$ into $B$, and assume that $A \subseteq B$.
We have two possible methods for constructing the "closure" $C$ of $A$ under
$f$.
First define $C^*$ to be the intersection of the closed supersets of $A$:
\begin{equation}
\hyperlabel{sub:exercise-4.9-eq1}
C^* = \bigcap\{X \mid
A \subseteq X \subseteq B \land \img{f}{X} \subseteq X\}.
\end{equation}
Alternatively, we could apply the recursion theorem to obtain the function $h$
for which
\begin{align*}
h(0) & = A, \\
h(n^+) & = h(n) \cup \img{f}{h(n)}.
\end{align*}
Clearly $h(0) \subseteq h(1) \subseteq \cdots$; define $C_*$ to be
$\bigcup\ran{h}$; in other words
\begin{equation}
\hyperlabel{sub:exercise-4.9-eq2}
C_* = \bigcup_{i \in \omega} h(i).
\end{equation}
Show that $C^* = C_*$.
[\textit{Suggestion}:
To show that $C^* \subseteq C_*$, show that $\img{f}{C_*} \subseteq C_*$.
To show that $C_* \subseteq C^*$, use induction to show that
$h(n) \subseteq C^*$.]
2023-08-09 13:39:41 +00:00
\begin{proof}
2023-08-09 13:39:41 +00:00
We show that $C^* \subseteq C_*$ and $C^* \supseteq C_*$.
2023-08-09 13:39:41 +00:00
\paragraph{($\subseteq$)}%
2023-08-09 13:39:41 +00:00
It suffices to show $\img{f}{C_*} \subseteq C_*$ since then $C_*$ is a
member of the family of sets being intersected in
\eqref{sub:exercise-4.9-eq1}.
Let $t \in \img{f}{C_*}$.
By definition of the \nameref{ref:image} of a set, there exists some
$u \in C_*$ such that $f(u) = t$.
By \eqref{sub:exercise-4.9-eq2}, there exists some $i \in \omega$ such
that $u \in h(i)$.
Then $t \in \img{f}{h(i)} \subseteq h(i) \cup \img{f}{h(i)} = h(i^+)$.
Therefore \eqref{sub:exercise-4.9-eq2} indicates $t \in C_*$.
\paragraph{($\supseteq$)}%
Define $$S = \{n \in \omega \mid h(n) \subseteq C^*\}.$$
We prove that (i) $0 \in S$ and (ii) if $n \in S$ then $n^+ \in S$.
Afterward we prove that (iii) $C_* \subseteq C^*$.
\subparagraph{(i)}%
\label{spar:exercise-4.9-i}
By construction, $h(0) = A$.
It trivially follows that $A \subseteq C^*$ by
\eqref{sub:exercise-4.9-eq1}.
Thus $h(0) \subseteq C^*$ meaning $0 \in S$.
\subparagraph{(ii)}%
\label{spar:exercise-4.9-ii}
Suppose $n \in S$.
That is, $h(n) \subseteq C^*$.
We must prove that $h(n^+) \subseteq C^*$.
Let $$t \in h(n^+) = h(n) \cup \img{f}{h(n)}.$$
Then either $t \in h(n)$ or $t \in \img{f}{h(n)}$.
If $t \in h(n)$, it immediately follows $t \in C^*$.
If $t \in \img{f}{h(n)}$, then the definition of the \nameref{ref:image}
of a set implies there exists some $u \in h(n)$ such that $f(u) = t$.
Since $h(n) \subseteq C^*$, \eqref{sub:exercise-4.9-eq1} indicates that
$$\forall X, A \subseteq X \subseteq B \land
\img{f}{X} \subseteq X \Rightarrow u \in X.$$
But then closure under image yields
$$\forall X, A \subseteq X \subseteq B \land
\img{f}{X} \subseteq X \Rightarrow f(u) \in X.$$
Since $f(u) = t$, $t \in C^*$.
Thus $h(n^+) \subseteq C^*$, i.e. $n^+ \in S$.
\subparagraph{(iii)}%
By \nameref{spar:exercise-4.9-i} and \nameref{spar:exercise-4.9-ii},
$S \subseteq \omega$ is an \nameref{ref:inductive-set}.
By \nameref{sub:theorem-4b}, $S = \omega$.
That is, for all $n \in \omega$, $h(n) \subseteq C^*$.
Thus $$C_* = \bigcup_{i \in \omega} h(i) \subseteq C^*.$$
2023-08-09 13:39:41 +00:00
\paragraph{Conclusion}%
2023-08-09 13:39:41 +00:00
Since $C^* \subseteq C_*$ and $C_* \subseteq C^*$, it follows $C^* = C_*$.
2023-08-09 13:39:41 +00:00
\end{proof}
\subsection{\unverified{Exercise 4.10}}%
\hyperlabel{sub:exercise-4.10}
2023-08-09 13:39:41 +00:00
In Exercise 9, assume that $B$ is the set of real numbers, $f(x) = x^2$, and
$A$ is the closed interval $\icc{\frac{1}{2}}{1}$.
What is the set called $C^*$ and $C_*$?
2023-08-09 13:39:41 +00:00
\begin{proof}
By \nameref{sub:exercise-4.9}, $C^* = C_*$.
By definition,
$$C^* = \bigcap \{X \mid
\icc{\frac{1}{2}}{1} \subseteq X \subseteq \mathbb{R} \land
\img{f}{X} \subseteq X\}.$$
Since $f(x)$ converges to $0$ for all values $x < 1$, it follows
$C^* = C_* = \ioc{0}{1}$.
\end{proof}
\subsection{\unverified{Exercise 4.11}}%
\hyperlabel{sub:exercise-4.11}
2023-08-09 13:39:41 +00:00
In Exercise 9, assume that $B$ is the set of real numbers, $f(x) = x - 1$, and
$A = \{0\}$.
What is the set called $C^*$ and $C_*$?
2023-08-09 13:39:41 +00:00
\begin{proof}
By \nameref{sub:exercise-4.9}, $C^* = C_*$.
By definition, $$C_* = \bigcup_{i \in \omega} h(i)$$ where
\begin{align*}
h(0) & = A = \{0\}, \\
h(n^+) & = h(n) \cup \img{f}{h(n)}.
\end{align*}
Thus $C_* = C^* = \{x \in \mathbb{Z} \mid x \leq 0\}$.
\end{proof}
\subsection{\sorry{Exercise 4.12}}%
\hyperlabel{sub:exercise-4.12}
2023-08-09 13:39:41 +00:00
Formulate an analogue to Exercise 9 for a function
$f \colon B \times B \rightarrow B$.
2023-08-09 13:39:41 +00:00
\begin{proof}
TODO
\end{proof}
\subsection{\verified{Exercise 4.13}}%
\hyperlabel{sub:exercise-4.13}
2023-08-09 13:39:41 +00:00
Let $m$ and $n$ be natural numbers such that $m \cdot n = 0$.
Show that either $m = 0$ or $n = 0$.
2023-08-09 13:39:41 +00:00
\code*{Bookshelf/Enderton/Set/Chapter\_4}
{Enderton.Set.Chapter\_4.exercise\_4\_13}
2023-08-09 13:39:41 +00:00
\begin{proof}
Suppose $m \cdot n = 0$.
For the sake of contradiction, assume $m \neq 0$ and $n \neq 0$.
By \nameref{sub:theorem-4c}, there exists some $p, q \in \omega$ such that
$p^+ = m$ and $q^+ = n$.
Thus
\begin{align*}
m \cdot n
& = m \cdot q^+ \\
& = m \cdot q + m & \textref{sub:theorem-4j} \\
& = m \cdot q + p^+ \\
& = (m \cdot q + p)^+. & \textref{sub:theorem-4i}
\end{align*}
By definition of a \nameref{ref:successor},
$m \cdot n = (m \cdot q + p)^+ \neq \emptyset = 0$, a contradiction.
Therefore our original assumption was wrong.
Hence $m = 0$ or $n = 0$.
\end{proof}
\subsection{\verified{Exercise 4.14}}%
\hyperlabel{sub:exercise-4.14}
2023-08-09 13:39:41 +00:00
Call a natural number \textit{even} if it has the form $2 \cdot m$ for some
$m$.
Call it \textit{odd} if it has the form $(2 \cdot p) + 1$ for some $p$.
Show that each natural number is either even or odd, but never both.
2023-08-09 13:39:41 +00:00
\code*{Bookshelf/Enderton/Set/Chapter\_4}
{Enderton.Set.Chapter\_4.exercise\_4\_14}
2023-08-09 13:39:41 +00:00
\begin{proof}
2023-08-09 13:39:41 +00:00
Let $$S = \{n \in \omega \mid n \text{ is even or odd but not both}\}.$$
We show that (i) $0 \in S$ and (ii) if $n \in S$ then $n^+ \in S$ as well.
Afterward we prove (iii) that the theorem statement holds.
2023-08-09 13:39:41 +00:00
\paragraph{(i)}%
\hyperlabel{par:exercise-4.14a-i}
2023-08-09 13:39:41 +00:00
$0$ is even since $2 \cdot 0 = 0$ by \nameref{sub:theorem-4j}.
Furthermore, $0$ is not odd since that would imply there exists some
$p$ such that $(2 \cdot p)^+ = 0$.
By definition of \nameref{ref:successor}, this is not possible.
Thus $0 \in S$.
2023-08-09 13:39:41 +00:00
\paragraph{(ii)}%
\hyperlabel{par:exercise-4.14a-ii}
2023-08-09 13:39:41 +00:00
Suppose $n \in S$.
Then $n$ is even or odd but not both.
\subparagraph{Case 1}%
Suppose $n$ is even and not odd.
Then there exists some $m \in \omega$ such that $2 \cdot m = n$.
Therefore $(2 \cdot m)^+ = n^+$.
Hence $n^+$ is odd.
For the sake of contradiction, suppose $n^+$ is even.
Then there exists some $p$ such that $2 \cdot p = n^+$.
We consider two additional cases:
\vspace{8pt}\quad
\textbf{Case 1a}: Suppose $p = 0$.
Then, by \nameref{sub:theorem-4j}, $2 \cdot p = 0 = n^+$.
By definition of \nameref{ref:successor}, this is not possible.
\vspace{8pt}\quad
\textbf{Case 1b}: Suppose $p \neq 0$.
Then \nameref{sub:theorem-4c} implies there exists some $q$ such that
$q^+ = p$.
Thus
\begin{align*}
n^+
& = 2 \cdot p \\
& = 2 \cdot q^+ \\
& = q^+ + q^+ \\
& = (q^+ + q)^+. & \textref{sub:theorem-4i}
\end{align*}
By \nameref{sub:theorem-4d}, the \nameref{ref:successor} operation is
one-to-one meaning $n = q^+ + q$.
But then
\begin{align*}
n
& = q^+ + q \\
& = q + q^+ & \textref{sub:theorem-4k-2} \\
& = (q + q)^+ & \textref{sub:theorem-4i} \\
& = (2 \cdot q)^+,
\end{align*}
indicating $n$ is odd.
This is a contradiction.
\vspace{8pt}\quad
\textbf{Conclusion}: Since the above two cases are exhaustive, it
follows our original assumption is wrong.
That is, $n^+$ is odd but not even.
\subparagraph{Case 2}%
Suppose $n$ is odd and not even.
Then there exists some $p \in \omega$ such that $(2 \cdot p)^+ = n$.
Therefore $(2 \cdot p)^{++} = (2 \cdot p^+) = n^+$.
Hence $n^+$ is even.
For the sake of contradiction, suppose $n^+$ is odd.
Then there exists some $q$ such that $(2 \cdot q)^+ = n^+$.
By \nameref{sub:theorem-4d}, the \nameref{ref:successor} operation is
one-to-one meaning $2 \cdot q = n$.
But this implies $n$ is even, a contradiction.
Thus our original assumption is wrong.
That is, $n^+$ is even but not odd.
\subparagraph{Conclusion}%
Since the foregoing cases are exhaustive, it follows $n^+ \in S$.
\paragraph{(iii)}%
By \nameref{par:exercise-4.14a-i} and \nameref{par:exercise-4.14a-ii},
$S$ is an \nameref{ref:inductive-set}.
By \nameref{sub:theorem-4b}, $S = \omega$.
Thus every natural number is either even or odd, but not both.
2023-08-09 13:39:41 +00:00
\end{proof}
\subsection{\verified{Exercise 4.15}}%
\hyperlabel{sub:exercise-4.15}
2023-08-09 13:39:41 +00:00
Complete the proof of \nameref{sub:theorem-4k-1}.
2023-08-09 13:39:41 +00:00
\begin{proof}
Refer to \nameref{sub:theorem-4k-1}.
\end{proof}
\subsection{\verified{Exercise 4.16}}%
\hyperlabel{sub:exercise-4.16}
2023-08-09 13:39:41 +00:00
Complete the proof of \nameref{sub:theorem-4k-5}.
2023-08-09 13:39:41 +00:00
\begin{proof}
Refer to \nameref{sub:theorem-4k-5}.
\end{proof}
\subsection{\verified{Exercise 4.17}}%
\hyperlabel{sub:exercise-4.17}
2023-08-09 13:39:41 +00:00
Prove that $m^{n+p} = m^n \cdot m^p$.
\code*{Bookshelf/Enderton/Set/Chapter\_4}
{Enderton.Set.Chapter\_4.exercise\_4\_17}
\lean{Data/Nat/Lemmas}{Nat.pow\_add}
2023-08-09 13:39:41 +00:00
\begin{proof}
2023-08-09 13:39:41 +00:00
Let $m$ and $n$ be \nameref{ref:natural-number}s and define
\begin{equation}
\hyperlabel{sub:exercise-4.17-eq1}
S = \{p \in \omega \mid m^{n+p} = m^n \cdot m^p\}.
\end{equation}
We prove that (i) $0 \in S$ and (ii) if $p \in S$ then $p^+ \in S$.
Afterwards we show that (iii) our theorem holds.
2023-08-09 13:39:41 +00:00
\paragraph{(i)}%
\hyperlabel{par:exercise-4.17-i}
2023-08-09 13:39:41 +00:00
Consider $m^{n+0}$:
\begin{align*}
m^{n+0}
& = m^n & \textref{sub:theorem-4i} \\
& = m^n \cdot 1 & \textref{sub:right-multiplicative-identity} \\
& = m^n \cdot m^0. & \textref{ref:exponentiation}
\end{align*}
Thus $0 \in S$.
2023-08-09 13:39:41 +00:00
\paragraph{(ii)}%
\hyperlabel{par:exercise-4.17-ii}
2023-08-09 13:39:41 +00:00
Suppose $p \in S$.
Now consider $m^{n+p^+}$:
\begin{align*}
m^{n+p^+}
& = m^{(n + p)^+} & \textref{sub:theorem-4i} \\
& = E_m(n + p) \cdot m & \textref{ref:exponentiation} \\
& = m^n \cdot m^p \cdot m & \eqref{sub:exercise-4.17-eq1} \\
& = m^n \cdot (m^p \cdot m) & \textref{sub:theorem-4k-4} \\
& = m^n \cdot m^{p^+}. & \textref{ref:exponentiation}
\end{align*}
Thus $p^+ \in S$.
2023-08-09 13:39:41 +00:00
\paragraph{(iii)}%
2023-08-09 13:39:41 +00:00
By \nameref{par:exercise-4.17-i} and \nameref{par:exercise-4.17-ii},
$S \subseteq \omega$ is an \nameref{ref:inductive-set}.
By \nameref{sub:theorem-4b}, $S = \omega$.
Thus for all $m, n, p \in \omega$, it follows that
$m^{n+p} = m^n \cdot m^p$.
2023-08-09 13:39:41 +00:00
\end{proof}
\subsection{\unverified{Exercise 4.18}}%
\hyperlabel{sub:exercise-4.18}
2023-08-09 13:39:41 +00:00
Simplify $\img{\in_\omega^{-1}}{\{7, 8\}}$.
2023-08-09 13:39:41 +00:00
\begin{proof}
By definition,
$$\in_\omega = \{\tuple{m, n} \in \omega \times \omega \mid m \in n\}.$$
Thus, the \nameref{ref:inverse} of $\in_\omega$ is
$$\in_\omega^{-1} =
\{\tuple{n, m} \in \omega \times \omega \mid m \in n\}.$$
Therefore $$\img{\in_\omega^{-1}}{\{7, 8\}} = \{6, 7\}.$$
2023-08-09 13:39:41 +00:00
\end{proof}
\subsection{\verified{Exercise 4.19}}%
\hyperlabel{sub:exercise-4.19}
2023-08-09 13:39:41 +00:00
Prove that if $m$ is a natural number and $d$ is a nonzero number, then there
exist numbers $q$ and $r$ such that $m = (d \cdot q) + r$ and $r$ is less
than $d$.
\code*{Bookshelf/Enderton/Set/Chapter\_4}
{Enderton.Set.Chapter\_4.exercise\_4\_19}
2023-08-09 13:39:41 +00:00
\begin{proof}
Let $d \in \omega$ such that $d \neq 0$.
Define
\begin{equation}
\hyperlabel{sub:exercise-4.18-eq1}
S = \{m \in \omega \mid
(\exists q, d \in \omega) m = (d \cdot q) + r \land r < d\}.
\end{equation}
We prove that $S$ is an \nameref{ref:inductive-set} by showing
(i) $0 \in S$ and (ii) if $m \in S$, then $m^+ \in S$.
Afterward we prove (iii) the theorem statement.
\paragraph{(i)}%
\hyperlabel{par:exercise-4.19-i}
Let $q = 0$ and $r = 0$.
We note $r < d$ by \textref{sub:zero-least-natural-number}.
Furthermore,
\begin{align*}
(d \cdot 0) + 0
& = d \cdot 0 & \textref{sub:theorem-4i} \\
& = 0. & \textref{sub:theorem-4j}
\end{align*}
Thus $0 \in S$.
\paragraph{(ii)}%
\hyperlabel{par:exercise-4.19-ii}
Suppose $m \in S$.
Then there exists $q, d \in \omega$ such that $m = (d \cdot q) + r$ and
$r < d$.
Then
\begin{align*}
m^+
& = ((d \cdot q) + r)^+ \\
& = (d \cdot q) + r^+. & \textref{sub:theorem-4i}
\end{align*}
By \nameref{sub:trichotomy-law-natural-numbers}, there are three cases to
consider:
\subparagraph{Case 1}%
Suppose $r^+ \in d$.
Then it immediately follows $m^+ \in S$.
\subparagraph{Case 2}%
Suppose $r^+ = d$.
Then
\begin{align*}
m^+
& = (d \cdot q) + r^+ \\
& = (d \cdot q) + d \\
& = d \cdot q^+ & \textref{sub:theorem-4j} \\
& = (d \cdot q^+) + 0. & \textref{sub:theorem-4i}
\end{align*}
Hence $m^+ \in S$.
\subparagraph{Case 3}%
Suppose $d \in r^+$.
Then, by definition of the \nameref{ref:successor}, $d \in r$ or
$d = r$.
But $r \in d$.
Thus, by \nameref{sub:trichotomy-law-natural-numbers}, a contradiction
is introduced.
Hence this case is not possible.
\subparagraph{Conclusion}%
Since the above cases are exhaustive, it follows $m^+ \in S$.
\paragraph{(iii)}%
By \nameref{par:exercise-4.19-i} and \nameref{par:exercise-4.19-ii},
$S \subseteq \omega$ is indeed an inductive set.
By \nameref{sub:theorem-4b}, $S = \omega$.
Thus for all $m, d \in \omega$ where $d \neq 0$, there exist numbers $q$
and $r$ such that $m = (d \cdot q) + r$ and $r < d$.
2023-08-09 13:39:41 +00:00
\end{proof}
2023-08-12 18:54:22 +00:00
\subsection{\unverified{Exercise 4.20}}%
\hyperlabel{sub:exercise-4.20}
2023-08-09 13:39:41 +00:00
Let $A$ be a nonempty subset of $\omega$ such that $\bigcup A = A$.
Show that $A = \omega$.
2023-08-09 13:39:41 +00:00
\begin{proof}
2023-08-12 18:54:22 +00:00
For the sake of contradiction, suppose $\omega - A$ is nonempty.
By \nameref{sub:well-ordering-natural-numbers}, there exists a least number
$m \in \omega$ of the set.
Let $n \in \omega$ such that $n > m$.
By \nameref{sub:members-natural-numbers}, $n$ is the set of all smaller
natural numbers.
Thus $m \in n$.
Therefore $n \not\in A$ since otherwise $m \in \bigcup A$ but $m \not\in A$.
Hence $\omega - A$ consists of all numbers greater than or equal to $m$.
There are now two cases to consider:
\paragraph{Case 1}%
Suppose $m = 0$.
Then $A = \emptyset$ by \nameref{sub:zero-least-natural-number}.
$A$ is assumed nonempty so this is a contradiction.
\paragraph{Case 2}%
Suppose $m \neq 0$.
Then \nameref{sub:theorem-4c} implies there exists some $p \in \omega$
such that $p^+ = m$.
Because $m$ is the least element of $\omega - A$, it follows $p \in A$ is
the largest element of $A$.
But since $p \not\in p$ by \nameref{sub:lemma-4l-b},
$p \not\in \bigcup A$, a contradiction.
\paragraph{Conclusion}%
Our above two cases are exhaustive.
Since both lead to contradictions, it follows our original assumption
must be wrong.
That is $\omega - A$ is empty, meaning $A = \omega$.
2023-08-09 13:39:41 +00:00
\end{proof}
\subsection{\unverified{Exercise 4.21}}%
\hyperlabel{sub:exercise-4.21}
2023-08-09 13:39:41 +00:00
Show that no natural number is a subset of any of its elements.
2023-08-09 13:39:41 +00:00
\begin{proof}
Let $n$ be a natural number.
Suppose $m \in n$.
By \nameref{sub:trichotomy-law-natural-numbers}, $n \neq m$ and
$n \not\in m$.
By \nameref{sub:corollary-4m}, $n \not\subseteq m$.
2023-08-09 13:39:41 +00:00
\end{proof}
\subsection{\verified{Exercise 4.22}}%
\hyperlabel{sub:exercise-4.22}
2023-08-09 13:39:41 +00:00
Show that for any natural numbers $m$ and $p$ we have $m \in m + p^+$.
\code*{Bookshelf/Enderton/Set/Chapter\_4}
{Enderton.Set.Chapter\_4.exercise\_4\_22}
2023-08-09 13:39:41 +00:00
\begin{proof}
Let $m$ be a natural number and $$S = \{p \in \omega \mid m \in m + p^+\}.$$
We prove that (i) $0 \in S$ and (ii) if $p \in S$, then $p^+ \in S$.
Afterward we prove (iii) the theorem statement.
\paragraph{(i)}%
\hyperlabel{par:exercise-4.22-i}
By definition of the \nameref{ref:successor}, $m^+ = m \cup \{m\}$.
Thus $m \in m^+ = m + 1$ (by \nameref{sub:successor-identity}).
Hence $0 \in S$.
\paragraph{(ii)}%
\hyperlabel{par:exercise-4.22-ii}
Suppose $p \in S$.
That is, $m \in m + p^+$.
By definition of the \nameref{ref:successor},
\begin{align*}
m + p^+
& \in (m + p^+) \cup \{m + p^+\} \\
& = (m + p^+)^+ \\
& = m + p^{++}. & \textref{sub:theorem-4i}.
\end{align*}
By \nameref{sub:theorem-4f}, $m + p^{++}$ is a
\nameref{ref:transitive-set}.
Therefore $m \in m + p^+ \in m + p^{++}$ implies $m \in m + p^{++}$.
Hence $p^+ \in S$.
\paragraph{(iii)}%
By \nameref{par:exercise-4.22-i} and \nameref{par:exercise-4.22-ii},
$S \subseteq \omega$ is an inductive set.
Thus \nameref{sub:theorem-4b} implies $S = \omega$.
Hence for any natural numbers $m$ and $p$, we have $m \in m + p^+$.
2023-08-09 13:39:41 +00:00
\end{proof}
\subsection{\verified{Exercise 4.23}}%
\hyperlabel{sub:exercise-4.23}
2023-08-09 13:39:41 +00:00
Assume that $m$ and $n$ are natural numbers with $m$ less than $n$.
Show that there is some $p$ in $\omega$ for which $m + p^+ = n$.
(It follows from this and the preceding exercise that $m$ is less than $n$ iff
2023-08-09 13:39:41 +00:00
$(\exists p \in \omega)m + p^+ = n$.)
\code*{Bookshelf/Enderton/Set/Chapter\_4}
{Enderton.Set.Chapter\_4.exercise\_4\_23}
2023-08-09 13:39:41 +00:00
\begin{proof}
Let $$S = \{n \in \omega \mid
(\forall m \in n, \exists p \in \omega) m + p^+ = n\}.$$
We prove that (i) $0 \in S$ and (ii) if $p \in S$, then $p^+ \in S$.
Afterward we prove (iii) the theorem statement.
\paragraph{(i)}%
\hyperlabel{par:exercise-4.23-i}
It vacuously holds that $0 \in S$.
\paragraph{(ii)}%
\hyperlabel{par:exercise-4.23-ii}
Suppose $n \in S$.
Let $m \in n^+$.
By definition of the \nameref{ref:successor}, $m \in n$ or $m = n$.
If $m \in n$, then there exists some $p \in \omega$ such that
$m + p^+ = n$.
But then \nameref{sub:theorem-4i} implies
\begin{align*}
n^+
& = (m + p^+)^+ \\
& = m + p^{++}.
\end{align*}
If instead $m = n$, then \nameref{sub:successor-identity} implies that
$$n^+ = m^+ = m + 1 = m + 0^+.$$
Hence $n^+ \in S$.
\paragraph{(iii)}%
By \nameref{par:exercise-4.23-i} and \nameref{par:exercise-4.23-ii}, $S$
is an \nameref{ref:inductive-set}.
By \nameref{sub:theorem-4b}, $S = \omega$.
Thus for all $m, n \in \omega$ such that $m \in n$, there exists some
$p \in \omega$ such that $m + p^+ = n$.
2023-08-09 13:39:41 +00:00
\end{proof}
\subsection{\verified{Exercise 4.24}}%
\hyperlabel{sub:exercise-4.24}
2023-08-09 13:39:41 +00:00
Assume that $m + n = p + q$.
Show that $$m \in p \iff q \in n.$$
\code*{Bookshelf/Enderton/Set/Chapter\_4}
{Enderton.Set.Chapter\_4.exercise\_4\_24}
2023-08-09 13:39:41 +00:00
\begin{proof}
Let $m, n, p, q \in \omega$ such that $m + n = p + q$.
\paragraph{($\Rightarrow$)}%
Suppose $m \in p$.
By \nameref{sub:theorem-4n}, $m + n \in p + n$.
By hypothesis, $m + n = p + q$ meaning $p + q \in p + n$.
By \nameref{sub:theorem-4k-2}, $q + p \in n + p$.
Therefore another application of \nameref{sub:theorem-4n} implies
$q \in n$.
\paragraph{($\Leftarrow$)}%
Suppose $q \in n$.
By \nameref{sub:theorem-4n}, $q + p \in n + p$.
By \nameref{sub:theorem-4k-2}, $p + q \in p + n$.
By hypothesis, $p + q = m + n$ meaning $m + n \in p + n$.
Therefore another application of \nameref{sub:theorem-4n} implies
$m \in p$.
2023-08-09 13:39:41 +00:00
\end{proof}
2023-08-12 18:54:22 +00:00
\subsection{\verified{Exercise 4.25}}%
\hyperlabel{sub:exercise-4.25}
2023-08-09 13:39:41 +00:00
Assume that $n \in m$ and $q \in p$.
Show that $$(m \cdot q) + (n \cdot p) \in (m \cdot p) + (n \cdot q).$$
[\textit{Suggestion:} Use \nameref{sub:exercise-4.23}.]
2023-08-12 18:54:22 +00:00
\code*{Bookshelf/Enderton/Set/Chapter\_4}
{Enderton.Set.Chapter\_4.exercise\_4\_25}
2023-08-09 13:39:41 +00:00
\begin{proof}
2023-08-12 18:54:22 +00:00
Let $n \in m$ and $q \in p$.
By \nameref{sub:exercise-4.23}, there exists some $r \in \omega$ such that
$q + r^+ = p$.
Then
\begin{align*}
& \qquad\quad n \in m \\
& \iff n \cdot r^+ \in m \cdot r^+ & \textref{sub:theorem-4n} \\
& \iff (n \cdot r^+) + ((m \cdot q) + (n \cdot q))
& \textref{sub:theorem-4n} \\
& \qquad\qquad \in (m \cdot r^+) + ((m \cdot q) + (n \cdot q)) \\
& \iff ((m \cdot q) + (n \cdot q)) + (n \cdot r^+)
& \textref{sub:theorem-4k-2} \\
& \qquad\qquad \in ((m \cdot q) + (n \cdot q)) + (m \cdot r^+) \\
& \iff (m \cdot q) + ((n \cdot q) + (n \cdot r^+))
& \textref{sub:theorem-4k-1} \\
& \qquad\qquad \in ((m \cdot q) + (n \cdot q)) + (m \cdot r^+) \\
& \iff (m \cdot q) + ((n \cdot q) + (n \cdot r^+))
& \textref{sub:theorem-4k-2} \\
& \qquad\qquad \in ((n \cdot q) + (m \cdot q)) + (m \cdot r^+) \\
& \iff (m \cdot q) + ((n \cdot q) + (n \cdot r^+))
& \textref{sub:theorem-4k-1} \\
& \qquad\qquad \in (n \cdot q) + ((m \cdot q) + (m \cdot r^+)) \\
& \iff (m \cdot q) + (n \cdot (q + r^+))
& \textref{sub:theorem-4k-3} \\
& \qquad\qquad \in (n \cdot q) + (m \cdot (q + r^+)) \\
& \iff (m \cdot q) + (n \cdot p) \in (n \cdot q) + (m \cdot p) \\
& \iff (m \cdot q) + (n \cdot p) \in (m \cdot p) + (n \cdot q).
& \textref{sub:theorem-4k-2}
\end{align*}
2023-08-09 13:39:41 +00:00
\end{proof}
2023-08-12 18:54:22 +00:00
\subsection{\unverified{Exercise 4.26}}%
\hyperlabel{sub:exercise-4.26}
2023-08-09 13:39:41 +00:00
Assume that $n \in \omega$ and $f \colon n^+ \rightarrow \omega$.
Show that $\ran{f}$ has a largest element.
2023-08-09 13:39:41 +00:00
\begin{proof}
2023-08-12 18:54:22 +00:00
By construction, the \nameref{ref:domain} of $f$ is finite.
Therefore $\abs{\ran{f}} \leq \abs{\dom{f}}$, i.e. $\ran{f}$ is also a
finite set.
By the \nameref{sub:trichotomy-law-natural-numbers}, every member of
$\ran{f}$ relates to one another.
Thus there must exist a largest element.
2023-08-09 13:39:41 +00:00
\end{proof}
\subsection{\sorry{Exercise 4.27}}%
\hyperlabel{sub:exercise-4.27}
2023-08-09 13:39:41 +00:00
Assume that $A$ is a set, $G$ is a function, and $f_1$ and $f_2$ map $\omega$
into $A$.
Further assume that for each $n \in \omega$ both $f_1 \restriction n$ and
$f_2 \restriction n$ belong to $\dom{G}$ and
$$f_1(n) = G(f_1 \restriction n) \land f_2(n) = G(f_2 \restriction n).$$
Show that $f_1 = f_2$.
2023-08-09 13:39:41 +00:00
\begin{proof}
TODO
\end{proof}
\subsection{\sorry{Exercise 4.28}}%
\hyperlabel{sub:exercise-4.28}
2023-08-09 13:39:41 +00:00
Rewrite the proof of \nameref{sub:theorem-4g} using, in place of induction,
the well-ordering of $\omega$.
2023-08-09 13:39:41 +00:00
\begin{proof}
TODO
\end{proof}
\setcounter{chapter}{5}
\chapter{Cardinal Numbers and the Axiom of Choice}%
\hyperlabel{chap:cardinal-numbers-axiom-choice}
\section{Equinumerosity}%
\hyperlabel{sec:equinumerosity}
\subsection{\verified{Theorem 6A}}%
\hyperlabel{sub:theorem-6a}
\begin{theorem}[6A]
For any sets $A$, $B$, and $C$,
\begin{enumerate}[(a)]
\item $A \equin A$.
\item If $A \equin B$, then $B \equin A$.
\item If $A \equin B$ and $B \equin C$, then $A \equin C$.
\end{enumerate}
\end{theorem}
\code{Common/Set/Finite}
{Set.equinumerous\_refl}
\code{Common/Set/Finite}
{Set.equinumerous\_symm}
\code{Common/Set/Finite}
{Set.equinumerous\_trans}
\begin{proof}
Let $A$, $B$, and $C$ be arbitrary sets.
\paragraph{(a)}%
Consider \nameref{ref:function} $I_A \colon A \rightarrow A$ given by
$I_A(x) = x$.
$I_A$ is trivially a one-to-one correspondence between $A$ and $A$.
Thus $A$ is \nameref{ref:equinumerous} to $A$.
\paragraph{(b)}%
Suppose $A \equin B$.
Then there exists a one-to-one correspondence $F$ between $A$ and $B$.
Consider now \nameref{ref:inverse} $$F^{-1} = \{\tuple{u, v} \mid vFu\}.$$
By \nameref{sub:one-to-one-inverse}, $F^{-1}$ is a one-to-one function.
For all $y \in A$, $\tuple{y, F(y)} \in F$.
Then $\tuple{F(y), y} \in F^{-1}$ meaning $F^{-1}$ is onto $A$.
Hence $F^{-1}$ is a one-to-one correspondence between $B$ and $A$, i.e.
$B \equin A$.
\paragraph{(c)}%
Suppose $A \equin B$ and $B \equin C$.
Then there exists a one-to-one correspondence $G$ between $A$ and $B$ and
a one-to-one correspondence $F$ between $B$ and $C$.
By \nameref{sub:one-to-one-composition}, $F \circ G$ is a one-to-one
function.
Thus we're left with proving $F \circ G$ is onto $C$.
Let $y \in C$.
Since $F$ is onto $C$, there exists some $t \in B$ such that $F(t) = y$.
Likewise, since $G$ is onto $B$, there exists some $x \in A$ such that
$G(x) = t$.
Then $F(G(x)) = y$.
Thus $\ran{(F \circ G)} = C$ meaning $F \circ G$ is onto $C$.
Hence $F \circ G$ is a one-to-one correspondence function between $A$ and
$C$, i.e. $A \equin C$.
\end{proof}
\subsection{\verified{Theorem 6B}}%
\hyperlabel{sub:theorem-6b}
\begin{theorem}[6B]
No set is \nameref{ref:equinumerous} to its powerset.
\end{theorem}
\code*{Bookshelf/Enderton/Set/Chapter\_6}
{Enderton.Set.Chapter\_6.theorem\_6b}
\begin{proof}
Let $A$ be an arbitrary set and $f \colon A \rightarrow \powerset{A}$.
Define $\phi = \{a \in A \mid a \not\in f(a)\}$.
Clearly $\phi \in \powerset{A}$.
Furthermore, for all $a \in A$, $\phi \neq f(a)$ since $a \in \phi$ if and
only if $a \not\in f(a)$.
Thus $f$ cannot be onto $\powerset{A}$.
Since $f$ was arbitrarily chosen, there exists no one-to-one correspondence
between $A$ and $\powerset{A}$.
Since $A$ was arbitrarily chosen, there is no set equinumerous to its
powerset.
\end{proof}
\section{Finite Sets}%
\hyperlabel{sec:finite-sets}
2023-09-14 15:00:28 +00:00
\subsection{\verified{Pigeonhole Principle}}%
\hyperlabel{sub:pigeonhole-principle}
\begin{theorem}
No natural number is equinumerous to a proper subset of itself.
\end{theorem}
2023-09-14 15:00:28 +00:00
\code{Bookshelf/Enderton/Set/Chapter\_6}
{Enderton.Set.Chapter\_6.pigeonhole\_principle}
\lean{Mathlib/Data/Finset/Card}
{Finset.exists\_ne\_map\_eq\_of\_card\_lt\_of\_maps\_to}
\begin{proof}
Let
\begin{equation}
\hyperlabel{sub:pigeonhole-principle-eq1}
S = \{n \in \omega \mid
\forall M \subset n, \text{every one-to-one function }
f \colon M \rightarrow n \text{ is not onto}\}.
\end{equation}
We show that (i) $0 \in S$ and (ii) if $n \in S$, then so is $n^+$.
Afterward we prove (iii) the theorem statement.
\paragraph{(i)}%
\hyperlabel{par:pigeonhole-principle-i}
By definition, $0 = \emptyset$.
Then $0$ has no proper subsets.
Hence $0 \in S$ vacuously.
\paragraph{(ii)}%
\hyperlabel{par:pigeonhole-principle-ii}
Suppose $n \in S$ and $M \subset n^+$.
Furthermore, let $f \colon M \rightarrow n^+$ be a one-to-one
\nameref{ref:function}.
If $M = \emptyset$, it vacuously holds that $f$ is not onto $n^+$.
Otherwise $M \neq \emptyset$.
Because $M$ is finite, the \nameref{sub:trichotomy-law-natural-numbers}
implies the existence of a largest member $p \in M$.
There are two cases to consider:
\subparagraph{Case 1}%
2023-09-14 15:00:28 +00:00
Suppose $n \not\in \ran{f}$.
Then $f$ is not onto $n^+$.
\subparagraph{Case 2}%
2023-09-14 15:00:28 +00:00
Suppose $n \in \ran{f}$.
Then there exists some $t \in M$ such that $\tuple{t, n} \in f$.
Define $f' \colon M \rightarrow n^+$ given by
2023-09-14 15:00:28 +00:00
\begin{align*}
f'(p) & = f(t) = n \\
f'(t) & = f(p) \\
f'(x) & = f(x) & \text{for all other } x.
\end{align*}
That is, $f'$ is a variant of $f$ in which the largest element of its
domain (i.e. $p$) corresponds to value $n$.
Next define $g = f' - \{\tuple{p, n}\}$.
Then $g$ is a function mapping $M - \{p\}$ to $n$.
2023-09-14 15:00:28 +00:00
Since $f$ is one-to-one, $f'$ and $g$ are also one-to-one.
Then \eqref{sub:pigeonhole-principle-eq1} indicates $g$ must not be onto
2023-09-14 15:00:28 +00:00
$n$.
That is, there exists some $a \in n$ such that $a \not\in \ran{g}$.
By the \nameref{sub:trichotomy-law-natural-numbers}, $a \neq n$.
Therefore $a \not\in \ran{f'}$.
$\ran{f'} = \ran{f}$ meaning $a \not\in \ran{f}$.
Because $a \in n \in n^+$, \nameref{sub:theorem-4f} implies $a \in n^+$.
Hence $f$ is not onto $n^+$.
\subparagraph{Subconclusion}%
The foregoing cases are exhaustive.
Hence $n^+ \in S$.
\paragraph{(iii)}%
By \nameref{par:pigeonhole-principle-i} and
\nameref{par:pigeonhole-principle-ii}, $S$ is an
\nameref{ref:inductive-set}.
By \nameref{sub:theorem-4b}, $S = \omega$.
Thus for all natural numbers $n$, there is no one-to-one correspondence
between $n$ and a proper subset of $n$.
In other words, no natural number is equinumerous to a proper subset of
itself.
\end{proof}
\subsection{\verified{Corollary 6C}}%
\hyperlabel{sub:corollary-6c}
\begin{corollary}[6C]
No finite set is equinumerous to a proper subset of itself.
\end{corollary}
\code{Bookshelf/Enderton/Set/Chapter\_6}
{Enderton.Set.Chapter\_6.corollary\_6c}
\begin{proof}
Let $S$ be a \nameref{ref:finite-set} and $S'$ be a
\nameref{ref:proper-subset} of $S$.
Then there exists some set $T$, disjoint from $S'$, such that
$S' \cup T = S$.
By definition of a \nameref{ref:finite-set}, $S$ is
\nameref{ref:equinumerous} to a natural number $n$.
By \nameref{sub:theorem-6a}, $S' \cup T \equin S$ which, by the same
theorem, implies $S' \cup T \equin n$.
Let $f$ be a one-to-one correspondence between $S' \cup T$ and $n$.
Then $f \restriction S'$ is a one-to-one correspondence between $S'$ and a
proper subset of $n$.
By the \nameref{sub:pigeonhole-principle}, $n$ is not equinumerous to any
proper subset of itself.
Therefore \nameref{sub:theorem-6a} implies $S'$ cannot be equinumerous to
$n$, which, by the same theorem, implies $S'$ cannot be equinumerous to
$S$.
Hence no finite set is equinumerous to a proper subset of itself.
\end{proof}
\subsection{\verified{Corollary 6D}}%
\hyperlabel{sub:corollary-6d}
\begin{corollary}[6D]
\ % Force a newline.
\begin{enumerate}[(a)]
\item Any set equinumerous to a proper subset of itself is infinite.
\item The set $\omega$ is infinite.
\end{enumerate}
\end{corollary}
\code{Bookshelf/Enderton/Set/Chapter\_6}
{Enderton.Set.Chapter\_6.corollary\_6d\_a}
\code{Bookshelf/Enderton/Set/Chapter\_6}
{Enderton.Set.Chapter\_6.corollary\_6d\_b}
\begin{proof}
\paragraph{(a)}%
\hyperlabel{par:corollary-6d-a}
Let $S$ be a set \nameref{ref:equinumerous} to \nameref{ref:proper-subset}
$S'$ of itself.
Then $S$ cannot be a \nameref{ref:finite-set} by
\nameref{sub:corollary-6c}.
By definition, $S$ is an \nameref{ref:infinite-set}.
\paragraph{(b)}%
Consider set $S = \{n \in \omega \mid n \text{ is even}\}$.
We prove that (i) $S$ is \nameref{ref:equinumerous} to $\omega$ and (ii)
that $\omega$ is infinite.
\subparagraph{(i)}%
\hyperlabel{spar:corollary-6d-i}
Define $f \colon \omega \rightarrow S$ given by $f(n) = 2 \cdot n$.
Notice $f$ is well-defined by the definition of an even natural number,
introduced in \nameref{sub:exercise-4.14}.
We first show $f$ is one-to-one and then that $f$ is onto.
Suppose $f(n_1) = f(n_2) = 2 \cdot n_1$.
We must prove that $n_1 = n_2$.
By the \nameref{sub:trichotomy-law-natural-numbers}, exactly one of the
following may occur: $n_1 = n_2$, $n_1 < n_2$, or $n_2 < n_1$.
If $n_1 < n_2$, then \nameref{sub:theorem-4n} implies
$n_1 \cdot 2 < n_2 \cdot 2$.
\nameref{sub:theorem-4k-5} then indicates $2 \cdot n_1 < 2 \cdot n_2$,
a contradiction to $2 \cdot n_1 = 2 \cdot n_2$.
A parallel argument holds for when $n_2 < n_1$.
Thus $n_1 = n_2$.
Next, let $m \in S$.
That is, $m$ is an even number.
By definition, there exists some $n \in \omega$ such that
$m = 2 \cdot n$.
Thus $f(n) = m$.
\subparagraph{(ii)}%
By \nameref{spar:corollary-6d-i}, $\omega$ is equinumerous to a subset
of itself.
By \nameref{par:corollary-6d-a}, $\omega$ is infinite.
\end{proof}
\subsection{\verified{Corollary 6E}}%
\hyperlabel{sub:corollary-6e}
\begin{corollary}[6E]
Any finite set is equinumerous to a unique natural number.
\end{corollary}
\code{Bookshelf/Enderton/Set/Chapter\_6}
{Enderton.Set.Chapter\_6.corollary\_6e}
\begin{proof}
Let $S$ be a \nameref{ref:finite-set}.
By definition $S$ is equinumerous to a natural number $n$.
Suppose $S$ is equinumerous to another natural number $m$.
By \nameref{sub:trichotomy-law-natural-numbers}, exactly one of three
situations is possible: $n = m$, $n < m$, or $m < n$.
If $n < m$, then $m \equin S$ and $S \equin n$.
By \nameref{sub:theorem-6a}, it follows $m \equin n$.
But \nameref{sub:pigeonhole-principle} indicates no natural number is
equinumerous to a proper subset of itself, a contradiction.
If $m < n$, a parallel argument applies.
Hence $n = m$, proving every finite set is equinumerous to a unique natural
number.
\end{proof}
\subsection{\verified{Lemma 6F}}%
\hyperlabel{sub:lemma-6f}
\begin{lemma}[6F]
If $C$ is a proper subset of a natural number $n$, then $C \equin m$ for
some $m$ less than $n$.
\end{lemma}
\code{Bookshelf/Enderton/Set/Chapter\_6}
{Enderton.Set.Chapter\_6.lemma\_6f}
\begin{proof}
Let
\begin{equation}
\hyperlabel{sub:lemma-6f-eq1}
S = \{n \in \omega \mid \forall C \subset n,
\exists m < n \text{ such that } C \equin m\}.
\end{equation}
We prove that (i) $0 \in S$ and (ii) if $n \in S$ then $n^+ \in S$.
Afterward we prove (iii) the lemma statement.
\paragraph{(i)}%
\hyperlabel{par:lemma-6f-i}
By definition, $0 = \emptyset$.
Thus $0$ has no proper subsets.
Hence $0 \in S$ vacuously.
\paragraph{(ii)}%
\hyperlabel{par:lemma-6f-ii}
Suppose $n \in S$ and consider $n^+$.
By definition of the \nameref{ref:successor}, $n^+ = n \cup \{n\}$.
Let $C$ be an arbitrary, \nameref{ref:proper-subset} of $n^+$.
There are two cases to consider:
\subparagraph{Case 1}%
\hyperlabel{spar:lemma-6f-1}
Suppose $n \not\in C$.
Then $C \subseteq n$.
If $C$ is a proper subset of $n$, \eqref{sub:lemma-6f-eq1} implies $C$
is \nameref{ref:equinumerous} to some $m < n < n^+$.
If $C = n$, then \nameref{sub:theorem-6a} implies $C$ is equinumerous to
$n < n^+$.
\subparagraph{Case 2}%
Suppose $n \in C$.
Since $C$ is a proper subset of $n^+$, the set $n^+ - C$ is nonempty.
By \nameref{sub:well-ordering-natural-numbers}, $n^+ - C$ has a least
element, say $p$ (which does not equal $n$).
Consider now set $C' = (C - \{n\}) \cup \{p\}$.
By construction, $C' \subseteq n$.
As seen in \nameref{spar:lemma-6f-1}, $C'$ is equinumerous to some
$m < n^+$.
It suffices to show there exists a one-to-one correspondence between
$C'$ and $C$, since then \nameref{sub:theorem-6a} implies $C$ is
equinumerous to $m$ as well.
Function $f \colon C' \rightarrow C$ given by
$$f(x) = \begin{cases}
n & \text{if } x = p \\
x & \text{otherwise}
\end{cases}$$
is trivially one-to-one and onto as expected.
\paragraph{(iii)}%
By \nameref{par:lemma-6f-i} and \nameref{par:lemma-6f-ii}, $S$ is an
\nameref{ref:inductive-set}.
By \nameref{sub:theorem-4b}, $S = \omega$.
Therefore, for every proper subset $C$ of a natural number $n$, there
exists some $m < n$ such that $C \equin n$.
\end{proof}
\subsection{\verified{Corollary 6G}}%
\hyperlabel{sub:corollary-6g}
\begin{corollary}[6G]
Any subset of a finite set is finite.
\end{corollary}
\code{Bookshelf/Enderton/Set/Chapter\_6}
{Enderton.Set.Chapter\_6.corollary\_6g}
\begin{proof}
Let $S$ be a \nameref{ref:finite-set} and $S' \subseteq S$.
Clearly, if $S' = S$, then $S'$ is finite.
Therefore suppose $S'$ is a proper subset of $S$.
By definition of a finite set, $S$ is \nameref{ref:equinumerous} to some
natural number $n$.
Let $f$ be a one-to-one correspondence between $S$ and $n$.
Then $f \restriction S'$ is a one-to-one correspondence between $S'$ and
some proper subset of $n$.
By \nameref{sub:lemma-6f}, $\ran{(f \restriction S')}$ is equinumerous to
some $m < n$.
Then \nameref{sub:theorem-6a} indicates $S' \equin m$.
Hence $S'$ is a finite set.
\end{proof}
\subsection{\verified{Subset Size}}%
\hyperlabel{sub:subset-size}
\begin{lemma}
Let $A$ be a finite set and $B \subseteq A$.
Then there exist natural numbers $m, n \in \omega$ such that
$B \equin m$, $A \equin n$, and $m \leq n$.
\end{lemma}
\code{Bookshelf/Enderton/Set/Chapter\_6}
{Enderton.Set.Chapter\_6.subset\_size}
\begin{proof}
Let $A$ be a \nameref{ref:finite-set} and $B$ be a subset of $A$.
By \nameref{sub:corollary-6g}, $B$ must be finite.
By definition of a finite set, there exists natural numbers
$m, n \in \omega$ such that $B \equin m$ and $A \equin n$.
By \nameref{sub:trichotomy-law-natural-numbers}, it suffices to prove that
$m > n$ is not possible for then either $m < n$ or $m = n$.
For the sake of contradiction, assume $m > n$.
By definition of \nameref{ref:equinumerous}, there exists a one-to-one
correspondence between $B$ and $m$.
\nameref{sub:theorem-6a} indicates there then exists a one-to-one
correspondence $f$ between $m$ and $B$.
Likewise, there exists a one-to-one correspondence $g$ between $A$ and
$n$.
Define $h \colon A \rightarrow B$ as $h(x) = f(g(x))$ for all $x \in A$.
Since $n \subset m$ by \nameref{sub:corollary-4m}, $h$ is well-defined.
By \nameref{sub:one-to-one-composition}, $h$ must be one-to-one.
Thus $h$ is a one-to-one correspondence between $A$ and $\ran{h}$, i.e.
$A \equin \ran{h}$.
But $n < m$ meaning $\ran{h} \subset B$ which in turn is a proper subset
of $A$ by hypothesis.
\nameref{sub:corollary-6c} states no finite set is equinumerous to a
proper subset of itself, a contradiction.
\end{proof}
\subsection{\pending{Finite Domain and Range Size}}%
\hyperlabel{sub:finite-domain-range-size}
\begin{lemma}
Let $A$ and $B$ be finite sets and $f \colon A \rightarrow B$ be a function.
Then there exist natural numbers $m, n \in \omega$ such that
$\dom{f} \equin m$, $\ran{f} \equin n$, and $m \geq n$.
\end{lemma}
\begin{note}
This proof avoids the \nameref{ref:axiom-of-choice-1} because $A$ and $B$
are finite.
In particular, we are able to choose a "smallest" element of each preimage
set.
Contrast this to \nameref{sub:theorem-3j-b}.
\end{note}
\begin{proof}
Let $A$ and $B$ be \nameref{ref:finite-set}s and $f \colon A \rightarrow B$
be a function.
By definition of finite sets, there exists \nameref{ref:natural-number}s
$m, p \in \omega$ such that $A \equin m$ and $B \equin p$.
By definition of the \nameref{ref:domain} of a function, $\dom{f} = A$.
Thus $\dom{f} \equin m$.
By \nameref{sub:theorem-6a}, there exists a one-to-one correspondence $g$
between $m$ and $\dom{f} = A$.
For all $y \in \ran{f}$, consider $\img{f^{-1}}{\{y\}}$.
Let $$A_y = \{ x \in m \mid f(g(x)) = y \}.$$
Since $g$ is a one-to-one correspondence, it follows that
$A_y \equin \img{f^{-1}}{\{y\}}$.
Since $A_y$ is a nonempty subset of natural numbers, the
\nameref{sub:well-ordering-natural-numbers} implies there exists a least
element, say $q_y$.
Define $C = \{q_y \mid y \in \ran{f}\}$.
Thus $h \colon C \rightarrow \ran{f}$ given by $h(x) = f(g(x))$ is a
one-to-one correspondence between $C$ and $\ran{f}$ by construction.
That is, $C \equin \ran{f}$.
By \nameref{sub:lemma-6f}, there exists some $n \leq m$ such that
$C \equin n$.
By \nameref{sub:theorem-6a}, $n \equin \ran{f}$.
\end{proof}
\subsection{\verified{Set Difference Size}}%
\hyperlabel{sub:set-difference-size}
\begin{lemma}
Let $A \equin m$ for some natural number $m$ and $B \subseteq A$.
2023-11-12 18:44:57 +00:00
Then there exists some $n \in \omega$ such that $n \leq m$, $B \equin n$,
and $A - B \equin m - n$.
\end{lemma}
\code{Bookshelf/Enderton/Set/Chapter\_6}
{Enderton.Set.Chapter\_6.sdiff\_size}
\begin{proof}
Let
\begin{equation}
\hyperlabel{sub:set-difference-size-ih}
S = \{m \in \omega \mid
\forall A \equin m, \forall B \subseteq A, \exists n
\in \omega (n \leq m \land B \equin n \land A - B \equin m - n)\}.
\end{equation}
We prove that (i) $0 \in S$ and (ii) if $n \in S$ then $n^+ \in S$.
Afterward we prove (iii) the lemma statement.
\paragraph{(i)}%
\hyperlabel{par:set-difference-size-i}
Let $A \equin 0$ and $B \subseteq A$.
Then it follows $A = B = \emptyset = 0$.
Since $0 \leq 0$, $B \equin 0$, and $A - B = \emptyset \equin 0 = 0 - 0$,
it follows $0 \in S$.
\paragraph{(ii)}%
\hyperlabel{par:set-difference-size-ii}
Suppose $m \in S$ and consider $m^+$.
Let $A \equin m^+$ and let $B \subseteq A$.
By definition of \nameref{ref:equinumerous}, there exists a one-to-one
correspondence $f$ between $A$ and $m^+$.
Since $f$ is one-to-one and onto, there exists a unique value $a \in A$
such that $f(a) = m$.
Then $B - \{a\} \subseteq A - \{a\}$ and $f$ is a one-to-one
correspondence between $A - \{a\}$ and $m$.
By \ihref{sub:set-difference-size-ih}, there exists some $n \in \omega$
such that $n \leq m$, $B - \{a\} \equin n$ and
\begin{equation}
\hyperlabel{par:set-difference-size-ii-eq1}
(A - \{a\}) - (B - \{a\}) \equin m - n.
\end{equation}
There are two cases to consider:
\subparagraph{Case 1}%
Assume $a \in B$.
Then $B \equin n^+$.
Furthermore, by definition of the set difference,
\begin{align}
(A - \{a\}) & - (B - \{a\}) \nonumber \\
& = \{x \mid
x \in A - \{a\} \land x \not\in B - \{a\}\} \nonumber \\
& = \{x \mid
(x \in A \land x \neq a) \land
\neg(x \in B \land x \neq a)\} \nonumber \\
& = \{x \mid
(x \in A \land x \neq a) \land
(x \not\in B \lor x = a)\} \nonumber \\
& = \{x \mid
(x \in A \land x \neq a \land x \not\in B) \lor
(x \in A \land x \neq a \land x = a)\} \nonumber \\
& = \{x \mid
(x \in A \land x \neq a \land x \not\in B) \lor F\} \nonumber \\
& = \{x \mid
(x \in A \land x \neq a \land x \not\in B)\} \nonumber \\
& = \{x \mid x \in A - B \land x \neq a\} \nonumber \\
& = \{x \mid x \in A - B \land x \not\in \{a\}\} \nonumber \\
& = (A - B) - \{a\}.
\hyperlabel{par:set-difference-size-ii-eq2}
\end{align}
Since $a \in A$ and $a \in B$, $(A - B) - \{a\} = A - B$.
Thus
\begin{align*}
(A - \{a\} - (B - \{a\})
& = (A - B) - \{a\} & \eqref{par:set-difference-size-ii-eq2} \\
& = A - B \\
& \equin m - n & \eqref{par:set-difference-size-ii-eq1} \\
2023-11-12 18:44:57 +00:00
& \equin m^+ - n^+.
\end{align*}
\subparagraph{Case 2}%
Assume $a \not\in B$.
Then $B - \{a\} = B$ (i.e. $B \approx n$) and
\begin{align*}
(A - \{a\}) - (B - \{a\})
& = (A - \{a\}) - B \\
& \equin m - n. & \eqref{par:set-difference-size-ii-eq1}
\end{align*}
The above implies that there exists a one-to-one correspondence $g$
between $(A - \{a\}) - B$ and $m - n$.
Therefore $g \cup \{\tuple{a, m}\}$ is a one-to-one correspondence
between $A - B$ and $(m - n) \cup \{m\}$.
2023-11-12 18:44:57 +00:00
Hence $$A - B \equin (m - n) \cup \{m\} \equin m^+ - n.$$
\subparagraph{Subconclusion}%
The above two cases are exhaustive and both conclude the existence of
some $n \in \omega$ such that $n \leq m^+$, $B \equin n$ and
$A - B \equin m^+ - n$.
Hence $m^+ \in S$.
\paragraph{(iii)}%
By \nameref{par:set-difference-size-i} and
\nameref{par:set-difference-size-ii}, $S \subseteq \omega$ is an
\nameref{ref:inductive-set}.
Thus \nameref{sub:theorem-4b} implies $S = \omega$.
Hence, for all $A \equin m$ for some $m \in \omega$, if $B \subseteq A$,
then there exists some $n \in \omega$ such that $n \leq m$,
$B \equin n$, and $A - B \equin m - n$.
\end{proof}
\subsection{\sorry{Theorem 6H}}%
\hyperlabel{sub:theorem-6h}
Assume that $K_1 \equin K_2$ and $L_1 \equin L_2$.
\begin{enumerate}[(a)]
\item If $K_1 \cap L_1 = K_2 \cap L_2 = \emptyset$, then
$K_1 \cup L_1 \equin K_2 \cup L_2$.
\item $K_1 \times L_1 \equin K_2 \times L_2$.
\item $^{(L_1)}{K_1} \equin ^{(L_2)}{K_2}$.
\end{enumerate}
\begin{proof}
TODO
\end{proof}
\subsection{\sorry{Theorem 6I}}%
\hyperlabel{sub:theorem-6i}
For any cardinal numbers $\kappa$, $\lambda$, and $\mu$:
\begin{enumerate}
\item $\kappa + \lambda = \lambda + \kappa$ and
$\kappa \cdot \lambda = \lambda \cdot \kappa$.
\item $\kappa + (\lambda + \mu) = (\kappa + \lambda) + \mu$ and
$\kappa \cdot (\lambda \cdot \mu) = (\kappa \cdot \lambda) \cdot \mu$.
\item $\kappa \cdot (\lambda + \mu) =
\kappa \cdot \lambda + \kappa \cdot \mu$.
\item $\kappa^{\lambda + \mu} = \kappa^\lambda \cdot \kappa^\mu$.
\item $(\kappa \cdot \lambda)^\mu = \kappa^\mu \cdot \lambda^\mu$.
\item $(\kappa^\lambda)^\mu = \kappa^{\lambda \cdot \mu}$.
\end{enumerate}
\begin{proof}
TODO
\end{proof}
\subsection{\sorry{Theorem 6J}}%
\hyperlabel{sub:theorem-6j}
Let $m$ and $n$ be finite cardinals.
Then
\begin{align*}
m + n & = m +_\omega n, \\
m \cdot n = m \cdot_\omega n, \\
m^n = m^n,
\end{align*}
where on the right side we use the operations of $\omega$ defined via
recursion and on the left side we use the operations of cardinal arithmetic.
\begin{proof}
TODO
\end{proof}
\subsection{\sorry{Corollary 6K}}%
\hyperlabel{sub:corollary-6k}
If $A$ and $B$ are finite, then $A \cup B$, $A \times B$, and $^B{A}$ are also
finite.
\begin{proof}
TODO
\end{proof}
\section{Exercises 6}%
\hyperlabel{sec:exercises-6}
\subsection{\unverified{Exercise 6.1}}%
\hyperlabel{sub:exercise-6.1}
Show that the equation $$f(m, n) = 2^m(2n + 1) - 1$$ defines a one-one-one
correspondence between $\omega \times \omega$ and $\omega$.
\begin{proof}
We prove that (i) $f$ is one-to-one and (ii) $f$ is onto $\omega$.
\paragraph{(i)}%
Let $y \in \ran{f}$.
Then there exists $(m_1, n_1) \in \omega \times \omega$ such that
$f(m_1, n_1) = y$.
Suppose there exists $(m_2, n_2) \in \omega \times \omega$ such that
$f(m_2, n_2) = y$.
We show that $m_1 = m_2$ and $n_1 = n_2$.
By \nameref{sub:trichotomy-law-natural-numbers}, we know either
$m_1 \leq m_2$ or $m_2 \leq m_1$ and either $n_1 \leq n_2$ or
$n_2 \leq n_1$.
WLOG, assume $m_1 \leq m_2$ and $n_1 \leq n_2$.
Then by \nameref{sub:exercise-4.23}, there exists some $p \in \omega$ such
that $m_1 + p = m_2$.
Likewise, there exists some $q \in \omega$ such that $n_1 + q = n_2$.
Therefore
\begin{align*}
& f(m_1, n_1) = f(m_2, n_2) \\
& \iff 2^{m_1}(2n_1 + 1) - 1 = 2^{m_2}(2n_2 + 1) - 1 \\
& \iff 2^{m_1}(2n_1 + 1) = 2^{m_2}(2n_2 + 1) \\
& \iff 2^{m_1}(2n_1 + 1) = 2^{m_1+p}(2(n_1 + q) + 1) \\
& \iff 2^{m_1}(2n_1 + 1) = 2^{m_1}2^p(2n_1 + 2q + 1) \\
& \iff 2n_1 + 1 = 2^p(2n_1 + 2q + 1) \\
& \iff 2n_1 + 1 = 2^{p+1}n_1 + 2^{p+1}q + 2^p.
\end{align*}
Notice though that the right-hand side of the above equality is smallest
when $p = 0$ and $q = 0$.
Since when evaluated at $p = 0$ and $q = 0$, the equality holds, it
follows this value of $p$ and $q$ are the only possible values that
$p$ and $q$ can take on.
Thus $m_1 + p = m_1 = m_2$ and $n_1 + q = n_1 = n_2$.
\paragraph{(ii)}%
We show $\ran{f} = \omega$ by the
\nameref{sub:strong-induction-principle-natural-numbers}.
It is clear that $\ran{f}$ is a subset of $\omega$.
Let $x \in \omega$ and suppose every number less than $x$ is in $\ran{f}$.
All that remains to be shown is that $x \in \ran{f}$.
By \nameref{sub:exercise-4.14}, there are two cases to consider:
\subparagraph{Case 1}%
Suppose $x$ is an even number.
Then $x = 2 \cdot y$ for some $y \in \omega$.
Then $f(0, y) = 2^0(2y + 1) - 1 = 2y = x$.
Hence $x \in \ran{f}$.
\subparagraph{Case 2}%
Suppose $x$ is an odd number.
Then $x = (2 \cdot y) + 1$ for some $y$.
It immediately follows $x - 1$ is an even number, meaning there exists
some $z$ such that $x - 1 = 2z$.
Since $z \in x$, the induction hypothesis states that there exists
some $m, n \in \omega$ such that $$f(m, n) = z = 2^m(2n + 1) - 1.$$
Therefore
\begin{align*}
f(m + 1, n)
& = 2^{m+1}(2n + 1) - 1 \\
& = (2)(2^m)(2n + 1) - 1 \\
& = 2z + 1 \\
& = x.
\end{align*}
Hence $x \in \ran{f}$.
\end{proof}
\subsection{\unverified{Exercise 6.2}}%
\hyperlabel{sub:exercise-6.2}
Show that in Fig. 32 we have:
\begin{align*}
J(m, n)
& = [1 + 2 + \cdots + (m + n)] + m \\
& = \frac{1}{2}[(m + n)^2 + 3m + n].
\end{align*}
\begin{proof}
\begin{figure}[ht]
\label{sub:exercise-6-2-fig1}
\includegraphics[width=0.6\textwidth]{fig-32}
\centering
\end{figure}
Let $m, n \in \omega$.
We note the next point following $(m, n)$ that coincides with the $x$-axis
is $(m + n, 0)$.
We can then formulate the sum of points seen as
\begin{equation}
\hyperlabel{sub:exercise-5-2-eq1}
\left[ \sum_{i=0}^{m + n} (i + 1) \right] - n.
\end{equation}
All that remains is showing \eqref{sub:exercise-5-2-eq1} identifies with
the equation for $J$.
\eqref{sub:exercise-5-2-eq1} is an arithmetic series.
By \nameref{S:sub:sum-arithmetic-series},
\begin{align*}
\left[ \sum_{i=0}^{m + n} (i + 1) \right] - n
& = \frac{[(m + n) + 1][1 + (m + n + 1)]}{2} - n \\
& = \frac{[(m + n) + 1][(m + n) + 2]}{2} - n \\
& = \frac{(m + n)^2 + 2(m + n) + (m + n) - 2n}{2} \\
& = \frac{1}{2}[(m + n)^2 + 3m + n] \\
& = J(m, n).
\end{align*}
Hence $J$ is correctly defined.
\end{proof}
\subsection{\unverified{Exercise 6.3}}%
\hyperlabel{sub:exercise-6.3}
Find a one-to-one correspondence between the open unit interval $\ioo{0}{1}$
and $\mathbb{R}$ that takes rationals to rationals and irrationals to
irrationals.
\begin{proof}
\begin{figure}[ht]
\label{sub:exercise-6-3-fig1}
\includegraphics[width=0.6\textwidth]{exercise-6.3.png}
\centering
\end{figure}
Consider function $f \colon (0, 1) \rightarrow \mathbb{R}$ given by
$$f(x) = \begin{cases}
\frac{1}{2x} - 1 & \text{if } x \leq \frac{1}{2} \\
\frac{1}{2(x - 1)} + 1 & \text{otherwise}.
\end{cases}$$
We prove that (i) $f$ is a one-to-one into $\mathbb{R}$, (ii) $f$ is onto
$\mathbb{R}$, and (iii) $f$ takes rationals to rationals and irrationals
to irrationals.
\paragraph{(i)}%
Before proceeding, consider the solutions to the following identity:
\begin{align*}
\frac{1}{2x} - 1 = \frac{1}{2(x - 1)} + 1
& \iff -8x^2 + 8x - 2 = 0 \\
& \iff 4x^2 - 4x + 1 = 0.
\end{align*}
Applying the quadratic equation shows $x = 1 / 2$ is the only solution.
Thus for any $x_1, x_2 \in \ioo{0}{1}$ such that $f(x_1) = f(x_2)$, it
must be that $x_1, x_2 \leq 1 / 2$ or $x_1, x_2 > 1 / 2$.
We now prove $f$ is one-to-one.
Let $y \in \ran{f}$.
Then there exists some $x_1 \in \ioo{0}{1}$ such that $f(x_1) = y$.
Suppose there exists some $x_2 \in \ioo{0}{1}$ such that $f(x_1) = f(x_2)$.
We prove that $x_1 = x_2$ by case analysis:
\subparagraph{Case 1}%
Suppose $x_1, x_2 \leq 1 / 2$.
Then $$\frac{1}{2x_1} - 1 = \frac{1}{2x_2} - 1$$ which straightforwardly
simplifies to $x_1 = x_2$.
\subparagraph{Case 2}%
Suppose $x_1, x_2 > 1 / 2$.
Then $$\frac{1}{2(x_1 - 1)} + 1 = \frac{1}{2(x_2 - 1)} + 1$$ also
straightfowardly simplies to $x_1 = x_2$.
\subparagraph{Subconclusion}%
The above cases are exhaustive.
Therefore $f(x_1) = f(x_2)$ implies $x_1 = x_2$.
Hence $f$ is one-to-one.
\paragraph{(ii)}%
Let $y \in \mathbb{R}$.
We prove that there exists an $x \in (0, 1)$ such that $f(x) = y$.
There are three cases we consider:
\subparagraph{Case 1}%
Suppose $y = 0$.
Then $f(1 / 2) = 0 = y$ is a readily identifiable solution.
\subparagraph{Case 2}%
Suppose $y > 0$.
Consider $x = \frac{1}{2(y + 1)}$.
We note that $x < 1 / 2$ meaning
\begin{align*}
f(x)
& = f\left(\frac{1}{2(y + 1)}\right) \\
& = \frac{1}{2(\frac{1}{2(y + 1)})} - 1 \\
& = (y + 1) - 1 \\
& = y.
\end{align*}
\subparagraph{Case 3}%
Suppose $y < 0$.
Consider $x = \frac{1}{2(y - 1)} + 1$.
We note that $x > 1 / 2$ meaning
\begin{align*}
f(x)
& = f\left(\frac{1}{2(y - 1)} + 1\right) \\
& = \frac{1}{2((\frac{1}{2(y - 1)} + 1) - 1)} + 1 \\
& = (y - 1) + 1 \\
& = y.
\end{align*}
\subparagraph{Subconclusion}%
The above three cases are exhaustive.
Thus $\ran{f} = \mathbb{R}$, i.e. $f$ is onto $\mathbb{R}$.
\paragraph{(iii)}%
Let $x \in (0, 1)$.
There are two cases to consider:
\subparagraph{Case 1}%
Suppose $x$ is a rational number.
Then there exist integers $m$ and $n$ such that $x = m / n$.
If $x \leq 1 / 2$, then
\begin{align*}
f(x)
& = f(m / n) \\
& = \frac{1}{2\left(\frac{m}{n}\right)} - 1 \\
& = \frac{n}{2m} - 1 \\
& = \frac{n - 2m}{2m}.
\end{align*}
Since $n - 2m$ and $2m$ are integers, $f(x)$ is a rational number.
If instead $x > 1 / 2$, then
\begin{align*}
f(x)
& = f(m / n) \\
& = \frac{1}{2\left(\frac{m}{n} - 1\right)} + 1 \\
& = \frac{n}{2(m - n)} + 1 \\
& = \frac{n + 2(m - n)}{2(m - n)}.
\end{align*}
Since $n + 2(m - n)$ and $2(m - n)$ are integers, $f(x)$ is again a
rational number.
Thus $f$ maps every rational number to a rational number.
\subparagraph{Case 2}%
Suppose $x$ is an irrational number.
First, consider the case where $x \leq 1 / 2$ and, for the sake of
contradiction, suppose $f(x)$ was a rational number.
Then there exist integers $m$ and $n$ such that
$$f(x) = \frac{1}{2x} - 1 = \frac{m}{n}.$$
But this would imply $$x = \frac{n}{2(m + n)},$$ a contradiction.
Thus $f(x)$ must be irrational.
Likewise, consider the case where $x > 1 / 2$.
Again, for the sake of contradiction, suppose $f(x)$ was a rational
number.
Then there exist integers $m$ and $n$ such that
$$f(x) = \frac{1}{2(x - 1)} + 1 = \frac{m}{n}.$$
But this would imply $$x = \frac{n}{2(m - n)} + 1,$$ a contradiction.
Thus $f(x)$ must be irrational.
Hence $f$ maps every irrational number to an irrational number.
\end{proof}
\subsection{\sorry{Exercise 6.4}}%
\hyperlabel{sub:exercise-6.4}
Construct a one-to-one correspondence between the closed unit interval
$$\icc{0}{1} = \{x \in \mathbb{R} \mid 0 \leq x \leq 1\}$$
and the open unit interval $\ioo{0}{1}$.
\begin{proof}
TODO
\end{proof}
\subsection{\verified{Exercise 6.5}}%
\hyperlabel{sub:exercise-6.5}
Prove \nameref{sub:theorem-6a}.
\begin{proof}
Refer to \nameref{sub:theorem-6a}.
\end{proof}
\subsection{\unverified{Exercise 6.6}}%
\hyperlabel{sub:exercise-6.6}
Let $\kappa$ be a nonzero cardinal number.
Show there does not exist a set to which every set of cardinality $\kappa$
belongs.
\begin{proof}
Let $\kappa$ be a nonzero cardinal number and define
$$\mathbf{K}_\kappa = \{ X \mid \card{X} = \kappa \}.$$
For the sake of contradiction, suppose $\mathbf{K}_\kappa$ is a set.
Then the \nameref{ref:union-axiom} suggests $\bigcup \mathbf{K}_{\kappa}$ is
a set.
But this "set" is precisely the class of all sets, which is \textit{not} a
set.
Thus our original assumption was incorrect.
That is, there does not exist a set to which every set of cardinality
$\kappa$ belongs.
\end{proof}
\subsection{\pending{Exercise 6.7}}%
\hyperlabel{sub:exercise-6.7}
Assume that $A$ is finite and $f \colon A \rightarrow A$.
Show that $f$ is one-to-one iff $\ran{f} = A$.
\code*{Bookshelf/Enderton/Set/Chapter\_6}
{Enderton.Set.Chapter\_6.exercise\_6\_7}
\begin{proof}
Let $A$ be a \nameref{ref:finite-set} and $f \colon A \rightarrow A$.
\paragraph{($\Rightarrow$)}%
Suppose $f$ is one-to-one.
Then $f$ is a one-to-one correspondence between $A$ and $\ran{f}$.
That is, $A \equin \ran{f}$.
Because $f$ maps $A$ onto $A$, $\ran{f} \subseteq A$.
Hence $\ran{f} \subset A$ or $\ran{f} = A$.
But, by \nameref{sub:corollary-6c}, $\ran{f}$ cannot be a proper subset of
$A$.
Thus $\ran{f} = A$.
\paragraph{($\Leftarrow$)}%
If $A = \emptyset$, then $f$ is trivially one-to-one.
Assume now $A \neq \emptyset$ and $\ran{f} = A$.
Let $x_1, x_2 \in A$ such that $f(x_1) = f(x_2) = y$ for some $y \in A$.
We must prove that $x_1 = x_2$.
Let $B = \img{f^{-1}}{\{y\}}$.
Then $x_1, x_2 \in B$.
Since $B \subseteq A$, \nameref{sub:subset-size} indicates that there
exist natural numbers $m_1, n_1 \in \omega$ such that $B \equin m_1$,
$A \equin n_1$, and $m_1 \leq n_1$.
Define $f' \colon (A - B) \rightarrow (A - \{y\})$ as the
\nameref{ref:restriction} of $f$ to $A - B$.
Since $\ran{f} = A$, it follows $\ran{f'} = A - \{y\}$.
Since \nameref{sub:corollary-6g} implies $A - B$ and $A - \{y\}$ are
finite sets, \nameref{sub:finite-domain-range-size} implies the
existence of natural numbers $m_2, n_2 \in \omega$ such that
$\dom{f'} \equin m_2$, $\ran{f'} \equin n_2$, and $m_2 \geq n_2$.
Thus, by \nameref{sub:set-difference-size},
\begin{align*}
m_2 & \equin \dom{f'} \equin A - B \equin n_1 - m_1 \\
n_2 & \equin \ran{f'} \equin A - \{y\} \equin n_1 - 1.
\end{align*}
By \nameref{sub:corollary-6e}, $m_2 = n_1 - m_1$ and $n_2 = n_1 - 1$.
Since $m_2 \geq n_2$, $n_1 - m_1 \geq n_1 - 1$.
But $1 \leq m_1 \leq n_1$ meaning $m_1 = 1$.
Hence $B$ is a singleton.
Therefore $x_1 = x_2$, i.e. $f$ is one-to-one.
\end{proof}
\subsection{\pending{Exercise 6.8}}%
\hyperlabel{sub:exercise-6.8}
Prove that the union of two finite sets is finite, without any use of
arithmetic.
\begin{proof}
TODO
\end{proof}
\subsection{\pending{Exercise 6.9}}%
\hyperlabel{sub:exercise-6.9}
Prove that the Cartesian product of two finite sets is finite, without any use
of arithmetic.
\begin{proof}
TODO
\end{proof}
\subsection{\sorry{Exercise 6.10}}%
\hyperlabel{sub:exercise-6.10}
Prove part 4 of \nameref{sub:theorem-6i}.
\begin{proof}
TODO
\end{proof}
\subsection{\sorry{Exercise 6.11}}%
\hyperlabel{sub:exercise-6.11}
Prove part 5 of \nameref{sub:theorem-6i}.
\begin{proof}
TODO
\end{proof}
\subsection{\sorry{Exercise 6.12}}%
\hyperlabel{sub:exercise-6.12}
The proof to \nameref{sub:theorem-6i} involves eight instances of showing two
sets to be equinumerous.
(The eight are listed in the proof of the theorem as statements numbered 1-6.)
In which of these eight cases does equality actually hold?
\begin{proof}
TODO
\end{proof}
\subsection{\sorry{Exercise 6.13}}%
\hyperlabel{sub:exercise-6.13}
Show that a finite union of finite sets is finite.
That is, show that if $B$ is a finite set whose members are themselves finite
sets, then $\bigcup{B}$ is finite.
\begin{proof}
TODO
\end{proof}
\subsection{\sorry{Exercise 6.14}}%
\hyperlabel{sub:exercise-6.14}
Define a \textit{permutation} of $K$ to be any one-to-one function from $K$
onto $K$.
We can the define the factorial operation on cardinal numbers by the equation
$$\kappa! = \card{\{f \mid f \text{ is a permutation of } K\}},$$
where $K$ is any set of cardinality $\kappa$.
Show that $\kappa!$ is well defined, i.e. the value of $\kappa!$ is
independent of just which set $K$ is chosen.
\begin{proof}
TODO
\end{proof}
\end{document}